90% found this document useful (21 votes)
73K views580 pages

Introduction To Geometry

Uploaded by

sayem
Copyright
© © All Rights Reserved
We take content rights seriously. If you suspect this is your content, claim it here.
Available Formats
Download as PDF, TXT or read online on Scribd
90% found this document useful (21 votes)
73K views580 pages

Introduction To Geometry

Uploaded by

sayem
Copyright
© © All Rights Reserved
We take content rights seriously. If you suspect this is your content, claim it here.
Available Formats
Download as PDF, TXT or read online on Scribd
You are on page 1/ 580

~ Introduction

Geometry |
3

as | Richard Rusczyk ae
pec

© 2006, 2007, 2009 AoPS Incorporated. All Rights Reserved.

Reproduction of any portion of this book without the written permission of AoPS Incorporated is strictly
prohibited, except for “fair use” or other noncommercial uses as defined in Sections 107 and 108 of the
U.S. Copyright Act.

Published by: AoPS Incorporated


P.O. Box 2185
Alpine, CA 91903-2185
(619) 659-1612
[email protected]

ISBN: 978-1-934124-08-6

Visit the Art of Problem Solving website at http://www. artofproblemsolving.com

Cover image designed by Vanessa Rusczyk using KaleidoTile software.

Cover includes a satellite image of the Mississippi River Delta from NASA Earth Observatory and a
photo of protractor and compasses by Vanessa Rusczyk.

Printed in the United States of America.

Second Printing, Second Edition 2009.

eee
eee
ll
HOW TO USE THIS BOOK

How to Use This Book

Learn by Solving Problems

This book is probably very different from most of the math books that you have read before. We believe
that the best way to learn mathematics is by solving problems. Lots and lots of problems. In fact, we
believe that the best way to learn mathematics is to try to solve problems that you don’t know how to
do. When you discover something on your own, you'll understand it much better than if someone just
tells it to you.
Most of the sections of this book begin with several problems. The solutions to these problems will
be covered in the text, but try to solve the problems before reading the section. If you can’t solve some
of the problems, that’s OK, because they will all be fully solved as you read the section. Even if you
solve all of the problems, it’s still important to read the section, both to make sure that your solution is
correct, and also because you may find that the book’s solution is simpler or easier to understand than
your own.
If you find that the problems are too easy, this means that you should try harder problems. Nobody
learns very much by solving problems that are too easy for them.

Explanation of Icons

Throughout the book, you will see various shaded boxes and icons.

Concept: This will be a general problem-solving technique or strategy. These are the
O=s “keys” to becoming a better problem solver!

| Important: — This will be something important that you should learn. It might be a
VY - formula, a solution technique, or a caution.

ARN ING! ; Beeware if


‘yous
sea this bad : This will pointout a common mistake or
or
: a pitfall.
HOW TOih
tt
A USE THIS BOOK
5
eee ae Eee

|Game: ~ Remember, math is fun! This box will contain a game to think about.
|
|
|
|
(:, ||
|
|
et Ne NI ERERON UNTER e pete)

Sidenote: This box will contain “material | which, although interesting, is not part of |
| SN the main material of the text. It’s OK to skip over these boxes, but if you |
read them, you might learn something interesting!
|
|

Bogus Solution: Just like the ae cube shown to the left, there’s something —
WORE with any “solution” that appears in this box. |
7 pee

Exercises, Review Problems, and Challenge Problems

Most sections end with several Exercises. These will test your understanding of the material that was
covered in the section that you just finished. You should try to solve all of the exercises. Exercises
marked with a x are more difficult.
Most chapters have several Review Problems. These are problems which test your basic under-
standing of the material covered in the chapter. Your goal should be to solve most or all of the Review
Problems for every chapter — if you’re unable to do this, it means that you haven't yet mastered the
material, and you should probably go back and read the chapter again.
All of the chapters end with Challenge Problems. These problems are generally more difficult than
the other problems in the book, and will really test your mastery of the material. Some of them are very,
very hard — the hardest ones are marked with a x. Don’t necessarily expect to be able to solve all of
the Challenge Problems on your first try — these are difficult problems even for experienced problem
solvers. If you are able to solve a large number of Challenge Problems, then congratulations, you are on
your way to becoming an expert problem solver!

Hints

Many problems come with one or more hints. You can look up the hints in the Hints section in the
back of the book. The hints are numbered in random order, so that when you're looking up a hint to a
problem you don’t accidentally glance at the hint to the next problem at the same time.
It is very important that you first try to solve the problem without resorting to the hints. Only after
you've seriously thought about a problem and are stuck should you seek a hint. Also, for problems
which have multiple hints, use the hints one at a time; don’t go to the second hint until you've thought
about the first one.
HOW TO USE THIS BOOK

Solutions

The solutions to all of the Exercises, Review Problems, and Challenge Problems are in the separate
solution book. If you are using this textbook in a regular school class, then your teacher may decide not
to make this solution book available to you, and instead present the solutions him/herself. However,
if you are using this book on your own to learn independently, then you probably have a copy of the
solution book, in which case there are some very important things to keep in mind:

1. Make sure that you make a serious attempt at the problem before looking at the solution. Don’t
use the solution book as a crutch to avoid really thinking about a problem first. You should think
hard about a problem before deciding to give up and look at the solution.

2. After you solve a problem, it’s usually a good idea to read the solution, even if you think you know
how to solve the problem. The solution that’s in the solution book might show you a quicker or
more concise way to solve the problem, or it might have a completely different solution method
that you might not have thought of.

3. If you have to look at the solution in order to solve a problem, make sure that you make a note of
that problem. Come back to it in a week or two to make sure that you are able to solve it on your
own, without resorting to the solution.

Resources

Here are some other good resources for you to further pursue your study of mathematics:

e The Art of Problem Solving books, by Sandor Lehoczky and Richard Rusczyk. Whereas the book
that you’re reading right now will go into great detail of one specific subject area — geometry — the
Art of Problem Solving books cover a wide range of problem solving topics across many different
areas of mathematics.
e The www.artofproblemsolving.com website. The authors of this book are also the webmasters of
the Art of Problem Solving website, which contains many resources for students:

— a discussion forum
online classes
resource lists of books, contests, and other websites
— a JATRX tutorial
— and much more!

e You can hone your problem solving skills (and perhaps win prizes!) by participating in various
math contests. For middle school students in the United States, the major contests are MATH-
COUNTS, MOEMS, and the AMC 8. For U.S. high school students, some of the best-known contests
are the AMC/AIME/USAMO series of contests (which choose the U.S. team for the International
HOW TO USE THIS BOOK

Mathematics Olympiad), the American Regions Math League (ARML), the Mandelbrot Competi-
tion, the Harvard-MIT Mathematics Tournament, and the USA Mathematical Talent Search. More
details about these contests are on page vii, and links to these and many other contests are available
on the Art of Problem Solving website.

A Note to Teachers

We believe that students learn best when they are challenged with hard problems that at first they may
not know how to do. This is the motivating philosophy behind this book.
Rather than first introducing new material and then giving students exercises, we present problems
at the start of each section that students should try to solve before the new material is presented. The
goal is to get students to discover the new material on their own. Often, complicated problems are
broken into smaller parts, so that students can discover new techniques one piece at a time. Then the
new material is formally presented in the text, and full solutions to each problem are explained, along
with problem-solving strategies.
We hope that teachers will find that their stronger students will discover most of the material in this
book on their own by working through the problems. Other students may learn better from a more
traditional approach of first seeing the new material, then working the problems. Teachers have the
flexibility to use either approach when teaching from this book.
The book is linear in coverage. Generally, students and teachers should progress straight through the
book in order, without skipping chapters. Sections denoted with a * contain supplementary material
that may be safely skipped. In general, chapters are not equal in length, so different chapters may take
different amounts of classroom time.

Extra! Occasionally, you'll see a box like this at the bottom of a page. This is an “Extra!” and
“meme might be a quote, some biographical or historical background, or perhaps an interesting
idea to think about.

Vi
ACKNOWLEDGEMENTS

Acknowledgements

Contests

We would like to thank the following contests for allowing us to use a selection of their problems in this
book:

e The American Mathematics Competitions, a series of contests for U.S. middle and high school
students. The AMC 8, AMC 10, and AMC 12 contests are multiple-choice tests which are taken by
over 400,000 students every year. Top scorers on the AMC 10 and AMC 12 are invited to take the
American Invitational Mathematics Examination (AIME), which is a more difficult, short-answer
contest. Approximately 10,000 students every year participate in the AIME. Then, based on the
results of the AMC and AIME contests, about 250 students are invited to participate in the USA
Mathematical Olympiad (USAMO), a 2-day, 9-hour examination in which each student must
show all of his or her work. Results from the USAMO are used to invite a number of students to
the Math Olympiad Summer Program, at which the U.S. team for the International Mathematical
Olympiad (IMO) is chosen. More information about the AMC contests can be found on the AMC
website at http: //www.unl.edu/amc.

e MATHCOUNTSS®, the premier contest for U.S. middle school students. MATHCOUNTS is a na-
tional enrichment, coaching, and competition program that promotes middle school mathematics
achievement through grassroots involvement in every U.S. state and territory. President George
W. Bush and former Presidents Clinton, Bush and Reagan have all recognized MATHCOUNTS
in White House ceremonies. The MATHCOUNTS program has also received two White House
citations as an outstanding private sector initiative. Particularly exciting for MATHCOUNTS
Mathletes® were the hour-long ESPN programs on each of the past three National Competitions.
More information is available at www.mathcounts. org.

e The Mandelbrot Competition, which was founded in 1990 by Sandor Lehoczky, Richard Rusczyk,
and Sam Vandervelde. The aim of the Mandelbrot Competition is to provide a challenging,
engaging mathematical experience which is both competitive and educational. Students compete
both as individuals and in teams. The Mandelbrot Competition is offered at the national level
for more advanced students and the regional level for less experienced problem solvers. More
information can be found at www.mandelbrot.org.

e The Harvard-MIT Mathematics Tournament (HMMT), which is an annual math tournament for
high school students, held at MIT and at Harvard in alternating years. It is run exclusively by MIT
and Harvard students, most of whom themselves participated in math contests in high school.
More information is available at web.mit.edu/hmmt/.

vii
ACKNOWLEDGEMENTS

e The USA Mathematical Talent Search (USAMTS), which was founded in 1989 by Professor
George Berzsenyi. The USAMTS is a free mathematics competition open to all United States
middle and high school students. As opposed to most mathematics competitions, the USAMTS
allows students a full month to work out their solutions. Carefully written justifications are
required for each problem. More information is available at www.usamts.org.
e The American Regions Math League (ARML), which was founded in 1976. The annual ARML
competition brings together nearly 2,000 of the nation’s finest students. They meet, compete
against, and socialize with one another, forming friendships and sharpening their mathematical
skills. The contest is written for high school students, although some exceptional junior high
students attend each year. The competition consists of several events, which include a team
round, a power question (in which a team solves proof-oriented questions), an individual round,
and two relay rounds. More information is available at www. arml . com.

How We Wrote This Book

This book was written using the TEX document processing system. Specifically, this book was prepared
using the MikTexX installation of pdflatex on a PC running Microsoft Windows XP. We must thank the
authors of the various 4TRX packages that we used while preparing this book, and also the brilliant
authors of The ATeX Companion for writing a reference book that is not only thorough but also very
readable. The diagrams were prepared using METAPOST, a powerful graphics language which is based
on Knuth’s METAFONT.

About Us

This book is a collaborative effort of the staff of the Art of Problem Solving. Richard Rusczyk was the
lead author for this book, and wrote most of the text. Some of the Sidebar and Extra sections were
prepared by Ashley Reiter Ahlin, Vanessa Rusczyk, and Naoki Sato. The solutions were written by
Ruozhou Jia, Brian Rice, Richard Rusczyk, and Naoki Sato. Extensive proofreading of the manuscript
was done by Mathew Crawford, Lisa Davis, Amanda Jones, David Patrick, Tim Lambert, Naoki Sato,
and Jake Wildstrom. Vanessa Rusczyk designed the cover and also contributed greatly to the interior
design of the book. David Patrick, Naoki Sato, Ravi Boppana, Meena Boppana, Valentin Vornicu, Greg
Brockman, Larry Evans, and Joseph Laurendi contributed problems and proofreading to this second
edition.
The author would also like to thank Josh Zucker, whose comments about how he learned mathematics
inspired the questions-before-the-lessons approach of the text.

Websites & Errata

We used several websites as source material for the text. Links to these sites, as well as an errata list for
the text and solutions, are provided at

http: //www.artofproblemsolving. com/BookLinks/IntroGeometry/links. php

viii
CONTENTS

Contents

How to Use This Book ili

Acknowledgements Vii

1 What's in a Name? 1
ki} Woydvamecesne Symbols i... o.oo: oe ae ge eel ola ee 2
13 Poieeeieines aNd PIANCS os Gs hose sk he ce A ee ce eee 2
Le ta erate WeVale d (oy6hels Pee ae ee en oe Pear me) ro 5
Pe Consmucuon: (
opy a oepment 22.24 oi ess eye, ee ae Z,
To Penadumcden of Proofs... ao 3. hee a oe eee eee ee 10
1.6 ULV hae be Peete =. >.» + 0,a % 4. Shabba bags paler A awepene ys ae 10

2 Angles 13
Sy VWhatssemAn cletaiat Vogts ck cae ee weOy ss RR ree 13
ae MAPBSUCINE ANCES oie, nec. e,mom yA a ey AS Seem ER... Be 14
255 ) otraignt and, Vertical Angles. 00... 0.¢+ + ocencsla. cs, ee: Hee letee ote TE 21
Dd eee orallel LINCS. oe cs a)2,an te Shydade ES. 14 i)hoe, ee eee. Ee 24
Be 4 Sauelesmpiairanglemig si tiew......4..0.4
botatesS wihalints Sep aintireneD .. 28 OL
ZG Y RRIOR ATIC 1OS hy ib us Veit a ee tenka a ae teteee oe A OE. ee 36
2.7. Parallel Lines Revisited ........ wakes!. aetidauctesibaanss copietens 3. Re 39
Ze we IMDANAT) a ASE Ges NNisa sicvigsec iknip1 a redane Ov Slemoms ely agva ao oc 42

3 Congruent Triangles 49
6 Al Ride PENGEL OT) Sue rere cevk cae faSee ols gSO ess Sac CO EWS eeoelaces Daly Rabk, OR a 49
S22 oes, Gonemcncey Laake, ana Laie Mine 6 eos selene . 5.5 50
OO EBON ETUCICE MI wyopiwiet nonce CA any vita oe RES & RNS olgis/, 2 «0.5 DO
CONTENTS

340 VASA ‘anid2AIAS Conertience ss 2 tote ee re reine a ieae 59


3.5 SSA Not-Necessarily Congruence ~... 7...) se os 63
3.6 Isosceles and Equilateral Irianglesy, 7 50 2-2 ac) ee le 65
3.7. Construction: Equilateral Triangle and Perpendicular Bisector ..........-..-- yes
Bie DOULILMNALY cre cee as Gert ten ra en ee ore eee alc ace ae 73

4 Perimeter and Area 81


Ale sPerimeter ona chs eo Fe ee 8 ee oes me oun Be enn or ee 81
CN Neh ae ee See Oe ee eee oe eee ee ee En a gr SY Oe 83
ACB ee Sdiilte base Oallie Altes: os eae eh te en in es 88
AA Pee OUTIMTIALY. pe Sate eetog) aie Saws: eee ses a ee Rta esGo my ee eT 92

5 Similar Triangles 99
rly autos OUilalily) tae oe hn Oe eee Sd rere ee ee eeSee eee en 99
Doi ASLO AL ee Bae oc RE a olloO et Ty oat olene St ee Nee ee 101
De OAS. OLINATH Vien teiegie. te DAR tar UIE ct «is Pohel atet at viet eer ee eee ee 108
DIAM OS OTOLUN LAT ICYea tontts* one estate tarote a? tate AO SR ae ee 113
DD mee Using sumilarity in Problems vr.<.* 4. %.*.*.* Sarees ct ee ee ee ee 114
Oommen Olstruction:-Angles and-Parallels «.\.~ 20+ .+se 5.55.05 sat.) a ee 121
DM 2ULOTIL Benet oa Re eeseseli tue oktina aan 2 whe aw Se a aeSe 124

6 Right Triangles 133


Cree Nae Orean COTO
| a s8 hems 8 tas ap 5 Gulot5S Soessa tons ORL CD 134
62 ee wo. opectal Right Triangles ay.vac snes yeic>ameras satis WE A ae, AS et 140
bom, ely thagorean. triples. 5, Mage Saiue Make’ bicrkas ses, Wee rtans Linens tl 147
Oe = Gongruenceand SiuiilanitysRevisited +. =r. Sera, A ee a oe 152
Box melieroms OTmUlA Mids r.Cetin 8+ ceedaa brn ead lee ewes. ee ena ORS OO, oe 156
6 ome Construction:-Perpendicular Lines sis hier cede Weel nae ole eee 161
Omer ULLAL wig ene erst Ree eeSRE PERN eecE NEE PEON S t058Arcochad wena en cl se 162

vf Special Parts of a Triangle 170


Mele DISECtOLS TTSede eek ye, Re A Se Se LS, ee, 08Br ee 170
Zoe »betpendicular Bisectors of a Triangle .is.92.14. 1... ot.140 <0. ee 173
7.3. “Angle Bisectorsiofa Triangle. x. 00). RUE L ee DAR ee, 1e7
anne ee eee SS SSS SS SSS SSS SSS SSS SSS SSS SSS SSS SS
CONTENTS
SSS SS

eM Cla mm gh NA Weve gh AW ihah te ipirnNes 80418 ahh aceoReaen Task eo LR, GAD eT 183
IN OCS ee Oe i, a ee a ee eS re ssc eee oe 188
Ake een OO eis ok ee hoe Se Ae Sy a BA wk OES Ge RS ae 192
Fee Poe CENT, SRC OTS mms tree THER Ok Gr Ake 4h Go eS. Gold 24 BRC S Fate See ee 197
Fe edna Roe, eynee ey a Bek a SG, 8 So eee es. Bw teed eS + a 199

8 Quadrilaterals 206
8.1 ROAULEL
DET O PSRUIG cewana. e ocrrd a ime aSekdoe cathe date pees retake eee ate ca a 2 EER rr ae 206
Bias SEL ePIIIS: oo astta Bye icus+case back wucieed.Wapee nan aeoeaeOd neces RUCTIREUD Ls Ro ar 208
oe BIR OP CAIS siandrenac- cos «save yeskirt tom Sere omieyh aecath 1 maaan ce 215
SRY MU on Seen acs Xe, Xs ee tae Satis 1 nee agente te ie ee eee ee pale)
UV) Poe 1G Cy ee ee ee ae ee ae ee i mee Se ey 221
BO AUTOS on Sea ws be Se a suet s ae ele la OREO a Se eee a oe 224
Bef oe AE BOCA LE ins rasgode naeae valsc'slecy wotey + Pvtac-a 0 tgee ae eR ene ee 227,
8.6%> Quadrilateral Problems: ..2..... <.cc. nce rte wet baeitaeieae
ass 232
Oe UIST Yaeaia +i gojepll oh2asok ke a yond fie Save AT > esa cd rt 236

9 Polygons 244
ed ES TPOCUICHION 10 POW CONS beck. sey nea a ut dntale Sele pe ge 244
Bie o . Neles te POLSON as ses ceu tnodes vay seu A dycae} es wtetany Teka eo) ee ee 246
op RG Ae ae naeoot eect one bar te ee ee eg oe a ey ee 249
A, sek Oly Proper oe a oi vi Se in gaa onpu ao ws SS a 254
Ue weAonetriuction: Reolar FOL PONS cai vasecs shes caves chen ts252mru=> aan gle ee 27
TEP ety Pe eee See eae ee ee aa aan Ame 8 an kee 260

10 Geometric Inequalities 266


MU em CSS ATI AIS LESAN at LIAN CLE. Gs. acwatens 2bips sefaxntnlenassts tycech<ms 6% co: MOR 266
10.2., Pythagoras — Not Just For,Right Triangles? ...........05.5...00.5........eeaige
. Eat 270
Tea OO MUAEY oo canmsinnareatsicmaiaraa slataatewnremtine Bindacninicisn minha alinelARNO tig nes 274
MEMORIAL Y doy wow glia caineseva a ARS«ni Rapa Ghd 54+ pO4+ 1%Dean Pays roe en be ey EEO oe Seren 280

11 Circles 284
d11>) AraMeasure, Arc Length, and Circumference... © ..6.i.0.¢. 20005... CERAM. 284

WN INGE ag Be Ne ern ae ec anh nn So: Sr 290


CONTENTS
CN

P13. Puriky Areas. iti cle snr ioe oy ten ohPee le koe io 294
114 .Summtiiary ress that. 8 292

12 Circles and Angles 304


121, Inscribed Angles uae sie cintan ele ceisler
ose. steps 304
12.2") Angeles Inside and Outside Circles’. . 25. sn et 2 ee a ee 310
12,3 Tangents hae Oe 2 ce ea 2 we See 2 ee 316
19% ~ Problemisy 40s. scseek wa. sc sala aeostote Suvi daeySolon teat oeteoeaeoma A 322
1275, — G@omStruction:, Tan Sents 5 ervinecuishmt ingen2 syohsusp Segegeiey ohow Anh ay ey ee oat
12,69 30 ITO gerne we MEN ge cpallonaga oo oh2 Hnagoeahok> Oy2h ee 330

13 Power of a Point 340


TSR ine Hisee Oner.O fray? Olnit 2c ese cncxde nse oon seaman p0Oedene ones cwoeeek Re REE SRO ok ae 340
132r oePower of aeboint Problems. «0.0.8.0 52c. kav sana: a2a0 o>. PRS 6 ees . ee 345
BerS PROLIF ALY EMeeenr, <i ox nord <Auhoestat arwrSkah aiei anaeanon 'ara asBE Gea de es <a e 351

14 Three-Dimensional Geometry 356


ASR lanes teeta wes. Saee PSSh ATR, ci csgescan cna vee ons ANOS. SF BORSA 2 oe 356
MPA tah AAR STINS MR © 0 diag Bae ateove akaesphohseesea OHSaeedaveamadine dhSn-cu wyonORMOND 5 OR es 360
PAS ce Valli Seco, S.boh cmcin8 asa spam es Vine oe4Aswnr Whoh wa Neh Re eanoeos ee 366
PP Ame KeCUlar POLY NCCTAt aie... <.Lie eedi= at.mods yaind and teethet nibaw ens MORRO. ONIN IOS a ae 370
of MMS TUT VINIATMa ee 9 oa, 0 os «vod sawn woterres 480 cory ORM g Yan A Aa ERS DD 374

15 Curved Surfaces 380


orl Cylinders Ame wy Soren 2 as ss . POR BRR PAM BISNIS. . * 380
WORE ¢COTES PRM ac MPsiedi5sine. ceitneinscerveesicd opaes oner eo RSE. @ One boaeeke . LOE 385
10 Opneresieieat 2 wien a dee... eae Nighi jedel4 «=emuaadiyt ..oe 390
OAS meEODICIIS® c2iiay binain viiwiedinsse Aawnue Gad. SaMbAbaba oeAdley ada anere MESURE Beene OAT. eae 394
Samm OULTULLLLATVp ussisplis Valsairooreds wsSasseSle.s sey 'si's'sodUndssas v'swsWasweghh-aeae84a 402

16 The More Things Change... 408


lol . Translationsm: yatta ala. enderveww levi bestest ok Oren. 2 Et 408
LGe2pe IROLATION SMe Sie ee MRM. 54Sonn 5 <0Ubioy sean cabente Mink oedackae meat ana 411

XII
SS
SS SSS SS SSS sl SS SSS SSS CSS SSS EPS eS
CONTENTS
SS

Wook IMETEA see oR 2 gio a ee ne a es er ae ire 413


ee) Ls Ne eens Ree es eee Ge, cet, uss b suka > hha OeRR ee 418
Hees OMG Svelea
WV
ESaa OR GUUTS AAO ats a a a ne en: cee er 422
TO oem cot
eu Ge An eS
TOTIAMOIS 4.764) cys ee 5 Pas, peal tle con eae ees cae tee 427
Re CN RalP he
ThAFigco:gna an Rasa Rm a leah ae 2aaNet at fe ee naar meame o> CPI 429

17 Analytic Geometry 435


dligrd media Tpeocin ee ath NR a pas. lok GAS tain, RG os ep ee ie eens Sei ee eee 437
iyeee EUs Ce eh eg! © NTS gee Ae SD, Seated, See eke aos BURY rad, 444
io basic analyhicteometry Problems: oir. 24 Pi. Ie POR, AO eee 446
Ree is tu ANIRIVUC GseOMelty 2 4.8d ig‘orates wie eel a sak oe se peace ee 452
ite ist atice between a Foinband a Line: 5 one asc aes cee en eee ee eae 461
Pee cence Atayht GeoIicuy Problems... . cite es a ee ee 466
I ALL ac a dg ees eecn 0 oa ohae eas toe ee 473

18 Introduction to Trigonometry 478


DOr ee ATMO ey ANG NIPNt LULAN POS Ve, Sty n raters ake Ee en a ce ee 478
Mere USCS OL AIT AAT OS See coors Nan mele ents Roe ope ee ie ee 488
16,30 Laweot Sines and Law of Cosines: . 065 wees ene ee ad) ca 494
IB AD OUUUMALY? go igdoo DT eh she x, Se oy Shee, Oe ee a eS at a ee 500

1) Problem Solving Strategies in Geometry 505


Te ae i eer aie. ne ce ode OS ak Sree ee goes VES. Sy Beak Pana geek a ces ee 505
19:2 Assreningwvariables, ahd. 6 ela p oa Sar ye 56 « eee Oo oe 510
193 brooch
ee tee, oe ot ee. «ee 514
7 AS UTC en aR ce RR ems 8 Bg Ud OR, ep ee eg cae gall

Hints to Selected Problems 525

Index 552

xiii
For Professor Harold Reiter, who brought me back to education, and for Vanessa Rusczyk, whose
confidence in me and love of the desert has kept me here ever since.

aaa
The Golden Ratio Spiral

Do not laugh at notations; invent them, they are powerful. In fact, mathematics is, to a large extent, invention of
better notations. — Richard Feynman

CHAPTER

|S ace in a Name?

aS

Each of these images helps tell a story. Throughout this book we’ll share these stories with you, but
before we tell these stories, we have to name our characters.
CHAPTER 1. WHAT’S IN A NAME?

1.1 Why Names and Symbols?

To convince you that names and symbols are useful, we'll start at the end of the book instead of the
beginning. Here’s the final example problem in this book, written without any special symbols or names.

Draw three points and connect each to the other two with straight paths. Also, draw the
circle that passes through all three of these points. Then, draw a line through one of those
three points such that the line goes inside the region you just formed and is equally close
to the two other straight paths you formed initially through this point. Draw the circle that
goes through the one of your three first points you just drew a line through, through the
point where this line hits the straight path that connects the other two of your first three
points, and through the point that is half-way between these two other points.
Consider the two paths from the point we drew the extra line through to the other two of
our first three points. These paths hit our second circle before they hit these other two points.
Show that the distance from where the circle hits these paths to the points where these paths
end is the same for both paths.

If you can make much sense of this problem, you're a much more careful reader than Iam! We need some
special names and symbols so we can communicate mathematical ideas more simply.

1.2 Points, Lines, and Planes

Pe

Figure 1.1: A Point

A dot. A speck. In geometry, it’s a point. If you lived on a point, you’d be awfully bored. There would
be no up and down, no right and left. You couldn’t move any amount in any direction. Since you can’t
move on your point in any direction, we say a point has 0 dimensions. In order to tell one point from
another, we usually label them with capital letters, such as point P above.

Figure 1.2: A Segment

Now, say you got so bored on one point that you just had to go to another point. If there were a
straight path from one point to another, that path would be called a line segment, or just a segment.
The two points at the ends of a segment are cleverly called the endpoints of the segment. We use these
endpoints to label the segment. For example, AB is the segment from A to B. To denote the length of the
segment, we omit the bar. For example, AB equals 1.5 inches in Figure 1.2.
nn kc cnn cc
4
1.2. POINTS, LINES, AND PLANES

Zs W

Xx M ¥

Figure 1.3: A Midpoint and Marking Segments of Equal Length

The endpoints aren’t the only points on a segment. There are infinitely many points, since between
any two points on the segment, we can find another point. One special point on a segment is the
segment’s midpoint, which is the point halfway between the endpoints. Because the midpoint is the
same distance from both endpoints, we say it is equidistant from the endpoints. In Figure 1.3, M is the
midpoint of XY. We show that XM = MY in the diagram with the little tick marks along XM and MY.
If we have multiple sets of equally long segments, we use a different number of tick marks for each. For
example, our diagram above indicates that ZX = WY, and that these lengths need not be the same as
XM and MY.

as a
A
Figure 1.4: A Ray

If you're not happy just going from A to B, you can keep going past point B. If you keep going
forever, you will make a ray. We refer to the ray in Figure 1.4 as AB, where the starting point, or origin,
of the ray comes first. In the diagram, the little arrow indicates that the ray continues forever in that
direction.

Figure 1.5: A Line


a

As you might guess, we could continue forever in both directions. The result is a line. Line AB is
shown in Figure 1.5. We sometimes use a lowercase letter to describe a line, such as line k in the figure.
We often leave off the little arrows in the diagrams.
CHAPTER 1. WHAT’S IN A NAME?

(e

Figure 1.6: Three Collinear Points Figure 1.7: Three Concurrent Lines

If three or more points are all on the same line, we say the points are collinear, and if three or more
lines all pass through the same point, we say the lines are concurrent.
Segments, rays, and lines are all one-dimensional figures, since you have only one way you can move
along them. Roughly speaking, any path you can draw with a pencil is one-dimensional, meaning you
can either move ‘forward’ on the path or ‘backward’ on the path.
Once we have the freedom to go off our path and move around on a surface, we’re up to two
dimensions. On a surface like this page, we might call our dimensions left-right and up-down. If the
page extended forever in every direction, we'd call it a plane. Most of this book discusses planar figures,
which are figures that exist in planes.
However, in Chapter 14, we wander off the page and add a third dimension you might think of
as ‘above-below.’ The physical space we live in is effectively three-dimensional, and most of what we
experience is three-dimensional.
Although it’s much harder to think about, there’s a great deal of math in higher dimensions. But
that’s a story for another day.

“Txercises,
1.2.1 Alice is thinking of a line. How many points on that line does she need to show Bob in order for
Bob to know exactly which line she is thinking about?
1.2.2 Mis the midpoint of AB and N is the midpoint of BM. If BN = 4, then what is AB?
12 Oks ,and T are on line k such that Q is the midpoint of PT, Ris the midpoint of QT, and S is
the midpoint of RT. If PS = 9, then what is PT?
1.2.4x Points A, B, C, D, and E are five points on a line segment with endpoints A and E. The points are
in the order listed above from left to right such that CD = AB/2, BC = CD/2, AB = AE/2, and AE = 12.
What is the length of AD? (Source: MATHCOUNTS) Hints: 203

4

essences
1.3. ROUND AND ROUND

1.3 Round and Round

Problem 1.1: Mark a point on a piece of a paper and label it O. Use a ruler to find points on your
paper that are 1 inch away from the point O. If you draw all of these points, what figure would you
create?

Problem 1.2: The figure shown at right is called a circle.

(a) Is it possible to draw a line that does not hit the circle in any points?
(b) Is it possible to draw a line that hits the circle in exactly one point?
(c) Two points?
(d) Three or more points?

We have many fancy names for things in mathematics. The fancy name we have for a group of
points that satisfy certain conditions is a locus. While you may never have heard of that word, you’ve
certainly heard of the first locus we'll investigate. (And it’s no big deal if you forget the word ‘locus’
until Intermediate Geometry!)

Problem 1.1: Mark a point on a piece of a paper and label it O. Use a ruler to find points on your
paper that are 1 inch away from the point O. If you draw all of these points, what figure would you
create? .

Solution for Problem 1.1: When we draw all the points that are 1 inch A
away from O, we form a figure called a circle. The point O is called
the center of the circle. We often refer to a circle by its center, writing
‘circle O’ or ‘OO’, where the © symbol tells us that we’re dealing with
a circle. We say that OA is a radius of the circle because it is a segment
connecting the center to a point on the circle. We know that all points
on the circle must be 1 inch, from the center, so OA = 1 inch. The term
‘radius’ is also used to mean the length of a radius, so we could write:
‘The radius of OO is 1 inch.’
You'll notice that we didn’t use a big dot to mark point A. When
there’s a label near where two figures meet, the label refers to the point
where they meet. Therefore, A is the point where our radius hits the circle.
Much of our work in this book involves both lines and circles.

Problem 1.2: Cana line and a circle intersect in 0 points? 1 point? 2 points? 3 points? More?

Solution for Problem 1.2: Given ©X, we can clearly find a line that doesn’t hit X anywhere. Line k shown
below is such a line. Imagine sliding line k closer and closer to ©X until it touches the circle at exactly
CHAPTER 1. WHAT’S IN A NAME?

one point, such as line £ touches ©X at point P. We say that line ¢ is a tangent line to the circle. We can
also use ‘tangent’ as a adjective, and write, ‘Line ¢ is tangent to OX.’

Lines even closer to the center intersect the circle at two points, such as MN does. A line that hits a
circle at two points is a secant line. A segment that connects two points on a circle is a chord. MN isa
chord, while MN is a secant line. A chord that passes through the center of a circle is a diameter.
Finally, the portion of a circle that connects two points on a circle is called an arc of that circle. Of
course, we have a symbol for that too: MN is the shorter of the two arcs that connect M and N. We call
the shorter of the two arcs that connect two points on a circle a minor arc of the circle. The longer arc
that connects the two points is a major arc of the circle. We usually use three points to denote a major
arc: PNM is the longer arc connecting P to M, while PM is the smaller arc connecting them. 0

Exercises >

1.3.1 In the figure at right, identify whether each of the following is a secant D igi Tse
line, a chord, a radius, a diameter, or a tangent line of OO. (If multiple terms
are accurate, list all of the accurate terms.)

(a) CO B A
(b) EF
(Eo)
(d) AB
(e) CB
1.3.2 Suppose point P is outside a given circle. Is it always possible to draw a line through P that is
tangent to the circle? (No proof is necessary now; you'll have the tools to prove your answer later in the
text.)

1.3.3 What is the maximum number of possible points of intersection of a circle and a triangle? (Source:
AMC 10) (A triangle is formed by connecting three points with line segments.)
1.3.4 Two circles and three straight lines lie in the same plane. If neither the circles nor the lines
are coincident (meaning the two circles are different and the three lines are all different lines), what is

Ce ee
1.4. CONSTRUCTION: COPY A SEGMENT

the maximum possible number of points at which at least two of the five figures intersect? (Source:
MATHCOUNTS) Hints: 374

1.4 Construction: Copy a Segment

Classical construction problems are sprinkled throughout the book because a deep understanding of
constructions usually leads to a deep understanding of geometry. Construction problems ask us to create
precise geometric diagrams with two simple tools. These tools are a compass, to make circles, and a
straightedge, to make straight line segments. Notice that we don’t say ‘ruler’ to make line segments.
You don’t get to use your straightedge to measure lengths of segments — you can only draw lines.
Similarly, you aren’t allowed to use your protractor to measure or create angles.
So, what can you do?

That’s the goal of these construction sections: to start learning what you can do with only compass
and straightedge. The only operations you can perform with your compass and straightedge are the
following:

1. Given a point, you can draw any line through the point.

. Given two points, you can draw the line that passes through them both.

. Given a point, you can draw any circle centered at that point.

WwW
me
N . Given a point and a segment, you can draw the circle with its center at that point and with radius
equal in length to the length of the segment.
5. Given two points, you can draw the circle through one point such that the other point is the center
of the circle.

That’s not much, but with these simple operations we can construct an enormous range of diagrams.

f)__Probiems
ile
Problem 1.3: Use your compass to find a point Y on k such that AB = XY. You cannot simply use a
ruler to measure AB, then use that measurement to find Y!
. B
CHAPTER 1. WHAT’S IN A NAME?

Problem 1.4: Shown below are segments AB and MN.


(a) Use straightedge and compass to construct a line segment that has length AB + MN. (Reminder:
You can’t just measure with a ruler!!)
(b) Construct a line segment that has length AB — MN. (Even though we don’t say ‘with a straight-
edge and compass,’ you still can’t measure with a ruler! ‘Construct’ implies ‘straightedge and
compass’ construction.)
B

oe
N

We start our exploration of construction by learning how to copy a segment.

Problem 1.3: Use your compass to find a point Y on k such that AB = B


XY. You cannot simply use a ruler to measure AB, then use that A the
measurement to find Y! join =
speocaues

Solution for Problem 1.3: All we can do with a compass is draw circles B
or parts of circles. To find a point that is AB from X, we first open our ofos Seg ors
compass to a width of AB by putting the point of the compass at A and 4
the compass pencil at B (or vice versa). Then we make a circle with center ee SS
X and this opening as the radius. Since this OX has a radius equal to AB, \ 7 k
the two points where it hits k are AB away from X. We can take either one B Beats4
of these as our point Y. O

Co ncept: In nearly all construction problems in which we must make a point, we


find that point by constructing two figures that the point must be on. The
_ point we seek is then at the intersection of these two figures. For example,
in Problem 1.3, we have line k and construct OX that Y must be on. Their
intersection gives us the point Y we seek.

Let’s try a slightly more challenging construction.

Problem 1.4: Shown are segments AB and MN. Use straightedge and compass B
to construct a line segment that has length AB + MN, and a line segment that has
length AB — MN. MN,
A
1.4. CONSTRUCTION: COPY A SEGMENT

Solution for Problem 1.4: We start by drawing a line k and choosing a point P on line k. We find a point
that is AB+MN from P in two steps. First, we find a point a distance of AB from P using our construction
technique in Problem 1.3. We do so by opening our compass to width AB and using this radius to draw
a circle centered at P. We take one of the points where this circle hits k to be point Q.

Then we find a point that is MN from Q by opening our compass to width MN and using this radius
to draw a circle centered at Q. As shown, this circle hits m at two points, R and R’. To get to point R from
P, we go a distance of AB to get to Q, then MN more to reach R. Therefore, PR = AB + MN. Similarly,
to get to R’ from P, we first go a distance equal to AB to get to Q, then head back towards P a distance of
MN to get to R’. So, PR’ = AB— MN. 0

You might have noticed that we didn’t need the entire circles B
we drew in our constructions. We only needed enough of the M
circle to tell where the circle would hit the line. Typically, these er
little arcs are all we draw in our constructions. Therefore, our A
paper when constructing AB — MN in Problem 1.4 might look as k Paakoak
shown at right. |

1.4.1 Given the segments shown, construct segments with the following is
lengths: D
(aye Aba CD = EF,
(b) 2AB. C
(ic) AB = QE SCD:
A B
EB
CHAPTER 1. WHAT’S IN A NAME?

1.5 The Burden of Proof

Earlier we defined a line segment as the direct path that connects two points. It seems obvious that
any two points can be connected by a segment. In fact, it seems so obvious that it should be easy to
prove. However, it isn’t just hard to prove — it’s impossible. The statement that any two points can be
connected by a straight line segment must be simply accepted as a fact. We call such a statement that
must be regarded as fact without proof an axiom. Axioms are also sometimes called postulates.
When the world’s most famous geometer, Euclid, wrote his famous Elements, he stated five axioms:

1. Any two points can be connected by a straight line segment.


2. Any line segment can be extended forever in both directions, forming a line.
3. Given any line segment, we can draw a circle with the segment as a radius and one of the segment’s
endpoints as center.

4. Allright angles are congruent. (We'll talk about right angles and what we mean by ‘congruent’ shortly!)
5. Given any straight line and a point not on the line, there is exactly one straight line that passes
through the point and never meets the first line.

In Euclid’s Elements, he combined these axioms to prove ever more complicated mathematical
statements. We call such proven mathematical statements theorems. A mathematical statement that is
not an axiom but hasn’t been proved false or true is called a conjecture.
In this book, we don’t start from Euclid’s axioms and prove everything that follows step-by-step. It’s
a good thing, too! It turns out that even Euclid missed a few axioms. Mathematicians since have shown
that Euclid’s arguments, in order to be completely valid, would need many more axioms added to these
five. In other words, there are some things that even the great Euclid didn’t realize are so ‘obvious’
that they could not be proved. Often when we reach these items in this text, we give a ‘common sense’
explanation of why we accept these statements as facts. We note when these really are axioms, as
opposed to statements that we can prove using previous axioms or theorems.

You can use the proofs we present both as guides for writing your own proofs and as stepping stones
to prove interesting theorems of your own.

1.6 Summary
aon
Definitions:

e A point is, well, a point. Euclid called a point ‘that which has no part.’ We can’t do much
better than that vague description. We typically denote points with capital letters.
e A straight path connecting two points is called a segment, and our original two points are the :
endpoints of the segment. We refer to a segment by its endpoints, such as AB. We remove the |
bar to denote the length of the segment: AB. ;
SSSS
S SSS SS Us SS 5S SSS SS SOS
1.6. SUMMARY
SS AA

Definitions:

e The point on a segment that is halfway between the endpoints is the midpoint of the segment.
We also say that this point is equidistant from the endpoints.

If we start at a point, then head in one direction forever, we form a ray. Our starting point is
aed
the vertex of the ray, and we denote a ray as AB, where the first point is the vertex of the ray.

If we continue a line segment past its endpoints forever in both directions, we form a line,
which we write as AB.

If this page were continued forever in every direction, the result would be a plane. Since we
can move in two general directions, such as right-left and up-down, on a plane, we say the
plane has two dimensions.

If we add a third dimension, we are in three-dimensional space.

The set of all points that satisfy specific conditions is called a locus.

Definitions:

e The set of all points that are the same distance from a given point is a circle. The given point
is the center of the circle, and the fixed distance is the radius. We often refer to a circle by its
center using the symbol ©, so ©O refers to a circle centered at O.

A line that touches a circle at a single point is tangent to the circle, while a line that hits a circle
at two points is a secant line. A segment connecting two points on a circle is a chord, and
a chord that passes through the center of its circle is a diameter. The portion of a circle that
connects two points on the circle is an arc, which we denote with the endpoints of the arc: MN
is the shorter arc that connects M and N.

When performing constructions with a straightedge and compass, you can only draw line segments
and circular arcs. You cannot use a ruler to measure segments. The operations you can perform are:

1. Given a point, you can draw any line through the point.

. Given two points, you can draw the line that passes through them both.

. Given a point, you can draw any circle centered at that point.

~N Given a point and a segment, you can draw the circle with its center at that point and with radius
OO
equal in length to the length of the segment.
5. Given two points, you can draw the circle through one point such that the other point is the center
of the circle.

eee eee eee


eee eee

_ Extra! Logic is the art ofgoing wrong with confidence.


: al al all —Mortrris Kline

LS.
11
CHAPTER 1. WHAT’S IN A NAME?

Extra! At the top of the first page of each chapter in this book is an image illustrating an
>> interesting geometric fact. The image at the start of this chapter is of the Golden Ratio
Spiral. A Golden Ratio Spiral is inside a golden rectangle, whichis a rectangle that can be
divided into a square and another rectangle such that the ratio between the dimensions
of the new rectangle equals that of the original rectangle.

1 Pine |
A B

I Z 1

z 1 O-x=1 a

Shown above is golden rectangle ABCD with dimensions 1 and x. PQ divides the
rectangle into a square of side 1 and a rectangle with dimensions x — 1 and 1. Since the
ratio of the dimensions of ABCD equals the ratio of the dimensions of BCQP, we have

1 x-1
x 1

The positive value of x that satisfies this equation is

14-5
~ 1.618034.
2

This number is the golden ratio (also sometimes called the golden mean), and is often
referred to by the Greek letter @ (‘phi’).
When we divide a golden rectangle into a square and a rectangle, the ratio of the
dimensions of the smaller rectangle is the same as that of the original rectangle. Therefore,
the smaller rectangle is a golden rectangle too, so we can split it into a square and another
smaller golden rectangle. We can do this over and over indefinitely, forming the figure
shown below.

All of the squares in the diagram together make up our largest golden rectangle.
When we omit the largest square, we get our next golden rectangle. Then we omit the
next largest square to find the next golden rectangle, and so on. If we then draw a
quarter-circle in each of the squares, as shown above, we get the Golden Ratio Spiral.
The Lighthouse Theorem

We're going to turn this team around 360 degrees. — Jason Kidd

CHAPTER RGEC ns aye aaa

pee SE

2.1 What is an Angle?

When two rays share an origin, they form an angle.

x In the diagram at left, rays OX and OY share origin O. We can refer to the
angle they form as ZXOY. The Z symbol tells us we’re referring to an angle. The
common origin is called the vertex of the angle, and the rays OX and OY are
7 Y called the sides of the angle. Notice that when we write the angle as ZXOY, we
put the vertex in the middle. We could also refer to the angle as ZYOX, but not as ZXYO. When it’s very
clear what angle we’re talking about, we can just name it with the vertex: ZO.
Of course, two intersecting lines also make angles.

13
CHAPTER 2. ANGLES
ee

Lines AB and CD at right intersect at P. Here, we can’t just write ZP,


since there are many different possible angles this could mean, such as
ZAPC, ZAPD, ZDPB, or ZBPC. We might even be referring to ZAPB.

Now that we know what angles are, we need a way to measure them
so we can compare one angle to another.

2.2 Measuring Angles


TTT i vay 1 Mim,
u \ 4m
a p 80 90.ae Uy
Ph
x&
—~ ne 2 60Ne3p BD>
. hae & Dy
\ NO\
aN 2. “A
DB
QQ DB B

Deeaeaeaee nee
t eS)\ 2
eosZA
a
PS \ o\ =A

§ \ \e\a-2
\ \ eget

ae a Se ae

Figure 2.1: A Protractor

Just as we use a ruler to measure the lengths of segments, we can use a protractor to measure angles.
Roughly speaking, an angle’s measure is how ‘open’ the angle is. Our protractor above shows half a
circle (which we call a semicircle) divided into 180 equal pieces. Each of these little pieces is considered
one degree of the semicircle, so that an entire circle is 360 degrees. We use the symbol ° to denote
degrees, so that a whole circle is 360°.
We use a protractor to measure the number of degrees of a circle between the two sides of an angle
whose vertex is the center of the circle. For example, in Figure 2.1
2.1, the vertex of ZYOZ is placed at
the center of the semicircle. There are 62 degrees between sides OZ and OY of £YOZ, so we say that
LYOZ= 62°. Sometimes angles are written with an m before Z to indicate measure: mZYOZ= 62°.

Problems

Problem 2.1: Use your protractor to find ZYOZ, ZCXD, ZDXE, and ZCXE.
2.2. MEASURING ANGLES

Problem 2.2: The diagram below shows four common angles. In each case, point O is the center of|
the circle. ZAOB cuts off 1/4 of a circle, ZCOD cuts off 1/3 of a circle, ZEOF cuts off 1/12 of a circle, and

es Gre
ZGOH cuts off 1/8 of a circle.

(a) What is the measure in degrees of ZAOB?


(b) What is the measure in degrees of ZCOD?
(c) What is the measure in degrees of ZEOF?
(d) What is the measure in degrees of ZGOH?
(e) What's so special about 360; why do we use 360 for the number of degrees in a whole circle?
Do not use a protractor; use what you are told about the angles in the text.

Problem 2.3: Given that ZWOY = 60° and 2WOX = 20° below, find ZXOY.

Problem 2.4: Suppose instead of measuring an angle the ‘regular’ way, we go


the ‘long’ way around, as shown in the diagram. The ‘regular’ angle PQR has
measure 40°. What is the measure of the ‘long’ way around angle?

Problem 2.5: Use your protractor to create a 37° angle and a 143° angle.

Extra! We could use two Eternities in learning all that is to be learned about our own world and the
imine thousands of nations that have arisen and flourished and vanished from it. Mathematics alone
would occupy me eight million years.
— —Mark Twain

15
CHAPTER 2. ANGLES

Problem 2.1:Use your protractor to find ZYOZ, OX) LDXE, and ZCXE.

Solution for Problem 2.1: The protractor itself is half a circle (which we call a semicircle); we use it to
measure the number of degrees of a circle the angle cuts off. Here are the steps we follow to use our
protractor to measure angles:

1. Place the protractor on the angle so that the vertex of the angle is exactly where the center of the
circle would be if the protractor were a whole circle. Your protractor should clearly show this
center point; it’s near the middle of the straight side.
2. Turn the protractor so that one side of the angle is along the ‘zero line’; i.e., the line through the
center point along the straight edge of the protractor.
3. Find where the other side of the angle meets the curved side of the protractor. The number there
tells you the measure of the angle.

For ZYOZ, we put our protractor on the page as shown below. We line up side OZ of the angle with
the zero line of the protractor, placing the center point of the protractor over O. We find that side OY
hits the curved edge at 90°.

S
=
~l >
i=)
=
Oo

When we follow this procedure with ZCXD, we find that there are two numbers where XD meets
the curved edge in the following diagram. We
W know
w to take the smaller of these numbers- clearly there
are 40 degrees, not 140 degrees, between XC and XD. We can also note that ZCXD D
is
i less than half the
entire semicircle, so its measure must be the smaller of the two numbers where XD meets the curved
edge of the protractor.

16
2.2. MEASURING ANGLES

aw \ \’ , P { NH Df, 2

Ss SSq s0 1D ae? 70

We can also use the above diagram to find the measure of ZCXE. Once again, our angle hits a point
on the curved edge with two numbers, but this time we know the angle is greater than 90° (since the
angle is more than half the semicircle). Thus, we know that ZCXE = 115°.

Finally, we can place the protractor as in the diagram below to find that ZDXE = 75°.

yO’
9

0S
0€
oF
8oct
OST
oft

Notice that ZCXD + ZDXE = ZCXE. This isn’t an accident! Since ZCXD and ZDXE share a side and
a vertex, putting them together gives ZCXE. 0

We saw in Problem 2.1 that knowing whether an angle is greater than or less
than 90° is necessary for finding its measure using a protractor. This 90° is such an
important measure that angles that are 90° have a special name, right angles. We
usually mark right angles with a little box as shown in ZJKL at right. Two lines, rays,
or line segments that form a right angle are said to be perpendicular. JK and KL are
perpendicular; we can use the symbol to write this briefly: [K 1 KL. K if

E77
CHAPTER 2. ANGLES

P
ok

50° 120°
Q R Y Z

Angles that are less than 90° are called acute, and those that are greater than 90° but less than 180°
are called obtuse. Sometimes we write the measure of an angle inside the angle as shown above.

Problem 2.2: The diagram below shows four common angles. In each case, point O is the center of
the circle. ZAOB cuts off 1/4 of a circle, ZCOD cuts off 1/3 of a circle, ZEOF cuts off 1/12 of a circle,
and ZGOH cuts off 1/8 of a circle.

COGS:
(a) What is the measure in degrees of ZAOB?

(b) What is the measure in degrees of ZCOD?

(c) What is the measure in degrees of ZEOF?

(d) What is the measure in degrees of 4GOH?

(e) Why in the world do we use such a weird number, 360, for the number of degrees in a whole
circle?

Do not use a protractor; use what you are told about the angles in the text.

Solution for Problem 2.2: Since a whole circle is 360°, and ZAOB is 1/4 of a circle, we have

LAOB = ($) (360°) = 90°.


We can tackle the other three angles in exactly the same way:

ZCOD = (4) (360°) 120°


ZEOF” = (5 )(360") =< 30°
LGOH = (3%) (360°) 45°

The number 360 comes from the ancient Babylonians. The Babylonians used a number system with 60
digits, instead of our decimal system, which only has 10 digits. When choosing a number of degrees for
a whole circle, they were likely influenced by their number system and possibly by astronomy (a year
has around 360 days). However, a look at our answers above points to what might have been the largest
factor in choosing 360. The Babylonians, like most people today, probably hated fractions. Since 360 is
divisible by lots of different numbers, many common angles in geometry have integer measures. Had

| nas memeanmm emi


2.2. MEASURING ANGLES

we used 100 degrees instead, we’d have to deal with measures such as 125 degrees, 84 degrees, and so
on. O

hPrablent 2.3: area that /WOY = 60° Bre /WOX = 20°, fndexOy.

Solution for Problem 2.3: As we just saw in Problem 2.1, we can combine two angles that share a side and
a vertex to make a third whose measure equals the sum of the measures of the first two. Applying this
to the diagram in the problem, we see that ZWOX + ZXOY = ZWOY. Therefore, 20° + ZXOY = 60°, so
ZXOY = 40°. O
We call angles that share a side, like ZWOX and ZXOY in Problem 2.3, adjacent angles.

Problem 2.4: Suppose instead of measuring an angle the ‘regular’ way, we go the -
‘long’ way around, as shown in the diagram. The ‘regular’ angle PQR has measure
40°. What is the measure of the ‘long’ way around angle? 40°

Solution for Problem 2.4: If we imagine our ‘regular’ angle PQR cutting off a circle, we know it cuts off
40° of the circle. The ‘long’ way around then must be the rest of the circle. Since a whole circle is 360°,
the remainder of our circle is 360° — 40° = 320°. O

Angles that are greater than 180° are called reflex angles. They are rarely important in problems.
We finish our discussion of measuring angles by learning how to draw them given angle measure-
ments.
Problem 2.5: Use your protractor to create a 37° angle and a 143° angle.

Solution for Problem 2.5: We start with one side, OA, of the 37° angle, which we can draw anywhere. To
create the other side, we use our protractor to figure cout where the other side would have to go in order
to make a 37° angle. We place our protractor over OA as if we are measuring an angle with OA asa side.
We then find the 37° point on the curved side, since a 37° angle would have to go through this point. As
we've noticed before, our protractor has two 37’s, one on each side. Since a 37° angle is clearly acute,
we know to choose point B in the figure on the next page, thus making an angle that is less than 90°.
@

Extra! The golden ratio that we discussed on page 12 does not only appear in ee For
“a example, consider the Fibonacci sequence shown below.

1,1,2,3,5,8,
13,21,94, 95,89, 144,293...

The first two terms of the sequence are both 1, and each subsequent term is the sum of
the previous two terms. Calculate the ratio between each term and the term before it,
such as 34/21~ 1.619. ‘See anything interesting?

19
CHAPTER 2. ANGLES

TN rm
Ceu
Oo ea ee

Similarly, we have two choices when we build our 143° angle. This time we choose the one that
creates the obtuse angle, as shown in the diagram below.

nee
90 lo

D ae ss os Ms

3/3
S y 0 aX:
ofa S
v/s B\S
=/2/ i
2\3 re

olf s
Sa ab =i

LJ

Exercises eS

2.2.1 Use your protractor to make angles with the following measures

(a)90° — (b) 45° (c) 135° — (d) 220°

2.2.2 Use your protractor to measure the angles shown. Classify each angle as right, acute, or obtuse
(a) ZX.
(b) ZABC and ZDBC.
(c) ZPQR, ZPRQ, and ZRPQ.

20
2.3. STRAIGHT AND VERTICAL ANGLES

2.3 Straight and Vertical Angles

y
Proviems
Problem 2.6: In the figure below, AOB is a straight line. What is the measure of ZAOB?

A B
O

Problem 2.7: In the figure, lines VY and WZ meet at X, and ZYXZ = 55°.
What is the measure of ZWXY?

Problem 2.8: Lines LN and MO intersect at P such that ZMPN = 72°. Find LPO.

Problem 2.9: Lines LN and MO intersect at P. Prove that MPN = LPO.

Problem 26: In the figure, AOB is a straight line. What is the measure of ZAOB? A

Solution for Problem 2.6: If we don’t see the answer right away, we can try to figure out
what portion of a circle the angle cuts off. So, we draw a circle with center O as in the “oe
diagram to the right. Now we can see that the angle cuts off half a circle (whichever
side of the line we pick). So, ZAOB = (1/2)(360°) = 180°.
This one’s easy to remember: we call an angle that is really a straight line a straight angle. 0

Extra! What science can there be more noble, more excellent, more useful for people, more admirably
ammene high and demonstrative, than this of mathematics? —Benjamin Franklin

el
GHAPTERAWANGLESs Thole 28
ape an acne too mee to be useful, but often the anal tools are the best.
= SR

Problem 2. in the foe tines Wy id We meet attx and IYXZ = 55°, What
is the measure of LWXY?

Solution for Problem 2.7: Since ZWXY and ZYXZ together make WXZ, which is a straight angle, we
now that. ZWxXYe ZY XZ =180— Hence, ZWXY' = 160 — ZY X71

We call two angles that add to 180° supplementary angles. As we have seen, when two lines
intersect like VY and WZ in Problem 2.7, any two adjacent angles thus formed are supplementary
because together they make a straight line.
Similarly, we call angles that add to 90° complementary angles.

| Problem 2.8: Lines LN and MO intersect at P such that ZMPN = 72°. Find ZLPO. cE

Solution for Problem 2.8: Angle LPO sure looks equal to ZMPN, and it ‘makes sense’ that the two are
equal, but ‘makes sense’ isn’t good enough in mathematics. We need proof, which we’ll tackle in the
next problem. For now, we'll try to compute ZLPO.
Since it’s not obvious how to compute ZLPO, we start by finding angles we can measure. Since MPN
and ZNPO together make a straight angle, we have ZNPO + ZMPN = 180°. Thus, ZNPO = 180° — 72° =
108°.
Similarly, since ZLPO and ZNPO are supplementary, we have

LEPO = 180"— ZNPO = 180° = 108° = 72°.

a When you can’t find the answer right away, try finding whatever you can
— you might discover something that leads to the answer! Better yet, you
might learn something even more interesting than the answer. The best
_ problem solvers are explorers.
2.3. STRAIGHT AND VERTICAL ANGLES

Solution for Problem 2.9: What's wrong with this proof:

picehe Solution: Suppose ZMPN= 72°. Since ZMPO is a straight angle, we know
pid) that ZNPO= 180°— 72° = 108°. Similarly, we have ZLPO= 180°-
: ZNPO = 72°. Therefore, MPN = ZLPO.

Every statement in that ‘proof’ is true. However, it is not a complete proof because it only addresses
one case; it only shows that ZMPN = ZLPO when ZMPN = 72°.

i :NIING! An example iis nota Broo! =

While examples aren’t proofs, they can be useful as guides. Looking at our example, we can quickly
construct our proof.

Since MPO is a line, we have


ZMPN = 180° — ZNPO.
Since LPN is a line, we have
ZLPO = 180° — ZNPO.
Combining these two equations gives ZMPN = 180° — ZNPO = ZLPO. O
Notice that our proof does not depend at all on the measure of ZMPN. The proof works no matter
how the lines intersect.
Pairs of angles such as ZMPN and ZLPO in the diagram below are called vertical angles. As we
proved in Problem 2.9, vertical angles are always equal to each other.

We often use little arcs to mark equal angles. In the diagram to the right,
ZMPN and ZLPO each have a single little arc in them to show that they are
equal. Angles ZLPM and ZNPO also are vertical angles, so they are equal.
We put a little hash-mark on the arcs at these angles to show that these two
angles are equal to each other, but not necessarily equal to our first pair of
equal angles (which have arcs without hash-marks).
CHAPTER 2. ANGLES

a ~
2.3.1 Find x, y, and z in the diagram below.

115 /Z
X/Y

2.3.2 Find the measure of an angle that is supplementary to each of the following angles:
(a) ZAOB = 120°
(b) ZCOD = 45°
(ce) g4EOF =90°
2.3.3 Find the measure of an angle that is complementary to each of the following angles:
(a) ZGOM = 30°
(b) ZIOJ = 45°
(c) ZKOL = 75°

2.4 Parallel Lines

Having dissected what happens when two lines meet, we should wonder about what happens if they
don’t. If two lines do not meet, we say that they are parallel. If lines AB and CD are parallel, we write
AB ||CD.

Figure 2.2: Two Sets of Parallel Lines

Just as we use little arcs to mark angles that are equal, we can use little arrows to mark lines that are
parallel. In the diagram above, lines j, k, and € are marked parallel, as are lines m and n. You won't see
us use this notation all the time, though. Those little arrows can really clutter up a diagram.

Cha ee aa ae
2.4. PARALLEL LINES

[|
Problems
_ ila
Problem 2.10: Draw a pair of parallel lines like those shown below. Then draw a line that crosses
both of the parallel lines. Measure all the angles formed between your line and both of the parallel
lines. Write the angle measures in the angles you form. Try it again with a different crossing line.
Do you notice anything interesting?
nn

Problem 2.11: Lines m and n are parallel, and we are given the measure
of one angle in the diagram as shown. Find the values of a, b, c, w, x, y,
and z.

Problem 2.12: In the figure, we have AB || CD and AD ||BC. We are also given the measures of four
angles as shown in terms of x and y. Find x and y.

Problem 2.13: Given that WV ||YZ and WZ || VY in the diagram,


Bde |
e
GHABT EROWANGLES ee
Problem 2.14: In the diagram, || and the angles are as marked. Find x.

Back on page 10, we noted that one of our axioms (statements we must accept without proof) is

Given any straight line and a point not on the line, there is exactly one straight line that
passes through the point and never meets the first line.

This common-sense statement is sometimes called the Parallel Postulate. We'll start our exploration
of parallel lines by taking a look at the angles formed when a line intersects a pair of parallel lines.

Problem 2.10: Draw a pair of parallel lines, then draw a line that crosses both =<———_____
of the parallel lines. Measure all the angles formed between your line and 6
both of the parallel lines. Write the angle measures in the angles you form.
Try it for a second line (don’t worry about the angles where your two lines meet — just focus on the
angles between the lines you draw and the parallel lines). Do you notice anything interesting?

Solution for Problem 2.10: In the diagram to the right, we have parallel
lines € and m, and we have added line n, which meets € and m at
A and B, respectively. We call a line that cuts across parallel lines
a transversal. We measure ZHAG and find that it equals 40°. Since
ZHAG and ZCAB are vertical angles, we don’t even have to measure
ZCAB. We know that ZCAB = ZHAG = 40°.
Since ZHAG and ZBAG together make up a straight angle, we
don’t have to measure BAG. We know that ZBAG = 180° — ZHAG = 180° — 40° = 140°. Similarly,
AHAG= 140%

We might wonder if we need our protractor at all, but then we think about those angles around B.
They sure look equal to those around A, and common sense tells us that they are, but we measure to
make sure. We find that indeed ZABF = 40°, from which we deduce that ZDBE = 40° as well. We can
also quickly determine that ZABD = 180° — ZABF = 140° and ZEBF = 140°.
Seeing that ZHAG = ZABF, we wonder if it’s always true that a transversal will cut parallel lines at
equal angles like HAG and ZABF. Like the Parallel Postulate, this turns out to be one of those ‘obvious’
facts that cannot be proved. It must be assumed. As we have seen while finding the angles above, once
we know that these two are equal, we can quickly use lines and vertical angles to find the rest of the
angles.
Neen cc ccc nnn nnn nn nnn nen SSS

Extra! There is far more imagination in the head of Archimedes than in that of Homer.
1110 08-1
—Voltaire

ee es
2.4. PARALLEL LINES

Figure 2.3: Angles Between a Transversal and Two Parallel Lines

Thus, we see that when a parallel line is cut by a transversal, we have two groups of four equal
angles. Specifically, in Figure 2.3, we have

2 II ony |
= e Bes ee

Ch ee eas Pt

Furthermore, the angles in the first group are supplementary to those in the second.
Pairs of these angles have special names to describe their relationships. These names are not terribly
important, but you'll see them elsewhere.

a and e are corresponding angles.


d and f are alternate interior angles.
a and g are alternate exterior angles.
cand f are same-side interior angles.
b and g are same-side exterior angles.

Again, the names are not such a big deal. After doing enough geometry, you'll probably know them
anyway. Don’t bother memorizing them now. Just understand which angles are equal and which are
supplementary. 0

Problem 2.11: Lines m and n are parallel, and we are given the measure of
one angle in the diagram as shown. Find the values of a, b, c, w, x, y, and z.

Solution for Problem 2.11: We know that when a transversal cuts parallel lines, equal angles come
in groups of four as we saw in Problem 2.10. Therefore, we know that w = y = c = 113°. We
also know that each angle in the other ‘group of four’ has a measure that is supplementary to 113°:
x=z=a=b=180— 113° =67°.0
Now that we understand the relationships between angles when a parallel line is cut by a transversal,
let’s try a more challenging problem.

27
CHAPTER 2. ANGLES

Solution for Problem 2.12: There’s no obvious way to make an equation for x or y, so we start off by using
our parallel lines and vertical angles to write the measures of all the angles we know in terms of x and y.

After labeling the angles we know in terms of x and y, we look for ways to build equations. We can
use angles that together form straight angles at A and B:

LEAN + EAB = (3y + 15°) + (3x — 15°) = 180°


LFBA + ZFBG + ZGBH = (3y + 15°) +x + y = 180°.
Rearranging these gives

x+y = 60°
x dy -= 165°
eS ee Se er
28
2.4. PARALLEL LINES

Subtracting the first from the second gives us 3y = 105°, so y = 35°. We can then use substitution to find
x = 25°. Of course, we didn’t have to label every angle above — we could have stopped when we had
enough information to set up a pair of equations to solve for x and y.

Note that we could have used parallel line relationships to set up the equations, too. Since AB ||
CD, we have DAN = ZKDM, so 3x — 15° = x + y. Also, ZHBC and ZBCI are supplementary, so
(3y + 15°) +(3x—15°) = 180°. Solving these two equations gives us the same answer as before. (It better!)
O

Concept: Solving a problem with two different methods is an excellent way tocheck
O==2 _syour answer.
==

We'll finish with two more challenging problems that illustrate how useful parallel lines can be when
seeking angle measures.

Problem 2.13: Given that WV || YZ and WZ ||VY in the diagram,


find x.

Solution for Problem 2.13: We start by using what we


know about parallel lines to find as many angles as we
can. We find ZV = 180° — 140° = 40° since WZ || VY. Similarly, ZZ = 40°, and ZZYV = 180° — ZZ = 140°.
Since ZPYV = 3x, we have ZPYZ = LVYZ — ZPYV = 140° — 3x. Since this angle, the 90° angle, and
the angle with measure x together give us straight line PY, we have
LPYZ +90" + x2 = 180".
We then substitute ZPYZ = 140° — 3x into this equation, and we have 140° — 3x + 90° + x = 180°. We
solve this equation for x to find that x = 25°. O
Using information about angles to find information about other angles is often called angle-chasing.
We've already learned three important tools in angle-chasing: straight angles, vertical angles, and
parallel lines. Stay tuned. We’ll see plenty more!

Concept: Often when we're angle-chasing, our goal is to build an eqce to solve
- ©) = for one of the variables in our problem. a

Problem 2.14: In the diagram, € ||m and the angles are as marked. Find x.
CHAPTER 2. ANGLES

forProblem 2.14: We need to relate our desired angle to angles we


Solution
know, but neither AB nor BC cuts both parallel lines. However, if weadda
line k through B parallel to € and m, we can do some angle-chasing. Since
k || £, we have ZABX = 180° — 130° = 50°. Since k || m, we have ZXBC = x.
Since ZXBC + ZABX = ZABC, we can now write an equation for x:

Repo as
Therefore, x = 40°. 0

Concept: Parallel lines are so useful in problems involving angles that sometimes
@— we'll add new ones to a diagram to help us.

2.4.1 Find x and y in the diagram at left below.


A B
Liss

x
O)
D C

Figure 2.4: Diagram for Problem 2.4.1 Figure 2.5: Diagram for Problem 2.4.2

2.4.2 Show that ZA = ZC and ZB = ZD in the diagram at right above.

2.4.3 Find x in the diagram at left below.

3 m B
beget) 352
valor
2x — Y n 40°

Figure 2.6: Diagram for Problem 2.4.3 Figure 2.7: Diagram for Problem 2.4.4
2.4.4 Inthe diagram at right above, m || n, and the angles are as marked. Find x. Hints: 224

2.4.5 In the diagram at right, k || € and m || n. If the angles are as marked,


find x and y. Hints: 481
2.5. ANGLES IN A TRIANGLE

2.5 Angles ina Triangle


A

Figure 2.8: A Triangle

When we connect three points with line segments, we form a triangle, as shown in Figure 2.8 above.
We will often refer to the triangle as AABC, or sometimes just ABC. The points A, B, and C are called
vertices of the triangle, and the segments AB, BC, and AC are called sides. In this section, however, we
will investigate the angles of a triangle.

Problems

Problem 2.15:
(a) Find the measure of the three angles in each of the triangles below.
(b) Can you guess a statement that is always true about the sum of the angles in a triangle?

B z es

Problem 2.16: Given that ¢ ||m, find ZBCD and ZBDC.

Problem 2.17: Prove that the sum of the measures of the angles in a triangle is always 180°.
(a) Draw a triangle AABC, and a line k through point A such that k ||BC.
(b) Find angles in your diagram that are equal to 4B and ZC. Use little arcs as described on page 23
/ to mark the angles equal.
ee _ Prove that ZCAB + ZABC + ZBCA = 180°.

54°.What is the
Problem 2.18: One angle iin a triangle is twice another angle, and the third angle oe
sme~asure of the smallest anglein the triangle? ot _

31
CHAPTER 2. ANGLES

Problem 2.19: In the diagram, m ||, AB 1 m, ZADC = ZBCE = 3x,


ZCEB = 50°, and ZBCD = x. Find x.

We start our investigation of angles in a triangle by measuring angles in a few triangles.

Problem 2.15:
(a) Find the measure of the three angles in each of the triangles below.
(b) Can you guess a statement that is always true about the sum of the angles in a triangle?

Doe Bee a

pe
ees
eae7d
Q

Solution for Problem 2.15: Using our protractor, we can find the measures of the angles in each triangle
as below:

ADEF: D=60° ZE=90° ZF =30°


AXYZ: 4X=40° ZY =70° ZZ =70°
APQR: LP =130° £Q=30° /ZR=20°

In each case, we see that the sum of the angles is 180°. Hmmm... Is that always true? 0
Before tackling the question of whether or not the angles of a triangle always add to 180°, let’s try a
parallel line problem that includes a triangle.
Caen nnnrrrreeer reece ceeeeeeeeeeeeeeee reece SSS SSS SSS SSS SSS

Extra! The cowboys have a way of trussing up a steer or a pugnacious bronco which fixes the brute so
impimpnt that it can neither move nor think. This is the hog-tie, and it is what Euclid did to geometry.
-Eric Temple Bell

i aa ee
2.5. ANGLES IN A TRIANGLE

Problem 2.16: Given that € || m, find ZBCD and ZBDC.

Solution for Problem 2.16: We start as we usually do when the answer isn’t
immediately obvious — we find what we can. We can use vertical angles to
find x = 70° in the figure to the right. We can also use either line ¢ or BD to
find w. BB gives 70° + w+ 50° = 180°, so w = 180° — 70° — 50° = 60°.

Now we can use our parallel lines to find the angles we want. Since || m,
Wwenave ZECD =w=— 60 and¢BDC=<=70°: o

Notice that we don’t have to call ZBCD and CBE ‘alternate interior angles’ in order to say they are
equal. Once we state that ¢ || m, we can note that BCD = w without writing ‘alternate interior angles.’
However, if your teacher tells you that you have to include ‘alternate interior angles,’ you better do it!
Inspired by Problem 2.16, we can now prove that the sum of the angles in a triangle is always 180°.
rc

Problem 2.17: Prove that the sum of the measures of the angles in a triangle is always 180°.

Solution for Problem 2.17: We start by drawing a triangle and by writing what we want to A
prove:
ZABC + ZCAB + ZBCA ='180". C
B
We don’t know a whole lot about angles yet, but we can start by wondering ‘Where have we seen 180°
before?’ Answer: A straight angle. So, we’d like to find a straight line that has all three of our angles, just
like ZEBC, ZCBD, and ZDBF make up line @ in our solution to Problem 2.16. However, we don’t have
any such line yet, so we'll have to add a line somewhere.

We need a line that allows us to use what we know about angles. We A e's
don’t know much about angles, so we don’t have too many options to
investigate. So far, probably the most useful angle information we have
learned comes from parallel lines. So, we add a line through A parallel to B C
BC to create the diagram to the right, which looks curiously like the figure
from Problem 2.16.
Since XY ||BC, we have ZXAB = ZABC and /BCA = ZCAY. Since /XAY is a straight angle, we have
LXAB + LCAB + ZCAY = 180°.
Now we’re home! We can now use the equalities we found with the parallel lines to get:
ZABC + ZCAB + ZBCA = 180°.

33
CHAPTER 2. ANGLES
ee

iss always 180°.


|Important: The sum off the angles iinaa triangle i
|
| cheater
Siavennuo
CSE

There are few geometric relationships you will use more than this one!

Important: Don’t view the proof for Problem 2.17 as5 magic. We sseee 180°, and that _|
makes us think of finding useful lines. We need a line that has angles —
equal to the three in AABC. Equal angles make us think of parallel lines, |
which always give lots of equal angles when cut by a transversal line. _
Close the book and try to re-create this proof on your own!

Now let’s try solving a problem using the fact that the sum of the angles of a triangle is 180°.
Problem 2.18: One angle in a triangle is twice another angle, and the third angle is 54°. What is the
measure of the smallest angle in the triangle?

Solution for Problem 2.18: We know one angle is 54°, but all we know about the other two is that one is
twice the other. Therefore, we let the smaller angle be x, so the larger is 2x. Since the sum of the angles
of a triangle is 180°, we have
54° + +t 2x = 180".
Solving this, we find x = 42°. The angles of our triangle are 42°, 54°, and 84°. (Note we can make a
quick check by adding the three and making sure we get 180°.)
Our smallest angle is 42°. 0

ore The key to tackling word problems in geometry is the same as any other |
= _ kind of word problem— turn the words into math. Usually this means
| defining variables and using the words to write equations to solve for the
variables.

-WARNING!! Your last step should be to make sure you’ve answered the question.
ry thatisasked. |
One of the most common uses of the fact that the angles in a triangle add up to 180° is as an
angle-chasing tool. Let’s give it a try.

Extra! In thespace ofone hundred and seventy-six years the Mississippi has shortened itself two hundred
alededens and forty-two miles. Therefore, in the Old Silurian Period the Mississippi River was peat of
. one million three hundred thousand miles long. Seven hundred and forty-two years from now
: Mississippi will beonly a mile and three-quarters long. There is someth
2 he scinatingoe
- science. One gets such wholesome returns ofconjecture out ofsuch.a trifling i nvestment offact.
: ~ =Mark Twain
ee ee ee
2.5. ANGLES IN A TRIANGLE

Problem 2.19: In the diagram, m || n, AB 1 m, ZADC = ZBCE = 3x,


ACER =o and 2BCD = x) Find x.
|
|
|

Solution for Problem 2.19: We start with our parallel lines, and see that ZABE = 90° because m || n and
m 1 AB. Then, from AABE we have ZEAB + ZABE + ZAEB = 180°, so

ZEAB = 180° — ZABE — ZAEB = 40°.

We can then use AACD to find

ZACD = 180° — ZDAC — ZADC = 180° — 40° — 3x = 140° — 3x.

Concept: When angle-chasing, it’s best to write the values you find for angles on |
your diagram as you find them, even when these values include variables. —
Z J

Our diagram now looks like the figure to the right. This picture suggests
a way to finish the problem. We have three angles with vertices at C that
together make a straight line, so we have 140° — 3x- -C

ZACD + ZDCB + ZBCE = 180°.

Substitution gives
140° — 3x +x + 3x = 180°, A
SOX 405 as

I
txercises i
2.5.1 Two angles ina triangle have measures 30° and 57°. What is the measure of the third angle?
2.5.2 The angles in a triangle are in the ratio 1 : 2 : 3. What are the measures of the angles?

x 2.5.3 Find ZZYP in the diagram at the left.


: 2.5.4 One of the angles in a triangle is a right angle. Show that the other
oe, two angles are complementary.
xX 28 7 p 2.5.5* Using what you know about triangles, find a different solution to
Problem 2.14. Hints: 109

35
CHAPTER 2. ANGLES

2.6 Exterior Angles

[Problem 2.20: Find x given the angle measures shown.

|Problem 2.21: Prove that ZX + ZY = ZXZP in the diagram shown.

Problem 2.22: In the diagram, AB | DE LBAC.= 2x — 207


ZACB = 30°, and ZDEF = x + 55° as shown. Find ZCED.

Problem 2.23: Find x + y + z.

In the previous section, we examined the interior angles of a triangle. In this section, we will take a
look at the angles formed when we extend a side of a triangle past a vertex. We cleverly call these the
exterior angles of the triangle. For example, in the problem below, ZACD is an exterior angle of AABC.

Problem 2.20: Find x given the angle measures shown.


2.6. EXTERIOR ANGLES

Solution for Problem 2.20: From AABC, we have


poet) + ACE = 1 BO")
so ZACB = 66°. From line BD we have
LACB + x = 180°,
so x = 180° — 66° = 114°. Oo
That straightforward solution suggests we can prove something general about an exterior angle of a
triangle.

Problem 2.21: Prove that 2X + ZY = ZXZP in the diagram shown. X

Solution for Problem 2.21: Our solution to Problem 2.20 guides the way. From AXYZ, we have
LX + LY + LXZY = 180°.

From line YZ p we have


LXZY + £XZP = 180.
Subtracting the second equation from the first gives us
LR Ll),
which we can easily rearrange to the desired ZX + ZY = ZXZP.
Notice that our proof does not at all depend on the values of the angles! 0
And now, we have more things to name. ZX and ZY are called the remote interior angles of exterior
angle ZXZP of AXYZ. This name is really not important. We mostly give it a name so we can briefly
write what we just proved:

Important: Any exterior angle of a triangle is equal to the sum of its remote interior
Vv angles.

Problem 2.22: In the diagram, AB ||DE, YBAC = 2x — 20°,


LACB = 30°, and ZDEF = x + 55° as shown. Find ZCED.

Solution for Problem 2.22: We know we'll probably need to find x to answer the problem. We could label
every angle we know in terms of x, but first we take a minute to look for a faster way to get x.

37
CHAPTER 2. ANGLES

We have ZBAC and ZACB of AABC, so we know the exterior angle ZGBC = ZBAC + ZACB = 2x + T0¢.
From AB ||DE, we know that ZGBC = DEF = x + 55°. Hence, we have 2x + 10° = x + 55°, so x = 45°.
Our desired angle is the supplement of ZDEF, so our answer is

ZCED = 180° — ZDEF = 180° — (x + 55°) = 80°.

There are many other ways we could have approached this problem. This is almost always true
when we have problems involving exterior angles. Using exterior angles of a triangle is really just a
shortcut for using a line and what we learned in Section 2.5 about the guess of a triangle. 0

Problem 2.23: Find x + y + z.

Solution for Problem 2.23: We'll use ZA, ZB, and ZC to refer to the interior angles of AABC. Using what
we just learned about exterior angles, we have

ZB+ZC
ZA+ZC
Zz LA + ZB.

Adding these, and noting that ZA + ZB + ZC = 180°, gives:

cys (ZB + ZC) + (ZA + ZC) + (ZA + ZB)


= 2(ZA+ZB+ ZC)
= 2(180°)
= 360°.
You'll be seeing this again. Only, next time, we'll be dealing with a figure that has more sides than a
simple triangle! 0

For an extra Sen Eyto


Tee out the sumot the interiorpes
io
the sum.
s of ) ;
2.7. PARALLEL LINES REVISITED

I)__Exercises
2.6.1 Find y in the figure at left below.

A
110°
38° és

62° cd B CG

Figure 2.9: Diagram for Problem 2.6.1 Figure 2.10: Diagram for Problem 2.6.2

2.6.2 Find ZC in the figure at right above.

Y 2.6.3 Find ZY in the figure at left.

2.6.4 Must an exterior angle of a triangle always be greater than 90°?

Problems

Problem 2.24:
(a) In the diagram at left below, what can we say about lines k and m?
(b) Why? (Hint: What’s wrong with APQR in the second figure?)

angles have the measures indicated in ‘the diagram, -


Problem 2.25: Given that thee
a fiat ABaeCDand BC ah
AD.
CHAPTER 2MANGLESINIAG 38
In
In Section 2.4, we learned some useful relationships about angles when we have parallel lines.
this section, we investigate whether or not we can ‘go backwards’ by using angle relationships to prove
lines are parallel.

Problem 2.24: In the diagram to the right, what can we say about
lines k and m? Why can we say it?

Solution for Problem 2.24: What’s wrong with this solution:


j

Bogus Solution: If lines k and m were parallel, then ZXPY = 2XQZ. Since we know |
: k and m must be parallel. ||
|| Ob that ZXPY = £XQZ, lines
=
This ‘solution’ is exactly the same as saying the following:

If an animal is a cat, the animal must have four legs. My pet Spot has four legs. Spot must
be a cat.

Clearly, this is bogus, because Spot could be a dog. All we know about Spot is that she has four legs.
Our Bogus Solution to the problem has the same flaw!
We know for sure that if lines k and m were parallel, then ZXPY = ZXQZ. It is not logically valid to
just run that backwards and say ‘If 2XPY = ZXQZ, then lines k and m are parallel.’ We have to prove
this second statement separately. In Spot’s case, we saw that the ‘backwards’ version of ‘If an animal is a
cat, then the animal must have four legs’ is not even true! We call this ‘backwards’ version the converse
of the original statement.

-WARNING!! Suppose we have a true statement of the form:


oo ae :
If this, then that.
The converse of this statement is:

If that, then this.

Even if our original statement is true, the converse doesn’t have to be


true. We have to prove the converse separately.

And now, back to our story. We’d like to prove that lines k and m are parallel, and we can’t use
our angle relationships because we don’t know they are parallel. The only other thing we know about
parallel lines is that they never meet. So, we wonder if it is possible for the lines to meet if ZXPY = tXOZ.

eee
2.7. PARALLEL LINES REVISITED

Suppose k and m meet at R as shown in the figure at right. We have k


a triangle, so we try to use what we know about triangles. First, we note P
that 40°
ZRPQ = 180° — 40° = 140°. oth. SAG R
Now that we have two angles of APQR, we can find the third:

ZPRQ = 180° — ZRPQ — ZRQP = 180° — 140° — 40° = 0°.

Uh-oh. If k and m meet at R, then ZR must be 0°, which doesn’t make any sense. Therefore, we have
shown that it is impossible for k and m to meet to the right of PO. (We’ll leave the case of k and m meeting
to the left of ize) as an Exercise.)

Since it is impossible for k and m to meet, they must be parallel. 0

‘Important: Each of the angle relationships regarding parallel lines that we found ©
| in Section 2.4 can be used to prove that two lines are parallel. Using —
| these angle relationships is the most common method of proving that |
| two lines are parallel. |

Our solution to the previous problem is an example of proof by contra-


diction. To prove a statement by contradiction, we start by assuming the —
statement is false. Then we show that this assumption leads us to an
impossible statement (such as ZPRQ = 0° above), which tells us that the
assumption itself is false. Having proved the statement cannot be false,
we have shown it must be true. |

Problem 2.25: Given that the angles are as marked in the diagram, prove that A B
AB ||CD and BC ||AD. 70° 80°

Solution for Problem 2.25: We start by finding the angles we can find. Triangles AACD and AABC tell us
that
ZACD = 180° — 30° — 80° = 70° and ZACB = 180° — 70° — 80° = 30°.

Therefore, we have ZDAC = ZACB, which means that AD || BC. We also have ZBAC = ZDCA, which
tells us AB ||CD.
We also could have noted that 2D + ZDAB = 180°, so AB ||CD. Likewise, 2B + ZBAD = 180° gives us
AD ||BC. o

aes
$«« == = # #f
CHAPTER 2. ANGLES

|)Exercises
2.7.1 We only did half of the proof in Problem 2.24. Complete the proof by showing that the two lines
cannot meet to the left of PO.

2.7.2 For the diagram at left below, we have proved that if 1 cuts k and m such that x = y (that is,
corresponding angles are equal), then k || m. Use this fact to show that if y = z (that is, alternate interior
angles are equal), then k || m.

Figure 2.11: Diagram for Problem 2.7.2 Figure 2.12: Diagram for Problem 2.7.3

2.7.3 Find x in the diagram to the right above. Hints: 427,519


2.7.4 Write the converse of each of the following statements, then identify whether or not that converse
is true.

(a) If two teams are playing in the World Cup Finals, then the teams must be playing soccer.
(b) If two of the angles of a triangle add to 80°, then one angle of the triangle must be 100°.
(c) Ifa river is the longest river in the world, then it must be the Nile.
(d) If an animal is a duck, then it must be a bird.

2.8 Summary

Definitions: Two rays that share an origin form an angle. The common
origin of the rays is the vertex of the angle. We use the symbol Z to denote an
angle, and we use a point on each side and the vertex, or just the vertex, to
identify the angle, such as ZXOY at the right.

We can use a protractor to measure angles (see page 14). The semicircular arc of the protractor is divided
into 180 degrees, so that a whole circle is 360 degrees.

2 eS ee oe
42
2.8. SUMMARY

aye e |
Definitions: Angles can be classified by their measures.

e A 90° angle is a right angle. Lines, segments, or rays that form a right angle are said to be
perpendicular.

e Anangle smaller than 90° is an acute angle.

e Anangle between 90° and 180° is an obtuse angle.

An angle that measures 180° is a straight angle.

An angle of more than 180° is a reflex angle.

Definitions:

e Two angles whose measures add to 180° are supplementary angles. Angles that together
make up a straight angle form a particularly useful example of supplementary angles.

e Two angles whose measures add to 90° are complementary angles.

e When two lines intersect, they form two pairs of vertical angles, such as ZMPN and ZLPO
below. Vertical angles are equal. :

Angles ZLPM and ZNPM together form a straight angle, so they are supplementary (i.e. add to 180°).

Important: The concepts are more important than the words for solving problems. |
‘Angles like ZMPN and ZLPO above are equal’ means something with-
out any more information. “Vertical angles are equal’ doesn’t tell you
anything until you reach for your dictionary to look up vertical angles. —

The words, however, are important for communicating the concepts. For now, though, focus on the
ideas. The words will come naturally.

| Definitions: Two lines that do not intersect are parallel. A line that cuts across multiple parallel
lines is called a transversal line.

43
CHAPTER 2. ANGLES

"Important: The angles formed when a transversal cuts


VY? across two parallel lines come in two groups of
four equal angles as shown:

| a =
| b=ad=fe=h
i Sree a

Each of the first set of angles is supplementary to each of the second set ||
oSee me Page - oe au ibs eee names for pe of these a !
—|

Important: The relationships described above when a2 transversal cuts two lines can
also be used to show that two lines are parallel.

Definitions: When we connect three points with line segments, we form a triangle.
The points are the vertices of the triangle, and the segments are the sides of the triangle.
le 2
| The angles inside the triangle formed by the sides are the interior angles of a triangle.
| When we refer to the angles of a triangle, we mean the interior angles.

meen The sum of the angles in a triangle is always 180°.

Sano When we extend a side past a vertex of a triangle, we form an


| exterior angle of the triangle, such as 4XZP shown. We call 2X and ZY the
|remote interior angles of exterior angle 2XZP.

'|Important: Any exterior angle of a triangle is equal to the sum of its remote
Vv interior angles.

Problem Solving Strategies

e When you can’t find the answer right away, try finding whatever you
can — you might find something that leads to the answer! Better yet,
you might find something even more interesting than the answer.
The best problem solvers are explorers.
e Doing a problem two different ways is an excellent way to check
your answer.

Continued on the next page.


2.8. SUMMARY

'Concepts: . . . continued from the previous page


O==s e Often when we're angle-chasing our goal is to build an equation to
solve for one of the variables in our problem.

Parallel lines are so useful in problems involving angles that some- |


times we'll add new ones to a diagram to help us.

e When angle-chasing, it’s best to write the values you find for angles |
on your diagram as you find them, even when these values include —
variables. |
e The key to tackling word problems in geometry is the same as any _
other kind of word problem — turn the words into math. Usually this
means defining variables and using the words to write equations to
solve for the variables.

e Sometimes using a proof by contradiction is much easier than prov- |


ing a statement directly. To prove a statement by contradiction, we —
start by assuming the statement is false. Then we show that this —
assumption leads us to an impossible statement, which tells us that
the assumption itself is false. Having proved the statement cannot
be false, we have shown it must be true.
bes

Things To Watch Out For!

WARNING!! e Anexample is not a proof!


4.0
e Your last step should be to make sure you’ve answered the
question that is asked.

Proof is at the heart of mathematics. On page 40, we saw one of the most common logical errors
beginners make.

WARNING!! Suppose we have a true statement of the form


ry"
a *

If this, then that.

The converse of this statement is

lf that,then this.
“Even i our ‘original statement i
is; true,the converse doesn’t hae tobe |
true. We have to provethe converse separately.
CHAPTER 2. ANGLES

[Review Prosems ill


2.26 Using your protractor, determine the following angles in the diagram D C
to the right: B

(a) ZAOB
(b) ZAOC A
(c) ZBOC
(d) ZDOE E
2.27 How many seconds does it take the second hand of a clock to rotate through an angle of 72°?

2.28 Given AB ||CD in the diagram to the right, find: La

(a) ZCGE A Hf 3p
(b)* ZHGC 5 Bao
(Cee ia é

(d) ZBHG mp)

2.29 Two angles of a triangle are 30° and 70°. What is the third angle?
2.30 The angle ZB in AABC is 60°. If an exterior angle at A is 170°, what is ZC?

2.31 Find x in the diagram to the right. A 5 B E


x,
2.32 Lines PO and RS are parallel, and TV PO. If TV intersects PO at X
and RS at Y, find ZRYX.
Sx
2.33 In the diagram to the left below, the angles are as marked. Find x. C D

Baek
x
GX140°
110°
D a

Figure 2.13: Diagram for Problem 2.33 Figure 2.14: Diagram for Problem 2.34

2.34 The measures of the angles are as marked in the diagram to the right above. Find x.
2.35 The three angles A, ZB, and ZC have the property that ZA is complementary to ZB, ZB is comple-
mentary to ZC, and ZC is complementary to ZA. Prove that ZA = ZB = ZC.

46
REVIEW PROBLEMS

2.36 Let AABC have (interior) angles in the ratio 3 : 4 : 5. What is the measure of its smallest exterior
angle? Hints: 135
2.37 The exterior angles of a triangle are in the ratio 2 : 3: 4. What are the (interior) angles of the
triangle?

C x7 D 2.38 Is it possible for the angles in the diagram to the left to have the measures
indicated? Why or why not?
E<xty 2.39 Is it possible for two exterior angles of a triangle to be supplementary? Why
5 or why not? Hints: 491
A y B
2.40 Three straight lines intersect at O and ZCOD = ZDOE in the diagram at left
below. The ratio of ZCOB to ZBOF is 7 : 2. What is the number of degrees in ZCOD? (Source:
MATHCOUNTS)

Figure 2.15: Diagram for Problem 2.40 Figure 2.16: Diagram for Problem 2.41

2.41 Find w in the diagram to the right above. Hints: 248


2.42 One angle of a triangle is 20°. If the largest angle of the triangle has six times the measure of the
smallest, what are the angles of the triangle?

2.43 The measures of the angles in the diagram to the right are as marked. C
Find ZC.
2.44 The three angles of a triangle have measures ZA = x — 2y, ZB = 3x + 5y, Ay
and ZC = 5x — 3y. Find x. A B\2x D

2.45 It is not possible to find the value of y in the previous problem from the
given information. What if you are also told that one angle of the triangle is
10°? Is it now possible to compute y? What are the possible positive value(s)
of y?
2.46 Show that if a transversal cuts two lines such that the same-side interior angles are supplementary,
then the two lines are parallel.
CHAPTER 2. ANGLES

UL_ Challenge Problems

2.47 Three angles in the diagram to the right are marked as right angles. If @ = a7,
what is the value of 6? (Source: Mandelbrot)

2.48 Whatis the number of degrees of the angle formed by the minute and hour hands
of a clock at 11:10 PM? (Source: MATHCOUNTS) Hints: 52, 356, 282

2.49 Find x in the diagram at left below. Hints: 355

B G

LK

a E

Figure 2.17: Diagram for Problem 2.49 Figure 2.18: Diagram for Problem 2.50

2.50 The angles in the diagram to the right above are as marked. Find ZBEA. Hints: 8
2.51 One angle of a triangle is equal to the sum of the other two. Show that the sum of two exterior
angles of the triangle is 180° greater than the third. Hints: 151, 200
2.52 One of the angles of the triangle in the previous problem is equal to 40°. What are the measures
of the other two angles?
2.53 The angles of a triangle are in arithmetic progression. If one of the angles is 100°, what are the
measures of the other two angles? (An arithmetic progression is a sequence of numbers in which the
difference between each term and its preceding term is always the same.) Hints: 581
2.54 Itis possible for the interior angles of a triangle to be in the ratio 1 : 2 : 6, but is it possible for the
exterior angles of a triangle to be in the ratio 1 : 2 : 6? Prove your answer. Hints: 14, 444
2.55 Find ZA + 4B + ZC + 4D in the figure at left below. Hints: 358
A B

D
(S
Figure 2.19: Diagram for Problem 2.55 Figure 2.20: Diagram for Problem 2.56

2.56 Findw+x+y+zin the figure at right above. Hints: 17, 400


2.57x Point Z is on side PR of APQR such that ZPZQ = ZPQZ, and ZPQR —ZPRO = 42° >FindwZRO7Z.
Hints: 54, 441, 156

48
/

Outer Napoleon Triangle

Equality may perhaps be a right, but no power on earth can ever turn it into a fact. — Honore de Balzac

CHAPTER

Congruent Triangles

3.1 Introduction

Two figures are congruent if they are exactly the same - in other words, we can slide, spin, and/or flip
one figure so that it is exactly on top of the other figure.

Smee
oat
Figure 3.1: Congruent Figures

Figure 3.1 shows four pairs of congruent figures. In each case, we can take one of the figures and
slide it, spin it, and/or flip it so that it exactly coincides with the other.

49
CHAPTER 3. CONGRUENT TRIANGLES

D Q

E He F P i R

Figure 3.2: Congruent Triangles

We use the symbol= to denote that two figures are congruent. For example, to describe the congruent
triangles in Figure 3.2, we write ADEF = AQPR. Notice that we are careful to put the vertices in the
same order: D corresponds to Q, E to P, and F to R. We would not, for example, write ADEF =APQR
for the triangles in Figure 3.2.
If we have two congruent triangles, all the corresponding pairs of sides are equal, as are the corre-
sponding angles. Conversely, if all pairs of corresponding sides of two triangles have equal lengths, and
all the corresponding angles of the two triangles are equal, then the triangles are congruent.
Fortunately, to show that two triangles are congruent, we don’t have to go through the hassle of
proving each pair of sides and each pair of corresponding angles are congruent. In this chapter, we'll
cover various ways requiring considerably less information to show that two triangles are congruent.

3.2 SSS Congruence

|Problems
lg
Problem 3.1: In each of the three triangles below, the side lengths are 5, 7, and 8 units. Are the
measures of the
Manele the same in all three triangles? (Yes, you should use your protractor.)

Problem 3.2: In this problem we investigate how many different non-congruent triangles have sides.
with lengths 2.5 cm, 3.5 cm, and 4 cm by building mee ABC with these side lengths. Start by
drawing segment AB with length 4 cm.
eS Suppose C is 3.5m away from A. What points on your paper are 3.5 cm away from A? tee
___ the figure that consists of all these points.
) We have AB = 4. cm, and our answer to (a) tellsus hee Ccmust be so that AC Sa 5c We
~also want C to be 2.5 cm away from B. What ee on your paper are 2.5 cm as B? Draw the
figure that consists of all these points. :
: :)| ‘Where i
in your resulting diagram can Cc| possibly be?
a(dd) How many different non-congruent triangles have sides with lengths 2.5cm, 3 5cm, and 4cm?

See
eeea eae ee
3.2. SSS CONGRUENCE

Problem 3.3: In the figure, the sides and angle have measures as shown. L a7 M
Find MLN.
9 |

17

Problem 3.4: In this problem we will show that if a radius of a circle bisects (cuts in half) a chord of
the circle that is not a diameter, then the radius is perpendicular to the chord.
(a) Draw a circle with center O and a chord connecting points A and B on the circle.
(b) Draw a radius of the circle that passes through the midpoint, M, of AB, thereby bisecting AB.
(c) Find a pair of congruent triangles and use these triangles to show that ZOMA = ZOMB = 90°.

Often our best way to get started with a new geometric principle is to experiment. So, we start off
by examining some triangles that have the same side lengths and checking if the triangles are indeed
congruent.

Problem 3.1: In each of the three triangles below, the side lengths are 5, 7, and 8 units. Are the
measures of the angles the same in all three triangles? (Yes, you should use your protractor.)

Solution for Problem 3.1: Using our protractor, we find that the angles of each triangle are approximately
82°, 60°, and 38°. Each triangle has the largest angle opposite the largest side and the smallest angle
opposite the smallest side. Hence, the sides and the angles appear to be the same in all three triangles,
so it looks like they’re congruent. 0
Let’s see if we can get acommon-sense explanation for why the triangles of Problem 3.1 are congruent.

Problem 3.2: Investigate how many different non-congruent triangles have sides with lengths 2.5
cm, 3.5 cm, and 4 cm by building triangle ABC with these side lengths.

Solution for Problem 3.2: It doesn’t matter which side we set to 4 cm, so we just pick AB to be the long
side. We start building AABC by drawing a 4 cm long segment.

A B
Now we have to use our other side lengths to figure out where C can possibly be. Suppose AC = 3.5
cm. Then, C must be 3.5 cm away from A. So, C must be on a circle with center A and radius 3.5 cm.
Similarly, since BC = 2.5 cm, C must lie ona circle with center B and radius 2.5 cm. We draw both circles
and end up with the diagram below. Since C must be on both of the circles, C must be at one of the
intersection points. Our two options for C (labeled C; and C2) are merely mirror images of each other.

51
CHAPTER 3. CONGRUENT TRIANGLES

If we had instead assumed AC = 2.5 cm and BC = 3.5 cm, we would end up with essentially the
same picture, as shown below.

Notice that if we ‘flip’ our first diagram horizontally, we get the second one. In fact, we don’t really
even need this second diagram. We could have simply swapped the labels A and B in our first diagram
to investigate the possibility that AC = 2.5 cm and BC = 3.5cm. 0
3.2. SSS CONGRUENCE

i m7 nh YY Tey oe
}

Important: We call the principle illustrated in Problems 3.1 and 3.2 the Side-Side- |
Side Congruence Theorem, or SSS Congruence for short. SSS states:

If the lengths of the sides of one triangle equal the lengths of


the corresponding sides of another triangle, then the triangles
are congruent.

For example, in the diagram below, we have AABC = AXZY by SSS


Congruence.
x
| A Xx

|
G
|
ys |
L
B |
Our ‘proof’ that all triangles with side lengths 2.5 cm, 3 cm, and 4 cm are congruent skips over a few
very important points. First, we didn’t prove that triangles AABC; and AABC)? are congruent in each
of our cases. Nor did we prove that the triangles in the first case are congruent to the triangles in the
second. Intuitively, it seems clear that these triangles are all congruent, but we haven’t proved it. In
fact, with the tools we have at this point, we cannot prove SSS Congruence at all!
We must accept as an axiom one of the Congruence Theorems we will study in this chapter. We can
then use that one theorem, and some other tools, to prove the others. Since some of those other tools are
considerably more advanced than those we have now, we'll stick to our intuitive explanations for all of
the Congruence Theorems.

Solution for Problem 3.3: What’s wrong with this:

‘Bogus Solution: Since LM and NO are parallel, ZMLN = ZLNO = 25°.


a

This Bogus Solution assumes that LM ||NO. This might be true, but we have to prove it to use it.

Since LO = MN, ON = LM, and LN = LN, we have ALNO = ANLM by SSS. Since angles ZMLN and
ZONL are corresponding parts of these two triangles, they must be equal. Therefore,

MLN = ZONDS25-.

Notice that we can now use ZMLN = ZONL to prove that LM ||NO. o

53
Ee
CHAPTER 3. CONGRUENT TRIANGLES

Important: Problem 3.3 shows us a very important general use of congruent tri- |
angles. Once we determine that two triangles are congruent, we can
apply whatever we know about the sides and angles of one triangle
to the other triangle. This obvious principle often goes by the fancy
name ‘Corresponding Parts of Congruent Triangles are Congruent,’ or
CPCTC.

Unless your teacher tells you to, you don’t have to clutter your paper writing CPCTC all over.
Once you have proved that ALNO = ANLM, you can simply write, ‘Since ALNO = ANLM, we have
ZMLN = ZONL.’
Let’s try using congruent triangles in a proof.

|Problem 3.4: Prove that if a radius ofa circle bisects (cuts in half) a chord of the circle that is not a |
diameter, then the radius must be perpendicular to the chord.

Solution for Problem 3.4: We start with a diagram, drawing circle O, chord AB with “f C
midpoint M, and the radius through M. We then mark the equal pieces of the chord Sele
and the equal radii of the circle, and see that we have congruent triangles. (Radii is
the plural of radius.) Specifically, AAMO = ABMO by SSS. So, ZAMO = ZBMO. Since
ZAMO + ZBMO = 180°, we must have ZAMO = ZBMO = 90°. oO

If a radius of a circle bisects a chord of the circle that is not a diameter, |©


DE

Important:
Vv? then the radius must be perpendicular to the chord. |
|
As we'll see in Chapter 6, this works in reverse, too! A radius perpendicular to a chord must bisect
the chord.

3.2510 EE = Grand FG-= EH inthe diagram at right. E H


(a) Prove that AEFG = AGHE.
(b) Show that ZEGF = ZGEH.
(c) Show that HE ||FG.
(d) Show that HG ||EF.
3.2.2 In triangle ABC, AB = AC. Let M be the midpoint of side BC.
(a) A segment, line, or ray bisects an angle if it divides it into two angles with equal measure. Show
that AM bisects BAC by proving that ZBAM = ZCAM.
(b) Show that AM BC.

54
3.3. SAS CONGRUENCE

3.2.3. In the diagram at right, AB-= 7, AD = 4, CD = 4, and BC = 7. Prove that A


ZABD = ZCBD. Hints: 165

3.3. SAS Congruence

i]Problems il
Problem 3.5: In each of the triangles below, two sides have lengths 2.5 cm and 4 cm, and the angle
between these sides is 60°. Measure the third side and the other two angles of each triangle.
C hie

AH

A Gee D | C

Problem 3.6: Investigate why the triangles in the previous problem appear to be congruent by
constructing a triangle with side lengths 2.5 cm and 4 cm such that the angle between these two sides
is 60°.
(a) Draw segment AB such that AB = 4 cm.
(b) Let BC be the side that is 2.5 cm long; thus, C is 2.5 cm from B. Construct the figure that consists
of all points that are 2.5 cm from B.
(c) The angle between AB and BC must be 60°. Where can C be to make ZABC = 60°? (Forget about
BC = 2.5 cm for this part.)
Draw your answer to (c).
(e) Where can C possibly be located?
(f) Are all triangles with two sides of length 2.5.cm and 4 cm with an angle of 60° between them
congruent?

Problem 3.7: Point E is the midpoint of both AC and BD as shown. Prove that
AB ||CD.
CHAPTER 3. CONGRUENT TRIANGLES

Problem 3.8: In the figure below, AD = CD and AB = BC. Prove that DB 1 AC. Hints: 461
D

Showing that all the sides of one triangle equal those of another is not the only way to quickly prove
two triangles are congruent. In this section we investigate one way to use angles along with sides.

Problem 3.5: In each of thetriangles below, two sides have lengths 2.5 cm and 4 cm, and the angle
between these sides is 60°. Measure the third side and the other two angles of each triangle.

Solution for Problem 3.5: In each case, the third side is 3.5 cm and the other two angles are approximately
82° and 38°, with the 82° angle opposite the 4 cm side. The triangles sure look congruent. 0

Sidenote: Perhaps only the ancient Greeks have contributed as much to geometry |
as the German mathematician David Hilbert. Among his wide range of |
work was Grundlagen der Geometrie, which placed geometry in a formal
axiomatic framework. Instead of Euclid’s 5 axioms, Hilbert used 21 ax-
ioms, since Euclid’s work included many hidden assumptions that must
be accepted as axioms. Among those axioms is SAS Congruence, which,
in combination with other axioms, he used to prove the other Congruence
Theorems presented in this chapter.
Not only did Hilbert produce much great mathematics, but he also in-
spired and challenged others. In 1900 he gave an address to the Second
International Congress in Paris on 23 unsolved problems of great impor-
tance. Since then, some of the problems have been solved, but many still
puzzle mathematicians to this day. He also succinctly phrased the passion
that guides many great mathematicians and scientists when he closed an
| address with the words, “We must know, we shall know.”

56
3.3. SAS CONGRUENCE

Problem 3.6: Investigate why the triangles in the previous problem appear to be congruent by
constructing a triangle with side lengths 2.5 cm and 4 cm such that the angle between these two sides
is 60°.

Solution for Problem 3.6: We start off just like we did in Problem 3.2, by letting let AB = 4 cm. We want
BC = 2.5 cmand ZABC = 60°. Since C is 2.5 cm from B, it lies on a circle with center B and radius 2.5 cm.
Since ZABC = 60°, we know that C lies on a ray from B that forms a 60° angle with AB. We can use our
compass and protractor to build the circle and our two possible rays as shown below.

Gi

Co

In our diagram there are only two options for C: the two points where one of the rays meets the
circle. Just like we saw in Problem 3.2, these C’s are mirror images. AABC; and AABC) have the same
side lengths and angle measures.
Therefore, we find that no matter how we build the triangle given two sides and the angle between
them, we always get a triangle with the same sides and angles. 0

As with our illustration of SSS on page 51, our solution to Problem 3.6 is not a complete proof of the
SAS Congruence Theorem. It merely provides an intuitive explanation. Let’s see SAS in action.
CHAPTER 3. CONGRUENT TRIANGLES

[Problem 3.7: Point E is the midpoint of both AC and BD as shown. Prove that
AB ||CD.

Solution for Problem 3.7: We saw in Section 2.7 that we can use angle equality to prove that lines are
parallel. We can get the angle equality we need by using congruent triangles. Since AE = EC, BE = ED,
and ZAEB = /CED (because they are vertical angles), we have AAEB = ACED by SAS. Therefore,
ZABE = LCDE, so ZABD = ZCDB and we have AB ||CD. O
P Sec oe SOU RET IE eee
Problem 3.8: In the figure, AD = CD and AB = BC. Prove that DB 1 AC. D

Solution for Problem 3.8: We have to prove that two lines are perpendicular, but we don’t have any
information about angles. We look for congruent triangles since that will give us some angle information.
The given side equalities along with the obvious BD = BD give us ADAB = ADCB by SSS. This
doesn’t give us any information about angles at point E, but it does give us some angle equalities like
LADE = CDE. This, together with AD = DC and DE = DE, gives us AADE = ACDE by SAS. Therefore,
ZAED = CED. Since these two angles are also supplementary, we have ZAED = ZCED = 90°, so
BDeAGsg

Exercises

3.3.1 Which two of the triangles below must be congruent and why must they be congruent?

A D G J M
6 6 go 6 6 6

; B 2 E : H P K : N
€ F I L O
3.3.2 In triangle ABC, AB = AC. Let M be the point on BC such that AM bisects ZBAC (so that
LCAM = ZBAM).

(a) Show that M is the midpoint of BC.


(b) Show that AM . BC.

58
3.4. ASA AND AAS CONGRUENCE

3.3.3 Find VZ in the diagram at the left below. (Note: The diagram is not to scale.) Hints: 467

D es
3.3.4 In the figure to the right above, AB = CD and ZBAC = ZDCA. Prove that AD || BC.

3.4 ASA and AAS Congruence

Problems

Problem 3.9: Shown are three triangles with two angles measuring 60° and 40°, and each triangle has
a side of length 3 cm between these two angles. Measure the other two sides of each triangle. Make a
guess about triangle congruence given two angles and an included side.

B H

Problem 3.10: Investigate why the triangles in the previous problem are congruent by constructing
a triangle with angles 60° and 40° such that the side between the two angles has length 3 cm. Start by
drawing a segment AB such that AB = 3 cm.
(a) Let the angle between AB and BC be 60°. Where can C be to make ZABC = 60°?
(b) Draw your answer to (a). The angle between AB and AC must be 40°. Where can C be to make
ZBAC = 40°? (Forget about ZABC = 60° for this part.)
(c) Draw your answer to (b). Where can C possibly be located?
Are all triangles with a side of length 3 cm between angles with measures 60° and 40° congruent?

Problem 3.11: What if the equal sides are not between the equal angles?
For example, what if AB = DE = 6, 2B = ZE = 60°, and ZC = ZF = 70°
as shown? Can we conclude that AABC = ADEF? Why or why not?
CHAPTER 3. CONGRUENT TRIANGLES

D ; C
Problem 3.12: In the diagram, AD = BC, AD || BC, and E and F are
on AC so that ZADE= CBE. Prove that AB ||CD and DF= EB.

| E

‘Problem 3.9: Shown are three triangles with two angles measuring
60° and 40° and each triangle
has a side of length 3 cm between these two angles. Measure the other two sides of each triangle.
Make a guess about triangle congruence given two angles and an included side.

Solution for Problem 3.9: In each triangle, the side opposite the 40° angle‘has length about 2 cm, and the
side opposite the 60° angle has length around 2.6 cm. The triangles appear to be congruent. 0

Problem 3.10: Investigate why the triangles in the previous problem are congruent by constructing
a triangle with angles 60° and 40° such that the side between the two angles has length 3 cm.

Solution for Problem 3.10: We start with AB of length 3 cm. We let ZABC = 60°.
As in Problem 3.6, this means that point C is on one of two rays from B that
make an angle of 60° with AB. Similarly, we have ZBAC = 40°, which means
that C is on one of the two rays from A that make an angle of 40° with AB. Point
C must be at one of the two intersections of rays. The two potential C’s, marked
C; and C2, are mirror images of each other.

Therefore, we find that all the triangles we can build given two angles and
the side between them are congruent. 0

_ Extra! ‘Divide the figure at right into four oe pieces thatcan be

Source: ThinkFun Inc.'s Puzzles webpage. ee the links page men-


7 “tioned in
i the Eon eeorae fora link to Pe site.

60
3.4. ASA AND AAS CONGRUENCE

Important: We call the principle illustrated in Problem 3.10 the Angle-Side-Angle


Vv Congruence Theorem, or ASA for short. ASA states that

If two angles of one triangle and the side between them are
equal to the corresponding angles and side of another trian-
gle, then the two triangles are congruent.

For example, in the diagram below, we have AABC = ADFE.

F
C

Problem 3.11: What if the equal sides are not between the equal B
angles? For example, what if AB = DE = 6, ZB = ZE = 60°, and
60°
ZC = ZF = 70°? Can we conclude that AABC = ADEF? Why or why
not? 6

Solution for Problem 3.11: If we have two angles of a triangle, we have the third. In the diagram above
we can quickly see that ZD = ZA = 50°. Hence, we can conclude that AABC = ADEF by ASA. 0
When we have the situation illustrated above, we don’t have to find the third angle and invoke ASA.
This procedure always works, so it gets an unsurprising theorem name of its own.

Important: We call the principle illustrated in Problem 3.11 the Angle-Angle-Side_


vv Congruence Theorem, or AAS for short. AAS states:

If two angles and a side of one triangle equal the correspond-


ing angles and side in another triangle as shown below, then
the triangles are congruent.
CHAPTER 3. CONGRUENT TRIANGLES

“WARNING!! : When usinga AAe the equal sides oe be adjacent to corresponding |


“s equal angles. For example, the triangles shown below are not congru- |
ent because the equal sides are not adjacent to corresponding equal ©
| angles! |
| B E |
|
| 5 |
| 30 r

}
|

Problem 3.12: In the cee Le ‘BC, ADI BC ee E D &


and F are on AC so that ZADE= ZCBF. Prove that AB ||CD
and DF = EB.

D C Solution for Problem 3.12: We go hunting for equal angles to use to


prove that AB and CD are parallel. We start by marking the angles we
know are equal. We have a pair in our original diagram, and can add
ZDAC = ZACB due to AD ||BC, as shown to the left.
A B Now we can find some congruent triangles. Since AD = BC,
ZLDAC = ZBCA, and AC = AC, we have ACAD = AACB by SAS. Therefore, ZDCA = ZBAC, so AB || CD.

We also have another pair of congruent triangles: ADAE = D os


ABCF by ASA. It’s not immediately clear how this will help
us prove BE = DF, so we go ahead and mark the new equal
lengths we know from our new triangle congruence. Focusing
on segments BE and DF, we look for congruent triangles that
have these as sides. We start by looking for triangles we know
something about, which leads us to AADF and ACBE. We A B
already have a side and an angle, and AF = AE + EF = CF + EF = CE gives us a second side. So,
AADF = ACBE by SAS. Finally, we conclude that DF = BE.
We could also have used SAS to show ADFE = ABEF, so DF = BE. o

Concept: In complicated geometry problems, mark side and angle equalities as you
. find them (particularly when you find non-obvious ones!)

Extra! The real voyage of discovery consists not in seeking new ised. but in having new eyes.
DT dllndlledilind —Marcel Proust
62
3.5. SSA NOT-NECESSARILY CONGRUENCE

3.4.1 Find all pairs of triangles below that must be congruent. Write out the appropriate congruence
(make sure you have the vertices in the right order!), and explain why the triangles must be congruent.

A D G J M

5 5 Eye 5 5
C F La I ie O
B E H K N

3.4.2 In the diagram at right, AB || DC. If M is the midpoint of Geog


must it also be the midpoint of BD? Why or why not? Hints: 183 €
3.4.3 Use ASA Congruence to prove that AAS Congruence is a valid
Congruence Theorem. (Do not assume AAS Congruence is a valid the- A
orem for this part — you are asked here to show that any two triangles a
that satisfy the AAS criteria are indeed congruent without using AAS.) Hints: 463

3.4.4 Inthe figure, PQ = PR and ZPQY = ZPRX. ie


(a) Prove that QY = RX. aS
(b)x Prove that XN = YN. Hints: 158, 343 0 N R

3.5 SSA Not-Necessarily Congruence

Problem 3.13: In the diagram below, we have three triangles with two sides of length 2.5 cm and 3
cm, and an angle of 40°. The 40° angle has the 3 cm segment as a side, but does not have the 2.5cm
segment as a side. Are the triangles congruent? —_, :
CHAPTER 3. CONGRUENT TRIANGLES

Problem 3.14: In this problem we explore why there’s no ‘Side-Side-Angle Congruence Theorem’ by
building a triangle with AB = 1.8 cm, BC = 1.5 cm, and <BAC= 40°.

(a) Draw AB = 1.8 cm.

(b) We know that BC = 1.5 cm. Draw the points in the diagram that are 1.5 cm from B.
(c) Draw the points in the diagram that can be C such that ZBAC = 40°. (Forget BC = 1.5 cm for
now.)

(d) Use the previous two parts to determine where C can possibly be.
(e) Is SSA a valid congruence theorem?

(f) Do we ever have a case in which given the lengths of sides AB and BC, and ZBAC, there’s only
one possible length of AC?

Problem 3.13: Inthe diagram, we we have three triangles with twosides of length 2.5 cmand 3cm, and
an angle of 40°. The 40° angle has the 3 cm segment as a side, but does not have the 2.5 cm segment
as a side. Are the triangles congruent?

B EF

A . D

Solution for Problem 3.13: Um, no. Clearly HI is way smaller than EF and BC, so the triangles aren’t
congruent. 0

SSS works. SAS works. ASA, and AAS, too. But SSA, not so much. Why?

Problem 3.14: Figure out why there’s no ‘Side-Side-Angle Congruence Theorem’ by building a
triangle with AB = 1.8 cm, BC = 1.5 cm, and ZBAC = 40°.

Solution for Problem 3.14: We start as we have before, drawing AB with length Co
1.8 cm. Since BC= 1.5 cm, C must be on a circle with center B and radius 1.5
cm. Also, since ZBAC= 40°, C must be ona ray from A that makes a 40° angle ee
with AB. The ray and the circle are shown to the right. S|
Uh-oh. The ray hits the circle in two points, Cj and C2. Both AABC, and
AABC2 match the information we are given about AABC, but they are very
obviously not congruent. 0

64
3.6. ISOSCELES AND EQUILATERAL TRIANGLES

WARNING! — Side-Side-Angle (SSA) is not a valid congruence theorem. You cannot


4.4 : :
ry use it to prove that two triangles are congruent.

6 45°

You might have noticed that if we try to build a triangle given two sides and a non-included angle
as in Problem 3.14, we don’t always get two possible points. The left diagram above shows a case in
which we do get exactly one possible C, and the right figure above shows a case in which no triangles
are possible. We'll be revisiting the former case later when we have more tools to talk about this special
case.

3.6 Isosceles and Equilateral Triangles

Problems a

Problem 3.15: In the diagram, AB = AC = 5 and ZCAB = 30°.


(a) Let M be the midpoint of BC. Draw AM. Prove that AACM = AABM. 2
(b) Find ZAMB. A oe
(c) Find ZACB. 5

Problem 3.16: Prove that if ZPQR = ZPRQ, then PR = PQ.

Problem 3.17: In the diagram, AC = CD = DB, and ZB = 23°.


Find ZA.
CHAPTER 3. CONGRUENT TRIANGLES

Problem 3.18: In the diagram, XY = XZ = 8 and ZX = 60°. Find YZ.

Problem 3.19: Find ZPBD.

Problem 3.20: In the diagram, O is the center of the circle, AB || CD,


ZABO = 24°, and ZOBC = ZOCD. Find ZBOC.

Problem 3.21: In the diagram at right, AB = BC = ACand AE=EB= E


BF = FC = CD = DA such that AABC is completely inside ADEF.
Prove that DE = EF = DF.

In this section, we investigate triangles that have two or three sides equal in length.

Problem 3.15: In the diagram, AB = AC = 5and /CAB


= 30°. Find ZACB.
3.6. ISOSCELES AND ) EQUILATERAL -
a es
TRIANGLES

Solution for Problem 3.15: We're a little thin on information. We have a side C
equality that seers important. But we only have one triangle, so there’s 2
no obvious way to use the side equality. We might look for congruent 4 M
triangles, but we'll have to somehow make two triangles. Splitting AABC
by connecting A to the midpoint, M, of BC gives us our congruent triangles. 5 '
Since AB = AC, BM = CM, and AM = AM, we have AAMB = AAMC by SSS. This tells us that
2B = LC, which is just enough to finish the problem. Since the angles of AABC add up to 180°, we have
180° = LA + ZB + ZC = 30° + 2A(LC).
Solving for LC, we have ZC = (180° — 30°)/2 = 75°. 5
A triangle in which two sides are equal is called an isosceles triangle. The equal sides are sometimes
called the legs of the triangle, and the other side the base.
Our general approach in Problem 3.15 can be used to show that if two sides of a triangle are equal,
then the angles opposite those sides are equal. These two equal angles are often called the base angles
of the triangle, and the other angle the vertex angle. As we saw in the last problem, if AABC is isosceles
with 2B = LC, we have ZB = ZC = (180° — ZA)/2.
We might now wonder if this runs the other way: do slo rie ey, equal sides?

Problem 3.16: Prove that if POR =/PRQ, then PR =PQ. _ Q

Solution for Problem 3.16: What's wrong with this solution?

"Bogus Solution: We proceed as we did in Problem 3.15, by connecting P to M, the


midpoint of QR, as in Figure 3.6. Since
MR = MQ, ZR = 4Q, and
PM = PM, we have APMR = APMQ. Therefore, PR = PQ. |
Q |
|

P M

This failed solution uses ‘SSA’ Congruence, which we have shown in Section 3.5 doesn’t work! We
have to construct our triangles in a way that lets us usea valid congruence theorem.

67
CHAPTER 3. CONGRUENT TRIANGLES

Instead of picking the point on QR that cuts it into two equal lengths, we Q
make equal angles by connecting P to point X on QR such that PX 1 OR.
Now we have APXR = APXQ by AAS. Therefore, PQ = PR, as desired. (Note P x
also that QX = XR, so that X is the same as point M in our Bogus Solution.
This doesn’t make that Bogus Solution any more correct, though!) 0

Putting our last two problems together gives us a pair of powerful tools.

Important:
VY A

B Gc

1. If AB = AC in AABC, then ZB = ZC.

2. IE ZB = ZC in AABC, then AB = AC
|

We'll try these tools on a couple more problems, and discover another important type of triangle.

Problem 3.17: In the diagram, AC = CD = DB, and ZB = 23°. A


Find ZA.

Solution for Problem 3.17: Since DC = DB, we have ZDCB = ZB = 23°. We could find ZCDB to get ZADC,
or we can note that ZADC is an exterior angle of ABCD, so ZADC = ZDCB + ZB = 46°. Since AC = DC,
we have ZA = ZADC = 46°. o

Problem 3.18: In the diagram, XY = XZ = 8 and ZX = 60°. Find YZ. x

Solution for Problem 3.18: We can’t do much about YZ right away, but we can find ZY and ZZ. Since
LX = 60°, ZY + ZZ = 180° — 60° = 120°. Since XY = XZ, we have ZY = ZZ, so LY = LZ = 60°. Hence, all
the angles of AXYZ are equal. Specifically, 2X = ZY means that YZ = XZ,so YZ =8. O
We call the triangle in Problem 3.18 an equilateral triangle because all of its sides are equal. As
Exercises, you will use an approach a lot like the one above to show:

68
3.6. ISOSCELES AND EQUILATERAL TRIANGLES

|Important: If all three angles of a triangle are equal, then so are all thece des |
| VV Conversely, if all three sides are equal, then all three angles are 60°.
FEW RO ASAE. RE DEER AE SS NS a NNO EIR

Therefore, in order to prove a triangle is equilateral, we can either prove all the sides are equal, or
prove that all the angles are equal.
Let’s put our isosceles and equilateral triangles to work.

‘Problem 3.19: Find ZPBD.


| P
|
|

|
|

|
!

Solution for Problem 3.19: We start by drawing BD so we can see the angle we are after. Since B
APAB is equilateral, PBA = 60°. ABAD is isosceles, since AB = AD. Since BAD = 90°, the
other two angles of ABAD equal (180° — 90°)/2 = 45°. Therefore, we have A B

ZPBD= ZPBAUI/ABD =1105,.

Problem 3.20: In the diagram, O is the center of the circle, AB || CD,


ZABO = 24", and ZOBE = 7OCD. Find 2BOG.

Solution for Problem 3.20: We have parallel lines and an isosceles triangle (OB = OC yore sais
because they are radii of the same circle). So, we can do some angle-chasing. We B
let ZOBC = x, so that we also have ZOCD =x from the given information and
ZOCB = x from isosceles AOCB. Since AB || CD, we have ZABC + ZBCD = 180°.
Therefore,
ZABO + ZOBC + ZBCO + ZOCD = 180°. D SCG wets C
Substitution gives 24° + 3x = 180°, so x = 52°. Therefore, ZBOC = 180° = 2% = 76°. B

Extra! Mathematics is the gate and key to the sciences.


TLdlindlindlind —Roger Bacon

69
CHAPTER 3. CONGRUENT TRIANGLES

Concept: Assigning a variable to an angle will often help you with your angle- |
= chasing. If you're stuck on an angle problem, assign one of the angle |
measures a variable and find other angles in terms of that variable. Hope-_
fully, you'll eventually be able to build an equation you can use to solve —
for the variable. oS |

We'll end this chapter with a proof involving equilateral and isosceles triangles.
Problem 3.21: In the diagram at right, AABC is equilateral and E Ss D
AE = EB = BF = CF = AD = CD such that AABC is completely inside
ADEF. Prove that ADEF is equilateral. ve

Solution for Problem 3.21: In order to prove that ADEF is equilateral, we


must show that either all its sides have the same length, or all its angles
have the same measure. We start by marking all the given information.
Our most useful tool in showing side lengths are equal is congruent
triangles. Therefore, we look for triangles with DE, EF, and FD as
corresponding sides. The triangles that stand out are ADEA, AEFB,
and AFDC. If we can show these are congruent, we’ll have the desired
DE = EF = FD.
In order to show that ADEA = AEFB = AFDC, we need only show
that ZDAE = ZEBF = ZFCD, since we already know that the sides adjacent to these angles in our three
triangles are all equal. To learn more about these angles, we look at the other angles at A, B, and C.
We already have ZCAB = ZABC = ZBCA. Furthermore, isosceles triangles AEAB, AFBC, and ADCA
are all congruent by SSS, so their base angles are all equal. Now that we know three of the angles with
vertex A equal the corresponding angles with vertex B, we can show ZDAE = ZEBF:
ZLDAE = 360° — ZEAB — ZBAC — ZDAC = 360° — ZFBC — ZCBA — ZEBA = ZEBF.
Similarly, we have ZDAE = ZFCD. Therefore, by SAS we have ADEA = AEFB = AFDC, which gives us
the desired DE = DF = EF. So, ADEF is equilateral.
Try finding an alternate solution by showing that all the angles of ADEF are equal. 0
We typically prove a triangle equilateral by either proving all its side lengths are equal, or all its
angles are equal. This often involves finding congruent triangles.

Concept: _ Mark the information you have ina problem on your diagram, particularly
_/==8 ~~equal sides and equal angles. This will make congruent triangles easier to”
3.6. ISOSCELES AND EQUILATERAL TRIANGLES

We've explored isosceles and equilateral triangles, but we’re not finished naming triangles yet! We
have a name for two sides equal, and one for all three sides equal. If no two sides of a triangle are equal,
the triangle is scalene.

3.6.1 In APQR, PQ = PRand ZP = 43°. Find ZQ.


3.6.2. Prove that if AB = AC in AABC, then ZABC = ZACB. (Note: You cannot simply state that the
triangle is isosceles, so the base angles are equal. You are asked here to prove this fact.)
3.6.3

(a) Prove that if the three sides of a triangle are equal in length, then all three angles of the triangle
have measure 60°.
(b) Prove that if the three angles of a triangle are equal, then all three sides of the triangle have the
same length.
Note that for this problem, you are not allowed to state that the triangle is equilateral as your proof. You
must prove the facts about equilateral triangles that you learned in the text.
3.6.4 Two angles of an equilateral triangle have measures 3x + 27° and 2y — 4°. Find x + y.
3.6.5 Point O is the center of the circle in the diagram on the left below. Find ZAOB if ZOAB = 70°.
Hints: 79
A

B yy
/e\.

aN
a B c
Figure 3.3: Diagram for Problem 3.6.5 Figure 3.4: Diagram for Problem 3.6.6

3.6.6 Triangle ABC is equilateral in the diagram on the right above. Points D, E, and F are in triangle
ABC such that ZCAD = ZABE = ZBCF. D lies on CF, E lies on AD, and F lies on BE. Prove that triangle
DEF is equilateral. Hints: 192,574

3.6.7 In the diagram at right, VW = VX and WX ||YZ. V

(a) Prove that WY = XZ. Hints: 178,518 W x


(b) Prove that YX = WZ. Hints: 315 NA Da

Extra! | am interested in mathematics only as a creative art. :


Tea dediad -G. H Hardy

71
CHAPTER 3. CONGRUENT TRIANGLES

3.7. Construction: Equilateral Triangle and Perpendicular Bisector

As a reminder, we once more repeat the rules of the game for constructions. The tools of classical
construction problems are a compass and a straightedge. Notice that we don’t say ‘ruler.’ You don’t get
to use your straightedge to measure — you can only draw lines. Similarly, you aren't allowed to use your
protractor to measure or create angles.

The only operations you can perform with your compass and straightedge are:

1. Given a point, you can draw any line through the point.

. Given two points, you can draw the line that passes through them both.

. Given a point, you can draw any circle centered at that point.

WO
rE
N . Given a point and a segment, you can draw the circle with its center at that point and with radius
equal in length to the length of the segment.

5. Given two points, you can draw the circle through one point such that the other point is the center
of the circle.

i Problems >

/Problem 3.22: Construct an equilateral triangle with a compass and straightedge. |

|Preblem 3.23: Draw a segment AB. We call the line that is perpendicular to AB and passes through
the midpoint of ABthe perpendicular bisector of AB. Construct the perpendicular bisector of AB.

If you used your protractor to make a 60° angle to construct an equilateral triangle, go back to the
beginning of this section and READ THE RULES!
Problem 3.22: Construct an equilateral triangle. ;

Solution for Problem 3.22: We start off by drawing a segment AB. We want
to find point C such that AC = BC = AB. Now we have a situation a lot
like our SSS ‘proof’ of Problem 3.2. Since AC must equal AB, we know
that C must be on the circle with center A and radius AB. We can construct
this circle with our compass. Similarly, C is on the circle with center B
and radius AB. Drawing this circle as well gives us the diagram to the
right. Our two circles meet at C and C’. Both triangles ABC and ABC’ are
equilateral.
To prove that this construction works, we only have to prove that AC = AB = BC. Since each of these
segments is a radius of a circle that is defined to have radius AB, clearly all three segments are equal. 0

72
3.8. SUMMARY

Important: Construction problems are two-part problems. Find the construction, —


NW then prove that it works. The second part is usually pretty easy after |
| you've found the construction, but don’t forget to do it! |

You'll see that in this book we often leave the proof part for the reader. Sometimes we’ll ask you to
supply the proof that our construction works as an Exercise.

‘Problem 3.23: Given asegment AB, construct a line through the midpoint of AB that is pee
to AB. We call this line the perpendicular bisector of AB.

Solution for Problem 3.23: In our last construction, we found two points
that appear to be directly above and directly below the midpoint of AB.
Perhaps connecting these points will give us the line we want. We start
with the same construction we used to make an equilateral triangle. Then
we draw CC’, which meets AB at M as shown. By soo, ACAG = ACEC,
so ZACC’ = ZBCC’. Therefore, ZACM = /ZBCM. Since we also have
AC = BC and ZCAM = ZCBM as we found in the previous problem, we
have AACM = ABCM by ASA. So, AM = BM and ZCMA = ZCMB. Since
ZCMA+ZCMB = 180°, we must have ZCMA = ZCMB = 90°. Therefore, GG’ is the perpendicular bisector
of AB. 0

Concept: Always be thinking about what you already know how to do when trying
trying |
> to do something new!

RTOS
3.7.1 Given a segment AB, divide the segment into 4 equal pieces with a straightedge and compass.
3.7.2 Is it necessary when constructing the perpendicular bisector of AB to use circles with radius AB?
Suppose we instead draw two intersecting circles with centers A and B and the same radius. Is the
segment connecting the two points where these circles meet the perpendicular bisector of AB?
3.7.3. Construct a 90° angle. Hints: 94
3.7.4 Construct a 30° angle. Hints: 504

3.8 Summary

In this chapter we investigated the following theorems for proving that two triangles are congruent:

73
CHAPTER 3. CONGRUENT TRIANGLES

e SSS Congruence: If the sides of one triangle equal the sides of another triangle, then the triangles
are congruent. (Section 3.2)

e SAS Congruence: If two sides of one triangle and the angle between them are equal to those of
another triangle, then the two triangles are congruent. (Section 3.3)

e ASA Congruence: If two angles of one triangle and the side between them are equal to those of
another triangle, then the two triangles are congruent. (Section 3.4)

e AAS Congruence: If two angles and a side of one triangle equal the corresponding angles and
side in another triangle as shown below, then the triangles are congruent. (Section 3.4)

2
e

Important: Once we determine that two triangles are congruent, we can apply what- —
ever we know about the sides and angles of one triangle to the other trian- _
gle. This obvious principle often goes by the fancy name eas i |
Parts of Coneant Triangles are Congruent,’ or CPCTC.

Definitions: A triangle in which two sides are equal is an isosceles triangle. The equal A
sides are the legs of the triangle and the other side is the base. The angle between the
two equal sides is often called the vertex angle of the triangle, and the other two angles
are the base angles, which are equal to each other.

Important: 1. If AB = AC in AABC, then ZB = CC.


V 2. If ZB = ZC in AABC, then AB = AC

| Definition: A triangle iin which all three sides are equal is an equilateral triangle. All three angles
of an equilateral triangle are 60°.

Important: If all three angles of a triangle are equal, then so are all three sides.
Conversely, if all three sides of a triangle are equal, then all its angles
equal 60°.

Throughout the book we will use congruent triangles to prove many important results. One of the
first is:

eee eC ee A
74
REVIEW PROBLEMS

|Important: _ If a radius of a circle bisects a chord of the circle that C |


| wD is not a diameter, then the radius must be perpendic- A EN B |
ular to the chord. oe

Problem Solving Strategies

| Concepts: e Inmore complicated geometry problems, mark side and angle equal- |
| ities as you find them (particularly when you find non-obvious ones!) »
e Dividing isosceles triangles in half by drawing a segment from the |
vertex between the equal sides to the midpoint of the base can be >
very effective. |
e If you’re stuck on an angle problem, assign one of the angle measures |
a variable and find other angles in terms of that variable. Hopefully, |
you'll eventually be able to build an equation you can use to solve |
for the variable.
|
e Mark the information you have in a problem on your diagram, par- |
ticularly equal sides and equal angles. This will make congruent |
triangles particularly easy to find. |
e Always be thinking about what you already know how to do when |
trying something new!

Things To Watch Out For!


oe

WARNING!! — SSA (Side-Side-Angle) is not a valid congruence theorem. If two sides


“Ss of one triangle are equal to two sides of another, and the two triangles
have equal corresponding angles that are not the angles between the |
equal corresponding sides, then the two triangles are not necessarily
congruent!

REVIEW PROBLEMS

3.24 In each of the diagrams below, name all pairs of congruent triangles you can identify (without
drawing more segments or naming more points). Write the relevant triangle congruences and explain

Tas)
CHAPTER 3. CONGRUENT TRIANGLES

why the triangles are congruent. If there are not any pairs of triangles in a given diagram that must be
congruent, state so.

Gages eet. tests ree came


D

A C F i K N
L
(a) ae Olea (f)
A
D M
i . 2 J K N

(g) B D (h) G (i) K M

}: \
4 : F lain
= f J TE N

3.25 Find the vertex angle of an isosceles triangle if one base angle is 6° less than half the vertex angle.
3.26 Find x in the diagram shown at left below.
A G

2x

B D

S B A

Figure 3.5: Diagram for Problem 3.26 Figure 3.6: Diagram for Problem 3.27

3.27 Find ZA in the diagram at right above.


3.28 Segments AB and CD meet at point P such that AP = CP and BP = DP.
(a) Isit always true that AAPD =~ ACPB?
(b) Is it always true that AD ||BC? Hints: 547

——
ee eae
REVIEW PROBLEMS

3.29 Let PQR be a triangle as shown at right, and let S and T be points on P
sides PR and OR, respectively. Triangles PTS, PTQ, and RTS are congruent.

(a) Show that ZPST = 90°.


(b) Find the measures of the angles of the triangle PQR.
Q di R
3.30 In triangle NOK at left below, KO = KN. Let I be the point on KN such that OI = ON. Let A be the
point on NO such that JA = IN.

(a) Find three angles in the diagram equal in measure to ZN.


(b) Prove that IA || KO.

N A

A I

D ees
O K Cis
Figure 3.7: Diagram for Problem 3.30 Figure 3.8: Diagram for Problem 3.31

3.31 Inthe diagram at right above, O is the center of the circle, OA = CD, and ZAOB = ZOCD.

(a) Show that ABOA = AOCD.


(b) Prove that OC ||AB.
(c) Show that ZOCB = 2ZOBA.
(d) Find ZOBA.

3.32 In the diagram at left below, AD = BD = AE and DE = EC. Prove that AC = BE. Hints: 71
A

ae
D c >
A
4 ‘

Figure 3.9: Diagram for Problem 3.32 Figure 3.10: Diagram for Problem 3.33

3.33 Inthe figure at right above, BOC = 42°, Ois the center of the circle, CD ||OA, and BAO = 22BCD.

(a) Find ZCBO.


(b) Show that ZBOA = 180° — 4ZBCD.
(c) Find ZBOA. Hints: 145

eects
- -
CHAPTER 3. CONGRUENT TRIANGLES

3.34 ABand XY are diameters of OO. Prove that AX = BY.

3.35 In triangle ABC, let M be the midpoint of BC. Prove that if MA = MB = MC, then ZBAC = 90°.
Hints: 546

3.36 All four angles of a square are 90° and all four sides have the same length. Square A ied
EFGH is inscribed in square ABCD as shown at right, meaning that each vertex of EFGH H
is on a side of ABCD. E

(a) Show that ZAHE = ZBEF. Bag

(b) Prove that triangles AEH, BFE, CGF, and DHG are congruent.

3.37 In the figure at left below, AABC is equilateral, AE = DC = BF, and EB = CF = AD, such that
AABC is completely inside ADEF.

(a) Prove that ADEF is equilateral. Hints: 73


(b) Would ADFF still be equilateral if AABC were not fully inside ADEF?

le E

Figure 3.11: Diagram for Problem 3.37 Figure 3.12: Diagram for Problem 3.38

3.38 In ABCDEF at right above, AB = FA, BC = EF, CD = DE, and ZCDA = ZADE. Prove that
ZABC = ZEFA. Hints: 33, 500

3.39 NP in the diagram bisects both ZMNO and ZMPO. Which of the following e
statements must be true? For each statement that must be true, prove that it must be
true. For each statement that may not be true, explain why it may not be true.

(a) NO=MN.
(b) NO = OP.
(c) MP ||NO. N
(d)x MO 1 NP.

3.40 In the diagram at right, A is the center of the circle and CD 1 AB. We are given
that ZACD = 2ZDCB.

(a) Let ZDCB = x. Find expressions for ZABC and ZBAC in terms of x.
(b) Find ZABC. i
a,

78
CHALLENGE PROBLEMS

i]_Challenge Problems Ji
3.41 The measures of two angles of an isosceles triangle are 3x + 4° and x + 17°. Find all possible values
of x. Hints: 148
3.42 Triangle ABC is equilateral in the diagram at left below, and ABDE, BCFG, and CAHI are squares.
Prove that triangle DFH is equilateral. Hints: 274, 534

A 1®)

Se
Lx)
D as I
‘<i _/
SS
G ESF F

Figure 3.13: Diagram for Problem 3.42 Figure 3.14: Diagram for Problem 3.43

3.43 Let ABCD bea rectangle, meaning thatall four of its angles are right angles. Construct equilateral
triangles ABE and BCF externally on sides AB and BC as shown at right above. Prove that triangle DEF
is equilateral. Hints: 88, 482, 582

3.44 AABC hasaright angle at ZC. Points D and E are on AB as shown at left below such that AD = AC
and BE = BC. Find ZDCE. Hints: 401, 232

YN eee 8) B D ©

Figure 3.15: Diagram for Problem 3.44 Figure 3.16: Diagram for Problem 3.45

3.45x Let ABCD be a square, and let E, F be points such that DA = DE = DF = DC and ZADE =
ZEDF = FDC. Prove that triangle BEF is equilateral. Hints: 280, 86,575
3.46 In triangle ABC in the diagram below, D and E are points on side AB, and F and G are points on
side AC, such that AD = DG = GB = BC = CE = EF = FA. Find ZBAC. Hints: 436

79
CHAPTER 3. CONGRUENT TRIANGLES

3.47 Inthediagramatright, DZE = ZEZY, ZCYE = ZAYE, and ED ||YZ.


Prove CD = DZ — CY. Hints: 399

3.48 Describe under what conditions SSA Congruence ‘works’. In other


words, when is it true that if AB = XY and BC = YZ and LBAC = XZ;
that we can immediately conclude that AABC = AXYZ? (No proof is
necessary; give a clear description of when SSA works and an intuitive
A Y 7,
explanation why it works in these cases.) Hints: 142
3.49 In the diagram at left below, ZXYZ = ZAXZ = 90°, AX = XY, and ZXZD = ZYZD. Prove that
ZD 1 AY. Hints: 197,325
Q
A ih Rs
B g

xX BONS P.

Figure 3.17: Diagram for Problem 3.49 Figure 3.18: Diagram for Problem 3.50

3.50 Draw equilateral triangles BCP, CAQ, and ABR outside AABC as shown in the diagram at right
above. Prove that AP = BQ = CR. Hints: 455

3.51x There are two possible triangles with AB = 13, BC = 10, and ZA = 40°. What is the sum of the
two possible values of ZB? (Source: Mandelbrot) Hints: 9, 357, 163

Extra! Take a look at the nearest globe (or imagine one). A woman at the North Pole, looking for
‘>in =the shortest path to Quito, Ecuador, just barely south of the equator, would fly straight
down the 78° West line of longitude. Then, to fly to Bifoun, Gabon (which is a quarter of
the way around the world to the East), she turns 90° to face East, and fly straight along
the Equator. She then returns to the North Pole by turning 90° North and flying along
another line of longitude, thus completing a triangular trip. She could begin repeating
her trip by turning 90° to the left and then flying to Quito again.

North Pole

_ At each stop, you can see that she must turn 90° to head towards the next stop. But
that’s a total of 270° in this triangle on the surface of the globe! How can that be?
Research spherical geometry to answer this mystery.

80
The Pizza Theorem

[here is no transfer into another kind, like the transfer from length to area and from area to a solid. — Aristotle

CHAPTER LE nan ee ER

[Oe eee and Area

4.1 Perimeter

The perimeter of a closed figure is how far you travel if you walk along its boundary all the way around
it once. So, the perimeter of a triangle is simply the sum of the lengths of the sides of the triangle.

Problem 4.2: Find the perimeter of ADEF.

Problem 4.3: Given that each little square in the grid is a 1 x 1 square,
find the perimeter of ABCDEFGHI/.
PERIMETER AND AREA
CHAPTER 4. he
a

Problem 4.4: The length of each leg of an isosceles triangle is three times the length of the base of the
triangle. The perimeter of the triangle is 91. What is the length of the base of the triangle?

Questions about perimeter are usually just questions about segment lengths. We'll try a few.
late
‘Problem 4.1: AABC is equiFind ral.
AB given that the perimeter of AABC is 36.
Solution for Problem 4.1: Since AABC is equilateral, all of its sides are equal. Let each have length s. Then
the perimeter is 3s. Therefore, we have 65 = 56,808 = 12.0
a a — ———— ee

| Problem 4.2: Find the perimeter of ADEF. D

|
|
Solution for Problem 4.2: Since ZE = ZF, we have DE = DF = 8. Therefore, our perimeter is DE+ DF + EF =
16+8 v3.0
‘Problem 4.3: Given that each little square in the grid is 1 x 1 square,
find the perimeter of ABCDEFGHI]J.

Solution for Problem 4.3: We could simply find the lengths of the 10 sides and add up all the numbers, but
there’s a slicker approach. All of our sides are either vertical or horizontal. The horizontal side on the
bottom has length 8; therefore, the sum of the horizontal sides on top, which together cover the same
horizontal distance without backtracking, must be 8. Similarly, AB + CD = 5, so the vertical sides on
the left have total length 5. The ones on the right have the same total length, so they contribute 5 to the
perimeter as well. So, our perimeter is 2(8 + 5) = 26. O

Problem 4.4: The length of each leg of an isosceles triangle is three times the length of the base of
the triangle. The perimeter of the triangle is 91. What is the length of the base of the triangle?

Solution for Problem 4.4: Let the base of the triangle have length x. Then the length of each leg is 3x.
Since the perimeter of the triangle is 91, we must have x + 3x + 3x = 91. Solving this equation for x, we
find x = 13. The length of the base of the triangle is 13. O

|Concept: —The key to solving most geometric word problems is the same as for most
_(©=s2 _~—s‘non-geometric word problems: assign variables to unknown quantities
and use the words to make equations you can solve.
4.2. AREA

|
_Exercises
4.1.1 In the diagram at left below, XY = 5 V2, YZ = 10, and ZY = /Z = 45°. Find the perimeter of
DAY Z:

G
X
5 V2 ; : ite
E
G
: 10 B A

Figure 4.1: Diagram for Problem 4.1.1 Figure 4.2: Diagram for Problem 4.1.2

4.1.2 Triangles ABC, ADE, and EFG at right above are all equilateral. Points D and G are midpoints of
AC and AE, respectively. If AB = 4, what is the perimeter of figure ABCDEFG? (Source: AMC 8)
4.1.3

(a) Must two congruent triangles have the same perimeter? Why or why not?
(b) Must two triangles with the same perimeter be congruent? Why or why not?
(c)* Must two triangles with the same perimeter be congruent if the triangles have one angle measure
in common?

4.1.4 Each little square in the grid at right is a 1 x 1 square. Find the
perimeter of ABCDEFGH.
4.1.5 A triangle with perimeter 45 has one side that is twice as long as
the shortest side and another side that is 50% longer than the shortest
side. Find the length of the shortest side of the triangle. Hints: 480

4.2 Area

Figure 4.3: A Square Figure 4.4: A Rectangle

A square is a figure that has four sides equal in length and four right angles, as shown in Figure 4.3. We
call a four-sided figure with four right angles a rectangle, an example of which is shown in Figure 4.4.
(A square is a special type of rectangle.) The opposite sides of a rectangle have equal length, as shown.

83
CHAPTER 4. PERIMETER AND AREA

We’ll learn more about squares and rectangles in Chapter 8.

Roughly speaking, the area of a figure is the total number of 1 x 1 squares needed dame é
to cover the figure, where we add fractional squares when we only need a piece of a 2
square to do the job. For example, we need 35 squares to cover the region ABCDEF at
right, so we say the area of ABCDEF is 35. We will often use brackets to denote area, B ‘&
like this: [ABCDEF] = 35.

Problems : Be

Problem 4.5: Consider rectangle ABCD, which has AB = CD = 4,BC = AD=5, B 5


and right angles at A, B, C, and D. What is the area of ABCD? ee
-

A D

Problem 4.6: The length of one side of a rectangle is 4 less than 3 times an adjacent side. The perimeter
of the rectangle is 64. Find the area of the rectangle.

Problem 4.7: A triangle that has a right angle as one of its angles is calleda B
right triangle. 2

(a) Find the area of right triangle AABC, where AB = 4, BC = 6, and #


LB = 90°,
(b) Find a formula for the area of a right triangle given the lengths of its A
sides.

Problem 4.8:

(a) Find the area of ADEF given that J is on EF, DJ = 8, DJ 1 EF, and EF = 7.
(b) Use the previous part to come up with a way to find the area of any triangle.
Does your approach work for obtuse triangles?

Problem 4.9: Given that ZZXY = ZXWY = 90°, XY = 8, XZ = 6, and ZY


find XW.
4.2. AREA

Problem 4.5: Consider rectangle ABCD, which has AB = CD = 4,BC =AD=5, B 5 | C


and right angles at A, B, C, and D. What is the area of ABCD?

Solution for Problem 4.5: We can fill ABCD with four rows of five 1 x 1 squares each, as
shown to the right. So, the area is (4)(5) = 20. O

Finding the area of a rectangle by taking the product of consecutive sides works even
when the sides aren’t integers.

‘Important: A rectangle with two adjacent sides of lengths € and w has area fw. Since
}
|
|
WY a square is just a rectangle in which these lengths are the same, the area
of a square equals the square of the length of one of its sides. |
t
}

Let’s apply our knowledge of rectangles to a problem.

Problem 4.6: The length


of one side of a rectangleis4 less than 3 timesan adjacent side. The
perimeter of the rectangle is 64. Find the area of the rectangle.

Solution for Problem 4.6: Let the lengths of the two sides be x and y. We are given that x = 3y—4. Since the
opposite sides of a rectangle are equal, the perimeter of the rectangle is 2x + 2y = 23y — 4) + 2y = 8y—8.
We are given that the perimeter is 64, so we have 8y — 8 = 64. Therefore, y = 9 and x = 3(9) — 4 = 23, so
the area of the rectangle is 9(23) = 207. 0

Now let’s use our rectangle formula to find the area of a triangle. We'll start with a special kind of
triangle.

(a) Find the area of right triangle AABC, where AB = 4, BC = 6, and ZB = 90°.
(b) Find a formula for the area of a right triangle given the lengths of its sides.

Solution for Problem 4.7:


(a) We don’t know anything about finding the area of a triangle yet; all we know about are rectangles.
However, we know that if we draw a diagonal of a rectangle, we cut the rectangle into two
congruent right triangles. In this problem we have a triangle — perhaps we can make a rectangle
by just adding another right triangle as shown below.
In the diagram, AACD is a copy of ACAB. Since BAC + ZB + ZBCA = 180° B 6 c
and ZB = 90°, we have ZBAC + ZBCA = 90°. Since ACAD = AACB, we have
ZDAC = ZBCA. Therefore, 2BAC + ZDAC = 90°, so ZBAD is a right angle. 4 4
Similarly, BCD is a right angle, so ABCD is a rectangle.
6 D
Since ABCD is a rectangle with sides 4 and 6, we know that [ABCD] = 24. a
Our two right triangles are congruent and the sum of their areas is [ABCD]. So, each right triangle
has area 24/2 = 12.

85
CHAPTER 4. PERIMETER AND AREA

(b) Let a and b be the lengths of the two sides adjacent to the right angle of AABC. We call the sides
adjacent to a right angle in a right triangle the legs of the right triangle. As shown in the previous
part, we can build a rectangle with consecutive sides of lengths a and b out of two congruent
copies of AABC. The rectangle then has area ab, so each of the two triangles has area ab/2. So, the
area of any right triangle is half the product of the lengths of the legs of the triangle.

So that takes care of right triangles. But what if the triangle isn’t a right triangle?

Problem 4.8: so cee D


(a) Find the area of ADEF given that J is on EF, DJ = 8, DJ 1 EF, and EF = 7.
(b) Find a method to determine the area of any triangle.

Solution for Problem 4.8:


(a) We do know how to tackle right triangles, so we break ADEF into ADE] and AD JF:

[DEF] = [DEJ] + (DF = VOD


2
, VOD
2
«Mey
2
+jr)= Der
2
= (2)
2
7) =28.
(b) We call a perpendicular segment from a vertex of a triangle to the opposite side of the triangle an
altitude of the triangle. Sometimes an altitude is also called a height. The terms ‘altitude’ and
‘height’ can also refer to the length of the altitude. The side to which the altitude is drawn is called
the base that corresponds to the given altitude. For example, we say D] is the altitude to base EF
in ADEF above. (As with ‘altitude’, the term ‘base’ can also refer to the length of the base.) Point
J gets a special name, too. The point at which an altitude meets the base to which it is drawn is
sometimes called the foot of the altitude. Thus, ] is the foot of the altitude from D to EF.
Our work so far strongly suggests that the area of a triangle is one-half the product of an altitude
of a triangle and the length of the base to which the altitude is drawn. For example, in part (a),
the area is one-half the product of EF and DJ, and DJ is the altitude from D to base EF.
We have already shown that if our base is either leg of a right triangle, then the area is

base X altitude
5 :

Our work on the first part of this problem can be used to show that this formula works whenever
the two angles at the base are acute, as ZE and ZF are.
But what if
one of the angles at the base is obtuse? In the diagram to X
the right, let YZ be the base. Here, we can’t draw a line from X that is
perpendicular to YZ, so we extend side YZ and draw our altitude to the
extension as shown on the next page. Now we need to check whether 7
[XYZ] = (XP)(YZ)/2. Y

86
4.2. AREA

In our first part we added areas; here, we subtract: Xx

[XYZ] [XPZ] — [XPY]


(XP)(PZ) (XP)(PY)
AR eR) Pay
XP(PZ — PY)
2
(XP)(YZ)
ioe

We have to tweak our definition of ‘altitude’ a little, but our formula still works.

We can now summarize our findings about the area of a triangle.

"Important: To find the area of a triangle, we select one side to be R |


Vv the base. The perpendicular segment from the vertex |
opposite the base to the base (extended if necessary) is
the altitude. We then have:
_ base x altitude
Area 2

For example, in the triangle shown, we have:

PQXxRZ
[POR] = —=2
You'll get plenty of practice finding areas of triangles in the pages to come. But first, we see that
unlike perimeter, which is rarely a useful problem solving tool, area can be used to solve problems that
don’t appear to have anything to do with area.

Problem 4.9: Given that ZZXY = ZXWY = 90°, XY = 8, XZ =6,and ZY =10, X


find XW.

Solution for Problem 4.9: This problem only has information about lengths and angles, and it doesn’t even
ask for area. However, all those perpendicular lines make us think about area. We can quickly see that
Pena ee
= 24.

XW is the altitude to side YZ, so [XYZ] = (XW)(YZ)/2. Since [XYZ] = 24, we have the equation
24 = (XW)(10)/2 = 5XW, so XW = 24/5. 0
Sprinkled throughout this book will be many other uses of area as a problem solving tool.
I ra a ee
87
CHAPTER 4. PERIMETER AND AREA

[|
_Exercises ig
4.2.1 Find the area of each figure shown below.

A (c) C 14 4
(a)

ji 8

B C J I

4.2.2 What is the area of square ABCD if the perimeter of the square is 36?

4.2.3 Inthe diagram at right, KO = 6, LM = 7, and KM = 12.

(a) Find [KLM].


(b) Find LN. Hints: 506

4.2.4 One gallon of paint covers 80 square feet of wall. How many gallons of paint do I have to buy if
Iam going to paint two rectangular walls that are each 24 feet long and 8 feet high?
4.2.5 A square poster is replaced by a rectangular poster that is 2 inches wider and 2 inches shorter.
What is the difference in the number of square inches between the area of the larger poster and the
smaller poster? (Source: MATHCOUNTS) Hints: 102

4.2.6x The perimeter of a square garden is 64 meters. The path surrounding


the garden has uniform width and has an area of 228 square meters. How many
meters of fencing are needed to surround the outer edge of the path? (Source:
MATHCOUNTS) Hints: 494

4.3 Same Base/Same Altitude

Problem 4.10: Given that BC = 40 and CD= 20 in the figure, what are
[ABC]/[ACD] and [ABC]/ ee
4.3. SAME BASE/SAME ALTITUDE

Problem 4.11: Suba and Sam have been hired to paint a triangle with base
AB on the weirdly shaped wall in the diagram. They are told to choose
either C or D as the third vertex of the triangle. Suba thinks using D will
make the triangle look cooler, but Sam thinks using C will result in a smaller
triangle to paint. How can Suba convince Sam to choose D? He

Problem 4.12: AC and BD meet at X as shown. Given [ABX] = 24,


[BCX] = 15, and [CDX] = 10, find [ADX].

In this section we examine a particularly useful sEneIe area concept.

Problem 4. 10: Given that Nee 40 and CD =203in the figure, what are A
|[ABC]/[ACD] and [ABC]/[ABD]?

|
Solution for Problem 4.10: The altitude from A is the same for both AACD and AABC. Let h be the
length of this altitude. Hence, [ABC] = (BC)(h)/2 = 20h and [ACD] = (CD)(h)/2 = 10h. Therefore,
[ABC]/[ACD] = 20h/10h = 2. Notice that [ABC]/[ACD] = (BC)/(CD).
Similarly, the altitude from A in each of AABC and AABD has length h. Therefore,
[ABC]. = HBO iz. BC 2
[ABD] (BD)(h)/2 BD 3°
A key step in our solution is noting that the three triangles have an altitude in common. Notice that in
each case the ratio of the areas of the triangles equals the ratio of the lengths of the sides to which this
common altitude is drawn. 0
Problem 4.11: Suba and Sam have been hired to paint a triangle with
base AB on the weirdly shaped wall in the diagram. They are told to
choose either C or D as the third vertex of the triangle. Suba thinks using
D will make the triangle look cooler, but Sam thinks using C will result in
a smaller triangle to paint. How can Suba convince Sam to choose D? 2

89
CHAPTER 4. PERIMETER AND AREA

Solution for Problem 4.11: To figure out which of AABC or AABD will use more paint, we must consider
the areas of the two triangles. Since

(AB)(BC)as
[ABD] = eg) —, and [ABC] = ay

that
eS te [ABD] _ (AB)(DE)/2 _ DE _ 3
[ABE] M(AB)O)/ 25 bea
So, Suba should point out that AABD will require less paint because its area is smaller than that of AABC.
O

The last two problems illustrate two useful problem solving principles:

| Important: 1. If two triangles share an altitude, then the ratio of their areas is |
the ratio of the bases to which that altitude is drawn. This is —
particularly useful in problems in which two triangles have bases |
along the same line. |

2. If two triangles share a base, then the ratio of their areas is the ratio |
of the altitudes to that base.

———

Problem 4.12: AC and BD meet at X as shown. Given [ABX] = 24,


[BCX] = 15, and [CDX] = 10, find [ADX]. 2 So ae

D
Solution for Problem 4.12: Since AABX and ACBX share an altitude from B, we have
AX _ [ABX] _ 8
Coe BCR a ©)
Turning to triangles AADX and ACDX, we have

[Aas =o
[COX eC 5
Therefore, [ADX] = (8/5)[CDX] = 16. o

Extra! Can you cut the figure at right into two congruent pieces that can be
“mii arranged to form a rectangle?

90
4.3. SAME BASE/SAME ALTITUDE

|_._Exercises ES
4.3.1 In the diagram at left below, PT = 6, TR = 3, and QV = 4.

(a) Find [POR], [PTO], and [QTR].


(b) Find TR/PT and [QTR]/[PTQ].
P A

4 Q

R G E ia

Figure 4.5: Diagram for Problem 4.3.1 Figure 4.6: Diagram for Problem 4.3.2

4.3.2 Inthe diagram at right above, AB = 2 and BE = 3.

(a) Find [BCD]/[ACD].


(b) Find [ADBC]/[ADC]. Hints: 255
(c) Find [BCD]/[ACBD]. Hints: 432
4.3.3 Euclid Apartment Building has a wall that is 30 feet long and 10 feet high. Jean is supposed to
paint a triangle with one vertex at the top of the wall, and a base that runs 8 feet along the bottom of
the wall. Jean wants to put the vertex at one corner and the base at the other side of the wall as shown
on the left. George, the building owner, wants to save money on paint. He insists that the top vertex
be right above the middle of the base, as shown on the right, so the triangle won’t be so big. However,
to show that he’s a nice guy, he says that she can make the base 10 feet wide instead. Jean argues that
George’s design will use more paint than hers. Is she right?

4.3.4 By what factor is the area of triangle multiplied if the length of its base is doubled and the height
is tripled? (Source: MATHCOUNTS)

4.3.5* Find QX in the diagram at right given that WX = 8 and [PQX] =[WXYZ]/6. Z ee
Hints: 244, 390 P
W ae Xx

Extra! For every complex problem there is an answer that is clear, simple, and wrong.
Te dodiod —H. L. Mencken

91
CHAPTER 4. PERIMETER AND AREA

4.4 Summary

Definition: The perimeter of a closed figure is how far you travel if you walk along its boundary
all the way around it once.

Definition: The area of a closed figure is the number of 1 x 1 squares (or pieces of squares) needed
to exactly cover the figure. We sometimes use brackets to denote area, so that [ABC] means the area
of AABC.

Definition: A rectangle has four sides and four right angles, as shown. Further-
more, opposite sides of a rectangle equal each other in length.

|Important: The area of a rectangle with consecutive sides of length f and w is fw.
Vv Since a square is just a rectangle in which these lengths are the same,
the area of a square equals the square of the length of one of its sides.

|Definitions: A triangle that has a right angle as one of its angles is called a right A
| triangle. The sides adjacent to the right angle of a triangle are called the legs of the
triangle.

|Important: The area of aright triangle is half the product of the legs of the triangle.
Vv? For example, in the right triangle shown, we have

sane) = 45X80,
Extra! Tedrick, trying to trick me into believing that 8 x 8 is 65, showed me how he could cut
“~~ an 8 x 8 square into 4 pieces that he could then reassemble into a 5 x 13 rectangle.

The 8x8 square has an area of 64, but the 5 x 13 rectangle has an area of 65. How e this
possible? i

COee ee
REVIEW PROBLEMS

‘Important: To find the area of a triangle, we select one side to be R


Vv the base. The perpendicular segment from the vertex
opposite the base to the base (extended if necessary) is
the altitude. We then have:

_ base x altitude
Area : aR Se

For example, in the triangle shown, we have:


PQx RZ
[PQR] = a

Area can be a very powerful problem-solving tool. One particularly useful pair of principles is:

|
Important: 1. If two triangles share an altitude, the ratio of their areas is the ratio |
NW of the bases to which that altitude is drawn. This is particularly —
useful in problems in which two triangles have bases along the -
| same line. |

2. If two triangles share a base, then the ratio of their areas is the ratio |
of the altitudes to that base. i

REVIEW PROBLEMS >

4.13 Find the area of each figure below.

(a) a 12 C (b) EB (c) H 12 i

iy
15 ;
B D F F G i

(d) K L
14

N M

4.14 A triangle has area 42 and one side of length 7. Find the length of the altitude to the side of length
7. Is it possible to find the perimeter of this triangle?

93
CHAPTER 4. PERIMETER AND AREA

4.15 Find the perimeter of a square that has an area of 75.


4.16 Find the perimeters of AABC and AXYZ shown below given that [ABC] = 24 and [XYZ] = 30.

A Je Y
=
6 ae
B G 4

4.17 The length of a given rectangle is 2 less than 4 times the width of the rectangle. Its perimeter is
51. What is its area?
4.18 Ihave a rectangular living room that is 24 feet long and 16 feet wide. I want to buy a carpet that
costs $2.50 per square foot, and I want to leave a 2 foot margin between the carpet and the wall all the
way around the room. How much will the carpet cost?

4.20 A rectangle has a perimeter of 28 cm and an area of 48 cm. Find


the dimensions of the rectangle.
4.21 If the height of a triangle is multiplied by 4, what must we do to
the base of the triangle in order to leave the area unchanged?
4.22 The altitude from A to BC in AABC has the same length as the
altitude from B to AC. Prove that BC = AC.

4.23 The area of rectangle ABCD shown at left below is 36 and DE = 2EC.

(a) What is [BCD]?


(b) What is [BED]?

Figure 4.7: Diagram for Problem 4.23 Figure 4.8: Diagram for Problem 4.24
4.24 Each of the outer triangles in the figure shown at right above has perimeter 23. The perimeter of
BDFHJ is 51. What is the perimeter of star ABCDEFGHI/J? Hints: 510

94
CHALLENGE PROBLEMS

4.25 In the figure at left below, AB = 12 cm and BC = AD = 8 cm. BC 1 ABand DA 1 AB. How many
Square centimeters are in the area of the shaded region? (Source: MATHCOUNTS)

Figure 4.9: Diagram for Problem 4.25 Figure 4.10: Diagram for Problem 4.26

4.26 Inthe figure at right above, [WOZ] = 12, [ZOY] = 18, and [WXYZ] = 50. Find [WOX].

Challenge Problems

4,27 Find the area of ABCD shown at right. Hints: 577 Sipea.
4.28 A gardener plans to build a fence to enclose a square garden plot. The
perimeter of the plot is 96 feet, and he sets posts at the corners of the square. The 6
posts along the sides are set 6 feet apart. How many posts will he use to fence the
entire plot? (Source: MATHCOUNTS) A 3 D
4.29 We begin with an equilateral triangle. We divide each side into three segments of equal length,
and add an equilateral triangle to each side using the middle third as a base. We then repeat this, to get
a third figure.

Given that the perimeter of the first figure is 12, what is the perimeter of the second figure? What is the
perimeter of the third figure? Hints: 273, 141
4.30x Suppose we continue the process described in Problem 4.29 forever. What is the perimeter of
the resulting figure?

4.31 Allsides of the building shownat right meet at right angles. If three
of the sides measure 2 meters, 7 meters, and 11 meters as shown, then 5)
what is the perimeter of the building in meters? (Source: Mandelbrot)
CHAPTER 4. PERIMETER AND AREA

4.32 In AABC at left below, CX = 2BX and AY = 3BY. Find [BXY] given that [ABC] = 144. Hints:
285, 89
A

C X B sie
Figure 4.11: Diagram for Problem 4.32 Figure 4.12: Diagram for Problem 4.33

4.33 The diagram at right above is formed by placing 4 squares together along a single line as shown.
Each square has a side length that is 1/2 the side length of the next larger square. The outer perimeter
of the figure is 115. What is the area of the whole figure? (Source: ARML) Hints: 329

4.34 Sue and Barry are trying to find the area of AABC. Sue mistakenly uses AB and the height from
A (instead of the height from C), and Barry mistakenly uses BC and the height from C (instead of the
height from A).

(a) Sue finds an area of 12 and Barry finds an area of 27. What is the area of AABC? Hints: 369, 38
(b)* Suppose instead that Sue finds an area of 120 and Barry finds an area of 150. Now what is the
area of AABC? Hints: 301

4.35 Show that [WPX] = [WPZ] in the diagram at left below. Hints: 426, 108

A(0,0) B(2,0)

Figure 4.13: Diagram for Problem 4.35 Figure 4.14: Diagram for Problem 4.36

4.36 In the diagram at right above, triangle ABC has vertex A at the origin, vertex B at the point (2,0),
and vertex C on the circle with center (3,2) and radius 1. What is the maximum possible area for such a
triangle? (Source: Mandelbrot) Hints: 75,553

4.37x AD, BE, and CF meet at X as shown at right. Prove that A


[AXC] _ AF
[BXC] FB’ E r
Hints: 231, 98
C D B

96
CHALLENGE PROBLEMS

4.38% In the diagram at left below, given [PQRS] = 5[PQA] and [PQRS] = 4[PBS], find [ABP]/[PQRS].
Hints: 583, 470, 361

P Q
A A B

2 B . D E e

Figure 4.15: Diagram for Problem 4.38 Figure 4.16: Diagram for Problem 4.39

4.39 Inrectangle ABCD at right above, point E is on side CD such that [CBE] — [ADE] = [AEB] —- [CBE].
What is the ratio of the area of the largest region to the area of the smallest region? (Source: ARML)
Hints: 324, 166

4.40x For Joe’s birthday, Will has bought a 7 x 7 inch square birthday cake with a flat top. It is a
chocolate cake with banana frosting on the top and on the sides. It turns out that seven people will
be present when the cake is cut, and each person will become quite envious if another person receives
more cake or frosting. Find a way to divide the cake among seven people, so that each receives an equal
amount of cake and frosting. Hints: 149, 67, 124

Extra! Perhaps surprisingly, it is possible to dissect some rectangles into squares of different
mimi sizes. The figure below shows a dissection of a 32 x 33 rectangle into squares, whose side
lengths are indicated.

t dissect aoo
t posible to
Isit into ee ofdifferentsizes? :
: : : Continued onthenext page. .

a lhCCC CCC CCC CCCT™~—,”:~—C«Ci


CHAPTER 4. PERIMETER AND AREA

Extra! ... continued from the previous page


im ~The answer is yes! The figure below shows such a dissection of a 112 x 112 square.
(The numbers indicate side lengths of the squares.)

12

112

A problem from the 2000 American Invitational Mathematics Examination asks the
following: “The diagram shows a rectangle that has been dissected into nine non-
overlapping squares. Given that the width and the height of the rectangle are positive
integers with greatest common divisor 1, find the perimeter of the rectangle.”

Try to solve it!

DB
The Euler Line

Allll cases
rnepe
are
nre
uniqu
1M +]
very similar
pris ciWiilnar
to s. —T.am
vo

CHAPTER |

5.1 What is Similarity?


CHAPTER 5. SIMILAR TRIANGLES

version
We call two figures similar if one is simply a blown-up, and possibly rotated and/or flipped,
of the other. Our first problem gives us an example of similar figures.
Problem5.1:
(a) Use a ruler to approximate the following ratios in Figure 5.1:

| oe HG
eeGF EE
ee
HF EG
EL

(b) Measure angles ZA through ZH.


(c) Do you find anything interesting in your answers to the first two parts?
G

A D E F

Figure 5.1: Two Similar Figures

Solution for Problem 5.1: Measuring each of the segments in the given ratios, we find that in each case,
the ratio is 1/2. When we measure the angles, we find that the angles of ABCD are equal to those in
EHGF (note the orders of the vertices!):

LA LE 0S
/Bis=4, 4H Se150*
Le see OU
Zi) te) (7 OOS

We write the similarity in Figure 5.1 as ABCD ~ EHGF since ZA corresponds to ZE, 2B corresponds
to 4H, etc. As with congruence, we have to be careful about the order of the vertices. For example, we
would not write ABCD ~ EFGH to describe Figure 5.1.
The ratio between corresponding lengths in similar figures is constant, and is equal to the ratio by
which one figure is ‘blown up’ to get the other. In Figure 5.1, we have

relbige WHEY AGEN Wo),|


EHC, Greer ee
All corresponding lengths of ABCD and EHGF follow this ratio. For example, we could include BD/HF
and AC/EG in that chain of equalities above.
As we saw in Problem 5.1, corresponding angles in similar figures are equal.

ie
er OO
5.2. AA SIMILARITY

Similar figures do not need to have the same orientation. The diagram
to the right shows two similar triangles with different orientations.
Speaking of triangles, we'll be spending the rest of this chapter dis-
cussing how to tell when two triangles are similar, and how to use similar
triangles once we find them. Below are a couple Exercises that provide practice using triangle similarities
to write equations involving side lengths.

Exercises ie

5.1.1. Given that AABC ~ AYXZ, which of the statements below must be true?

(ay VARY X=AC/YZ:


(nje48/BC = VXIXZ,
(oye Abs xs, = Bie] YX:
(d) (AC)(YX) = (YZ)(BA).
(e) BC/BA = XY/ZY.
5.1.2 AABC ~ AADB, AC = 4, and AD = 9. What is AB? (Source: MATHCOUNTS) Hints: 113

5.2 AA Similarity

In our introduction, we stated that similar figures have all corresponding angles equal, and that corre-
sponding sides are in a constant ratio. It sounds like a lot of work to prove all of that; however, just as
for triangle congruence, we have some shortcuts to prove that triangles are similar. We'll start with the
most commonly used method.

We'll explore why AA Similarity works in Section 5.5, but first we'll get some experience using it in
some problems.

a
CHAPTER 5. SIMILAR TRIANGLES

[Problem 5.2: Below are two triangles that have the same measures for two angles.

x E F
60°
50°

60°
B
2 DU;

Find the third angle in each, and find the ratios AB/DF, AC/DE, BC/EF by measuring the sides with a
ruler.

Problem 5.3: In this problem we try to extend AA Similarity to figures with A D


more angles by considering figures with four angles. Can you create a figure
EFGH that has the same angles as ABCD at right such that EFGH and ABCD are
not similar? (In other words, can you create EFGH so that the angles of EFGH_ p ie
equal those of ABCD, but the ratio of corresponding sides between EFGH and
ABCD is not the same for all corresponding pairs of sides?)

Problem 5.4: In the figure at right, MN || OP, OP = 12, MO = 10, and LM = 5. P


Find MN.

N 1

Wx.

Extra! My dad was going to cut down a dead tree in our yard one day, but he was afraid it
‘>it might hit some nearby power lines. He knew that if the tree were over 45 feet tall, the
tree would hit the power lines. He stood 30 feet from the base of the tree and held a ruler
6 inches in front of his eye.
Continued on the next page. . .

oe
5.2. AA SIMILARITY

Problem 5.6: Find BC and DC given AD = 3, BD = 4, and AB = 5.

Problem 5.7: Given that DE || BC and AY || XC, prove that A


BY
Fx
AD >
pp’ D i

x
B &

Problem 5.2: Below are twotriangles that have the same measures for twoangles.
A E i:
60°

60°

Find the third angle in each, and find the ratios AB/DF, AC/DE, BC/EF by measuring the sides with
a ruler.

Solution for Problem 5.2: The last angle in each triangle is 180° — 50° — 60° = 70°, so the angles of AABC
match those of ADFE. In the same way, if we ever have two angles of one triangle equal to two angles
of another, we know that the third angles in the two triangles are equal.
Measuring, we find that the ratios are each 1.5. It appears to be the case that if all the angles of two
triangles are equal, then the two triangles are similar. ‘0
We might wonder if two figures with equal corresponding angles are always similar. So, we add an
angle and see if it works for figures with four angles.

Extra! ... continued from the previous page


i> >i Fe lined the bottom of the ruler up with the base of the tree, and saw that the top of the
tree lined up with a point 8 inches high on the ruler. He then knew he could safely cut
the tree down. How did he know?

103
CHAPTER 5. SIMILAR TRIANGLES

|[Problem 5. 3: Doe,your wae ak oe foun a more than 3 aie Can A D


you create a figure EFGH that has the same angles as ABCD at right such that fe
EFGH and ABCD are not similar? (In other words, can you create EFGH so that
the angles of EFGH equal those of ABCD, but the ratio of corresponding sides
between EFGH and ABCD is not the same?)

Solution for Problem 5.3: We can quickly find such an EFGH. 4 D E A


The diagram to the right shows a square EFGH next to our
initial rectangle. Clearly these figures have the same angles,
but when we check the ratios, we find that

ae NS ee : : f G
EF FG
ABCD and EFGH are not similar, so equal angles are not enough to prove similarity here. 0
Let’s return to triangles and tackle some problems using AA Similarity.

Problem 5.4: In the figure at right, MN ||OP, OP = 12, MO = 10, and LM = 5. P


Find MN.

Solution for Problem 5.4: See if you can find the flaw in this solution:

“Bogus Solution: Since MN || OP, we have ZLMN = ZLOP ne ZLNM = -LLPO.


Therefore, ALMN~ ALOP, so LM/MO = MN/OP. Substituting our
ot
given side lengths gives 5/10 =MN/12, so MN = 6.

Everything in this solution is correct except for LM/MO = MN/OP. MO is not a side of one of our
similar triangles! The correct equation is LM/LO = MN/OP. Since LO = LM + MO = 15, we now have
5/15 = MN/12, so MN = 4. 0

Problem 5.5: The lengths in the diagram are as marked, and WX || YZ. Find PY
and WX.
~s

Solution for Problem 5.5: Where does this solution go wrong:

104
ARR STSPREE STO SRS CadSenN ge Pea am ARERR |Nemes ree re eee
0.2. AA SIMILARITY

[ SOR Ua
| Bogus Solution: Since WX || ZY, we have ZW = ZZ and ZX = ZY. Therefore,
lt AWPX ~ AYPZ, and we have

PX _ WX _ WP
P27 PY
Substitution gives
a Hee
FO U2 RY
We can now easily find YP = 50/3 and WX = 18/5.

___ This solution doesn’t get the vertex order in the similar triangles right, so it sets up the ratios wrong!
PX and PZ are not corresponding sides. PX in AWPX corresponds to PY in AZPY because ZW = ZZ.
Here’s what the solution should look like. Pay close attention to the vertex order in the similarity
relationship.
Since WX ||ZY, we have ZW = ZZ and ZX = ZY. Therefore, AWPX ~ AZPY. Hence, we have

PX _ WX _WP
BYE Y¥ZRAGPZ
Substitution gives
gen oe
er er:
We can now easily find PY = 6 and WX = 6. O
Perhaps you see a common thread in the last two problems. While you won’t always find parallel
lines in similar triangle problems, you'll almost always find similar triangles when you have parallel
lines.

| Important: Parallel lines mean equal angles. Equal angles mean similar triangles. |
| Vv The figures below show two very common set-ups in which parallel lines —
lead to similar triangles. Specifically, APQR ~ APST and AJKL ~ AMNL. ||
| N |

WARNING! Read the Bogus Solutions to Problems 5.4 and 5.5 again. These are |
2% very common errors; understand them so you can avoid them. |

105
CHAPTER 5. SIMILAR TRIANGLES

lem
‘Prob5.6: Find BC and DC given AD = 3, BD = 4, and AB =5.

Solution for Problem 5.6: Since LBAD = ZCAB and ZBDA = ZCBA, we have ABAD ~ ACAB by AA
Similarity. Therefore, we have BC/BD = AB/AD = 5/3, so BG= (0/3)(BD)= 20/3:

We can use this same similarity to find AC, and then subtract AD to get CD. We could also note that
ZBCD = ZBCA and ZBDC = ZCBA, so ABCD ~ AACB by AA Similarity. Therefore, CD/BD = BC/AB =
(20/3)/5 = 4/3, so CD = (4/3)(BD) = 16/3. 0
Similar triangles — they’re not just for parallel lines.

Important: Similar triangles frequently pop up in problems with right angles. The |
diagram in Problem 5.6 shows a common way this occurs. Make sure ©
you see that |
AABD ~ ABCD ~ AACB. |
As you'll see throughout the rest of the book, similar triangles occur in all sorts of problems, not just
those with parallel lines and perpendicular lines. They’re also an important step in many proofs.

Problem 5.7: Given that DE ||BC and AY ||XC, prove that A

EY _ AD
Ex. DB D E
:

Solution for Problem 5.7: Parallel lines mean similar triangles. The ratios of side lengths in the problem
also suggest we look for similar triangles.
Since AY || XC, we have AAYE ~ ACXE. Now we look at what this means for our ratios. From
AAYE ~ ACXE, we have EY/EX = AE/EC. All we have left is to show that AE/EC = AD/DB.

Since DE || BC, we have AADE ~ AABC. Therefore, AD/AB = AE/AC, which is almost what we
want! We break AB and AC into AD + DB and AE + EC, hoping we can do a little algebra to finish:
ADS si Ab
AD + DB * AE ¥EC
If only we could get rid of the AD and AE in the denominators — then we would have AD /DB =AE/EC.
Fortunately, we can do it. We can flip both fractions:
A + DB GAL BEG
ALD "iL
ies
C—O
5.2. AA SIMILARITY

Therefore, Aden DB.= VAP,


AD + AD EC 801+ Gy
AE + ag, DB = 1+ EC which
Zp, ; gives us

DETEC
AD™ AE
Flipping these fractions back over gives us AD/DB = AE/EC. Therefore, we have EY/EX = AE/EC =
AD/DB, as desired. 0

Our solution to the previous problem reveals another handy relationship involving similar triangles:

Important: If BC ||DE and BD and CE meet at A as shown, then A |

AB _ AC B C
BIT OCe
1B) E

ye

iE

12

fh 21
Ll ae
16
B D

(a) Find AC and BC. (b) Find HJ.

(O}

7 1.6 M Ny

(c) Find ON and MN. (d) Find RS.

5.2.2 If two isosceles triangles have vertex angles that have the same measure, are the two triangles
similar? Why or why not?
CHAPTER 5. SIMILAR TRIANGLES

5.2.3 Inthe diagram, WXYZ is a square. M is the midpoint of YZ,and AB 1 MX. W x

(a) Show that WZ ||XY. Hints: 182


(b) Prove that AZ = YB. a

(c) Prove that XB = XA. F ¥ Y

(d) Prove that AAZM ~ AMYX, and use this fact to prove AZ = XY/4. B

5.2.4 In triangle ABC, AB = AC, BC = 1, and ZBAC = 36°. Let D be the point on side AC such that
LABD.=7EBD:

(a) Prove that triangles ABC and BCD are similar.


(b)x Find AB. Hints: 150
5.2.5x Find x in terms of y given the diagram below. Hints: 258, 522

5.3 SAS Similarity

z Problems

Problem 5.8:
(a) Measure BC, EE and angles 2B, ZC, ZE, and ZF.
(b) Can you make a guess about how to use Side-Angle-Side for triangle similarity?

| Extra! Descartes commanded the future from his eka more than Napoleon from the throne.
dedi dliad —Oliver Wendell Holmes
108
5.3. SAS SIMILARITY

Problem 5.9: In the figure below on the left, we have ae = ae = 3 and clearly ZBAD = ZCAE. We
wish to prove that AABD ~ AACE. (Note that we cannot assume that BD ||CE! We have to prove it.)

A rg)

c u

(a) Suppose we draw a line through B parallel to CE that hits AE at X as in the diagram on the right.
What do we know about AABX and AACE?
(b) Given that AE = 15 in both diagrams above, what is AX?
(c) What can we conclude about D and X?
(d) What can we conclude about AABD and AACE?
What similarity rule can we create from this investigation?

Problem 5.10: Given AC = 4, CD = 5, and AB = 6 as in the diagram, find


BC if the perimeter of ABCD is 20. (Source: Mandelbrot)

Problem 5.8: leh


(a) Measure BC, EF, and angles ZB, ZC, ZE, and ZF.
(b) Can you make a guess about how to use Side-Angle-Side for triangle similarity?
D »
>
A
iS 4.5
15 os 3
| E

109
CHAPTER 5. SIMILAR TRIANGLES S
a
a

Solution for Problem 5.8: We aren’t surprised to find that BC appears to be half EF: BC is about 2 cm and
EF is around 4 cm. We also aren’t shocked to find that 4B appears to equal ZE and /C appears to equal
ZF.
This example suggests that if two sides in one triangle are in the same ratio as two sides in another
triangle (as AB/AC = DE/DF), and the angles between these sides are equal (as ZA = ZD), then the
triangles are similar. 0
No doubt, you know where this is headed. Time to develop a proof for our guess. As usual, we try
to use what we already know, AA Similarity, to prove our guess for ‘SAS Similarity’.

[Problem 5.9: In ino figure Of the right, we have A

AB_
AC
AD
AB
_45: 8 12
and clearly ZBAD = ZCAE. Prove that AABD ~ AACE.

Solution for Problem 5.9: What did we do wrong here:

Bogus Solution: Since BD || CE, we have ZABD = ZACE and ZADB = ZAEC, so |
pia AABD ~ AACE by AA Similarity. =

There’s not a single false statement in that solution. However, the assertion that BD || CE needs to
be proved, and our Bogus Solution merely states it without justification.
In the solution below, we take the clever tactic of considering the point X on AE such that BX || CE.
Then we prove that X is in fact D.

We'd like to prove that BD ||CE, but there’s no obvious way to even A
start. We seem stuck, so we try to go a different direction. We create a
point X on AE as shown at right, such that BX ||CE. Our goal now is to 8
show that X must be D. Notice that we are not assuming that BD || CE.
We are taking some other point, X, such that BX || CE, then trying to
prove that X must be D. B xX
oy 1 FE 2
Since BX || CE, we have ZABX = ZACE and ZAXB = ZAEC, so C E
AABX ~ AACE by AA Similarity. Therefore,

AX _ AB_4
AE AG = 5"
so AX = (4/5)(AE) = 12. Hence, X is on AE 12 units fromA. But that’s where point D is! Therefore, D
must
be the same point as point X; i.e., D is the point on AE such that BD ||CE. Now that we’ve proved
BD ||CE, we can conclude that AABD ~ AACE. g

110
5.3. SAS SIMILARITY

We have established another way to prove two triangles are similar.

‘Important: Side-Angle-Side Similarity (SAS Similarity) tells us that if two sides _


Vv in one triangle are in the same ratio as two sides in another triangle (as |
AB/AC = DE/DF below), and the angles between these sides are equal —
(as ZA = ZD below), then the triangles are similar.

eS 4
A > :

C E
B
3 |

Note that we can also write that ratio equality as the ratio of correspond-
ing sides in the triangles: AB/DE = AC/DF.

You may be wondering how our solution to Problem 5.9 can be used to prove SAS Similarity in
general, since Problem 5.9 only deals with the case of two triangles that share an angle, as AABD and
AACE share ZA. We can use this approach generally because if an angle in one triangle equals an angle
in another, we can always slide (and/or flip) one triangle until it’s on top of the other, as shown in
Figure 5.2.
A D 1BASy

a
| ‘ B
B
E
aa Saas
Figure 5.2: Sliding Triangles to Prove Similarity

SAS Similarity is most often used in diagrams like the one shown in Problem 5.9. However, it does
come up in less obvious situations.

Problem 5.10: Given AC = 4, CD = 5, and AB = 6 as in the diagram, find


BC if the perimeter of ABCD is 20. (Source: Mandelbrot)
a

Solution for Problem 5.10: Since


ACM 4e2 ABO me2
mnie
6eS we MAD ig993"
we have AACB ~ AABD by SAS (since the angle between the sides in each ratio above is ZA). Since the
sides of AABD are 3/2 the corresponding sides of AACB, we have BD = 3BC/2. Now we can use that
er
bt
CHAPTER 5. SIMILAR TRIANGLES

perimeter information. Since BC + CD + DB = 20, we have

BC +5+ == =20

Therefore, BC = 6. 0

B 4.5 E E
D C M
3
E G H

Figure 5.3: Diagram for Problem 5.3.1 Figure 5.4: Diagram for Problem 5.3.2

5.3.2 In the figure at right above, M is the midpoint of EH and of FG. E and F are midpoints of IM and
MJ, respectively. Prove that IJ || GH.
5.3.3 Show that if WZ* = (WX)(WY) in the diagram at left below, then ZWZX = ZWYZ. Hints: 147

EB

W Q R
xX
B
Zz Y A

Figure 5.5: Diagram for Problem 5.3.3 Figure 5.6: Diagram for Problem 5.3.4

5.3.4 In the diagram at right above, ZPRQ = ZPQA = 90°, QR = QA, and ZQPC = ZRPC.

(a) Prove ZQCB = ZQBC. Hints: 202


(b)x Prove
RA ||PB. Hints: 388

Extra! [ must study politics and war that my children may have liberty to study mather
oe philosophy. My children ought to study mathematics and philosophy, geograph 1 sal
1 history,
oe naval architecture, navigation, commerce, and agriculture,in order to give their children aAe
to leo painting, poetry, music, matey, statuary, ee andge |

4
5.4. SSS SIMILARITY

5.4 SSS Similarity

We use SSS Similarity less often than AA and SAS.

|Important: Side-Side-Side Similarity (SSS Similarity) tells us that if each side of


Vv one triangle is the same constant multiple of the corresponding side of
another triangle, then the triangles are similar. (And therefore, their —
corresponding angles are equal.)

For example, in the diagram, we have

AB _ AC _ BC |
| DE DE EP |
| so
| AABC ~ ADEF. |
|
Therefore, ZA = 2D; 7B = 26 and ZC = /F.

|Problems
_ i
Problem 5.11: Given the side lengths shown in the diagram, prove that
AE || BC and
AB ||DE.

As we noted, few problems require SSS Similarity. We may, however, consider it in problems in
which all we are given is lengths, but we have to prove something about angles.

Extra! Mathematics is the art ofgiving the same name to different things.
: a a as
| bao —Henri Poincaré
a ——~« a
CHAPTER 5. SIMILAR TRIANGLES

a -- ae ~

‘Problem 5.11: Given the side lengths shown in the diagram, prove that AE ||BC
and AB ||DE.

Solution for Problem 5.11: We need to use angles to show the segments are parallel, but all we have are
sides. We look for similarity, and see that
ABD VACHE BCU
DEW AD AE wa?
so AABC ~ ADEA by SSS Similarity. Therefore, BAC = ZEDA, so AB || DE. Also, ZDAE = ZACB, so
AE ||BC. o

_txercises me
5.4.1 Two isosceles triangles have the same ratio of leg length to base length. Prove that the vertex
angles of the two triangles are equal. Hints: 314

5.5 Using Similarity in Problems

In this section we explore some challenging problems that are solved with similar triangles, and we
discover why AA Similarity works.

Problems

Problem 5.12: In the diagram, DE || BC, and the segments have


the lengths shown in the diagram. Find x, y, and z.

Problem 5.13: As shown in the diagram, ZA = 90° and ADEF is a square.


Given that AB = 6 and AC = 10, find AD.
5.5. USING SIMILARITY IN PROBLEMS

Problem 5.14: In the diagram, PX is the altitude from right angle ZQOPR of rE
right triangle POR as shown. Show that PX* = (QX)(RX), PR* = (RX)(RQ), and
PQ?= (QX)(QR).
R

Problem 5.15: AABC ~ AXYZ, AB/XY= 4, and [ABC]= 64. In this problem we will find [XYZ].
(a) Let hc be the altitude of AABC to AB, and let hz be the altitude of AXYZ to XY. What is hc /hz?
(b) Find [XYZ].
Extra challenge: What general statement about the areas of similar triangles can you make?

Problem 5.16: In the diagram, ZACQ= ZQCB, AQ 1 CQ, and P is the midpoint A
of AB. Prove that PQ ||BC. Hints: 35, 417 a
PLE

Problem 5.17: Flagpole CD is 12 feet tall. Flagpole AB is 9 feet tall. Both


flagpoles are perpendicular to the ground. A straight wire is attached A
from B to D, and another from A to C. The flagpoles are 40 feet apart, and E
the wires cross at E, which is directly above point F on the ground. We 9
wish to find EF.

(a) Use similar triangles to find ratios of segments that equal EF/AB.
(b) Use similar triangles to find ratios of segments that equal EF/DC.
(c) Cleverly choose one ratio from each of the first two parts and add them to get an equation you
can solve for EF.

Problem 5.18: In this problem we will explore why AA Similarity works. Do not use AA Similarity to
solve the problem!
In the diagram below, we have two triangles (AABE and AACD) with equal angles, and sides with
lengths as marked. Our goal in this problem is to find BE and DE, and discover a process to prove that
if the angles of one triangle equal those of another, then the corresponding sides of the two triangles
are in constant proportion. We will make heavy use of the Same Base/Same Altitude principle we
discovered in Section 4.3, so you might want to review that section if you get stuck.

(a) What are [ABE]/[ACE] and [BEC]/[BED]?_


| (b) Use theprevious part to show that [ACE] = [ABD].
(©) Whatis[ABE]/[ABD]?
: (d) Use the previous part to find AD.
L(e). What is BE? C 14 D
:0) Can we use our work in this problem to prove that if two angles of one triangle equal those of
another triangle, then the triangles are similar?

115
CHAPTER 5. SIMILAR TRIANGLES

We start off with some warm-ups involving parallel and perpendicular lines.

‘Problem 5.12: in the diagram, DE ||BC, and the segments have A


the lengths shown in the diagram. Find x, y, and z.

Solution for Problem 5.12: Since ED ||BC, we have AFBC ~ AFED by AA Similarity. Therefore, we have
FC BC FB 64 16
ED DERE O6uos
Solving for x and y, we find x = BC = (16/9)(DE) = 80 and y = FC = (16/9)(DF) = 48.

Since AADE ~ AABC by AA Similarity, we have AE/AC = DE/BC = 45/80 = 9/16. Since AE = z and
AC = AE+ EC =z +60, we have z/(z + 60) = 9/16. Cross-multiplying gives 16z = 9z + 540, so z = 540/7.
i=
~
f

Problem 5.13: As shown in the diagram, ZA = 90°, and ADEF isasquare. B


Given that AB = 6 and AC = 10, find AD. E

Solution for Problem 5.13: Since ZA = ZEFC = 90°, we have EF ||AB; similarly, DE || AC. Therefore, this
problem has both right triangles and parallel lines. Our parallel lines quickly tell us that by AA, we
have
ABDE ~ ABAC ~ AEFC.
If we let each side of ADEF be x, we have BD = 6 — x and FC = 10 — x. Our similar triangles can then be
used to solve for x. From ABDE ~ AEFC, we have BD/DE = EF/FC. Substitution gives
6= 2) ae
pe NSS
Cross-multiplying and solving the resulting equation for x gives x = 15/4. Therefore, AD = x = 15/4. O

Problem 5.14: In the diagram, PX is the altitude from right angle ZQPR of P
right triangle PQR as shown. Show that PX? = (QX)(RX), PR? = (RX)(RQ), and
PQ? = (QX)(QR).

Solution for Problem 5.14: Right triangles mean similar triangles. ZPXR = ZQPR and ZPRX = ZPRQ,

116
5.5. USING SIMILARITY INPROBLEMS

so APXR ~ AQPR. Therefore, we have PR/RX = RQ/PR, so PR? = (RX)(RQ). Similarly, we can show
APQX ~ ARQP, so PQ/QX = QR/PQ, and we have PQ? = (QX)(QR).

Combining the two triangle similarities (or by noting that ZXPQ = 90° — ZXPR = ZXRP and /PXQ =
ZPXR), we find APXQ ~ ARXP. Therefore, PX/QX = RX/PX,so PX* = (RX)(QX). O

The square root of the product of two numbers is called the geometric mean of the two numbers.
The previous problem suggests where the name ‘geometric mean’ comes from. For example, what is
the geometric mean of QX and RX?

Problem 5.15: Given that AABC ~ AXYZ, AB/XY = 4, and [ABC] = 64, find [XYZ].
Solution for Problem 5.15: Since AABC ~ AXYZ and AB/XY = 4, the ratio of corresponding lengths in the
triangles is 4/1. Therefore, the altitude of AABC to AB is 4 times the corresponding altitude to XY in
AXYZ.
For a quick proof, consider the diagram to the right, C
in\which we’ve drawn the aforementioned altitudes to
AB and XY. Since AABC ~ AXYZ, we have ZA = LX.
Combining this angle equality with ZCPA = ZZVX Z
gives AAPC ~ AXVZ by AA, so CP/ZV = AC/XZ. TAS
Since AC and XZ are corresponding sides of our orig- eV UY.
inal triangles, their ratio is 4/1, so CP/ZV = 4/1.

Finally, we can find the ratio [ABC]/[XYZ]. Since A P :


both the base and the altitude of AABC are 4 times the corresponding base and altitude of AXYZ, we

as = (ABNER)
)(CP)/ (29° A B Cr ee 4\2 aes
BCU DOS = (XY)(ZV)/2 — ew) inp - @ aah
So, we have [XYZ] = [ABC]/16 = 4. o

The same procedure we used to solve this problem can be used to find an important relationship
between the areas of two similar triangles.

Important: If two triangles are similar such that the sides of the larger triangle are
k times the sides of the smaller, then the area of the larger triangle is k*
times that of the smaller.
This relationship holds for any pair of similar figures, not just for
triangles. x

Problem 5.16: In the diagram, ACQ = QCB, AQUCOyand P is-the A


midpoint of AB. Prove that PQ || BC. Ah
iL 7i

ee
Solution for Problem 5.16: If we could show that ZQPA = ZB, then we could use that to prove PQ || BC.

i,We
CHAPTER 5. SIMILAR TRIANGLES

Unfortunately, there are no obvious similar triangles or congruent triangles we can use to show that
LOPAI==LB.

A We extend segment AQ to point Z on BC because we’d like to create triangles


that might be similar (namely, AAPQ and AABZ). We’d also like to use the angle
equalities at C, which we can now do by noting that ZAQC = ZCQZ, CQ = CQ,
and ZACQ = ZQCZ, so ACQZ = ACQA by ASA. Therefore, we know that
Bees BS AQ = QZ, so AQ = AZ/2.
We might seem stuck here, but then we remember the last bit of information we haven’t used. Since
P is the midpoint of AB, we have AP = AB/2, so APAQ ~ ABAZ by SAS Similarity. Thus, ZAPQ = ZB,
so PQ ||BC. O

When you're stuck on a problem, ask yourself, ‘What piece of information —


|
have I not used?’

In many problems, there’s more than meets the eye. Extending segmentsSs
that seem to end abruptly (particularly in the middle of a triangle) some-~
times gives useful information.

Problem 5.17: Flagpole CD is 12 feet tall. Flagpole AB is 9 feet tall.


Both flagpoles are perpendicular to the ground. A straight wire is 4
attached from B to D, and another from A to C. The flagpoles are 40
feet apart, and the wires cross at E, which is directly above point F on E 12
the ground. Find EF. ?

Solution for Problem 5.17: We start off by noticing that AB ||EF || CD since all three are perpendicular to
BC. By now you know the drill: parallel lines mean similar triangles. We look first for similar triangles
that include EF, and we see ACEF ~ACAB and AEBF ~ ADBC. Therefore, we have
Bip. nC d EE GiB
Ea BE
AB SIGR
intA coe en GE CH wenial
We see CB in both groups, so we investigate the ratios involving CB more closely. We see that we have
CF + BF = CB, so
EF FFL ee ie Ce Bee Ce
ABs) CB GB CRigg eC hal
Now we can find EF:

Notice that the length of BC is irrelevant! O

a
5.5. USING SIMILARITY IN PROBLEMS

_WARNING!! In the last solution we didn’t spell out exactly why AEBF ~ ADBC, |
‘“S since we've gone through those steps several times already. When |
you are writing solutions for your class or for a contest, you should |
include the steps we left out here (cite which angles are equal and —
why, then invoke AA). Only start leaving out the simple steps if you |
are certain that it is O.K. to do so. |

We finish this section by exploring why AA Similarity works.

‘Problem 5.18: In the diagram we have two triangles (AABE and


-AACD) with equal angles, and sides with lengths as marked. Find
BE and DE without using AA Similarity. Can you use your method
to prove why AA Similarity works?

Solution for Problem 5.18: We would like to show that AABE ~ AACD, so
we start thinking about side length ratios. The only ratio tool we have
that doesn’t depend on having similar triangles already is our Same
Base/Same Altitude technique of Section 4.3, so we try that. We don’t
have any triangles to use our technique on, so we draw BD and CE as
shown to the right.
AABE and AAEC share an altitude from E, so

LACE SAP
eee,EEG. =2 5A
[AGElmn AG ag) 3 od)
Similarly, AABE and AABD share an altitude from B, so

[ABET S2AE ore) 8 (5.2)


[ABD] AD 8+DE’
We suspect that AB/AC = AE/AD because we suspect AABE ~ AACD. From (5.1) and (5.2), we have

AE _(ABE]
AD [ABD]
4, } AB _ [ABE]
ACM ALL|

Since the numerators in our area ratios are the same, we need only show that [ABD] = [ACE]. These
two areas share [ABE], so we need only show that [BEC] = [BED].

Since ZABE = ZACD, we know BE || CD. Therefore, the altitudes from C and D to BE must be the
same. Hence, triangles BEC and BED have the same base (BE) and the same length altitudes to that base,
so [BEC}"= [BED].

Finally, we can find DE. We have:

[AEC] = [AEB] + [BEC] = [AEB] + [BED] = [ABD],

Th)
CHAPTER 5. SIMILAR TRIANGLES

so we can use our area ratios above. Since [ABE]/[ACE] = [ABE]/[ABD], we have AB /AC = AE/AD, so

2/3 = 8/(8 + DE).

Solving this equation for DE, we find that DE = 4.


Now we very strongly suspect that BE/CD = AE/AD. To prove it, we use the same process we just
followed.

We consider AA’B’D where A’ is on AD and B’ is on CD such that A’D = AE and BE = B’D. Since
BE ||CD, we have ZAEB = ZADC = ZA’DB’. Therefore, AA’B’D = AABE by SAS Congruence. (You can
also think of AA’DB’ as the result of sliding AABE along AD until side BE is on CD.) Since A’D = AE,
we have AA’ = ED = 4. We also have ZDA’B’ = ZEAB = ZDAC, so AC ||A’B’.
We can chase areas around as before to show that B’D/CD = A’D/AD, so B’D = (2/3)(14) = 28/3.
Since B’D = BE, we have BE = 28/3. Note that because B’D = BE and A’D = AE, we have shown that
BE/CD = AE/AD, as suspected. 0

x x Whenever we have a two triangles that have two angle


Q measures in common, we can slide (and possibly flip) one
R a R’ triangle onto the other so that we get a diagram like that
[ea in Problem 5.18. For example, APQR and AYZX in the
P diagram to the left have two angle measures in common
y y YP Z=0' (and consequently the third angles are equal, too). We can
r therefore move APQR on top of AYZX such that two of the
sides of the ‘moved’ triangle coincide with sides of AYZX, as AP’Q’R’ in the diagram shows.
We can use the exact same approach as we used in Problem 5.18 to show that if two angles of one
triangle equal the corresponding angles of the other, then each pair of corresponding lengths in the two
triangles has the same ratio.

:“Area ca
can. be a very useful problem solving tool even in problems that
app ear i have nothing |to do withsarea. ;

5.5.1 X and Y are on sides PQ and PR, respectively, of APQR such that XY || QR. Given XY = 5,
OR 15 yandsy k= '8; tind PY:

| a aaa ETT
5.6. CONSTRUCTION: ANGLES AND PARALLELS

5.5.2 In the figure, the area of AEDC is 25 times the area of ABFD. A
E
(a) Find CD/DB. Hints: 350
(b) Find [EDC]/[ABC]. Hints: 171
(c)x Find [AFE]/[ABC]. Hints: 321 Ee
B dD é

5.5.3 In the diagram, WZ || XY and WX || ZY. WA and WB hit XZ at C and D, W X


respectively, such that ZC = XD. sy
B
(a) Prove that ZC/XC = AC/WC. F
(b) Prove that XD/ZD = DB/WD. Ls
(c) Prove
CD ||AB. Hints: 462

5.5.4 In the diagram at left below, PQ = PR, ZX || QY, QY + PR, and PQ is extended to W such that
WZ 1 PW.
(a) Show that AQWZ ~ ARXZ. Hints: 360
(b)x Show that YQ = ZX — ZW. Hints: 172, 550

P Q
Figure 5.7: Diagram for Problem 5.5.4 Figure 5.8: Diagram for Problem 5.5.5

5.5.5x PA and BQ bisect angles ZRPQ and /RQP, respectively. Given that RX 1 PA and RY 1 BQ,
prove that XY ||PQ. Hints: 584, 152, 254

5.6 Construction: Angles and Parallels


In this section, we take a look at how to use our knowledge of similar triangles for geometric construc-
tions. But first, since parallel lines and similar triangles are so closely related, we'll have to learn how to
construct a parallel line.
Don’t forget the construction rules! Straightedge and compass only. No protractor. No measuring
with your ruler.

Extra! The shortest distance between two points is under construction.


OO So —-Noelie Altito
121
CHAPTER 5. SIMILAR TRIANGLES

Problem 5.19: Shown below are angle 2X and line m with point Y on it. Construct a line through Y
that makes an angle with m that is equal to 2X.

m
pS Ee AS
xX ¥

(a) Draw a circle with center X and another circle with the same radius and center Y.
(b) Use your circles from the first part to construct a point Z on the circle centered at Y such that YZ
makes an angle with m equal to 2X.

Problem 5.20: Given a line n and a point A not on n, construct a line through A that is parallel to n. |

Problem 5.21: Draw a segment AB. In this problem we learn how to divide AB into 3 equal pieces.
(a) Draw a line through A but not through B. Construct a segment on this line that has A as one
endpoint. Call the other endpoint P.
(b) Construct a segment that has A as one endpoint, has P on the segment, and that is 3 times as
long as AP. Call the other endpoint of this segment R.
Draw RB. Construct point X on AB such that AX/AB = 1/3.

Equal angles are very important in our study of similar triangles and parallel lines, so we'll start our
constructions by learning how to copy an angle.

Problem 5.19: Given ZX and line m with point Y on it, construct a line through Y that makes an angle
with m that is equal to 2X.

Solution for Problem 5.19: Since we measure angles by the portion of a circle that the angle cuts off, we
start by making a circle with center X. Let the points where this circle hits the sides of ZX be A and B.
We then make a circle with center Y with the same radius, since we want to cut off the same amount of
this circle that 2X cuts off its circle. Let P be one of the points where this circle hits m.
We can’t use a ruler to measure AB to tell how much of circle X that ZX cuts off, but we can use our
compass! We open our compass to a width equal to AB, then draw an arc with center P and radius AB.
Call the point where this arc meets circle Y point Q. Drawing YQ gives us angle ZQYP equal to ZX.

122
5.6. CONSTRUCTION: ANGLES AND PARALLELS

Are you convinced that this construction works? I’m not. We’ll have to prove that ZY = ZX to see
why the construction works. Since OX and OY have the same radius, we have XB = YP and XA = VO)
Since we opened the compass to width AB to make our arc centered at P, we have AB = PQ. Therefore,
we have ABXA = APYQ by SSS, so we have ZX = ZY! Now, I’m convinced. 0

As we've seen, we can use equal angles to prove that two lines are parallel. Let’s see if we can use
equal angles to construct two parallel lines.

Problem 5.20: Given line 1 and point A not on n, construct a line through A that is parallel to n. j

Solution for Problem 5.20: We know how to copy an angle, and equal P
angles give parallel lines, so we try copying an angle along line n to A
point A. But first, we’ll need a line through A and an angle along n, so ie Q
we draw a line through A that hits n at point X. We then copy angle X V
to point A just as we copied an angle in Problem 5.19. We draw circles
with the same radii with centers A and X. We then set our compass to rata | i
width UV and draw an arc centered at P with that radius. This arc hits
OA at point Q such that ZPAQ = ZVXU. Therefore, AQ l|n. O

We haven’t used parallel lines just for problems involving angles — we’ve also used them in problems
involving similar triangles and ratios of lengths.

Problem 5.21: Given segment AB, construct points X and Y on AB such that AX = XY = YB = AB/3.

Solution for Problem 5.21: We know how to divide a segment in half, but cutting it by a ratio like 1/3 calls
for more advanced tools than simple midpoints. The best geometry tool we have for ratios is similar
triangles, but it’s not yet clear how we can use similar triangles.
Unsure how to deal with trisecting (dividing into three pieces) AB, we try a simpler problem. Can
we create a segment with A as an endpoint such that the segment is cut into three equal pieces? This
isn’t so hard — we start with a line through A, pick a different point P on the line, then copy AP twice
along the line to get Q and R such that AP = PQ = QR = AR/3.

So, we have a trisected segment with A as an endpoint, but unfortunately, it isn’t AB that we trisected.
However, we do have that 1/3 ratio, so perhaps we can now use similar triangles. Similar triangles call
for parallel lines. We draw RB, then construct lines through P and Q that are parallel to RB. These lines
hitAB at X and Y.
R

A x 4 B

We can use similar triangles to prove that X and Y are the points that divide AB into three equal
pieces. Since PX || QY || RB, we have APAX ~ AQAY ~ ARAB. Therefore AX/AB = AP/AR = 1/3 and
AY/AB = AQ/AR = 2/3. Since AX = AB/3 and AY = 2AB/3, we have AX = XY = YB = AB/3, as desired.
O

123
CHAPTER 5. SIMILAR TRIANGLES

Notice that our first key step in finding this solution was thinking about an easier related problem.

Concept: When stuck ona problem, try solving an easier related problem. One way
©=== todo this with a construction problem is relaxing one of the constraints of
the problem.

For example, in Problem 5.21, we relaxed the constraint that the trisected segment has both A and B
as endpoints. Instead, we just created a trisected segment with A as an endpoint.
Although this is the end of the similar triangles chapter, this won’t be the end of your study of
similar triangles. You'll see them pop up in many more problems, and you'll find another whole chapter
devoted to an application of similar triangles with the lofty name Power of a Point.

Exercises i

5.6.1 Given a segment of length 1, construct a segment with length 1/5. Construct a segment with
length 22.
5.6.2 Given a triangle AABC, construct a triangle AXYZ that is similar to AABC, but has 9 times the
area of AABC.

5.7 Summary

Definition: Two figures are similar if one is simply a blown-up, and possibly rotated and/or
flipped, version of the other.
a . |Important: Corresponding angles in similar figures are equal, and the ratio of the
Vv lengths of corresponding sides of similar triangles is always the same.

H
‘@

A
aN D in F

In similar quadrilaterals ABCD and EHGF, we have ZA = ZE, ZB = Lhe bs


ZC = ZG, and ZD = ZF. We also have

AB _ BC _CD_DA_ AC_BD
EH °°HG*?SGFY PPE PEG? PA
We denote these figures as similar by writing ABCD ~ EHGEF.

124
0.7. SUMMARY

There are three main ways to show that two triangles are similar:

e AA Similarity. If two angles of one triangle equal two angles of another, then the triangles
are similar. This is by far the most commonly used method to prove two triangles are similar.
(Section 5.2)

e SAS Similarity. If two sides in one triangle are in the same ratio as two sides in another triangle,
and the angles between the sides in each triangle equal each other, then the triangles are similar.
(Section 5.3)

e SSS Similarity. If each side of one triangle is the same constant multiple of the corresponding
side of another triangle, then the triangles are similar. (Section 5.4)

Parallel lines and perpendicular lines are clues to look for similar triangles. Three very common
set-ups that contain similar triangles are shown below.
N
A P
J

Q
D
R ae M
B G S ‘i K

AABC ~ ABDC ~ AADB APQR ~ APST AJLK ~ AMLN

Important: If BC ||DE and BD and CE meet at A as shown, then A

AB _ AC
Bid? CE
a
D is

‘Important: If two triangles are similar such that the sides of the larger triangle are ~
| k times the sides of the smaller, then the area of the larger triangle is ke |
| times that of the smaller. |
This relationship holds for any pair of similar figures, not just for
triangles. : |

Problem Solving Strategies

‘© When you're stuck on aproblem,


askyourself, ‘Whatpiece
ofinfor-
mation have I not used?’ ||
|
|
|
|

Continued on the next page. . . |

125
CHAPTER 5. SIMILAR TRIANGLES

Concepts: ... continued from the previous page

| : e In many problems, there’s more than meets the eye. Extending


segments that seem to end abruptly (particularly in the middle of a
triangle) can often yield quick solutions.
e When stuck on a problem, try solving an easier related problem. For
constructions, useful easier related problems often involve relaxing
one of the constraints of the problem.

e Consider using similar triangles in problems involving ratios of seg-


ment lengths.

Things To Watch Out For!

WARNING! Below are shown two common situations that lead to mistakes. The |
= diagram on the left may lead you to write ‘AABC ~ AADE,so AB/BD =
BC/DE.’ The one on the right might lead to ‘AJKL ~ ANLM, so
JL/NL = KL/ML.’ Both of these are incorrect! Make sure you see |

| - 1
{

|
N

C
| i= |
ee G K #a

REVIEW PROBLEMS >

5.22 In each of the parts below, either identify all pairs of similar triangles or state that there are not
any pairs of triangles that are necessarily similar. For each pair of similar triangles you find, state why
the triangles are similar.

126
REVIEW PROBLEMS

(f) "

hv
pe a E
F 6x3

5.23 Find x and y in the diagram at right, given the angle equalities C x B
and side lengths shown in APQR and AABC. Q y
5.24 Points Pand QareonABand AC, respectively, such that PQ ||BC. 3
Given AB = 12, PB = 9, and AC = 18, find QA. 6 8 A
5.25 The side lengths of a triangle are 4 centimeters, 6 centimeters, fi :
and 9 centimeters. One of the side lengths of a similar triangle is 36
centimeters. What is the maximum number of centimeters possible in the perimeter of the second
triangle? (Source: MATHCOUNTS)
5.26 What’s wrong with the diagram shown at left below?

Figure 5.9: Diagram for Problem 5.26 Figure 5.10: Diagram for Problem 5.27

5.27. Find DE in the diagram at right above.


5.28 Why is the diagram shown at left below impossible?

P
8 a

Rew2
iaag T
10
3 V

Figure 5.11: Diagram for Problem 5.28 Figure 5.12: Diagram for Problem 5.29

5.29 In the diagram at right above, find WY and YV.

127
CHAPTER 5. SIMILAR TRIANGLES

5.30 AABC at right is equilateral. M is on AB and N on AC such that BM = CN. A

(a) Prove that AM = AN. N aA

(b) Prove that AAMN is equilateral. C B

5.31 Given AABC ~ AYZX, [ABC] = 40, [YZX] = 360, AB = 9, and BC = 12, find the following:

(a) SY Ze
(b) The length of the altitude to side XZ of triangle AYZX.
5.32 Let ABCD be a rectangle as shown at left below, with AB = 25 and BC = 12. Let E be a point on
AB, such that AE < BE and triangles AED and BCE are similar. Find AE.
P

ae xe
A E B Ww

D C Q R

Figure 5.13: Diagram for Problem 5.32 Figure 5.14: Diagram for Problem 5.33

5.33 In the diagram at right above, PW = 6 and WX = 4. Find QX.


5.34 (Try this without looking back in the text first!) In the diagram at left below, AP ||BQ ||CR. Prove that

ee yet eee
CR AP BQ

Q <

A 6 B 20

Figure 5.15: Diagram for Problem 5.34 Figure 5.16: Diagram for Problem 5.35

5.35 Two of the sides in the right triangle at right above have length 12 cm and 20 cm, as shown. What
is the number of centimeters in the length of the altitude h drawn to the side with length 20 cm? (Source:
MATHCOUNTS) (If you know the Pythagorean Theorem, try doing this problem without it!)
5.36 Let ABC be a triangle, and let D and E be points on sides AB and AC, respectively, such that
DE || BC. Prove that
NRE SMO}s:
eae Cid
(Try to do this one without looking back in the text for the proof!)

eo
°C ee ee
CHALLENGE PROBLEMS

i]L_Challenge Problems _[E


5.37 Let ABCbea triangle, and let D and E be points on AB and AC, respectively, such that AD/AE =
BD/EC. Prove that DE || BC. Make sure you see why this differs from the previous problem! Hints: 363, 179
5.38 If the sum of one of the base angles and the vertex angle is the same for two different isosceles
triangles, must the triangles be similar? Hints: 196
5.39 In the figure at left below, isosceles AABC with base AB has altitude CH = 24 cm. DE = GF,
HF = 12 cm, and FB = 6 cm. Find the area of CDEFG. (Source: MATHCOUNTS) Hints: 490

C A

D
E
D G
B
S E H c ‘i

Figure 5.17: Diagram for Problem 5.39 Figure 5.18: Diagram for Problem 5.40

5.40 In triangle ABC at right above, D and E are points on sides AC and AB, respectively, such that
(AD)(AC) = (AE)(AB). Prove that ZCDE + ZCBE = 180° and ZADB + /BEC = 180°. Hints: 269, 70
5.41 D,E,and F are on sides BC, AC, and AB, respectively, of AABC such that DE ||AB, DF ||AC, and
BC ||EF. Prove that D, E, and F are the midpoints of the sides of AABC. Hints: 24
5.42 In the diagram at left below, PS || QT and PQ || ST. Prove that SU/SP = QP/QR.

OAR

P 4h
S

u ae
Figure 5.19: Diagram for Problem 5.42 . Figure 5.20: Diagram for Problem 5.43

5.43 The area of triangle XYZ at right above is 8 square inches. Points A and B are midpoints of
congruent segments XY and XZ. Altitude XC bisects YZ. What is the area of the shaded region?
(Source: AMC 8) Hints: 77, 162
5.44 The midpoints of the three sides of an equilateral triangle are connected to form a second triangle.
A third triangle is formed by connecting the midpoints of the second triangle. This process is repeated
until a tenth triangle is formed. What is the ratio of the perimeter of the tenth triangle to that perimeter
of the third triangle? (Source: MATHCOUNTS) Hints: 72

129
CHAPTER 5. SIMILAR TRIANGLES

5.45 Inrectangle ABCD at left below, AB = 5 and BC= 3. Points F and G are on CD so that DF = 1 and
GG= 24 kines AF and BC intersect at E. Find the area of AAEB. (Source: AMC 10) Hints: 99

Ie
B
D1 Zn Ch
Cs
3

A 5 B G Cry A

Figure 5.21: Diagram for Problem 5.45 Figure 5.22: Diagram for Problem 5.46

5.46 In triangle ABC at right above, AC = 4, BC = 3, AB = 5, and ZACB= 90°. The infinite sequence
of points Cy, Co,,C3,/ Gi, 22, Is generated as
as follows: C; is the foot of the altitude from C to side AB, C2 is
the foot of the altitude from C, to side AC, C3 is the foot of the altitude from C2 to side AB, and so on.
Calculate the sum CC; + CyC2 + CoC3 + C3C4 +---. Hints: 138, 307

5.47x In the figure shown, we have taken an equilateral triangle and divided each side into four
segments of equal length. We have then connected these points to form smaller equilateral triangles.

Consider instead dividing each side into n segments of equal length, where n is some positive integer,
then connecting these points as before to form smaller equilateral triangles. Use this dissection of the
original equilateral triangle to prove that the sum of the first n positive odd integers is n*. Hints: 367,475
5.48x Figure ABCD at left below has sides AB = 6, CD = 8, BC = DA = 2, and AB ||CD. Segments are
drawn from the midpoint P ofAB to pointsQ and R on side CD so that PQ and PR are parallel to AD
and BC, as shown. Diagonal DB intersects PQ at X and PR at Y. Evaluate PX/YR. (Source: Mandelbrot)
Hints: 58, 176
A £ B E

D hana G POLE :
Figure 5.23: Diagram for Problem 5.48 Figure 5.24: Diagram for Problem 5.49

5.49x In rectangle ABCD at right above, points Fand G lie on AB so that AF = FG = GB and E is the
midpoint of DC. Also, AC intersects EF at H and EG at J. The area of rectangle ABCD is 70. Find the
area of AAHF. (Source: AMC 12) Hints: 257, 319, 393
5.50x Given an acute triangle AABC, construct with straightedge and compass square DEFG such that
D and E are on BC, Gis on AB, and F is on AC. Hints: 201, 132

foo
ae °° ea eee
CHALLENGE PROBLEMS

55lxe Fomish, ho, 1, and U are on the sides of triangle ABC, as shown,
such that line segments UR, OT, and SP all pass through point X, and are
parallel to BC, CA, and AB, respectively. Prove that

PQ RS TU
Tea ror Wu ae
Hints: 353, 223, 261

Extra! A polyomino, a term coined by USC mathematics professor Solomon W. Golomb in


‘ur wet 1953, is simply a piece consisting of a number of connected squares. Evidently, it is a
generalization of a domino, a piece consisting of two connected squares.
The number of squares is indicated by the prefix, so a monomino consists of one
square, a domino has two, a triomino has three, a tetromino has four, a pentomino has
five, and so on. The five tetrominoes are shown below. (You might recognize them from
the video game Tetris!)

ce
The first obvious question to ask is, “How many polyominoes are there that contain
a given number of squares?” The answer is partially answered by the following table.

Number of squares Number of polyominoes

et)og

SOMONDURWONHY

__ At the time of writing, the number of polyominoes is known up to56 squares,butno


general formula is known. :
The next obvious question is to ask is, “What interesting Mice can thesesicrine
mak e?” For example, can the five tetrominoes be used to cover a4x5 chessboard? Try to.
use the tetrominoes to cover such a chessboard, or prove it is npr to> dosbe before
. re the page. :
Coniauel on thenextpage. .

131
CHAPTER 5. SIMILAR TRIANGLES

Extra! ... continued from the previous page


i mw The answer turns out to be “no.” But how can we prove this? Just saying that we have
tried many different ways and given up is not a very satisfactory answer, and ultimately
not rigorous. It turns out there is a simple proof, using a common problem solving
technique. We color the 4 x 5 chessboard as shown.

Let’s take another look at those tetrominoes again.

Pt cEk) Gael Gale


If we place the first piece on the rectangle, how many black and white squares will
it cover? It’s not hard to see that it will always cover two black squares and two white
squares.

a es
The same holds for the fourth and fifth pieces.

-
And there are two different combinations of colored squares the second piece can cover,
but both still have two black squares and two white squares.

-_
That means that these four pieces, together, will always cover eight black squares and
eight white squares. But what about the third piece? It can be placed two different ways.

oo os
One way covers three white squares and one black square, and the other way covers
one white square and three black squares. This means that the five pieces, together, will
always cover eleven white squares and nine black squares, or nine white squares and
eleven black squares, which means in particular they can never cover ten white squares
and ten black squares, as shown. Thus, we have used parity (even-ness and odd-ness)
and chessboard coloring to prove that a covering cannot exist.
The twelve pentominoes, on the other hand, can fit snugly inside a 6 x 10 rectangle.
See if you can figure out how. (Answer on page 285.)

GZ ee eee ee re
The Pythagorean Theorem

Choose always the way that seems the best however rough it may be; custom will soon render it easy and agreeable.
— Pythagoras

CHAPTER Sea eS

Me ye Right Triangles

In this chapter we study right triangles. As a quick refresher, a right triangle A


is a triangle that has a right angle among its angles. The side opposite the right
angle is called the hypotenuse, and the other two sides are called legs. In the figure
to the right, AB is the hypotenuse, while AC and BC are both legs. As you'll see
throughout the book, many problems are solved by building right triangles and B
using the principles you'll learn in this chapter, particularly one of the most famous math theorems of
all: the Pythagorean Theorem.

Extra! It has long been a sport among lovers of mathematics to find new proofs of the
mimi Pythagorean Theorem. Most engaging are the proofs that can be expressed simply
as a diagram, challenging the viewer to fill in the details and learn how the diagram pro-
vides a proof of the Pythagorean Theorem. Pure mathematicians are not the only people
who get to join in this game; proofs have been attributed to President James A. Garfield
and Leonardo da Vinci (among many others). We'll share several of these ‘proofs without
words’ with you in this book. You can find a long list of Pythagorean Theorem proofs
at Alexander Bogomolny’s excellent Interactive Mathematics Miscellany website and in
_ Roger Nelsen’s two Proofs Without Words books.

133
CHAPTER 6. RIGHT TRIANGLES

6.1 Pythagorean Theorem

Problem 6.1: In this problem we will prove one of the most famous G
theorems of mathematics. The diagram shows right triangle AABC with 7
hypotenuse AB = c and legs AC = b and BC =a. a b
(a) Prove that a? = cd.
B bs A
(b) Prove that b? = ce.

Problem 6.2: Find the missing sides in the triangles below.

Problem 6.3: Find r.

Problem 6.4: Isosceles triangle AMNO has MN = NO = 7 and MO = 10.


Find [MNO]. :

Problem 6.5: Two vertical poles are 14 feet apart. One is 16 feet tall, and the
other is 9 feet tall. A rope extends straight from the top of one poetto the ee of
a

the otherhere Bow Lossis the rope?


16]

_Q
134
6.1. PYTHAGOREAN THEOREM

Problem 6.6: Let P be a point and m be a line that doesn’t pass through P. B
Let X be on m such that PX m. The distance from a point toa line is the jy -
length of the shortest segment from the point to the line. In this problem,
we prove that the distance from P to m is PX.

(a) Let B bea point on m besides X. What kind of triangle is APXB?


(b) Use the Pythagorean Theorem to get an expression for PB?.
(c) Why does the previous part show that PX is the shortest possible distance from P to line m?

Problem 6.7: In this problem we investigate whether AABC must be a right triangle given that BC = a,
AC = b, AB =c, anda* +b? =c?.
(a) Suppose we have a triangle ADEF such that ZF = 90°, EF = a, and DF = b. (These are the same
lengths as BC and AC in AABC.) What is DE?
(b) What can we say about triangles AABC and ADEF?
(c) Must AABC bea right triangle?

Problem 6.8: The side lengths are as marked in the diagram. Find CD.

We start off with a proof of one of the most famous theorems in mathematics.

Problem 6.1: Right triangle AABC has hypotenuse AB = c and legs


AC = band BC = a. Use similar triangles formed by altitude CD to prove
thata tec.

Solution for Problem 6.1: As we showed in Problem 5.14, we can use AADC ~ AACB to show that b/e = c/b,
so b? = ce. Similarly, ABDC ~ ABCA gives us a/d = c/a, so a” = cd. Adding these expressions for a” and
b* gives us
e+e =cdice=cidt+e)=c’.
O

Extra! Mighty is geometry; joined with art, resistless.


la dlndlndind 4 : —Euripides

135
CHAPTER 6. RIGHT TRIANGLES

|Important: ‘Inany right triangle, the sum of the squares of the A


Vv? legs equals the square of the hypotenuse. In the : |
figure to the right, we have b |

| Cree c ; B

This is the famous Pythagorean Theorem.

‘Problem 6.2: Find the missing sides in the triangles below. —


: G 14 7
A : i
: ~ E Fas

B C L
17 jaar,
fo 8

x
Solution for Problem 6.2:
(a) AABC: AB? + BC? = AC’, so 64 + BC? = 128. Therefore, BC? = 64 and BC = 8:
(b) ADEF & AFEX: From ADEF we have DE? + EF* = DF?, so 36 + EF* = 100, and EF = 8. Now we
can tackle EX with AFEX: EX? + FE? = FX, so EX? = FX* — EF* = 225. So, EX = 15.
(c) AGHI: HI? + GP = GH?, so HI? = 196 — 49 = 147. Therefore, HI = ¥147 = 7 ¥3.
(d) AJKL: KL? = Lj? + JK? =9 + 64 = 73. So, KL = 73.
ix

Problem 6.3: Find r.

Solution for Problem 6.3: We have a right triangle, so we can apply the Pythagorean Theorem:

PMS (Fe De
Therefore, 49 + 77 = 72 +2r+1,sor=24.0
Sometimes it’s not the sides of a right triangle we are missing, but the whole right triangle itself! In
these cases, we have to build the right triangle ourselves, then apply the Pythagorean Theorem.
———
ee ee ee
136
6.1. PYTHAGOREAN THEOREM

Problem 6.4: Isosceles triangle AMNO has MN = NO = 7 and MO = 10. Find [MNO].
Solution for Problem 6.4: Finding an altitude will allow us to find the area. When N
we first studied isosceles triangles on page 67, we saw that we can cut an isosceles 7 7
triangle in
half with an altitude. We do so here in the figure to the right, in which
altitude NX cuts MO in half so that MX = XO = 5. The Pythagorean Theorem
applied to AMNX gives
MX? + NX? = MN’,
so 25 + NX? = 49. Therefore, NX = ¥24 =2 V6, and our area is

[MNO] = aa = 10 V6.

In our next example, it’s not as clear initially how to build a useful right triangle.

Problem 6.5: Two vertical poles are 14 feet apart. One is 16 feet tall, and P_
the other is 9 feet tall. A rope extends straight from the top of one pole to a
the top of the other pole. How long is the rope?

Solution for Problem 6.5: We don’t have a right triangle, but the right angles at Q
and R have us thinking about building our own. We draw ST perpendicular to
PQ, and we have a right triangle with hypotenuse PS! Since ZQ+ZSTQ = 180°,
we have OR ||TS. Therefore, TSR + ZR = 180°, so ZTSR = 90°, and STQRisa
rectangle. Since STQR is a rectangle, we have TS = QR = 14and TQ = SR=9.
Therefore, PT = PQ-— SR =7, and we can now use the Pythagorean Theorem
to find PS:

PS? = PT* + TS* = 49+ 196 = 245.


Therefore, the length of our rope is ¥245 = 7 V5. 0

be ofthe most common ways to find lengths. This iscpacinas true in


.: problems that already involve right angles, such as Problem 6. 5.

Problem 6.6: The distance from a point to a line is the length of the shortest segment from the point
to the line. Prove that this shortest segment is perpendicular to the line.

Solution for Problem 6.6: This is a great example of proving the obvious. Sometimes the obvious is so
obvious that people will propose solutions like this:

137
CHAPTER 6. RIGHT TRIANGLES

Bogus Sol ution: The shortest distance from P to the line m will be when we go
straight from P to the line. This would make a right angle with m
because we get a right angle when we go straight from the point to
the line. |

This ‘proof’ doesn’t say anything at all. There aren’t any false statements, but there aren't any useful
statements either. It essentially argues ‘the angle is a right angle because that’s what you get when you
make a right angle.’ Here’s why we get a right angle:

xX B We start by drawing a diagram. Let P be our point and m be our line. Let X be
m on m such that PX m. We must show that PX is the shortest segment from P to
m, So we must show that PX is shorter than any other segment from P to m. We let
B be another point on m, and compare PX to PB.
Now we have a right triangle, APXB, so we can use the Pythagorean Theorem: PB? = PX? + XB?.
Since XB? is clearly greater than 0, we must have PB2 > PX2,so0 PB > PX. So, the perpendicular segment
from P to m is shorter than any other segment from P to m. 0

Important: The distance from a point to a line is the length of the segment from the |
point to the line that is perpendicular to the line. |

You'll have a chance to use this definition of the distance from a point to a line to prove the following
as an Exercise:

Important: If AB ||CD, then A and B are the same distance from CD. — |

A B |
| |

Ci =
D aes

Throughout this section we have used the Pythagorean Theorem to find information about the sides
of right triangles. Perhaps you're wondering if we can use the Pythagorean Theorem ‘in reverse’ to
determine if a triangle is a right triangle.

Problem 6.7: Suppose we have AABC such that BC = a, AC = b, AB = c, and a2 +b? = c2. Is AABC
necessarily a right triangle?

Solution for Problem 6.7: If a* + b? = c*, we do know that there is some triangle with side lengths a, b,
c that is a right triangle. Specifically, consider ADEF with ZF = 90°, EF = a, and DF = b. Then, the
Pythagorean Theorem tells us that DE? = EF? + DF* = a? + b*. Since we know that a2 + b? = c2, we have
DE = c. Therefore, ADEF is a right triangle with sides of length a, b, and c. Since AABC has the same
side lengths as ADEF, we have AABC = ADEF by SSS. Therefore, AABC must also be a right triangle. 0

DC
6.1. PYTHAGOREAN THEOREM

Important: If the sides of a triangle satisfy the Pythagorean Theorem, then the tri-
VY angle must be a right triangle. Any time you see triangle sides that
satisfy the Pythagorean Theorem, you should mark the angle opposite
the longest side as a right angle.

Sidenote: The ancient Egyptians used this method of determining a triangle is right
S in construction. They used loops of rope consisting of 12 rope segments
of equal length. When pulled taut to form a triangle with sides equal to 3,
4, and 5 of these segments, they had a right angle opposite the 5-segment |
side! They could then use this to make sure angles on buildings or bricks,
etc., were right angles.

Let's put this principle to work in a simple problem.

Problem 6.8: The side lengths are as marked


inthediagram. Find CD. _ A

Solution for Problem 6.8: Since AB* + BC? = AC*, we know that AABC is a right triangle with right angle
at B. Therefore, ACBD is a right triangle with hypotenuse CD, so we can use the Pythagorean Theorem:
CD? = BC? + BD* = 16+ 81 = 97,
so CD = 97.5

yy
fxercises
hg
6.11 Find the missing side length in each of the diagrams below:

@) , o) ©) aK
6

17

A a4
is c- 1s
E ? Z H

(d) | bagless”See 4 (mary (f) a i;


R

L = O
CHAPTER 6. RIGHT TRIANGLES
a

6.1.2 Prove that the two non-right angles of a right triangle are complementary.
6.1.3 One leg of a right triangle is 3 cm more than 3 times the other leg, and the hypotenuse is 1 cm
longer than the longest leg. Find the area of the triangle. Hints: 276
6.1.4 Prove that in a right triangle, the hypotenuse is the longest side.
6.1.5 Which of the following groups of three numbers can be the side lengths of a right triangle?
(a) 6,8, 10. (d) 5/8, 3/2, 13/8.
(b) 4,5, 6. (ce) 22, 32,5.
(c) 9,3 V3, 6 V3. (f) 1.2, 3.5, 3.7.
6.1.6 Find the area of a triangle with sides of length V6, V7, and v13.

6.1.7 Find the area of an isosceles triangle with two legs of length 8 and base of length 6. Hints: 61

6.1.8 In this problem, we show that if AB ||CD, then A and B are the same distance from CD.

(a) Let X and Y be on CD such that AX 1 CD and BY 1 CD. Show that ZXAB = 90°.
(b) Prove that AAXB = AYBX, and use this to prove that A and B are equidistant from CD.

6.1.9 Initially, a 25-foot ladder rests against a vertical wall such that the top of the ladder is 24 feet from
the ground. Then, Nathan moves the base of the ladder farther out from the wall so that the top of the
ladder slides down until resting against the wall at a point 20 feet above the ground. Given that the wall
is perpendicular to the ground, how far did Nathan move the base of the ladder?
6.1.10 A triangle has sides measuring 13cm, 13cm, and 10cm. A second triangle is drawn with sides
measuring 13cm, 13cm and xcm, where x is a whole number other than 10. If the two triangles have
equal areas, what is the value of x? (Source: MATHCOUNTS) Hints: 161, 326

6.2 Two Special Right Triangles

|Problem 6.9: AXYZ is an isosceles right triangle as shown with ZY = 90°. Given that X
XZ = 6, find 2X and XY.

Problem 6.10:
(a) If a right triangle is isosceles, must it be the two legs that are the equal sides?
.(b) Show that in any isosceles right triangle, the length of the hypotenuse is V2 times the length of
ales.
mG ee aes

140
6.2. TWO SPECIAL RIGHT TRIANGLES

Problem 6.11: In this problem we will derive a formula for the area of an
equilateral triangle. We'll start with AABC, an equilateral triangle with side
length 4.
(a) Let AX be an altitude of AABC as shown. Find BX.
(b) Find AX.
(c) Find the area of AABC.
(d) Let APQR be an equilateral triangle with side length s. Find [PQR] in terms of s.

Problem 6.12: Our goal in this problem is to find YZ and YX in the triangle shown. X
(a) Suppose we extend YZ past Y to a point D such that YD = YZ. How are
triangles AXYZ and AXYD related?
30° 12
(b) What kind of triangle is AXZD?

(c) Find YZ and YX.


(d) How are the sides of a triangle that have angle measures 30°, 60°, and 90° L 60°
related? Y Zz
(e) Extra challenge: Suppose APQR has sides 3, 3 V3, and 6. Can we deduce that the angles of
APQR are 30°, 60°, and 90°?

Problem 6.13: Find all the missing sides in the diagram.

We know a special side relationship when a triangle is isosceles, and another special relationship
when a triangle is a right triangle. What if the triangle is both?

Problem 6.9: AXYZ is an isosceles right triangle with ZY = 90°. Given that XZ =6, X
find ZX and XY. a

Solution for Problem 6.9: Since the triangle is isosceles, the angles opposite the equal sides are equal.
Therefore, 2X = ZZ. Since ZY is a right angle, 2X and ZZ must add to 90°. So, 2X = ZZ = 90°/2 = 45°.

To find XY, we use the Pythagorean Theorem: XY* + YZ* = XZ*. Since YZ = XY and XZ = 6, we
have 2(XY2) = 36,so XY = V18 =3 v2. 0

141
CHAPTERee
eee 6. RIGHTee
TRIANGLES

This problem suggests we might be able to find a general relationship among the sides in an isosceles
right triangle.

Problem 6.10:
| (a) Ifa right triangle is isosceles, must the two sides that are equal be the legs of the triangle?
|
(b) Show that in any isosceles right triangle, the length of the hypotenuse is V2 times the length of
a leg.

Solution for Problem 6.10:


(a) The Pythagorean Theorem quickly shows us that it must be the two legs that are equal in an
isosceles right triangle. Suppose our triangle is AABC with right angle at ZC. The Pythagorean
Theorem gives us AC? + BC? = AB. So, if AB equals AC or BC, then the other leg must be 0, which
is impossible. Therefore, in any isosceles right triangle, it is the two legs that are equal.
(b) If legs AC and BC each equal x, then we can find hypotenuse AB with the Pythagorean Theorem:
AB? = x? + x* = 2x”. Therefore, AB = x V2, so the hypotenuse must be V2 times a leg.

"Important: In any isosceles right triangle, the legs areequalandthe A


hypotenuse is V2 times the leg. Because the angles of
an isosceles right triangle are always 45°, 45°, and 90°, x |45°\* V4
an isosceles right triangle is sometimes called a 45-45-90
triangle. If we know the length of any side of sucha 4
triangle, we can quickly find the others using the side
i relationships shown in the figure at the right.

|Sidenote: Due to the theorem that bears his name, Pythagoras may be an even more
_ well-known name to the general public than the widely acknowledged —
‘father’ of geometry, Euclid. Pythagoras lived during the 6th and 5th |
centuries BC (nearly 200 years before Euclid), and was the founder of a
secret society known, unsurprisingly, as the Pythagoreans. It is unclear }
i
whether Pythagoras was the first to prove the Pythagorean Theorem, but
he and his organization were definitely responsible for a number of other
___ discoveries, including one that dramatically shook their understanding of
the world.
_ The Pythagoreans had a deeply held belief that every phenomenon in
nature could be described by whole numbers. However, their study of
isosceles right triangles led them to the startling conclusion that the ratio
_ of the length of the hypotenuse of an isosceles right triangle to the length
of a leg of the triangle cannot be expressed as a ratio of two integers. As
we have seen in this section, this ratio equals v2.
V2
| Continued onn thenext‘page. .
6.2. TWO SPECIAL RIGHT TRIANGLES

Sidenote: . . continued from the previous page


ay “To show that ¥2 cannot be written as a ratio of two integers, we use proof—|
by contradiction. This means we assume the opposite of what we want to
prove, and show that this assumption leads to an impossible conclusion.
Here’s what the proof looks like:
We assume that V2 = p/q, where p and q are integers and p/q is in lowest
terms. Squaring V2 = p/q gives 2 = p*/q?, so we have p” = 2q*. Therefore,
p must be even, so we can write p = 2m for some integer m. This gives
(2m)*= 2q*, so 2m* = q?. Therefore, q is even, too. But p and q can’t both
be even if p/q is in lowest terms!
Since our assumption that V2 can be written as a ratio of integers in
lowest terms leads to the impossible conclusion that the ratio is also not in |
lowest terms, we have found our contradiction that forces us to conclude
that our assumption is wrong. Therefore, V2 cannot be written as a ratio
of integers.
We call a number that can be written as a ratio of integers a rational
number. A number like 2 that cannot be so written is called an irrational
number. Some sources claim that the Pythagorean responsible for proving |
the existence of irrational numbers, Hippasus, was rewarded for his insight
by being murdered. Mathematical discoveries are considerably more well-
rewarded now! (See page 216.)
are |

Problem 6.11: Find the area of an equilateral triangle with side length 4, then find a formula for the
area of an equilateral triangle with side length s.

Solution for Problem 6.11: To find the area, we will need to find the length of an
altitude. Therefore, we draw altitude AX as shown. Since ZB = ZC, ZAXB =
ZAXC, and AB = AC, we have AAXB = AAXC. Therefore, BX = CX = BC/2 = 2.
We can now use the Pythagorean Theorem to find AX. Since AX? + BX? = AB’,
we have:
— VAB2 — BX2 = V¥16 — 4 = V12 = 2 V3.
The area of AABC is
pane] = BAX) _WANS) _ 4 vg
We can use the exact same procedure to derive a formula for the area of an equilateral triangle with side
length s. If we let the sides of AABC each be s and draw altitude AX as before, we have BX = CX = s/2.
The Pythagorean Theorem then gives
2 ee
Avie VAB2 = Bx2 = 2-5 = eee Ne
Now we can find our area:
[ABC] =
(BC)(AX) _ s(s ¥3/2) _ s*V3
bale
hr candy Samia Tale

143
CHAPTER 6. RIGHT TRIANGLES

Important: The area of an equilateral triangle with side length s is s? ¥3/4. If you |
WW, understand how we found this formula, you shouldn’t need to ae
it!

Our derivation of the formula for the area of an equilateral triangle suggests a notable relationship
among the sides of a triangle with angles 30°, 60°, and 90°.

Problem 6.12:
(a) How are the sides of a triangle that have angle measures 30°, 60°, and 90° related?
(b) Suppose APQR has sides 3, 3 3, and 6. Can we deduce that the angles of APQR are 30°, 60°,
and 90°?

Solution for Problem 6.12:


(a) We found the area of an equilateral triangle by cutting the triangle into x
two right triangles that have acute angles of 30° and 60°. Here we go v
backwards — we build an equilateral triangle with two of these right ‘
triangles. In the diagram to the right, congruent right triangles AXYZ ~ 4308
and AXYD together form equilateral triangle XDZ. mi
Suppose the length of XZ is s. Since ZD = XZ = s and DY = YZ, we 2 60° }, 60°
have YZ = s/2. As we saw in Problem 6.11, we can use the Pythagorean D 16 Z
Theorem on AXYZ to find that XY = s V3 /2. Therefore, the sides in any triangle with angles 30°,
60°, and 90° come in the ratio:

Leg opposite 30° : Leg opposite 60° : Hypotenuse = 1 : V3: 2.

In other words, the hypotenuse is double the leg opposite the 30° angle, and the leg opposite the
60° angle is V3 times the other leg.
For example, if the hypotenuse is 12, then the leg opposite the 30° angle is Xx
12/2 = 6, and the leg opposite the 60° angle is 6 V3 as shown in the figure to the
right. 12
We have just discovered the relationship among the sides of another common 6 v3 /°"
type of right triangle. 60°
6
Important: Ina right triangle with acute angles of 30° and 60°, the
Y _ Sides are in the ratio 1 : V3 : 2. as shown to the right.
Such a triangle is often called a 30-60-90 triangle.

(b) Having found this relationship among the sides given the angles, we wonder if all triangles that
have sides in these ratios are 30-60-90 triangles. We'll consider one such triangle: APQR with sides
3,3 V3, and 6. What’s wrong with this ‘proof’ that APQR is a 30-60-90 triangle:

144
6.2. TWO SPECIAL RIGHT TRIANGLES

Bogus Solution: We extend RQ to S as shown such that Pp


| QOS = OR =3. Since QS =OR and PO = s
PQ, we have APQR = APQS by SAS Con- x
gruence. Therefore, PS = PR = 6. Clearly, 6 \6 |
RS = 6 also, so APQS is equilateral. So, 3 V3 * |
ZR = 60° and ZRPQ = 90° — ZR = 30°, and Yaad
APQR is 30-60-90. She
| R SR eis: S|
Unfortunately, we have a big problem here: ‘Since Qs = OR and PQ = PQ, we have APOR =
APQS by SAS Congruence.’ This statement assumes that ZPQR = ZPQS, which in turn assumes
that ZPQR is a right angle. But we can’t assume that, because that’s what we’re trying to prove!
There are a couple ways we can show that APQR is a 30-60-90 triangle.
Solution 1: Patch the hole in the Bogus Solution. We show that APQR is a right triangle by showing
that its sides satisfy the Pythagorean Theorem:
3? + (3 V3)? =9 +27 = 36 =6.
Therefore, APQR is a right triangle with right angle opposite the side with length 6. From here,
we can use the Bogus Solution to complete our proof that APQR is a 30-60-90 triangle.
Solution 2: We can use triangle congruence another way to write an even simpler proof! We
compare our APQR to 30-60-90 triangle AABC, which has hypotenuse 6. Because AABC is 30-60-
90, we know that its legs have lengths 3 and 3 V3. Therefore, AABC = APQR by SSS Congruence,
so the angles of APQR are the same as those of AABC. In other words, APQR is a 30-60-90 triangle.

Important: _ If the side lengths of a triangle are in the ratio 1 : V3 : 2, then the triangle |
Vv is a 30-60-90 triangle, with the right angle opposite the longest side and
the 30° angle opposite the shortest side. |
You've seen two proofs for one specific triangle above; you'll have a chance to write a proof for
all such triangles in the Exercises. (You knew that was coming!)

Let’s use our knowledge of 45-45-90 and 30-60-90 triangles to find some side lengths in a problem.

Problem 6.13: Find all the missing sides in the diagram.

Solution for Problem 6.13: Because AABC is a 30-60-90 triangle and AB is opposite the 30° angle, we have
AC = AB V3 = 6 V3 and BC = 2(AB) = 12.
Because ABDC is a 45-45-90 triangle, we have BD = BC = 12 and CD = BC Np PAS,

145
CHAPTER 6. RIGHT TRIANGLES

Because AEDC is a 30-60-90 triangle and CD is the longer leg,


we have CD = DE Y3. Since CD = 12 V2, we have

DE=
CD 2 2oeel2 Noo Nour) Came
~B 8 WB vB 3
We also have EC = 2ED = 8 V6. Finally, AECF is a 45-45-90
triangle with hypotenuse EC = 8 V6. Therefore, the legs each
have length EC/V2 = 8 V3. All our lengths are shown in the
completed figure at right. 0

Exercises

6.2.1

20

A 10 G

(a) Find AB, ZB, and ZC. (b) Find DE, EF, and ZF.

J
G I D

V6 V6 : t .
H Geek.
(c) Find GI, ZG, and ZI. (d) Find JK, JL, LM, ZLJM, and 2M.

6.2.2 Prove that any triangle whose sides have lengths in the ratio 1 : V3 : 2 is a right triangle. Hints:
413

6.2.3 Let ABCD bea square of side length 3, and let E, F, G, and H be points on sides
AB, BC, CD, and DA, respectively, such that AE = CF = CG = AH = 1.

(a) Prove that EFGH is a rectangle by showing that all of its angles are right angles.
(b) Find the area of EFGH.
6.2.4 Find the area of APQR given that PQ = QR = 8 and ZPQR = 120°. Hints: 312
6.2.5x In AABC, AB = 20, ZA = 30°, and ZC = 45°. Find BC. (Source: MATHCOUNTS) Hints: 469

146
6.3. PYTHAGOREAN TRIPLES

6.3 Pythagorean Triples

A Pythagorean triple is a set of three integers that satisfy the Pythagorean Theorem. For example,
{3,4,5} is a Pythagorean triple because 32 + 42 = 5*. However, {1,2,3} is not a Pythagorean triple since
there’s no way to add two of 12, 27, and 32 to get the third.

Before proceeding with the problems, play around a little and try to find some Pythagorean triples
on your own. Can you find any patterns in your triples?

Problem 6.14:
(a) Find the missing side length in each right triangle shown below.
(b) Given that {a,b,c} is a Pythagorean triple, is {2a,2b,2c} a Pythagorean triple? How about
{na,nb,nc}, where n is a positive integer?

Problem 6.15:
(a) Find the last side in each right triangle below. Do you see a pattern?

i pet eee
4 15

B 26 ; < 37

24

(b) Complete the table below.

(c) Isa triangle with sides 2n, n? — 1, and n? + 1, where n > 1, always a right triangle?

147
CHAPTER 6. RIGHT TRIANGLES

an |
Problem 6.16: Find the missing side in each of the triangles below without using the Pythagore
Theorem directly. Just use your knowledge of Pythagorean triples!
Ee
A

R
Q
20 x 72 y

B sg) C ees
Z

We begin our discussion of Pythagorean triples with the most common triple.

Problem 6.14:
(a) Find the missing side length in each right triangle shown below.
(b) Given that {a,b,c} is a Pythagorean triple, is {2a,2b, 2c} also a Pythagorean triple? How about
{na,nb,nc}, where n is a positive integer?

x
D
L G
: 10 20 333
3
Se Bo HS 2546262 1

Solution for Problem 6.14:


(a) Wecan apply the Pythagorean Theorem to find

AG f= NAB + BCZ= VOI 6= 5


EE) =, NDP DE = Vo4.=8
GHiee=— NG HPN oa ao
XZ = VXY?+ YZ? = \/some big numbers = 555

OK, we got a little lazy on that last one. We noted in the first three that the sides are always in the
ratio 3: 4: 5,so we took a stab at 555 for XZ. We're right, of course, but why?
There are a couple ways we can prove that we are right. First, we could note that AXYZ ~ AABC
due to SAS Similarity. Second, we could use algebra, as we'll see in the second part of the problem.

Extra! Mathematics consists of proving the most obvious thing in the least obvious way. .
> (10> —George Polya

TAR eee
6.3. PYTHAGOREAN TRIPLES

(b) Weare given that {a,b,c} is a Pythagorean triple, so a? + b* = c*. To test {2a, 2b, 2c}, we consider the
sum of the squares of the first two terms:
(2a)* + (2b)? 4aq* + 4b?
= 4(a° +0’)
= 4c* (because a? + b* = c*)
Bee
Therefore, {2a, 2b, 2c} is a Pythagorean triple. There’s nothing special about 2; we can run through
this with any number:
(na)* + (nb)* = na? + n7b? = (a2 + 0°) = 2c? = (nc).

So, {na,nb,nc} is also a Pythagorean triple. Notice also that n doesn’t have to be an integer to
produce side lengths of a right triangle! For example, since a triangle with sides {3, 4,5} is a right
triangle, a triangle with sides {3(1/7), 4(1/7),5(1/7)} = {3/7,4/7,5/7} is a right triangle.
You will often see triangles with sides in this ratio referred to as 3-4-5 triangles. Triangles
whose sides are in the ratios of other Pythagorean triples are sometimes similarly identified, such
as 5-12-13 triangles or 7-24-25 triangles.

Important: If {a,b,c} are the sides of a right triangle, then so are {na,nb,nc} for any
VY positive number n.

Therefore, we can generate whole groups of Pythagorean triples from a single triple:
{3,4,5} — {6,8, 10} — (9,12, 15} — --- — {333,444,555} > ---
There’s nothing special about integer multiples of our {3,4,5} triple. We can generate all sorts of other
sets of three numbers that are the sides of a right triangle. Here are a few:
{0.3,0.4,0.5} {3/11,4/11,5/11} ~~ {6 ¥5,8 V5, 10 V5}

Recognizing common Pythagorean triples, or multiples of those triples, is a very useful problem
solving tool. The following problem exhibits some common triples, as well as a neat pattern for
generating more.

Problem 6.15: ; iv,


(a) Find the last side in each right triangle. 3 EX [ ae [ =
4 A)
(b) Do you seea pattern? Can you prove the pattern always

24

Solution for Problem 6.15:


(a) We can use the Pythagorean Theorem to find the rest of the sides as shown below.

149
CHAPTER 6. RIGHT TRIANGLES

(b) If we take the smallest even leg, halve it, then square the 5 NS 6 10 8 S 17.
result, we get the number that is one more than the other leg j >»
and one less than the hypotenuse. Putting this in math terms, 4
we let our smallest even leg have length 2n. Then the other 10 e 26 12 a oF
two sides are n* — 1 and n? + 1. We can make a table that
shows what happens for different values of the even leg.
Smallest Even Leg |Other Leg |Hypotenuse
n 2n bent zal las n>+1
2 4 Bier: 5
E 6 aRCe: 10
4 8 15 ies
8 10 24 26
Gi 12 cen LE
7 14 48 50
All the triangles described in the table are indeed right triangles. We can show that any triangle
with sides 2n, n2 — 1, and n2 +1, wheren > 1,isa right triangle with some algebra:

(net (2 = 1) S42 ad
= ni +2n*+1
= (n* +1).

Sidenote: Choose three positive integers k, m, and n, where m > n. Compute the three
: numbers k(m* — n7), 2kmn, and k(m* + n?). Try it for several different values
of k,m,and n. You should find that the three numbers you generate in each
_case form a Pythagorean triple. See if you can figure out why this works.
Even more surprising is the fact that every single Pythagorean triple
fits this mold. In other words, for every single Pythagorean triple, can
we always find some k, m, and n such that {2kmn,k(m? — n2),k(m2 + n2)}
generates that triple.

Problem 6.16: Find the missing side in each of the triangles below
without using the Pythagorean Theorem directly. Just use your knowl- A
edge of Pythagorean triples!

Solution for Problem 6.16: We note that the legs of AABC are in the ratio 15/20 = 3/4. Therefore, our legs
are in the same ratio as the legs in a 3-4-5 triangle. We can then conclude that the hypotenuse is 5/3 of
the smallest leg, or (5/3)(15) = 25.

150
6.3. PYTHAGOREAN TRIPLES

We might also have found AC by noting that the legs of AABC are 20 = 4(5) and 15 = 3(5), so this is
a 3-4-5 triangle. Therefore, we know that AC = 5(5) = 25.

In APQR, the ratio of the given leg to the hypotenuse is 0.7/2.5 = 7/25. This makes us think of the
Pythagorean triple {7, 24,25}. Since 0.7 = 7(0.1) and 2.5 = 25(0.1), we know the other leg is 24(0.1) = 2.4.

In AXYZ, the ratio of the given leg to the hypotenuse is 72/78 = 12/13, which reminds us of the
Pythagorean triple {5, 12, 13}. Since our leg is 72 = 12(6) and the hypotenuse is 78 = 13(6), the other leg
must be 5(6) = 30

Notice our general tactic for finding the missing length in each triangle in Problem 6.16 — we take the
ratio of the sides we know and then try to recognize a simple Pythagorean triple that has the numbers
in our ratio. O

| WARNING! Make sisure you ssee » that it’S important that ‘the 2: 5length ir
in1 APQR of |
“S Problem 6.16 is the hypotenuse and not a leg. A triangle with legs|
0.7 and 2.5 does not have third side 2.4. In fact, it doesn’t match any ©
simple Pythagorean triple. Therefore, you must be careful when using |
your knowledge of Pythagorean triples to make sure your given sides —
| match corresponding sides (legs or hypotenuse) of the Pythagorean
triple you would like to use. |
Ss $$!

Sidenote: Perhaps inspired by Pythagorean Triples, the great mathematician Pierre |


de Fermat explored the equation
ok |

for values of n besides n = 2 (n = 2 gives the Pythagorean Theorem, of |


course). That there are no integers x, y and z that satisfy this equation when —
nis an integer larger than 2 is known as Fermat's Last Theorem. Although |
it seems like a simple proposition, Fermat’s Last Theorem evaded proof for
hundreds of years until Andrew Wiles proved it in 1994. Alas, his proof is
far too long to fit in the margins of this book.

I rxercises leg
6.3.1 Which of the following are side lengths of a right triangle? (Try using your knowledge ofPythagorean
triples!)

(a) 300, 400, 500.


(b) 36, 48, 60.
(cla v5 av
i2-V 13:
(d) 20, 37.5, 42.5.
(e) 1.44, 1.96, 2.4.
GR
154
CHAPTER 6. RIGHT TRIANGLES

(a) Is there a right triangle such that all three side lengths are odd integers? Hints: 523
(b) Is there a right triangle such that two of the side lengths are even integers and the other is an odd
integer? Hints: 143
(c) Is there a right triangle in which two side lengths are simple fractions (ratios of integers, such as
2/3 or 3/7), and the other side length is an integer?
(d) Is there a right triangle in which one of the side lengths is a simple fraction and the other two side
lengths are integers?
6.3.3. Susie rides her bike north 3 miles, then east 8 miles, then south 13 miles, then east 16 miles. How
far is she from where she started?

6.3.4 Use SSS Similarity to show that if {a,b,c} is a Pythagorean triple, then so is {na,nb, nc}, where n is
a positive integer.
6.3.5 In this problem we find all Pythagorean triples in which 18 is one of the legs.
(a) Let b and c be the other two sides, where c is the hypotenuse. Show that c? — b? = 187.
(b) Factor the right side of the equation from part (a) and use the factors of 187 to solve for all possible
pairs of integers of integers (b,c) that satisfy the equation.
6.3.6 Find the Pythagorean triple that has 97 as the length of the hypotenuse without using a computer
or calculator. Hints: 101

6.4 Congruence and Similarity Revisited

Problems :

Problem 6.17: Are the two triangles shown congruent? Why or


why not?

Problem 6. 18: In this problem we will prove thatif radius OC of e001is ig) wales to chord AB of
20, then radius OC bisects chord AB. | a
o@ ‘Draw a diagram to use in your proof. Include both radius OC and chord AB.
(by : ‘Find a pair of congruent triangles that you can use to complete the proof.

2,what7
Problem 6.19: The center of a circle isi 4 units awayfrom a chord ayofthe circle.- Pgs
the radius of the circle?

TED
6.4. CONGRUENCE AND SIMILARITY REVISITED

Problem 6.20: Are the two triangles in the diagram similar? Why or why A
not?
12 15

In this section we discuss a special congruence theorem and a special similarity theorem that only
work for right triangles.

Problem 6.17: oe the two


t D triangles 4in1 the dincrarn congruent? G fe
Why or why not?

Solution for Problem 6.17: What's wrong here:

Bogus Solution: Two sides and an angle of one triangle equal two sides and an angle
gia of the other, so the triangles are congruent.

The problem here is that SSA is not in general a valid congruence theorem, as we found in Section 3.5.
However, if we note that the angles that are equal are right angles. ..
We can use the Pythagorean Theorem to find AB and FG. Inboth cases, the length is ¥100 — 81 = 19.
Therefore, by SSS (or SAS), we have AABC = AFGE. 0

In the same way, any time the hypotenuse and a leg of one right triangle equal those of another,
we can use the Pythagorean Theorem to show that the third sides of each triangle are equal. In other
words, SSA works for right triangles. Rather than give it an unwieldy name like ‘SSA, but just for right
triangles’, we call it:

“Important: Hypotenuse-Leg Congruence (HL ¢Congruence) states that if the hy-


ae VY potenuse and a leg of one right rape equal those of another, ten the
triangles are congruent.
ie

As you might guess, there’s an Leg-Leg Congruence (LL Congruence), but that’s just SAS Congru-
ence (make sure you see why), so there’s really no need to give this a special name.

Problem 6.18: Prove that if radius OC of OO is perpendicular to chord AB of @0O, then radius OC
bisects chord AB.

‘ommon sense is thecollection ofprejudices acquired by age eighteen.


en ee | _ Albert Einstein

153
CHAPTER 6. RIGHT TRIANGLES

Solution for Problem 6.18: As usual, we start with a diagram, including both our radius 'e
and our chord. We add the radii to point A and B so we have right triangles to work 4 ras B
with. Since OA = OB and OX = OX, we have AOXA = AOXB by HL Congruence. es
Therefore, AX = BX, so radius OC bisects chord AB. O

Problem 6.18, combined with Problem 3.4 on page 54, gives us a very useful tool:

3 Roo FA esoaea Re sit |


f

|Important: _ Ifa radius of a circle bisects a chord of the circle, it is perpendicular to the |
| chord. Conversely, if a radius of a circle is perpendicular to the chord, it _
pisses
bisects the chord. |

We can use this tool on a variety of circle problems. For example:

|
Problem 6.19: The center of a circle is 4 units away from a chord PQ of the circle. If PQ = 12, what
is the radius of the circle?

Solution for Problem 6.19: We let point M be the point on PQ closest to the center
of the circle, which we'll call O. Since M is the point on PQ closest to O, we have
OM 1 PQ. Because OM is part of a radius that is perpendicular to chord PQ, OM
bisects chord PQ. So, we have PM = MQ = 6. Finally, we apply the Pythagorean
Theorem to AOPM to find OP = VPM? + OM? = V36 + 16 = 2V13. 0
Unsurprisingly, the Pythagorean Theorem can be used to give us a Hypotenuse-
Leg Similarity Theorem as well.

Problem 6.20: Are the two triangles in the diagram similar? Why or A R
why not?

Solution for Problem 6.20: What's wrong with this:

Bogus Solution: Since 20/15 = 16/12, we have two pairs of sides in a common ratio
and 4A = ZQ, so by SAS Similarity, the triangles are similar.

SAS Similarity requires the angle to be between the two sides that have the constant ratios, so we
can’t use SAS Similarity like this.
We recognize these two triangles as 3-4-5 triangles (or we use the Pythagorean Theorem), and we
find that AB = 9 and QR = 12. Therefore, we have

AB _ AC _ BC
RQ. PQ” PR’
so AABC ~ AQRP by SSS Similarity. o

= SS eee
6.4. CONGRUENCE AND SIMILARITY REVISITED

In much the same way, we can use the Pythagorean Theorem to show that if a hypotenuse and a leg
of one right triangle are in the same ratio as the hypotenuse and leg of another right triangle, then the
other legs are also in that ratio. So, the two triangles are similar. Therefore, we have a new similarity
theorem just for right triangles.

Important: Hypotenuse-Leg Similarity (HL Similarity) states that if the hypotenuse |


and a leg of one right triangle are in the same ratio as the hypotenuse ©
and leg of another right triangle, then the triangles are similar.

As with congruence, there is a Leg-Leg Similarity Theorem that states that if the legs of one right
triangle are in the same ratio as the legs of another right triangle, then the two triangles are similar.
However, this is just the same thing as SAS Similarity. (Make sure you see why!)

Exercises
la
6.4.1 Which of the following pairs of triangles are similar? For the pairs that are similar, write both the
similarity relationship and why the two triangles are similar.

(a) 2.4 (b) 8


P A B W ae 63 :

Q P R Y F

oes E
(d)
5 Q 100
12 C M O
a G 7 FA 28
B is N P

6.4.2. Points A and B are ona circle with radius 9. Given that the center of the circle is 3 units from AB,
find AB.

6.4.3 In this problem we prove the HL Similarity Theorem we discovered in Problem 6.20. Let AABC
and AMNO be right triangles with right angles at ZA and 2M. Furthermore, suppose AB/BC = MN/NO.
We wish to prove that AABC ~ AMNO.

(a) Use the Pythagorean Theorem to find expressions for AC? and MO?.
(b) Use the given equation AB/BC = MN/NO to find an expression for AB. Write an expression equal
to AC?/MO?.
UE EEE =a
#55
CHAPTER 6. RIGHT TRIANGLES

Prove that AC?/MO? =


(c) Substitute your expression for AB into your expression for AC?/MO*.
BC?/NO*. Hints: 345
(d) Prove that AABC ~ AMNO. Hints: 507

6.4.4 AD and BC are both perpendicular to AB in the diagram at left below, and CD 1 AC. If AB=4
and BC = 3, find CD. (Source: HMMT)

D Sg

Me

A
:
B *
\N C

Figure 6.1: Diagram for Problem 6.4.4 Figure 6.2: Diagram for Problem 6.4.5

6.4.5 ABCD isa square in the diagram at right above. M is the midpoint of AD and BM 1 MN.

(a) Prove that AMDN ~ ABAM.


(b)x Prove that ZABM = ZMBN. Hints: 189

6.5* Heron’s Formula

As we saw when discussing SSS Congruence, all triangles with the same three side lengths are congruent.
Therefore, they all have the same area. So, we wonder, ‘Since the area is fixed given the three side lengths,
is there a way to figure out what the area is?’
In this section we'll answer this question with the Pythagorean Theorem and a lot of algebra.

Problem 6.21: In this problem, we find the area of AABC, which has sides
of length 13, 14, and 15. We start by drawing altitude AX. Let it have length
hand let BX = x.

(a) What is XC in terms of x?


(b) Apply the Pythagorean Theorem to AABX to get an equation with h
and x in it.
(c) Apply the Pythagorean Theorem to AACX to get an equation with h
and x in it.
. (d) Subtract your equation in (c) from the one in (b) to find x. Use x to find h.
- (e) Find the area of the triangle.
(f) How could you use your knowledge of Pythagorean triples to avoid all this algebra?

156
6.5*. HERON’S FORMULA

Problem 6.22: In this problem we will try to find a formula for the area of A
a triangle in terms of only its sides. As usual, we let AB = c, BC = a, and
AC = b. We start again with altitude AX = h, and we let BX = x. Finally, we let C
$ = (a+b+c)/2 to simplify our algebra in the problem. (Warning: There’s a lot
of algebra in this problem. You will use the difference of squares factorization
many times: y* — z* = (y — z)(y +2).) B x
(a) Apply the Pythagorean Theorem to AABX to get an equation with a, b,c, h, and/or x in it.
(b) Apply the Pythagorean Theorem to AACX to get an equation with a, b, c,h, and/or x in it.
(c) Subtract your equation in (b) from the one in (a) to get an equation with just x, a, b, and c. Use
this equation to show that x = (a? + c* — b*)/(2a).
(d) Substitute your expression from (c) into one of your earlier equations to show that

ae 2 ys(s — a)(s — b)(s — c)


a

(e) Show that [ABC] = ys(s — a)(s — b)(s — c).

Problem 6.23: Find the area of each triangle below using the formula you derived iin the previous
problem.

Before we tackle finding a formula for the area of a triangle in terms of its sides, we try finding the
area of a specific triangle.

Concept: Often it’s best to try a few examples before attempting to derive a general
| = formula. The coe can be a good guide when you try to derive your
formula. Os

Problem 6.21: Find the area of a triangle with sides of lengths 13, 14, and 15.

Extra! You've probably heard that music and math are related. You can thank Pythagoras for
“a this discovery, too! Pythagoras is credited with discovering that pleasant sounds are.
produced by vibrating strings when the ratio of the lengths of the strings is a whole
wumber. This discovery led to the development of the musical See wesovndee now as

To”
CHAPTER 6. RIGHT TRIANGLES

Solution for Problem 6.21: We start by drawing altitude AX = h of AABC as shown. A


We let BX = x, so that XC = 14-—.x. We have some right triangles, and lots of
lengths, so we try the Pythagorean Theorem. Applying the Pythagorean Theorem —_43 15
to triangles AAXB and AAXC yields:
Dear?
ita 100 B a
(14 ee 25 14

Seeing hi on the left of both equations, we think to subtract the second equation from the first to eliminate
h?. We get
x? — (14—x)* = 169 — 225.
Therefore, x? — 196 + 28x —x? = —56, so x = 5. We can substitute x = 5 into either of our original equations
to find h = 12. Therefore,
(BC)(AX) _ (14)(12) _ 8A
[ABC] = 5 5

Seeing that x and h are integers, we wonder if we could have found them without the algebra.
Triangles AABX and AACX share a leg and have 13 and 15 as their respective hypotenuses. This makes
us think of the Pythagorean triples 5-12-13 and 9-12-15, which have the needed hypotenuses and a leg
length in common. Fortunately, the two other legs, 5 and 9, add up to 14, the length of BC. Therefore,
when we glue a 5-12-13 triangle to a 9-12-15 along the leg with length 12, we get a triangle with sides
i3gl4 15.26
Unfortunately, we usually can’t find Pythagorean triples that fit together nicely to make our triangle.
However, we can follow our procedure from Problem 6.21 to find a formula for the area of a triangle in
terms of its sides.
Put on your algebra hat...

Problem 6.22: In triangle ABC, let AB = c, AC = b, and BC = aand lets = (a+b+c)/2. Finda formula
for [ABC] in terms of s, a, b, and c.

Solution for Problem 6.22: We set up the problem in the same way as the last A
problem, drawing altitude AX and letting AX = h and BX = x,so XC =a—-x.
We apply the Pythagorean Theorem to AABX and AACX to get: f

Cth eee Co O
(a—-x +h = B X q C

Again, we can subtract the second equation from the first to find x:

xt= ea

Therefore, x* 2 — a? + 2ax — x* = c? — b, so 2ax = c2 — b? + a2, and we have

a2 +2 —b
y=
2a

OG ee
6.5%. HERON’S FORMULA

That wasn’t so bad, but finding h looks a little scary. We can make the algebra somewhat nicer by solving
x* + h* = c? for h* and factoring using the difference of squares:
We =e =x? =(c—x)(c +2).

Substituting our expression for x into this equation gives

h2 Bra gtel, eee — 0


2a 2a
2ac —a* —c*+b*\ (2ac+a2+c2-b
2a 2a ;

Now we recognize —(a — c)? = —a* + 2ac — c* and (a + c)* = a? + 2ac + c? in the numerators:
jit aes) (a+c)*—b
a 2a ( 2a )
B (Po eaahaeeal (Ciena se*a)
2a 2a
(b+c-—a)a+b-—c)at+c—b)(at+b+c)
4a?
(Make sure you see how we used the factorization y? — z? = (y — z)(y + z) to simplify the algebra.)

We can write h* even more briefly with the substitution s = (a+b+c)/2. We call this s the semiperime-
ter, i.e. one-half the perimeter. From this, we have 2s = a+b+c,sob+c-—a = 2s — 2a. Similarly, we can
substitute for all the terms in the numerator of our h expression:

h2 (2s — 2aj(zs— 2c)(25— 2))(2s) 48(s — @)(s'= b) (see)


a ile ana. 768 Cai eae tly pala ney ae ST Mie
Now that we have an expression for h, we can find the area:

[ABC] = ae = a eae = ,/s(s — a)(s — b)(s — 0).

It’s not always the nicest way to find the area of a triangle, but if we have the lengths of the three
sides, we can find the area with:

Aside from algebra, the only tool we used in deriving Heron’s formula was the Pythagorean Theorem.

159
CHAPTER 6. RIGHT TRIANGLES

Here are a few triangles on which to try Heron’s formula. You should compare your answers to the
areas you have found with other methods.

‘Problem 6.23: Find the area of each triangle below using Heron’s formula.
K
A D 15 rc H
13 14
6 6 P fe ye
7
B 6 C is G 10 I J 15 is

Solution for Problem 6.23: In each case, there are other ways to find the area of the triangle. See if you can
find the area using these other ways and compare your result to Heron’s formula.
AABC: s = (6+6+6)/2
= 9, so [ABC] = V99 — 6)9 — 6)9 — 6) = 9 V3.
ADEF: s = (8 + 15 +17)/2= 20, so [DEF]= -¥20(20 — 17)(20 — 15)(20 — 8) =
AGHI: s = (7 + 7 + 10)/2 = 12, so [GHI] = -12(12 — 10)(12 — 7)(12 — 7) = 10 fe
AJKL: s = (13 +144 15)/2 = 21, so [JKL] = V21@1—
13)@1 — 14 (21 — 15) = 84. o
Lest you start to think Heron’s is always the way to go, try finding the area of a triangle with sides
2 V3, 4, and 2 V7, ora triangle with sides V¥13, V¥13, and V6. Heron doesn’t look so pretty in either case,
but the answers can be found rather quickly with other methods. See if you can figure out how.

Exercises >

6.5.1 Use Heron’s Formula to find the areas of triangles with the following side lengths:

(ayo, o:
(b) 4,5,6.
(c) 12,35, 37.
(d) 6 V2,7 V2,9 v2.
6.5.2 In AGHI, GH = 5, HI = 7, GI = 8. Find the height of AGHI from G to HI.

Extra! Chinese mathematician Liu Hui based a proof of the


ithe Pythagorean Theorem on the diagram at right in the third
century A.D. (Of course, he probably didn’t call the result
the Pythagorean Theorem!) See if you can fill in the details!

160
6.6. CONSTRUCTION: PERPENDICULAR LINES

6.6 Construction: Perpendicular Lines

I]
Proviems il s
Problem 6.24: Construct a line through A that is perpendicular to line n. (Hint: See Problem 3.23.)

Problem 6.25: Suppose the shown segment has length 1. Construct a segment with length 10.
Pe—e Q

With all this talk about perpendicular lines and right triangles, we should wonder how to construct
perpendicular lines with a ruler and compass.

Problem 6.24: Given line 1 and point A, construct a line through A that is perpendicular to n.

Solution for Problem 6.24: Back on page 73, we constructed a very


specific perpendicular line given a segment. Maybe we can use
what we learned there to make a line through A perpendicular
to n. To do so, we'll have to find two points on n such that A is
directly ‘above’ the midpoint of the segment connecting the two
points. So, we draw a circle with center A that hits n in two points,
X and Y. Therefore, AX = AY.

We can then mimic our construction on page 73 by drawing


circles with centers X and Y and radius AY. Since AY = AX, these circles will meet at A. Let point B be the
other point where these circles meet. Since AX = AY, XB = YB, and AB = AB, we have AAXB = AAYB.
Therefore, MAX = ZMAY. Together with AX = AY and AM = AM, this gives us AMAX = AMAY by
SAS Congruence. Finally, we have ZXMA = ZYMA, so these angles are each 90° because together they
make up a straight angle.

Therefore, AB passes through A and is perpendicular to n. O


Now that we can create perpendiculars, we can use right triangles in our constructions. Let’s give it
a try.

Problem 6.25: Suppose the shown segment has length 1. Construct a segment with length V10.
P-—+Q

Extra! The mind is not a vessel to be filled, it is a fire to be kindled.


£8 1011 —Plutarch

161
CHAPTER 6. RIGHT TRIANGLES

Solution for Problem 6.25: Seeing the square root, we think of the Pythagorean Theorem, ie
since we’ve seen a lot of square root signs in our right triangle problems. Since 17437 = "10, ais
we want to make a right triangle with legs of length 1 and 3. The hypotenuse of this
triangle will have length V10. We start by extending PQ, then constructing aline through p\
Q perpendicular to a as described in Problem 6.24. Specifically, our steps for constructing y
the perpendicular line are: é

(a) Draw a circle with center Q and radius QP. Name the second point where this circle hits ze) point
xe

(b) Draw circles with centers P and X, both with radius PX. Name the points where these circles meet
Y and Z.

(c) Draw wa which is the line through Q perpendicular to PQ. (Make sure you see why this line must
go through Q.)

Once we have this perpendicular line, we create a segment of length 3 along it by,
drawing a circle with center and
Q radius QP to get point R on the perpendicular such that ,{—¢ Q
QR = 1. Then we can copy QR a couple times to get S and T such that RS = ST = 1, so ;
QT = 3. Segment PT is then the hypotenuse of a right triangle with legs of lengths 1 and 3, R
so it has length V1? +32 = 10. o |S
r

Exercises >

6.6.1 Construct a 45-45-90 triangle and a 30-60-90 triangle.


6.6.2 Given a segment of length 1, construct a segment of length 3.

6.7 Summary

Definitions: A right triangle is a triangle that has a right angle among its angles. The side opposite
| the right angle is called the hypotenuse and the other two sides are the legs.

_ Extra! The dazzling tiling ‘proof without words’ of the Pythagorean


mpi wap Theorem shown at right comes from Annairizi of Arabia (circa
— - 900 AD). See if you can figure out how it works! Source: Proofs
Without Words II by Roger Nelsen

162
SRN
SS a ES WPCAER SE OC I ee
|Important: In any right triangle, the sum of the squares of the A
| legs equals the square of the hypotenuse. Or, in the
figure to the right, we have b

a+b? =c*, C a5 B

This is the famous Pythagorean Theorem. The Pythagorean Theorem


also runs in reverse: If the sides of a triangle satisfy the Pythagorean |
Theorem, then the triangle must be a right triangle. The longest side
of such a triangle is the hypotenuse, and the angle opposite it is a right.
angle.

Two special right triangles, the 45-45-90 triangle and the 30-60-90 triangle, are shown below. The
relationships among the sides of each triangle are as indicated.

Conversely, if you determine that the side lengths of a triangle are in the ratio 1: 1: V2, you can
deduce that the triangle is a 45-45-90 triangle. If the side lengths are in the ratio 1 : V3 : 2, then the
triangle is a 30-60-90 triangle with the 30° angle opposite the shortest side.

|Important: We used the Pythagorean Theorem toprove ‘the following useful facts: |
| |

e The distance from a point to a line is the length of the segment


from the point to the line that is perpendicular to the line. |

e The area of an equilateral triangle with side length s is s? ¥3/4. If


you understand how we found this formula, you shouldn't need
to memorize it!

e Heron’s Formula tells us that if the sides of a triangle are a, b, and |


c, and we let s = (a+b +c)/2, then the area of the triangle is |

4/s(s — a)(s — b)(s — c).

|‘Definition: | A Pythagorean triple is a set of three integers that satisfy the Pythagorean Theorem.
;‘Some common abe sccubias Whe are: |

(5,12,19} (724,25) 18,15,17)_19,40,41).


163
CHAPTER 6. RIGHT TRIANGLES

Important:
In If. b,chaare the sides ofa2 right triangle, then so are {na, nb, nc} for any
WW positive number n. |
J

We also investigated two new congruence theorems and two new similarity theorems specifically
for right triangles:

e HL Congruence. If the hypotenuse and a leg of one right triangle equal those of another, then the
triangles are congruent.

e LL Congruence. If the legs of one right triangle equal those of another, then the triangles are
congruent. (This is the same as SAS Congruence.)
e HL Similarity. If the hypotenuse and a leg of one right triangle are in the same ratio as the
hypotenuse and leg of another right triangle, then the triangles are similar.

e LL Similarity. If the legs of one right triangle are in the same ratio as the legs of another right
triangle, then the triangles are similar. (This is the same as SAS Similarity.)

We used HL Congruence to prove the following useful fact:

Parana If a radius of a circle bisects a chord of the circle, it is perpendicular to the ||


Vv chord. Conversely, if a radius of a circle is perpendicular to the chord, it _
| bisects the chord. |

Problem Solving Strategies

:Concepts: e Building right triangles and applying the Pythagorean Theorem is |


C= : one of the most common ways to find lengths. This is particularly |
— _ true in problems that already involve right angles.

e Often it’s best to try a few examples before trying to derive a general
formula — the examples can be a good guide when you try to derive
your formula.

Things To Watch Out For!

“WARNING!!! | Be careful when using your knowledge of Pythagorean triples to make


ee Ma Ai
_ sure your given sides match corresponding sides (legs or ee
_ of the Pythagorean triple you would like to use.

Extra! In mathematics you don’t understand things. You just get used to them.
09 —~John von Neumann

ee
REVIEW PROBLEMS

REVIEW PROBLEMS a

6.26 Find the missing side length in each of the triangles below:

H
(a) | (b) Cee 42 i
3y2| 7!
l\ V17
a eSB G I J

6.27 Find the missing side length in each of the triangles below by using your knowledge of Pythagorean
triples:

6.28 Find the missing side lengths in the triangles below:

M P
(a) (b)

N O ane R

6.29 For each of the following parts, state whether or not the three numbers listed could be the side
lengths of a right triangle.

(a) 11,16, 19.


(b) 1/5, 4/15, 1/3.
(c) 73,2 2,9.
(d) v0.5, V1.2, v1.3.
(e) 0.77, 2.64, 2.75.
6.30 Given that AB = 3, BC = 6, and ZABC = 60°, why must AABC be a right triangle? (You cannot
simply say ‘It is a 30-60-90 triangle; you must prove that it is!)

165
CHAPTER 6. RIGHT TRIANGLES

6.31 In this problem, we find yet another proof of the Pythagorean Theorem.

(a) Construct a square of side length a + b, and place four right triangles with legs of lengths a and b
in it as shown in the left diagram. Let K be the area of each shaded triangle. Express the area of
the whole square in terms of K and the areas of the two interior squares.
(b) Now place the four triangles as shown in the right diagram and let c be the length of the hypotenuse
of each of the triangles. Express the area of the large square in terms of K and the area of the
interior square.
(c) What do you conclude from parts (a) and (b)?

6.32 In the figure at right, AB = BC = 1. AD || BC and CD || AE. Find AE. (Source: ‘up|
MATHCOUNTS)

6.33 Find the perimeter of an isosceles triangle with base length 10 and area 60.
6.34 Using any method you like, find the areas of the triangles with the side lengths
below:

(a) 6,8, 10.


(b) 3,4,6.
(c) v6, ¥24, ¥30.
(d) 1, 24/7, 25/7.
(e)= D26272

6.35 In the figure shown, E is the midpoint of AB, D is the midpoint of AC, AB = 16, A E Ww
and AC = 12. (Source: MATHCOUNTS)
(a) Show that AEFB ~ ACAB. D 5
(b) Find DG and EF. cc’ G
(c) What is the area of AEFGD?
6.36 Find the area of an equilateral triangle with a height of length 8.
6.37 In AMNO, MN = 13, and NO = 37. P is on MO such that NP . MO and NP = 12. Find the area
of AMNO. (Source: MATHCOUNTS)

166
CHALLENGE PROBLEMS

6.38 Let ABC be a right triangle, with ZACB = 90°. Let D be the foot of the perpendicular from C to
side AB. Let x = AD and y = BD. Prove that CD = xy.

A
C

D E

A XD y B Ee LAN v7
Figure 6.3: Diagram for Problem 6.38 Figure 6.4: Diagram for Problem 6.39

6.39 In the diagram at right above, AB = AC, AF 1 BC, FD 1 AB, and FE 1 AC. Prove that FD = FE.

6.40 In the diagram at right, BD = 6, AB = 3, and DE = 5. What is the length of AE? A


(Source: MATHCOUNTS)

6.41 The length of one leg of a right triangle is 22, and the other two sides also have
integer lengths. Find the perimeter of the triangle.
6.42 Point A is on side XZ of AXYZ such that XA = XY. Given that ZYXZ = 90°, E
LYZX = 30°, and ZA = 6 — V12, find the area of AXYZ.
6.43 The sides of a triangle have lengths of 15, 20, and 25. Find the length of the shortest altitude of
the triangle. (Source: AMC 10)

Challenge Problems >

6.44 A right triangle has area 210 and hypotenuse 29. Find the perimeter of the triangle. Hints:
Lipsay2
6.45 What's wrong with the diagram at left below? Hints: 409

+ 390 12 2 p& OC

Figure 6.5: Diagram for Problem 6.45 Figure 6.6: Diagram for Problem 6.46

6.46 Inrectangle ABCD, AD = 1, P is on AB, and DB and DP trisect ZADC. (An angle is trisected when
it is divided in three equal angles.) What is the perimeter of ABDP? (Source: AMC 10) Hints: 421
6.47 The hypotenuse of a right triangle has length 8. The triangle’s area is also 8. Find the perimeter
of the triangle. Hints: 16

167
CHAPTER 6. RIGHT TRIANGLES

6.48 In the diagram at right ZB = ZD = 90°, AB = DC = 24, and BC = AD = 32. B : D


(Source: ARML) ae

(a) Prove that AABE = ACDE. A G


(b) Let M be the midpoint of AC. Prove that EM 1 AC.
(c) Find the area of AAEC. Hints: 296

6.49 Triangle ABC has AB = 12, BC = 16, and AC = 20. If D is on AC such that AD = 12, find the area
of AADB. (Source: ARML) Hints: 116, 424

6.50 Riders ona Ferris wheel travel in a circle in a vertical plane. A particular wheel has radius 20 feet
and revolves at the constant rate of one revolution per minute. How many seconds does it take a rider
to travel from the bottom of the wheel to a point 10 vertical feet above the bottom? (Source: AMC 10)
Hints: 465

6.51 Shownat right is rectangle ABCD. Angle C is trisected by CF and CE, Dp aC


where E is on AB, F is on AD, BE = 6, and AF = 2. (Source: AMC 12)
(a) pHind BC. :
(pnd Or.
A L CI
B
(c) Find [ABCD]. E
6.52 Initially, a fifty-foot ladder rests against a wall. As I start to climb it, the ladder slides down,
finally stopping such that it touches the wall at a point 8 feet below where it originally touched the wall.
During the slide, the base of the ladder slid 16 feet from its original position. How far is the top of the
ladder from the ground after the slide, given that the wall is perpendicular to the ground? Hints: 137
6.53 Using only a pencil and paper, find the Pythagorean triple with 73 as the length of the hypotenuse.
Hints: 40, 423

6.54x If the lengths of the three sides of a right triangle are whole numbers, how many such distinct
noncongruent right triangles exist having one leg with a length of 24 units?
Goo SlvARoT, KS= 13,7571 = 14 and Rf = 15:

(a) Find the length of the height from R to ST.


(b)x Let M be the midpoint of ST. Find RM. Hints: 563
6.56% We begin with an equilateral triangle with side length 1. We divide each side into three segments
of equal length, and add an equilateral triangle to each side using the middle third as a base. We then
repeat this, to get a third figure.

If we continue this process forever, what is the area of the resulting figure? Hints: 13, 398

168
CHALLENGE PROBLEMS

6.57x Let AXOY bea right-angled triangle with zXOY = 90°. Let M and N be the midpoints of legs
OX and OY, respectively. Given that XN = 19 and YM = 22, find XY. (Source: AMC 10) Hints: 290,505

6.58% In triangle ABC, AB = AC, BC = V3 —1, and ZBAC = 30°. Find the length of AB. Hints: 384

6.59x In AABC, AB = 6 and BC = 8. Find AC in each of the following cases:

(a) 2B = 30°. Hints: 57


(b) ZB = 45°. Hints: 129
(cy 2B ei30°. Hints: 215

Extra! The great modern mathematician Paul Erdés was fond of describing particularly beau-
‘wim §tiful proofs as being from ‘the book.’ For sport, many aesthetically inclined math fans
find beautiful ‘proofs without words’ by using diagrams designed to ‘show’ the truth of
a mathematical statement rather than ‘saying’ it. In our various proofs without words of
the Pythagorean Theorem, we’ve seen proofs without words applied to the most natural
area of mathematics for them, geometry. However, the diagrams below show that the
tools of geometry can be used to ‘prove without words’ statements from other fields of
mathematics, as well. Perhaps you'll agree that these proofs are indeed “from the book.’

re

a
>}
&
+
+
+
+

Match the images above with the following mathematical statements:

2. Mn+ 1)2n +1)


017427437
+4 4+---40 Sry er

i () (2 Gens3 n(n
+ 1)
e1724+3+44--"+0= ae

0 1434547494-:-4+(2n-1)
=n’.

Make sure you see how each statement is illustrated by its matching image!
Source: Proofs Without Words by Roger Nelsen

EE eC CG lcCC;C:C:C:;C:;<;7C;73 }Ehtséi<s;=<; CSC


St” CS
CHAPTER 7. SPECIAL PARTS OF A TRIANGLE

The Nine-Point Circle

Why isn’t there a special name for the tops of your feet? — Lily Tomlin

CHAPTER

Seer Parts of a Triangle

Most of mastering geometry is mastering triangles. In this chapter we explore several special points,
lines, and circles that can be found in any triangle.
In general, any line segment from a vertex of a triangle to a point on the opposite side is called a
cevian. Before we dive into the special parts of a triangle, we’ll investigate a couple of special lines.

7.1 Bisectors

The perpendicular bisector of a line segment is the line passing through the midpoint of the segment
such that the line is perpendicular to the segment. In Figure 7.1, line k is the perpendicular bisector of
AB.

Figure 7.1: A Perpendicular Bisector

170
7.1. BISECTORS

The angle bisector of an angle is the ray that divides the angle into two equal angles. In Figure 7.2,
ray m is the angle bisector of ZAOB.

Mm

O —o—

Figure 7.2: An Angle Bisector


I)
Problems
ils
Problem 7.1: Line m is the perpendicular bisector of AB. Use congruent triangles to show that if C is
on m, then CA = CB.

Problem 7.2: In this problem, we will show that if AC = BC, then C is on the perpendicular bisector
of AB.
(a) Let C bea point such that AC = BC and let M be the midpoint of AB. Draw a diagram.
(b) Draw CM, then use congruent triangles to prove that CM 1 AB. Why does this mean that C is
on the perpendicular bisector of AB?

Problem 7.3: In this problem we show that any point on the angle bisector of ZXYZ is equidistant
from YX and YZ.
(a) Let C be on the angle bisector of ZXYZ. Draw ZXYZ and segments from C to the sides of ZXYZ
that are perpendicular to YX and YZ, respectively.
(b) Use congruent triangles to prove that the two segments from C to the sides of the angle are equal
in length. eS

Problem 7.4: C is a point inside XYZ such that C is equidistant from YZ


and YX as shown. Show that C must be on the angle bisector of £XYZ.

Na
CHAPTER 7. SPECIAL PARTS OF A TRIANGLE

Before we start tackling problems involving special lines in triangles, we will use what we already
know about congruent triangles to learn about these special lines. We'll start with perpendicular
bisectors.

Problem 7.1: Line m is the perpendicular bisector of AB. Show that if C is on m, then CA = CB.

m Solution for Problem 7.1: Drawing AC and CB, we see that AAMC = ABMC by
SAS Congruence since AM = MB, ZAMC = ZBMC, and CM = CM. Therefore,
AG= Cb ta
B Having shown that every point on the perpendicular bisector of a segment is
equidistant from from the endpoints of the segment, we should wonder if every
point that is equidistant from these endpoints has to be on the perpendicular
C bisector of the segment connecting them.

Problem 7.2: Show that if AC = BC, then C is on the perpendicular bisector of AB.

Solution for Problem 7.2: To show that C is on the perpendicular bisector of AB,
we connect C to the midpoint of AB, which we'll call M, then show that CM 1 AB.
Since M is the midpoint of AB, we have AM = MB. We are given AC = CB, and
obviously CM = CM, so we have AACM = ABCM by SSS Congruence. Therefore,
ZAMC = ZBMC. Since these two angles must add to 180°, they must each equal
BOO; EM is the perpendicular bisector of AB. (This proof doesn’t address the
possibility that C is the midpoint of AB. We take care of this by noting that the
midpoint of AB is on the perpendicular bisector of AB by definition.) 0
Putting these last two problems together tells us something very important about the perpendicular
bisector of a segment.

ise ctor ofa segment isthestraight line consisting of


re juidistant from the endpoints of the segment.

Solution for Problem 7.3: We start by drawing the lengths we need to show x
equal. These are the perpendicular segments CP and CQ from C to YX and
YZ as shown. By AAS Congruence, we have ACPY = ACQY, so CP = CQ. 5
Therefore, any point on the angle bisector of an angle is equidistant from C
the sides of the angle. 0
As with the perpendicular bisector, we can ‘run this backwards,’ deducing
that any point that is equidistant from the sides of an angle must be on the Orr
angle bisector.

ne
7.2. PERPENDICULAR BISECTORS OF A TRIANGLE

Problem 7.4: nisisaSpain eee LXYZ such that C is eer uaa an x


and YX as shown. Show that C must be on the angle bisector of ZX YZ.

Solution for Problem 7.4: We have ACPY = ACQY by HL Congruence because P Xx


CP = CQ and CY = CY. Therefore, CYP = ZCYQ, so YC bisects ZXYZ. 0 C

Putting the last two problems together tells us a little more about angle
bisectors. y
QZ

Important: The angle bisector ofan angle consists of all points that are equidistant
Vv from the sides of the angle. |

Sa
7.1.1 Line m is the perpendicular bisector of both AB and CD. Which of the following must be true and
why (or why not)?
(a) AB=CD.
(b) AB|| CD.
(c)x AC = BD. Hints: 460

7.1.2 Describe all the points that are equidistant from two intersecting lines.
7.1.3 Describe all the points that are equidistant from two parallel lines.

7.2 Perpendicular Bisectors of a Triangle

Problems >

Problem 7.5: Let k and ¢ be perpendicular bisectors of the sides AB and


AC of AABC as shown.
(a) From what two points must every point on k be equidistant?
(b) From what two points must every point on ¢ be equidistant?
(c) Let O be the intersection of k and . Why must O be on the perpen-
dicular bisector of BC?
(d) If we draw a circle with center O and radius OA, will the circle go
through B and C as well? Why or why not?

173
CHAPTER 7. SPECIAL PARTS OF A TRIANGLE

Problem 7.6: Line m is the perpendicular bisector of side XY of right x


triangle AXYZ, and m meets XZ at L as shown.

(a) Show that AXKL ~ AXYZ.


(b) Use part (a) to show that L is the midpoint of XZ.
(c) Where do the perpendicular bisectors of the sides of a right triangle Be Z
meet?
(d) What is the radius of the circle that passes through X, Y, and Z?

Problem 7.7: Ariel and Zappa are playing ‘NAME THAT CIRCLE.’ In ‘NAME THAT CIRCLE,’ Ariel
imagines a circle drawn on her sheet of paper. Zappa can then ask for points that are on the circle.
Zappa wins when he can draw the whole circle.
(a) Can Zappa win the game if Ariel only gives him two points?
(b) Can Zappa win if Ariel gives him three points?
(c) Zappa asks for four points. Ariel isn’t really paying attention and just *
picks out four points at random on the piece of paper and ends up with
points shown in the diagram at right. Zappa tries and tries, but he can’t
NAME THAT CIRCLE. Finally, he insists that there is no such circle. ° :
Could he possibly be right or should he just try harder?
Ariel’s points

Problem 7.8: In this problem we find the radius of a circle that passes through all three vertices of an
equilateral triangle. Let AABC be our triangle, with AB = 6. Let P be the midpoint of BC and Q the
midpoint of AC, and let O be the center of the circle through the vertices of AABC.
(a) Draw a diagram. Include the perpendicular bisectors of AC and BC. Where must these perpen-
dicular bisectors intersect?
(b) Prove that AOCP is a 30-60-90 right triangle.
(c) Find CP, then find the radius of the circle.

Now that we've learned a little bit about perpendicular bisectors, we're ready to study the perpen-
dicular bisectors of the sides of a triangle.
Problem 7.5: Show that the three perpendicular bisectors of the sides of a triangle are concurrent,
‘meaning they meet at a single point. Show that there is a circle centered at this point that passes
‘through all three vertices of the triangle.

Extra! The mathematician’s patterns, like those of the painter or the poet, must be beautiful; the ideas,
i like the colors or the words, must fit together in a harmonious way. There is no permanent place
in the world for ugly mathematics.
—G. H. Hardy

174
7.2. PERPENDICULAR BISECTORS OF A TRIANGLE

Solution for Problem 7.5; We start by thinking about just two of the perpen-
dicular bisectors, k and €, shown in the figure at left. Let k and € meet at O.
We wish to show that O is on the perpendicular bisector of BC.
Since O is on the perpendicular bisector of AB, it is equidistant from A
and B, i.e. OA = OB. Similarly, since O is on the perpendicular bisector of
AC, we have OA = OC. 00, GA = OB = OC, Since OB = OC, O must be
on the perpendicular bisector of BC (see Problem 7.2), so the perpendicular
bisectors of the sides of a triangle are concurrent.
Seeing OA = OB = OC, we note that if we draw a circle with center O and radius OA, the circle will
pass through B and C as well. O
The point O and the circle described in the last problem are so useful in geometry problems that they
have their own names.

The perpendicular bisectors of the sides of a


triangle are concurrent at a point called the
circumcenter. The circle centered at the cir-
cumcenter that passes through the vertices
of the original triangle is called the circum-
circle of the triangle because it is circum-
scribed about the triangle (meaning it passes
through all the vertices of the triangle).
Finally, the radius of this circle is called
the circumradius, the circumcenter is usu-
ally labeled with the letter O, and the circumradius is usually called R.

In right triangles, the circumcircle is particularly easy to describe.

Problem 7.6: Where is the circumcenter of a right triangle? What is the circumradius of a right
triangle?

Solution for Problem 7.6: If we draw a few right triangles and their per- x
pendicular bisectors, we will begin to suspect that the midpoint of the
hypotenuse is the circumcenter. m L
We start our proof by drawing the perpendicular bisector of one of the
legs, suchas line m in the diagram. Let m meet XZ atL. Since mand YZ are
both perpendicular to XY, we have m || YZ. Therefore, AXKL ~ AXYZ, ¥ ott Zi
so XL/XZ = XK/XY. Since XK/XY = 1/2, we have XL/XZ = 1/2, so L is the midpoint of XZ. Clearly,
L is also on the perpendicular bisector of XZ (since L is the midpoint of XZ), so L must be where the
perpendicular bisectors of AXYZ intersect. Therefore, L is the circumcenter of AXYZ.
The endpoints of the hypotenuse are vertices of the triangle, so by definition they are on the circum-
circle. Since the midpoint of the hypotenuse of our right triangle is the circumcenter, the hypotenuse is
a diameter of the circumcircle. Therefore, the circumradius is half the length of the hypotenuse of the
right triangle. 0

175
CHAPTER 7. SPECIAL PARTS OF A TRIANGLE

“Important: The circumcenter of aright triangle is the midpoint |


WW, of the hypotenuse, and the circumradius equals |
one-half the length of the hypotenuse.

ie: a
t SS Ss BSS

Back in Problem 1.1, we saw that two points are enough to define a line. All this talk about circles
going through the vertices of a triangle should make us wonder — how many points of a circle do we
need to define a circle? Clearly, one won’t do, since we can draw all sorts of circles through a given
point.

Problem 7.7: Ariel and Zappa are playing ‘NAME THAT CIRCLE.’ In ‘NAME THAT CIRCLE,’
Ariel imagines a circle drawn on her sheet of paper. Zappa can then ask for points that are on the
circle. Zappa wins when he can draw the whole circle.
(a) Can Zappa win the game if Ariel only gives him two points?
(b) Can Zappa win if Ariel gives him three points?
(c) Zappa asks for four points. Ariel isn’t really paying attention and just picks out four points
at random on the piece of paper. Zappa tries and tries, but he can’t NAME THAT CIRCLE.
Finally, he insists that there is no such circle. Could he possibly be right or should he just try
harder?

Solution for Problem 7.7:


(a) Clearly two points isn’t enough — we can make all sorts of different circles given
two points. This is shown at right.
(b) Given three points, Zappa can NAME THAT CIRCLE! He can connect the points A B
to make a triangle. Then, he can draw the perpendicular bisectors of two of the
sides of the triangle. Where they meet is the circumcenter. He can then make the
circumcircle. Notice that the three points must be noncollinear, meaning that
it’s impossible to draw a single line that goes contains all three points.
The center of any circle that passes through all three points must be on all
three perpendicular bisectors of the sides of the triangle with these points as vertices. Therefore,
the circumcenter is the only possible center of a circle through the three points. Therefore, the
circumcircle is the only circle that passes through all three points.
(c) Let the points be A, B, C, and D. As described in the previous part, G *i)
there is only one circle through A, B, and C. But this circle doesn’t have
to go through D! Therefore, if we pick out four points at random, there
doesn’t have to be a circle that passes through all four points. Zappa B A
should give up once he sees that the circumcircle of AABC doesn’t pass
through D. One such arrangement of points is shown at right.

176
7.3. ANGLE BISECTORS OF A TRIANGLE

Important: Just as two points determine a line, we have now shown that three
noncollinear points determine a circle. This means that given any three
noncollinear points, there is exactly one circle that passes through all |
three.
|
Now let’s try a problem involving the circumradius.

‘Problem 7.8: Find thecircumradius of an equilateral triangle with side length6.


Solution for Problem 7.8: Let AABC be our equilateral triangle. We connect its
circumcenter to a vertex and draw a couple perpendicular bisectors as shown.
Since CQ = CP and OC = OC, we know that AOCP = AOCQ by HL Congruence.
Therefore, ZOCQ = ZOCP, so OC must bisect ZACB. Hence, ZOCP = ZACB/2 = 30°
and AOCP is a 30-60-90 triangle. Since P is the midpoint of BC, we have CP = 3, so

oc = cP (= = =2N8.
O

[|
Exercises
7.2.1 Where is the circumcenter of an acute triangle — in, on, or outside the triangle? How about an
obtuse triangle? (You do not need to prove your answer — you'll learn tools later that will make these
proofs easier.)

7.2.2 Find the radius of the shown circle with center O given that BC = 8andQO=2. A

7.2.3. Find the circumradius of an equilateral triangle with side length 18.
7.2.4 What is the circumradius of a right triangle with legs of length 6 and 8?
ey

7.3 Angle Bisectors of a Triangle

q)__Probiems
ile
Problem 7.9: Let k be the angle bisector of ZCAB and m be the angle bisector of
ZABC. Let k and m meet at I.
(a) Every point on k must be equidistant from what two lines?
(b) Every point on m must be equidistant from what two lines?
_(c) Why must Ibe on the angle bisector of £C?
CHAPTER 7. SPECIAL PARTS OF A TRIANGLE

Problem 7.10: The angle bisectors of AXYZ meet at I, and perpendicular


segments from I to the sides of the triangle are drawn as shown.
(a) Why is IR = IQ = IP?
(b) Suppose we draw a circle with center I and radius IP. Does the circle
hit YZ at any point besides P?
(c) Show that the circle from (b) is tangent to sides XZ and XY at Q and
R, respectively.

Problem 7.11: Segments GH, HI, and IG are tangent to the circle with center C as
shown. Given that ZGHI = 70° and /GIH = 50°, find the following:
(a) ZCIH.
(b) ZHCI.

Problem 7.12: In the diagram, BE is the angle bisector of ZB. In this problem, B
we will prove the Angle Bisector Theorem, which states that if BE is an angle
bisector as shown, then AB/AE = CB/CE.

(a) Ratios make us think of similar triangles, which make us think of 4 C


_ parallel lines. Draw a line through C parallel to AB, then extend BE 3
past E to hit this new line at X. What triangles are similar?
(b) What type of triangle is ABCX?
(c) Use your similar triangles from (a) and your observation in (b) to prove that AB/AE = CB/CE.

Problem 7.13: Find AC and XQ in the triangles shown. (XZ = 12)

Problem 7.14: Back in Problem 7.10, we discovered a circle that is tangent


to all three sides of AXYZ. In this problem, we will discover a formula for
the area of AXYZ in terms of the radius of this circle and the perimeter of
the triangle.

(a) Break AXYZ into three triangles: AXYI, AYZI, and AZXI. Find the
area of each triangle in terms of one of the sides of AXYZ and 1, the
radius of the circle in the diagram.
(b) Find the desired formula for [XYZ] by adding [XYI], [YZI], and [ZXI].

178
7.3. ANGLE BISECTORS OF A TRIANGLE

Problem 7.15: In this problem we find the radius of a circle that is tangent to all three sides of AABC
given that the sides of AABC have lengths 7, 24, and 25.
(a) What kind of triangle is AABC? What are the area and perimeter of AABC?
(b) What is the radius of the circle tangent to all three sides of AABC?

Having found that the perpendicular bisectors of the sides of a triangle are concurrent, we wonder
if a similar proof can be used to show that the bisectors of the angles of a ely are also concurrent.

Problem 7.9: Prove that the angle bisectors of a1 triangle arare concurrent.

Solution for Problem 7.9: We start much as we did with the perpendicular bisectors. C
We consider the intersection of two of our angle bisectors, point I in the diagram.
Since I is on the bisector of ZCAB, it is equidistant from AC and AB. Since it is on the ie k
angle bisector of ZABC, it is equidistant from AB and BC. Therefore, I is equidistant
from all three sides of the triangle. Specifically, since I is equidistant from AC and
BC, I is also on the angle bisector of ZACB, as we proved in Problem 7.4. 0 A B

We found a special circle centered at the intersection of the perpendicular bisectors. Let’s see if the
intersection of the angle bisectors is the center of a special circle, too!

Problem 7.10: The angle bisectors of AXYZ meet at I, and perpendicular


segments from I to the sides of the triangle are drawn as shown.
(a) Why is IR =1Q = IP?
(b) Suppose we draw a circle with center J and radius IP. Does the circle
hit YZ at any point besides P?
(c) Show that the circle from (b) is tangent to sides XZ and XY at Q and
R, respectively.

Solution for Problem 7.10:


(a) In the previous problem, we found that I, the intersection of the angle bisectors of AXYZ, is
equidistant from the sides of AXYZ. Since IR, IQ, and IP are the distances from I to the sides of
AXYZ, we have IR = IQ = IP.
(b) One way we can show that the circle with center J and radius IP doesn’t hit YZ anywhere else is
to show that all the other points on YZ are outside the circle.
As shown, point V is a point on YZ besides point P. From right Xx
triangle AIPV, we have IV?= IP? + VP?, so IV > IP. Our circle consists
of all points exactly IP from I. Since V is farther than IP from I, it must R
be outside our circle. There’s nothing special about V; all points on YZ Q
besides P are outside our circle. Therefore, it is impossible for the circle
with radius IP to hit YZ at a second point. So, the circle must be tangent y = VA
to YZ.

179
CHAPTER 7. SPECIAL PARTS OF A TRIANGLE

(c) Since IP = IQ = IR, the circle with center I and radius IP goes through Q and R, too. For the same
reason as in the previous part, this circle is tangent to all three sides of AXYZ.

We have found the center of a special circle at the intersection of the angle bisectors of a triangle.

“Important: The angle bisectors of a triangle are concur- x


VY rent at a point called the incenter. This point
is equidistant from the sides of the triangle.
This common distance from the incenter to ae .
the sides of a triangle is called the inradius, ve <
because the circle with center] and thisradius y ped, tN Le
is tangent to all three sides of the triangle. This
circle is unsurprisingly called the incircle because itis inscribed in the tri-
angle (meaning it is tangent to all the sides of the triangle). The incenter
is usually denoted I, and the inradius is usually written as r. |

Each triangle has exactly one incircle. (You'll be asked to prove this in Problem 12.43.) Therefore, if
a circle is tangent to all three sides of a triangle, its center is the intersection of the angle bisectors of the
triangle.

Problem 7.11: Segments GH, HI, and IG are tangent to the circle with center C as
shown. Given that ZGHI = 70° and ZGIH = 50°, find ZHCI.

Solution for Problem 7.11: Since the circleistangent to all three sides of AGH, it is the incircle of AGHI.
Therefore, C is the incenter of AGHI, so CI and CH are bisectors of angles ZGIH and ZGHI, respectively.
ZCHI = (4GHI)/2 = 35° and 2CIH = (ZGIH)/2 = 25°, so we have ZHCI = 180° — 35° — 25° = 120°. of
The incenter and the incircle aren’t the only useful aspects of angle bisectors.

Problem 7.12: Given that E is on AC such that BE is the bisector of ZABC, prove that AB/AE = CB/CE.

Solution for Problem 7.12: Seeing ratios, we think of similar triangles. Similar B
triangles make us think of parallel lines. We draw a line through C parallel to
AB and we extend BE past E to hit this new line at X. We pick this parallel
line to draw because it lets us use the angles that are formed by the angle bisector.
Specifically, we have 4BXC = ZABX. ZABX also equals ZXBC, so AXCB is isosceles 5 &
and CB = CX.
Our parallel line gives us similar triangles: AABE ~ ACXE. This similarity
gives us AB/AE = CX/CE. Since CB = CX, we have the desired AB/AE = CB/CE.
O

180
7.3. ANGLE BISECTORS OF A TRIANGLE

Important: The> Angle Bisector Theorem states thatiffis . B. . 4


Vv on AC such that BE is the angle bisector of ZB in |
triangle AABC, then

|
AB _ CB
PV SRR alsh
A Saycne Fa
| We should think of this any time we have a problem involving lengths
and angle DiseciOre.

For example, try using the Angle Bisector Theorem on thises,

‘Problem 7.13: Find AC and XQ in the diagram. (XZ=12) x


|
|

| 9

Solution for Problem 7.13: From the Angle Bisector Theorem applied to AABC, we have AB/BD = AC/CD,
so 12/6 = AC/7 and AC = 14.

Finding XQ is a little more challenging. We let XQ = x,so QZ = 12—x. The Angle Bisector Theorem
tells us
106 WAS
KOMZO"
so we have
BALLS J
abi ore
Cross-multiplying gives 9(12 — x) = 8x, so x = 108/17. Therefore, XQ = x = 108/17. Note that
XQ = (YX)(XZ)/(YX + YZ). Is this a coincidence? O

But wait, there’s more. We can even relate an incircle to the area of its triangle!

Problem 7.14: Find the area of AXYZ in terms of its inradius and its side lengths.

Solution for Problem 7.14: Those radii of the incircle perpendicular to the sides x
make us think of altitudes. Indeed, if we break AXYZ into three triangles
with I as a vertex, these radii are altitudes. Adding the areas of these three R
triangles gives us [XYZ]: Q

[XYZ] = [XY] + [YZ] + [XZ]] =


(IR(XY) | EP)YZ) | UQ)XZ)
2 2 2 ¥ P Z
Since IR = IP = IQ =r, the inradius, and XY + YZ + XZ = 2s, where s is the semiperimeter of AXYZ, we
have:
r(XY) : r(YZ) F XZ) : cs AY 7. J a
[XYZ] =
2 2 Dias, 2 -

181
CHAPTER 7. SPECIAL PARTS OF A TRIANGLE

Important: The area of a triangle equals its inradius times its semiperimeter. |

This [ABC] = rs can be a useful tool in problems involving the area of a triangle or the inradius (or
both).

‘Problem 7.15: of a circle that is tangent to all three sides of AABC given that the
Find the radius
sides of AABC have lengths 7, 24, and 25.

Solution for Problem 7.15: Since 7* + 24* = 257, the triangle is a right triangle with legs of length 7 and 24.
(This problem is an example of why it’s useful to recognize Pythagorean triples.) So, we can easily find
[ABC] = (7)(24)/2 = 84. Since we can also find the semiperimeter, we can find the inradius. Because
s =(7+24+4 25)/2 = 28, we have
_ [ABC] _ 84
s = one

Exercises -

7.3.1 AB, AC, and BC are tangent to OO. If ZA = 70°, ZB = 72°, and ZC = 38°, what is B
the measure of ZOAB? Hints: 228

7.3.2 Is the incenter of an obtuse triangle inside, outside, or on the triangle?

7.3.3 Can the incenter and the circumcenter of a triangle ever be the same point?

7.3.4 PS and RT are angle bisectors of APQR, and ZPIR = 130°. Ly


(a) Given ZRPQ = 30°, find ZPRT, ZPRQ, and ZQ. 2
(b) Given ZRPQ = 50°, find ZPRT, ZPRQ, and ZQ. R
(c) Given ZRPQ = 80°, find ZPRT, ZPRQ, and ZQ. Q S
(d) Do you notice anything unusual in your answers for ZQ? Why does this unusual pattern occur?

7.3.5 Find RZ in the figure at right if PR = 9, QZ = 4, PQ = 6, and PZ bisects ZOPR. i


7.3.6 In AABC, AB = 10, AC = 12, and BC = 8. Point M is on side BC such that
ZBAM = CAM. Find BM.
7.3.7 For each part below, find the length of the inradius of a triangle with the given
numbers as side lengths. Z R

(a) 3,4,5
(b) 6,6, 6
(cy /7 7,10
(cl) 5Or6, 7

182
7.4. MEDIANS

7.4 Medians

A median of a triangle is a segment from a vertex to the midpoint of the opposite side. In the figure
below, AD, BE, and CF are all medians.

Eee <a | a
Important: The medians of a triangle are concurrent at a C
Vv point called the centroid of the triangle. The
centroid of the triangle is usually labeled G.

A E B |
}
bE

In each of the problems in this section, you can assume the medians of a triangle are concurrent.
We'll prove that the medians of a triangle are concurrent in Chapter 17. You'll also have a chance to do
it yourself in the Challenge Problems. (Hint: We suggest re-reading Section 4.3.)

|_Proviems ite
Problem 7.16: In this problem we will prove that the medians of a trian-
gle divide the triangle into 6 triangles of equal area. We will make heavy
use of the same base/same altitude principle we studied in Section 4.3.

(a) Let [DGB] = x, [CGE] = y, and [AGF] = zas shown in the diagram.
Prove that [DGC] = x, [AGE] = y, and [BGF] = z.
(b) Show that [ABD] = [ACD] and use this to prove that y = z.
(c) Prove that the six triangles formed by drawing the medians of
AABC have equal area.

Problem 7.17: Use triangles AAGB and ADGB in the previous problem to prove that the centroid cuts
each median in a 2 : 1 ratio, specifically AG/GD = 2. .....-—s—s=sidi“césss

Problem 7.18: APQR is an equilateral triangle with sidelengthis 12 .0


also a median. _ | :
lay. Show that each angle bisector of an equilateral triangle
of APQR. LO
_&) Find the length of the inradius

Problem 7.19: Points D, E, and F are the midpoints of the sides of AABC .
as shown below. |
/ (a) Show that ABDF ~ ABCA.

Pe ee
ow that each ofthe four small triangles in the diagram has area equal to 1/4 the area ofAABC.

183
CHAPTER 7. SPECIAL PARTS OF A TRIANGLE

Problem 7.20: AABC has sides of length AB = 9, BC = 12, and CA = 15. Find the length of the median
from B to the midpoint of AC. Hints: 229, 537

Problem 7.21: In AJKL, JK = JL = 10 and KL = 16. J


(a) Find the length of median JM. N
(b) Let medians JM and KN meet at G. Find KG. K L
(c) Find the length of median KN.

Medians create triangles with equal bases; therefore, they create triangles with equal areas. We can
use these equal areas to learn even more about the medians of a triangle.

Problem 7.16: Prove that the medians of a triangle divide the triangle into six triangles with equal
area.

Solution for Problem 7.16: Let our triangle be AABC with midpoints D, E, G
and F, and with centroid G, as shown. We have [BGD] = [CGD] because
these triangles have the same altitude from G to equal bases BD and CD.
Similarly, [AGE] = [CGE] and [AFG] = [BFG]. We assign variables to these D
areas as shown in the diagram at right.

The little triangles aren’t the only triangles that share altitudes. Since POO
AABD and AACD have the same altitude from A to equal bases BD and, B
DC, we have [ABD] = [ACD]. Therefore, x+2z = x+ 2y, so y = Z. Similarly, F
we can use [AFC] = [BFC] to show that x = y, so we have x = y = z and all six little triangles have the
same area. 0

“Important: —_ The medians of a triangle divide the triangle into six little triangles of
VY equal area

We can use this property of medians to discover an interesting property of the centroid.

Problem 7.17: Prove that the centroid of any triangle cuts each of the triangle’s median in a 2 : 1
ratio, with the longer portion being the segment from the centroid to the vertex.

Solution for Problem 7.17: Our work with areas gives us the solution right away. &
Since AAGB consists of two of the little equal-area triangles and AGDB is only
one of them, we have [AGB]= 2[GDB]. These two triangles share an altitude
from B to bases AG and GD, so Ec D

AG _ [AGB] _
GDM IGD. aes A : B
Clearly this works for any median. 0

184
7.4. MEDIANS

Important: The centroid ofa triangle cuts its medians into a2 : 1 C


NW ratio. For example, for the triangle shown, we have
D
Bn OG tee @
ie SAYoat heCFSN ve B

This simple fact can be useful in a variety of problems. Here’s an example.

Problem 7.18: APQR is an equilateral triangle with side length 12. Find the length of theinradius
of
APQR.

hs Solution for Problem 7.18: Let PL and RN be angle bisectors of APQR. Since the
angle bisectors of an equilateral triangle are also the medians of the triangle, L is
the midpoint of RQ and the incenter, I, is also the centroid. So, I divides median
PL such that PI/IL = 2/1. Therefore, IL = PL/3.

APLR is a 30-60-90 triangle, so RL = PR/2 = 6 and PL = RL Y3 = 6 V3. Finally,


R L Q we have IL = PL/3 = 2 V3. (Note: We could also have solved this problem by
noting that AIRL is a 30-60-90 triangle.)O
Instead of connecting the midpoints to the opposite vertices, suppose we connect them to each other,
as shown in Figure 7.3. ADEF is called the medial triangle of AABC.
Cc

A r B

Figure 7.3: Medial Triangle

Problem 7.19: Prove that the four small triangles in Figure 7.3 are congruent, and that each is similar
to AABC.

Solution for Problem 7.19: AFAE and ABAC have an angle in common, but we don’t know anything
about the other angles of the triangles. So, we look to SAS Similarity to prove similarity. Since
AF/AB = AE/AC = 1/2, we have AFAE ~ ABAC by SAS Similarity. Similarly, we canshow AFBD ~ AABC
and ACED ~ ACAB.
Since each of the small triangles above have lengths that are 1/2 the sides of AABC, we have
PEP DE DE)
BeAr AC 2.
Therefore, ADEF ~ AABC by SSS Similarity.
Since all four little triangles are similar to AABC with the same side ratio, the four little triangles
must be congruent. 0

185
CHAPTER 7. SPECIAL PARTS OF A TRIANGLE

Notice that we can also use AAFE ~ AABC to show that ZAFE = ZABC, from which we have FE || BC.
Similarly we can show that the other two sides of the medial triangle are parallel to their respective sides
of the original triangle.
Ee Sa we

Important: Given that ADEF is. the medial triangle of AABC as shown below,w
Vv have: 3
C

A r B

I. ADEF ~ AABC, ADEF = AFBD = AAFE = AEDC.

EF _DE_DF1
BC. AR AC
Ill. DF ||AC, EF ||BC, and DE ||AB.

Together, facts II and ul are sometimes called the Midline Theorem.

Let’s try a couple problems using the properties of medians we have just found.

Problem 7.20: AABC has sides of length AB = 9, BC = 12, and CA = 15. Find the length of the
median from B to the midpoint of AC.

Solution for Problem 7.20: First we notice that AABC has side lengths in the B
ratio 3 : 4 : 5, so it is a right triangle with hypotenuse AC. Therefore, S
we are looking for the length of the median to the hypotenuse of a right
triangle. Since the midpoint of the hypotenuse of a right triangle is also
the circumcenter of the right triangle (see Problem 7.6), the median to the
midpoint of the hypotenuse is a radius of the circumcircle as shown in the
diagram. Therefore, the length of our median equals the circumradius,
which is half the hypotenuse. So, our answer is CA/2 = 15/2. 0

Concept: When given the lengths of the sides of a triangle iinaa problem, always take
O= the time to check if itis a right triangle. Special Peper of right tenes
often aot problems.

: Important: The median to the hypotenuse of a right triangle is ¢ jual in length to.
Y half the eee

186
7.4. MEDIANS

16. alas - a
e
Problem 7.21: In AJKL, JK = JL m (Gh sad
(a) Find the length of median JM. N
(b) Find the length of median KN. a :

Solution for Problem 7.21:

(a) Since JK = JL, median JM is also an altitude (since AJKM = AJLM). Therefore, AJMK is a right
triangle with hypotenuse JK = 10 and one leg of length KM = 8. So, JM = 6.
(b) Let JM and KN meetat G, which is the centroid of AJKL. It’s not obvious J
how to get KN, but we do have right triangle AKGM, which we can use N
to find KG. Since G is the centroid and JM is a median, GM = JM/3 = 2.
Therefore, KG = VGM2 + KV2 = V4 + 64 = 2 V17. Since KN is a median K L
and G the centroid, KG/KN = 2/3. Therefore, KN = (3/2)KG = 3 v17. M

Exercises i
7.4.1 Dand E are the midpoints of sides BC and AC of AABC in the diagram at A
right. AD and BE meet at G, AG = 9, and GE = 4. Find GD and BG.
B
7.4.2 Right triangle APQR has PQ = 5, QR = 12, and RP = 13. The midpoints of
OR, PR, and PQ are A, B, and C, respectively. ies C

(a) Draw adiagram. Mark the right angle. B D


(b) Find the length of median QB.
(c) Find the lengths of medians PA and RC.

7.4.3 The three medians of right triangle AABC intersect at G as shown. C


Given that AC = 8 and AB = 15, find the following:

(a) [ABC]. : D
(b)a[AFCh
(CAC 7 . .
(d) [AEG].
(e) [EGDC].
(f) [AFGE].
(g) [DEF].
(h) [BED].
(i)* CG. Hints: 403
7.4.4x Medians XA and YB of AXYZ have the same length. Prove that XZ = YZ. Hints: 167,375

ees
- « . == © ez
CHAPTER 7. SPECIAL PARTS OF A TRIANGLE

7.5 Altitudes

We’ve already seen altitudes in our investigation of triangle area. You can probably guess by now what
we'll prove first about altitudes.

__Proviems ia
Problem 7.22: In this problem we will prove that the lines containing the altitudes of any triangle are
concurrent. This is a pretty slick proof. Before trying it, you might want to review what we learned
about the medial triangle on page 186.
(a) Our diagram shows altitudes AD, BE, and CF of AABC. Draw a line ®
through A parallel to BC, a line through B parallel to AC, and a line
through C parallel to AB. The intersections of these three lines form
another triangle. Call this triangle AJKL, with A on KL and B on JL. D
(b) Show that ACAK = AACB.
(c) Use part (b) and similar congruences to prove that A, B, and C are
the midpoints of KL, JL, and JK, respectively.
(d) How are lines AB, BE, and CF related to A JKL?
Prove that the altitudes of AABC are concurrent.

Problem 7.23: Where do altitudes of a right triangle intersect — inside, outside, or on the triangle?
How about an acute triangle? An obtuse triangle? (If necessary, extend the altitudes to the point
where they all meet.)

Problem 7.24: Altitudes OZ and XP intersect at N as shown. Given that


ZYXZ = 70° and £XZY = 45°, find each of the following:

(a) ZZXP.
_)) 20
| (c) ZYXP.
| (d)x ZNYZ. Hints: 194

Problem 7.25: The altitudes of AABC meet at point H. At what point do the altitudes of AABH meet?
How about the altitudes of AACH? ABCH? (As usual, extend the altitudes if necessary.)

We start, as you probably guessed, by proving that the altitudes of a triangle are concurrent. This
is a bit of a magical proof, and it uses what we have already learned about perpendicular bisectors and
the medial triangle.
Problem 7.22: Prove that the lines containing the altitudes of a triangle are concurrent.

Solution for Problem 7.22: Our altitudes make us think of perpendicular lines. Concurrent perpendicular

TERee ees!
7.5. ALTITUDES

lines make us think of perpendicular bisectors. Unfortunately, there’s no simple solution like we had
with perpendicular bisectors. So, we put our wishful thinking hat on, and wonder if we could possibly
show that the altitudes of our triangle are also the perpendicular bisectors of the sides of some other
triangle. If so, we would know they are concurrent, since the perpendicular bisectors of our ‘other
triangle’ must be concurrent. But what's our ‘other triangle’?
We want our A, B, and C to be the midpoints of the sides of our other triangle (since then the altitudes
of AABC would be perpendicular bisectors of the sides of the other triangle). This means we want AABC
to be the medial triangle of some other triangle. Since we know that the sides of a triangle are parallel
to the sides of its medial triangle, we know how to make a triangle starting with its medial triangle.

We draw lines IK, Ki, and TL through the vertices of AABC parallel
to the sides of AABC as shown. To prove that AABC is indeed the
medial triangle of AJKL, we must show that the vertices of AABC are
the midpoints of the sides of AJKL. Since AB || JK, ZCAB = ZACK.
Similarly, ZCAK = ZBCA,so ACAK = AACB. Likewise, ABLA = AACB,
so ABLA = ACAK. Therefore, AL = AK, so A is the midpoint of DR

In the same way, we find that B and C are the midpoints of LJ


and JK, respectively. Therefore, AABC is the medial triangle of AJKL.
Since KL || BC, altitude AD of AABC is perpendicular to KL. Since
AD is perpendicular to KL and passes through its midpoint, it is the
perpendicular bisector of KL. Similarly, CF and BE are also perpendicular bisectors of the sides of AJKL.
Lines AD, BE, and CF are concurrent because they are the perpendicular bisectors of the sides of AJKL.
These lines also contain the altitudes of AABC, so we have proved that the lines containing the altitudes
of AABC are concurrent. 0

Important: The altitudes of any triangle are concurrent at a point called the ortho-
NW center. We usually denote the orthocenter with the letter H.

To get a feel for the orthocenter, we look for it in a few different types of triangles.
Problem 7.23: Where do altitudes of a right triangle intersect— inside, outside, or on the triangle?
How about an acute triangle? An obtuse triangle?

189
CHAPTER 7. SPECIAL PARTS OF A TRIANGLE

Solution for Problem 7.23: We start with right triangle AABC with right angle at B. Point B is the foot of
of a right
the altitude from A to BC and the foot of the altitude from C to AB; therefore, the orthocenter
triangle is the vertex of the right angle.

In acute ADEF, we see that all the altitudes go inside the triangle, so the orthocenter is inside the
triangle.
In obtuse APOR, the altitudes from the two acute angles are entirely outside the triangle (except
where they pass through a vertex). Therefore, the orthocenter of an obtuse triangle is outside the
triangle. 0
Let’s try using the orthocenter in a problem.

Problem 7.24: Altitudes QZ and XP of AXYZ intersect at N. Given that Xx


ZYXZ = 70° and ZXZY = 45°, find the following:
(a) ZZXP. Q 7
(b) 2X20: va
(c) ZY XP. Y 2 7.
(d)x ZNYZ.

Solution for Problem 7.24:


(a) From right triangle AXPZ, we have ZZXP = 90° — ZXZP = 45°.
(b) From right triangle AXZQ, we have 2XZQ = 90° — ZZXQ = 90° — ZZXY = 20°.
(c) We could use either right triangle AXYP, or we could use ZZXP and ZYXZ:

ee SN Lee 7) ee ee
(d) Since two altitudes meet at N, we know that N is the orthocenter. Therefore, Xe
if we continue YN to meet XZ at R, we know that YR is also an altitude. R
From right triangle ARYZ, we find that ZNYZ = 90° — ZRZY = 45°. Q
Oo

vy P Z,
Notice that recognizing the orthocenter was a key step in the solution to our last part. This is just
one small example of how the awareness of the special points, lines, and circles of a triangle can help.

190
SS
SS sss SSS SSS SSS SSS SSS ss
7.5. ALTITUDES
sss

Concept: If we have the altitudes from two vertices of a triangle, then we know that |
sp the line through the intersection point of these two altitudes and the third
vertex is also an altitude. Once you have two altitudes, you should almost |
always think about this third altitude. |
This, of course, also works with our other special lines — if you have |
two medians, you have the third; if you have two angle bisectors, you —
have the third, etc. With these other special lines you also get a little more,
since we have special knowledge about the centroid (the 2 : 1 ratio), the
incenter (center of the inscribed circle), and the circumcenter (center of the
circumscribed circle). |
Again, if you have two of any of these special lines, you should consider |
the thind lineand remember what you know about Ms intersection point

Here’s an interesting tidbit about altitudes and orthocenters:

Problem 7.25: The altitudes of AABC meet at point H. At what point do the altitudes of AABH meet? |
How about AACH? ABCH?

Solution for Problem 7.25: Our diagram shows that the altitudes of AABH are HZ Cc
(perpendicular to side AB), AC (perpendicular to BH), and BC (perpendicular
to AH). These lines clearly all pass through C, so C is the orthocenter of AABH. Y
Similarly, B is the orthocenter of AAHC and A is the orthocenter of ABHC. 0 xX

And here’s an interesting fact about how the orthocenter, circumcenter, and
centroid of a triangle are related: A 7 B

Sidenote: In any triangle, the centroid, G, isonyn the line segment connecting the ortho-
ay center, H, to the circumcenter, O, such that 20G = GH. This line through ©|
these three points is called the Euler line, after the great mathematician —
Leonhard Euler. One such line is shown below.

Euler was one of the most prolific mathematicians ever, producing no-
table work in nearly every field of mathematics, and creating new fields
himself. The Euler line is just one of many mathematical results that have
been named after Euler. His impact on mathematics was so great that much
of our notation today was inspired by him, such as f(x) for functions, ifor
a = e for the base of common logs, & for summation, and even 7 for, well,
He linked three of these with his famous statement e” = —1.

191
CHAPTER 7. SPECIAL PARTS OF A TRIANGLE

[Exercises
7.5.1 RC and QB are altitudes of APQR as shown, ZQORP = 20°, and ZQPR = :
32°. Find the following: eb?

(a) ZPOB: Pp SR
(b) -ZGRE:
(Gn ZGOR:
(d)EZEOB:

7.5.2 Inthe diagram, YZ = 5, YB = 3, and XA = 4. Xx

(a) Find [XYZ].


(b) Find BZ. B
(Cj Eindex Z.
(d) Find XB. Z A a
(eye bind),
7.5.3 AABC is isosceles with AB = AC. Must the altitudes from B and C have the same length? Why or
why not?
7.5.4 Why must all the altitudes of an equilateral triangle have the same length?
7.5.5 Altitude AD of AABC is also an angle bisector of AABC. Must D be the midpoint of BC? Why or
why not?

7.6x Challenging Problems

In this section we tackle several challenging problems in which we use what we’ve learned in this
chapter about the special points, lines, and circles of a triangle. You may wish to try your hand at the
Review Problems at the end of this chapter before attempting the problems in this section.

Problems | >

Problem 7.26: In this problem, our goal is to find the area of ABCD, given that AB = BC = 6,
CD = 3 V2, ZABC = 90°, and that there is a circle passing through all four vertices of ABCD.
(a) Where is the center, O, of the circle that passes through all four vertices?
(b) What is the radius of this circle?
(c) What kind of triangle is AOCD?
(d) What are the angles in AOAD? What is ZCDA?
(e) Find the area of ABCD.

192
7.6%. CHALLENGING PROBLEMS

Problem 7.27; Given AB = BC = 10 and AC = 12, find the circumradius and the inradius of AABC.
Hints: 39, 322, 549

Problem 7.28: In the diagram, AD bisects ZBAC and 1 is the incenter A


of AABC. Furthermore, AB = 7, BC = 8, and AC = 11. Find AI/ID.

(a) Use the Angle Bisector Theorem to find another pair of seg-
ments that have a ratio equal to AI/ID. E
(b) Use the Angle Bisector Theorem again to find another pair of
lengths (besides AI and ID) that have a ratio equal to the ratio
you found in part (a).
(c) Let BD = x. Set up and solve an equation for x using the ratios B C
you found in the previous two parts. Then finish the problem
by finding AI/ID.

Problem 7.29: Is it possible for there to be points E and F on side BC of AABC such that BE = EF = FC
and ZBAE = ZEAF = ZFAC? Why or why not? Hints: 377

Problem 7.30: Point E is the midpoint of AC, and AD is a median of AABC. Fis on A
AB such that AF = AB/4. EF and AD meet at X. Find AX/AD. Hints: 95 Pp

Extra! The incircle and the circumcircle are not the only notable circles that can be found in a
im imi §=triangle. For any triangle, a single circle passes through the midpoints of the sides, the
feet of the altitudes, and the midpoints of the segments connecting the orthocenter to the
vertices. This circle is cleverly called the nine-point circle of the triangle.

The nine-point circle is tangent to the incircle, has a radius equal to half the circum-
radius (you have the tools to prove this—see if you can!), and its center is the midpoint
of the segment connecting the orthocenter and the circumcenter, which we discussed on
page 191. We’ll prove the existence of the nine-point circle, and some of its properties,
during our deeper investigations of triangles in Intermediate Geometry.

193
CHAPTER 7. SPECIAL PARTS OF A TRIANGLE

HProblen 7.26: Find the area of ABCD, given that AB = BC = 6, CD =3 2, ZABC = 90°, and that
there is a circle passing through all four vertices of ABCD.

Solution for Problem 7.26: We don’t know much about figures with four sides at this point, but we
now know a whole lot about triangles. We start with right triangle AABC. The area of this triangle
is (AB)(BC)/2 = 18, so all we have to find to finish is [ACD]. Unfortunately, we don’t have much
information about AACD.

We do, however, know that the circumcenter of AABC is the midpoint ‘é


of AC. Therefore the center of the circle, O, is the midpoint of AC. Breaking
the problem into triangles worked well once, so we try again by drawing
OD. Since AABC is an isosceles right triangle, AC = 6 V2. So, the radius of
the circle is 3 V2. We label all the lengths we know, and we see that AOCD
is an equilateral triangle.
Since AOCD is equilateral, its angles are all 60°. Furthermore, ZAOD =
180°—ZCOD = 120°. Since AAOD is isosceles, OAD = ZODA = 30°. Now,
we see that AACD is a 30-60-90 triangle, so AD = CD V3 = 3 V6. Finally, A
[ACD] = (CD)(AD)/2 = 9 V3.
Finally, we have [ABCD] = [ABC] + [ACD] = 18 + 9 V3. In Section 12.1, we will learn a much faster
way to deduce that ZADC is a right angle. 0
In this solution, we seemed to stumble on a surprising equilateral triangle and a useful right angle.
We were able to find them because we kept track of everything we learned in our diagram.

Concept: Label your diagram with all the lengths and angles you can find. This
will help you find relationships in the diagram that might be hard to find
otherwise. ZZ

Problem 7.27: Given AB = BC = 10 and AC = 12, find the circumradius and the inradius of AABC.

Solution for Problem 7.27: Since we're looking for the circumradius and the B
inradius, we’re thinking about perpendicular bisectors and angle bisectors.
Our triangle is isosceles, so we'll start with the special lines from the vertex 10
angle (ZB) since they’re all the same line.
BP splits our triangle in two congruent halves, so AP = 6. Since ABPA is A rely €
right, we have BP = 8 from the Pythagorean Theorem. To find the circumradius, eee A)
we build a right triangle with the circumradius as a side by connecting the circumcenter, O, to A. Since
OB = x and BP = 8, we have OP = 8 — x and we can apply the Pythagorean Theorem to AOAP to find
OA? = OP? + AP. Substituting our expressions for the sides, we have

x? =(8 x)? + 36.


A little algebra then yields 16x = 100, so x = 25/4.

194
7.6*. CHALLENGING PROBLEMS

Since the area of a triangle is its semiperimeter times its inradius, we can quickly find the inradius
of AABC:
Ps. [ABC] . ~ (AC)(BP)/2 _ 48
= 3.
e © (10s 10relo)oe ao
Challenge: See if you can find the inradius using a tactic like the one we used for the circumradius! 0

Concept: Need to find a length in a problem? Try building right triangles.

and J's the incenter of ABC.


Problem 7.28: In the diagram, AD bisects /BAC AI/ID.
Furthermore, AB = 7, BC = 8, and AC = 11. Find
A

Solution for Problem 7.28: We're looking for a ratio and we have an angle bisector, so we try using the
Angle Bisector Theorem. Since I is the incenter of AABC, BE bisects ZABC. Therefore, BI is an angle
bisector of AABD. From the Angle Bisector Theorem, we have

AI _ AB
1D--BDa
We know AB = 7, so if we can find BD, we’re finished. The Angle Bisector Theorem also gives us
AB/BD = AC/CD when applied to angle bisector AD of AABC. Since BD + DC = BC = 8, we have:
Des aoe Wich
BD 8-BD
Cross-multiplying gives 7(8—BD) = 11BD, from which we find BD = 28/9. Therefore, AI/ID = AB/BD =
9/4. 0

Concept: Keep your eye on the ball! Often, working backwards from what you want |
will guide you to the solution.

Problem 7.29: Is it possible for there to be points E and F on side BC of AABC such that BE = EF = FC
and ZBAE = ZEAF = ZFAC? Why or why not?

Solution for Problem 7.29: We have angle bisectors and lengths, so we try the Angle A
Bisector Theorem. AE and AF trisect BAC, but we can also look at them as the
angle bisectors of ZBAF and ZEAC, respectively. The Angle Bisector Theorem
applied to ABAF and AEAC gives us

AB _ BE _ LEA fies aaatame


and AGC ECE
APR ABF
Therefore, we have AB = AF and AE = EC.

195
CHAPTER 7. SPECIAL PARTS OF A TRIANGLE

A Marking these equalities in our diagram, congruent triangles jump out.


Specifically, SSS tells us AABE = AAFE = AAFC. However, this means that
ZAEB = LAEF = 90° and ZAFE = ZAFC = 90°. Therefore AAEF has two right
angles, which is impossible.
So, it is not possible for there to be points E and F on side BC of AABC such
Boe on eee © sthat BE = Pr Ee and 2BAn = /EAi= 76 AG ie
Problem 7.30: Point E is the midpoint of AC, and AD is a median of AABC. F is on AB such that
AF = AB/4. EF and AD meet at X. Find AX/AD.

Solution for Problem 7.30: We start with a diagram. We go ahead and draw in A
the other two medians, hoping we can use what we know about the centroid.
Since K is the midpoint of AB and AF = AB/4, F is the midpoint of AK. So, F
AAFE ~ AAKC by SAS Similarity. Therefore, ZAFE = ZAKC, so FE || KC. This
gives us AAFX ~ AAKG, so K E

AXKMEARel
AG AK
Since G is the centroid of AABC, we have AG/AD = 2/3, so

(Sigeye.
AD SNAG MAD 8:

Exercises lo

7.6.1 P,Q, and R are the midpoints of sides YZ, XZ, XY, respectively, and ZA is Y
the altitude to side XY. A

(a) Show that AP = RQ. Hints: 298


(b)x Show that ZPRQ = ZPAQ. Hints: 47 x O Z
7.6.2 Inright triangle ABC, let F be the midpoint of hypotenuse AB, and let D be the foot of the altitude
from C to AB. Let E be on AB such that CE is the angle bisector of ZACB. Prove that ZDCE = ZECE.
Hints: 136

7.6.3
In APQR, PQ = 8, PR = 10, and QR = 9. PD bisects ZQPR, QA 1 PD, ‘g
and BC passes through A such that BC ||PR.

(a) Prove that AC is a median of APAQ. E


(b)x Find DB. Hints: 532, 226, 125 PS
piesaDB R
Extra! [fat first the idea is not absurd, then there is no hope for it.
{1888111118108 eesAlbert Einstein

196
7.7. CONSTRUCTION: BISECTORS

7.7 Construction: Bisectors

In this section we’ll construct some of the special points, lines, and circles of a triangle.

|
Probiems
Problem 7.31: Given 2X shown, construct a ray from X that bisects 2X.

Problem 7.32: Construct the incircle of AABC shown.

Problem 7.33: Yes, it’s a circle. But you'll note that we haven’t marked the center.
Your job is to construct it.

We already know how to make perpendicular lines, midpoints, and perpendicular bisectors. But we
haven't learned how to make angle bisectors, so we'll start there.

Problem 7.31: Given 2X, construct a ray from X that bisects ZX.

Solution for Problem 7.31: Since we measure angles as a


portion of a circle, we start with a circle centered at X. We
call the points where the circle meets the sides of the angle
points A and B. Since XA = XB, AXBA is isosceles. This
makes us happy because we know that the angle bisector
of the vertex angle of an isosceles triangle is also a median,
an altitude, and a perpendicular bisector.
Therefore, all we have to do is construct the perpendic-
ular bisector of AB to have the angle bisector of 2X. We can
quickly construct this by drawing arcs centered at A and
B with radius AX. These arcs meet at X and a point we'll
call Y. XY bisects ZAXB. We'll leave the proof that this
construction works as an Exercise. 0
Now that we can construct the special lines of a triangle,
we can construct special circles. For example:

17
CHAPTER 7. SPECIAL PARTS OF A TRIANGLE

Problem 7.32: Givena triangle AABC, construct the incircle of the triangle.

Solution for Problem 7.32: To construct a circle, we need both its center and its
radius. We can find the center of the incircle by constructing the bisectors of two of
the angles of the triangle. We can perform these constructions exactly as described
in Problem 7.31. Where these rays meet is our center, point I.

Now that we have the center, we just need the radius. The radius of the incircle
drawn to a point where the incircle touches a side of the triangle is perpendicular
to that side. Therefore, we can find the radius by constructing this perpendicular
segment from I. Specifically, we construct a line through I that is perpendicular to AB as described in
Problem 6.24. Suppose this line meets AB at Z. IZ is the length of our radius.
We then construct our incircle by drawing a circle with center J and radius IZ. 0
We'll finish this chapter with two clever solutions to a challenging construction problem.

Problem 7.33: Given a circle but not its center, construct the center of the circle.

Solution for Problem 7.33: Solution 1: We don’t know a whole lot about x
constructing parts of a circle, but we know a ton about constructing parts bd
of a triangle. Specifically, we know how to construct the circumcircle
of a triangle — its center is just the intersection of the perpendicular
bisectors of the sides.
Therefore, we just pick three points A, B, and C on the circumference
of our circle. We then construct the perpendicular bisectors of two of
the sides of AABC as shown. The intersection of these perpendicular
bisectors, point O, is the circumcenter of AABC. Since our starting circle
is the circumcircle of AABC, the circumcenter of AABC is the center of
the starting circle.

Solution 2: Another useful circumcenter fact we know is that the cir-


cumcenter of a right triangle is the midpoint of its hypotenuse. Inspired by
this observation, we create right APQR by choosing point P on the circle,
drawing line n through P to meet the circle again at Q, then constructing
line m through P perpendicular to n. Where line m hits the circle again
gives us point R.
Since APQR is a right triangle, the midpoint of its hypotenuse is the
center of its circumcircle. Therefore, we find the center of our original
circle by constructing O, the midpoint of RQ. o

7.7.1 Given AABC, construct the circumcircle of the triangle.

198
7.8. SUMMARY

7.7.2 Construct a 30° angle.


7.7.3. Construct a 45° angle.

7.7.4 Construct a 120° angle.


7.7.5 Given AABC, construct the orthocenter of the triangle.

7.7.6 Prove that the construction described in the solution to Problem 7.31 really does produce an angle
bisector.

7.8 Summary

Definitions:

e The perpendicular bisector of a line segment is the line through the midpoint of the segment
that is perpendicular to the segment.

Important: The perpendicular bisector of a segment is a straight line consisting


VV of all points that are equidistant from the endpoints of the segment.

e The angle bisector of an angle is the line that divides the angle into two equal angles.

Important: The angle bisector of an angle consists of all points that are equidistant
WW from the sides of the angle.

Definition: A cevian is a line segment from a vertex of a triangle to a point on the line containing
opposite side of the triangle.

We explored important properties of several sets of cevians:

Definitions: The perpendicular bisectors of the sides of a triangle are


concurrent at a point called the circumcenter. The circle centered at the
circumcenter that passes through the vertices of the original triangle is
called the circumcircle of the triangle because it is circumscribed about
the triangle (meaning it passes through all the vertices of the triangle).
Finally, the radius of this circle is called the circumradius, the cir-
cumcenter is usually labeled with the letter O, and the circumradius is
usually called R.

Extra! There is geometry in the humming of the strings.


{> 1261108 (11> —Pythagoras

199
CHAPTER 7. SPECIAL PARTS OF A TRIANGLE

Definitions: The angle bisectors of a triangle are concurrent at a point


called the incenter. This point is equidistant from the sides of the triangle.
This common distance from the incenter to the sides of a triangle is called
the inradius, because the circle with center I and this radius is tangent
to all three sides of the triangle. This circle is unsurprisingly called the
incircle because it is inscribed in the triangle (meaning it is tangent to
all the sides of the triangle). The incenter is usually denoted I, and the
inradius is usually written as r.

Definitions: A median of a triangle connects a vertex of a triangle to the


midpoint of the opposite side. The medians of a triangle are concurrent at a
point called the centroid of the triangle. The centroid of the triangle is usually
labeled G.

Important: e The medians of a triangle divide the triangle into six little trian-
Vv gles with equal area.

e The centroid of a triangle cuts its medians into a 2 : 1 ratio. For


example, for the triangle shown, we have

AG Gyre Guene
e GD CE seCh eels

Definition: The altitudes of a triangle (extended if necessary) are concurrent at the


| orthocenter of the triangle, which is usually denoted H.

|Important: The orthocenter of an acute triangle is inside the triangle. The or-
Vv thocenter of a right triangle is the vertex of the right angle. The
orthocenter of an obtuse triangle is outside the triangle.

Important: Right triangles offer a few important relation-


: Ae Ss

e The circumcenter of a right triangle is


the midpoint of the hypotenuse, and
the circumradius equals one-half the hy-
potenuse.

- :_¢ The median to the hypotenuse of a right triangle is equal in ban |


to halfthe hypotenuse. |

Our exploration of these special points, lines, and circles uncovered a few more important facts:

200
7.8. SUMMARY

Important: Given any three noncollinear points, there is exactly one circle that passes
through all three.

Important: The Angle Bisector Theorem states that if E is B


on AC such that BE is the angle bisector of ZB in
triangle AABC, then

ae OB i E :
Ab Ck
We should think of this any time we have a problem involving lengths
and angle bisectors.

Important: The area of a triangle equals its inradius times half its perimeter.

Vv
Important: If we connect the midpoints of the sides of a triangle, we divide the
Vv triangle into four congruent triangles, each of which is similar to the
original triangle. The central triangle is called the medial triangle of our
original triangle.

Problem Solving Strategies

Concepts: e If we have the altitudes from two vertices of a triangle, then we know
@—-- that the line through the intersection point of these two altitudes and
the third vertex is also an altitude. Once you have two altitudes, you
should almost always think about this third altitude.
This, of course, also works with our other special lines — if you —
have two medians, you have the third; if you have two angle bisec-
tors, you have the third, etc. With these other special lines you also
get a little more, since we have special knowledge about the centroid _
(the 2 : 1 ratio), the incenter (center of the inscribed circle), and the
circumcenter (center of the circumscribed circle).
Again, if you have two of any of these special lines, you should
consider the third line and remember what you know about the
intersection point.

Continued on the next page. . .

~
201
CHAPTER 7. SPECIAL PARTS OF A TRIANGLE

|Concepts: .. . continued from the previous page

: e When given the lengths of the sides of a triangle in a problem, always


take the time to check if it is a right triangle. If it is, this fact may
simplify the problem considerably.

e Label your diagram with all the lengths and angles you can find.
This will help you find relationships in the diagram that might be
hard to find otherwise.
e Need to find a length in a problem? Try building right triangles.
e Keep your eye on the ball! Often working backwards from what you
want will guide you to the solution.

REVIEW PROBLEMS

7.34 In triangle APQR, PQ = 12, ZQ = 90°, and QR = 16. Find the area, the circumradius, and the
inradius of APOR.

7.35 Points T and U are on OK such that K is 2 units from TU. If TU = 14, what is the radius of the
circle?

7.36 AXYZ is an equilateral triangle with side length 12. M is the midpoint of side YZ and N is the
midpoint of XZ. YN and XM meet at E.

(a) What kind of triangle is AXYM?


(b) Find XM.
(c) Find [XYZ].
(d) Find XE/EM.
(e) Find the inradius of AXYZ.
(f) Find the circumradius of AXYZ.

7.37 Altitudes AD and CF of acute triangle AABC meet at H. Prove that ZCHD = ZABC.
7.38 If the altitudes of a triangle all have the same length, must the triangle be equilateral? Why or
why not?
7.39 Point D is onside YZ of AXYZ such that ZYXD = ZZXD. Given that XZ = O12 = 3, end Ds = 4
find XY. ,
7.40 Medians AD, BE, and CF meet at G. The area of AABC is 48. Find the areas of AADC, AAGC,
AGFB, ADEF, and AAEF.

aaa
EN 5
REVIEW PROBLEMS

741 ATUV is isosceles with TU = UV = 30 and TV = 36. UY bisects ZTUV and TM is a median of
ATUV. UY and TM meet at X.

(a) Show that Y is the midpoint of TV.


(b) Show that UY 1 TV.
(c) Find TY and UY.
(d) Find XY and XU.
(e) Find XT and XM.

7.42 In Problem 7.8 we found the circumradius of an equilateral triangle with side length 6, while in
Exercise 7.2.3, we found the circumradius of an equilateral triangle with side length 18. Given that the
first circumradius is 2 V3, how could we have quickly found the second without going through the same
laborious procedure we used to solve Problem 7.8?
7.43 In triangle ABC, altitude AD intersects angle bisector BE at point X. If ZBAC = 117° and ZACB =
35°, then determine ZDXE.

7.44 Point O is the circumcenter of APOR, ZQPR = 45°, and ZQPO = 23°.

(a) Find ZRPO and ZOQP.


(b)* Find ZOQR.

7.45 Mand N are the midpoints of CA and CB, respectively, as shown at right. BM C
and AN meet at X. Given that XM = 3.5 and XA = 7.2, find XN and XB. N

7.46 Medians AX and BY of triangle AABC are perpendicular at pointO. AX=12 M


and BC = 10. x B

(a) Find AO and BY.


A
(b)x Find the length of median CZ.

7.47 Cevians AD and BE meet at point X inside AABC. CX extended meets AB at F. If [AEX] = [CEX] =
[CDX] = [BDX] = [BXF] = [AXF], must X be the centroid of AABC?

7.48 BE and CF are angle bisectors that meet at J as shown at right. CE = 4, AE = 6, a


and AB = 8.

(a) Prove that ZEIC = 90° - #.


(b) Find BC. B C
(c) Find BF.

7.49

(a) Show that if the centroid and the orthocenter of a triangle are the same point, then the triangle is
equilateral.
(b) Show that if the incenter and the centroid of a triangle are the same point, then the triangle is
equilateral.
De
TT
203
CHAPTER 7. SPECIAL PARTS OF A TRIANGLE

|| Challenge Problems

7.50 The angle bisector of ZG of AGHI passes through the circumcenter of AGHI. Show that GH = GI.
Hints: 454
7.51 Let H and O denote the orthocenter and circumcenter of acute triangle ABC, respectively. AH
meets BC at P and AO meets BC at Q. Prove that BAP = ZCAQ. Hints: 499, 288
7.52 In AJKL, we have JK = JL = 25 and KL = 40. Find the following:

(a) the area of AJKL.


(b) the inradius of AJKL.
(c) the circumradius of AJKL. Hints: 305

7.53 Let I be the incenter of triangle ABC, and let Al meet BC at A’.

(a) Prove that A’I/IA = BA’/AB.


(b)x Prove that A’I/IA = BC/(AB + AC). Hints: 372

7.54 In triangle ABC, ZA = 100°, 2B = 50°, and ZC = 30°. H is on BC and M on AC such that AH is an
altitude and BM is a median. Find ZMHC. (Source: AHSME) Hints: 464
7.55 Is it possible for two angle bisectors in a triangle to be perpendicular? Why or why not? Hints:
509
7.56 In AABC, /C isa right angle. Point M is the midpoint of AB, point N is the midpoint of AC, and
point O is the midpoint of AM. The perimeter of AABC is 112 and ON = 12.5. What is the area of MNCB?
(Source: MATHCOUNTS)

7.57 Fora given triangle ABC, extend AB and AC as shown below. We will prove that there is a circle
tangent to BC and the extensions of AB and AC. We start by drawing the bisectors of two of the exterior
angles of AABC as shown with the grey lines below.

(a) Let he a point on the external angle bisector of angle ZB. Prove that P is equidistant from AB
and BC.
(b)s. Let rk a point on the external angle bisector of angle ZC. Prove that Q is equidistant from AC
and BC.

204
CHALLENGE PROBLEMS

(c) Let I, be the intersection of the external angle bisectors of angles ZB and ZC, and let X, Y, and Z
be the feet of the altitudes from I, to BC, AB, and AC. Prove that I,X = I,Y = I,Z. What does this
tell us about the circle with center I, and radius I, X? This circle is called an excircle of AABC; I, is
called an excenter, and r, is an exradius. Every triangle has three excircles, one tangent to each
side of the triangle.
(d) Prove that J, lies on the internal angle bisector of angle ZA.
[ABC]
(e)x Let r, = 1,X. Prove that r, = 5 BC’ where s is the semiperimeter of AABC. Hints: 44, 474

7.58 Points M and N are the midpoints of sides PA and PB of APAB. As P moves along a line that is
parallel to side AB, which of the four quantities listed below change? (Source: AMC 10)
(a) the length of segment MN.
(b) the perimeter of APAB.
(c) the area of APAB.
(d) the area of ABNM.

7.59 Let AXYZ have 2X = 60° and ZY = 45°. A circle with center P passes through A and B on side XY,
C and D on side YZ, and E and F on side ZX. Suppose AB = CD = EF.

(a) Prove that P is the incenter of AXYZ.


(b) Find ZXPY. (Source: HMMT)

7.60x A ladder is initially resting vertically against a wall that is perpendicular to the ground. It begins
to slip and fall to the ground, with the bottom of the ladder moving directly from the wall, and the top
of the ladder always touching the wall. What path does the midpoint of the ladder trace? (Don’t forget
to prove that the midpoint of the ladder really does hit every point on the path!) Hints: 271, 347

7.61 ‘Triangle AABC has area 48. Let P be the midpoint of median AM and let N A
be the midpoint of side AB. If G is the intersection of MN and BP, find the area of
AMGBP. (Source: Mandelbrot) Hints: 212, 407 N
7.62x In this problem we will prove that the medians of a triangle are concurrent.
&
Let AABC be our triangle. Medians AD and BE meet at point G. CG meets AB at M
point F.

(a) Show that [ACG] = [GCB] = [AGB]. Hints: 557


(b) Use your result from part (a) to show that AF = FB.
(c) Do the previous parts show that the medians of any triangle are concurrent?

7.63x Point N is on hypotenuse BC of AABC such that ZCAN = 45°. Given AC = 8 and AB = 6, find
AN. Hints: 448, 560, 585

7.64x The circle at right has radius 1 and is circumscribed about equilateral triangle A
ABC. If X is the midpoint of AC and Y is on are AC such that ZYXA is right, then find 4
length XY. (Source: Mandelbrot)
B CG
CHAPTER 8. QUADRILATERALS

The Eight Point Circle Theorem

It is easier to square the circle than to get round a mathematician. - Augustus De Morgan

CHAPTER

Quadrilaterals

8.1 Quadrilateral Basics


ce
H

D
G

A B E F

Figure 8.1: A Quadrilateral Figure 8.2: A Concave Quadrilateral

Figure 8.1 shows a quadrilateral, namely ABCD. A quadrilateral has four sides, four vertices, and
four angles. All the quadrilaterals we will study in this text have all four vertices in the same plane.
Nearly all quadrilaterals we deal with in this book are convex, like ABCD above, meaning that all of the
interior angles of the quadrilateral are less than 180°. There are also concave quadrilaterals, like EFGH
in Figure 3-2.
In this section we address basic definitions and facts that hold for all quadrilaterals. In later sections,
we turn to specific types of quadrilaterals.

206
1. QUADRILATERAL BASICS

Problems |

Problem 8.1: Use what you know about the sum of the angles in a triangle to find ZD in the figure
below.
B

As you've probably already considered, we can connect two more pairs of vertices in a quadrilateral
by connecting the opposite vertices.

C At left, we have added AC and BD to quadrilateral ABCD. We call these segments


connecting the opposite vertices of a quadrilateral the diagonals of a quadrilateral.
D
Let’s see how we can use a diagonal in a problem.
A B

Problem 8.1: Find ZD in the figure.

Solution for Problem 8.1: We don’t yet know anything about angles of a quadri-
lateral, but we know about triangles, so we add diagonal AC, thus making two
triangles, AABC and AADC. The sum of the angles in each of these triangles is
180°, so the sum of the angles in the whole quadrilateral ABCD is 2(180°) = 360°.
Now we can quickly find 2D:

ZD = 360° — ZDAB — ZABC — ZBCD = 70°.

Important: The interior angles of any quadrilateral add to 360°.

ts a riangles, triangles, triangles! As you'll see in the coming> sections


ge we can derive for Lies eee offaunas

207
CHAPTER 8. QUADRILATERALS

exercises ill
8.1.1 In quadrilateral ABCD, ZA = 100°, 2B = 50°, and /C is 30° more than ZD. Find ZD.

8.1.2 The angles of a quadrilateral are x, 3x — 10°, 27°, and 4x — 30°. Find the measure of the largest
angle of the quadrilateral.
8.1.3

(a) If two of the angles of a quadrilateral are equal, must the other two be equal?
(b) Is it possible for three of the angles of a quadrilateral to equal each other, but the fourth angle be
different?
(c) If all of the angles of a quadrilateral have the same measure, what is this measure?

8.1.4 How many different quadrilaterals can be formed by connecting the four points
shown at right?
8.1.5 We showed earlier that the interior angles of a convex quadrilateral add to 360°. » °
Show that the interior angles of a concave quadrilateral add to 360° as well. Hints: 391

8.2 Trapezoids

A trapezoid is a quadrilateral in which two sides are parallel. Figure 8.3 shows trapezoid WXYZ, in
which WX ||ZY.
W e¢

M N

Z ¥

Figure 8.3: A Trapezoid

These parallel sides are often called the bases of the trapezoid, with the other sides called the legs.
The segment that connects the midpoints of the legs is the median of the trapezoid; MN is the median
of WXYZ above.
A B

D CG

Figure 8.4: Is This a Trapezoid?


: You might be wondering ‘What if there are two pairs of opposite sides that are parallel like in
Figure 8.4; is the quadrilateral still a trapezoid?’ Good question! Unfortunately, there isn’t a good
answer. Some people define a trapezoid as having exactly one pair of opposite sides, so Figure 8.4

208
8.2. TRAPEZOIDS

would not be a trapezoid. Other people define a trapezoid as having at least one pair of opposite sides;
to these people, Figure 8.4 would be considered a trapezoid.
In this book, we will be careful to present proofs and definitions based on trapezoids as being valid
for either of these definitions. In this section, we will focus on quadrilaterals with exactly one pair of
parallel sides.

a} Problems ie

Problem 8.2: In trapezoid ABCD, we have AB || CD, ZA = 150°, and


ZB = 80°. Find ZC and ZD. oo
z OU: 80°|
B

DD) = i”

Problem 8.3: In trapezoid ABCD, AB || CD, diagonals BD and AC meet at AioB


N,and side lengths BN, CN, DN, and CD are as shown in the diagram. Find Me
AN and AB.

D 30 C

Problem 8.4: In trapezoid EFGH, we have EF ||GH, EF = 6, FG = 10, GH = 12, i ce


and EH = 8. Our goal is to find the length of median MN.
N M
(a) Extend EH and FG so that the two meet at X.
(b) Use similar triangles to find XE and XF, then find XN and XM. H C
(c) Show that AXEF ~ AXNM. Is the median parallel to the bases of the trapezoid?
(d) Find MN.

Problem 8.5: Bases PQ and SR of trapezoid PQRS have lengths 5 and 9, respectively, and are 8 units
apart.
(a) Find [PQS] and [QRS].
(b) Find [PQRS].
(c) Can you find a general formula for the area of a trapezoid given its base lengths and the distance
between the bases?

Problem 8.6: In trapezoid WXYZ, we have WX || YZ, WX = 17, XY = 13, YZ = 21, and XZ = 20. In
this problem we seek the area of WXYZ.
(a) We build right triangles by drawing an altitude from X to YZ. How do we know this altitude
meets YZ between Y and Z?
(b) Let the length of the altitude we draw in part (a) be h, and let it split YZ into segments of
lengths x and 21 — x. Use either the Pythagorean Theorem and algebra or your knowledge of
Pythagorean triples to find x and h.
(c) Find the area of WXYZ.

209
CHAPTER 8. QUADRILATERALS

Le 8.7: In trapezoid ABCD, we have AD || BC, 2B = ZC, and 2B < 90°. Prove that AB = CD.
Hints: 131

Problem 8.8: In trapezoid GHIJ, GH ||IJ, GH = 8, HI = GJ = 6, and IJ = 10. In this problem we will
find HJ.
(a) Draw heights HX and GY. Find JY, YX, and XI.
(b) Find HX, then find HJ.

Many trapezoid problems are solved using the basic properties of parallel lines.

|Probleat 62 in trapezoid ABCD, we have AB || CD, ZA = 150°, and A B |


|ZB = 80°. Find ZC and ZD. 150°~ 80°
D C

Solution
forProblem 8.2: Since
AB ||CD, we have ZA+ ZD = 180°, so ZD = 30°. Similarly, ZC = 180°— 2B =
100°. oO

Problem 8.3: In trapezoid ABCD, AB || CD, diagonals BD and AC meet at A_B


N, and lengths BN, CN, DN, and CD are as shown in the diagram. Find AN SA
and AB. a

D 30 C

Solution for Problem 8.3: Parallel lines mean similar triangles: AB || CD gives AABN ~ ACDN, so we
have
AN _ BN _ AB
CN HON SC"
Since we have BN/DN = 1/4, we find AN = CN/4 = 3 and AB = CD/4=5.0

Now we turn our attention to the median and the area of a trapezoid. The median of a trapezoid
connects the midpoints of the legs of the trapezoid.

Problem 8.4: In trapezoid EFGH, we have EF ||GH, EE ae FG = 10; GH =-19; E F


and EH = 8. Find the length of median MN.
N M

H G

Solution for Problem 8.4: Make sure you understand why this solution is incomplete:

Bogus Solution: Since M is midway between the lines, and so is N, MN must be


parallel to the bases and midway between the lengths of the bases.
Therefore, MN is the average of the bases, so MN = (EF +GH)/2 = 9.

210
8.2. TRAPEZOIDS

This solution has several problems. It consists of a series of statements without sufficient justification.
For example, it doesn’t explain what ‘M is midway between the lines’ means, nor why this leads to MN
being parallel to the bases. We also have absolutely no justification for the claim that MN is the average
of EF and GH.
Many students new to writing mathematics write solutions that look much like this Bogus Solution.
Strive to explain all your steps in mathematical terms, and to justify your claims. Showing exactly why
MN is the average of EF and GH is not a trivial task. MN looks parallel to the bases, but we'll have to
go through a few steps to prove it. We want to use triangles since we know a lot about triangles, so we
extend EH and FG to meet at X. With this inspiration as a starting point, here’s what a complete solution
looks like:

The extensions of the legs meet at X. Since AEXF ~ AHXG, we have x

AE oloEP
XH GH E F
So, substitution gives N M
ie pal t
xEG8 3 A G
Solving this equation, we find XE = 8. Similarly, since XF/XG = 1/2, F is the midpoint of XG, so
XF = FG = 10. Mis the midpoint of FG, so XM = XF + FG/2 = 15. Similarly, we have XN = 12. Since
XN/XE = 12/8 = 3/2 and XM/XF = 15/10 = 3/2, we have AXEF ~ AXNM by SAS Similarity. Therefore,
NM/EF = XN/XE = 3/2, so NM = (3/2)(EF) = 9. Notice also that ZXNM = ZXEF from our similarity, so
NM || EF. o

You'll have a chance at the end of this section to use the same process we used on Problem 8.4 to
prove these assertions for all trapezoids.
We used triangles to learn about the median; now we use them to learn about the area of a trapezoid.

Problem 8.5: Bases PQ and SR have lengths 5 and 9, respectively, and are 8 units apart. What is the
area of trapezoid PQRS?

Solution for Problem 8.5: We know how to find the area of triangles, so we dividethe Y_P 5 Q
trapezoid into two triangles by drawing diagonal QS. We then draw the altitudes QX =
and SY of the two triangles. Since the parallel lines are 8 units apart, QX = SY = 8. P!
We can now find the area of the trapezoid by adding the areas of the triangles: !
PQ\SY) , (SRQX) _, (PQ+SR
ee ioMe as (“ee : )= 56. :! es
[PORS] = [PQS] + [QRS] = PLMEY
O

Since the median of a trapezoid is the average of the bases of the trapezoid, we can use our last
problem to write a formula for the area of a trapezoid in terms of the height and median and a trapezoid.

211
CHAPTER 8. QUADRILATERALS

Important: The area of a trapezoid equals the height of the trapezoid times the length |
Vv of the median of the trapezoid. |

Problem 8.6: In trapezoid WXYZ, we have WX || YZ, WX = 17, XY = 13, YZ = 21, and XZ = 20.
Find the area of WXYZ.

Solution for Problem 8.6: We need the height, so we try to add a height to the the diagram in a way that
forms useful right triangles. The altitude from X to YZ is the most promising candidate, but we first
have to figure out where the altitude from X will hit YZ. It’s not immediately clear if the foot of the
altitude, which we'll call H, is on YZ or not. (It might be beyond Y on ZY for example.) Wherever H
is on YZ, AXZH is a right triangle. Since XZ is the hypotenuse and ZHa leg of this triangle, we have
HZ < XZ. Since XZ < YZ, we know that HZ < YZ. Similarly, we have HY < XY < YZ. Since HZ and
HY are both less than YZ, H must be on YZ. Now we can confidently draw our diagram with H on YZ.

We let XH = h and HY = x, so that ZH = 21 — x and we can now use the WwW 17 x


Pythagorean Theorem on AXHZ and AXHY:

+h? = 169 13
(1-—x)* +h = 400
Sele
Subtracting the first equation from the second allows us to eliminate hi and
find x. We get
[21 ea)> he ence tdi) = 400s 10o,
Therefore, x? — 42x + 441 + h? — x* — h? = 231, so —42x = —210, which gives x = 5.
Substitution then gives us h = 12. Seeing such nice neat numbers, we realize that we could have
guessed ht = 12, since a leg of length 12 goes with hypotenuses XZ = 20 and XY = 13 nicely. (Notice that
just because h = 12 fits these hypotenuses nicely doesn’t mean that must be 12. We still have to check
that ZY = 21 if we guess h = 12.)

Now that we have h = 12, we can find [WXYZ] = (h)(WX + YZ)/2 = (12)(38)/2 = 228. o

Extra! Consider a quadrilateral with perpendicular diagonals as


imp ei §~Shownat right. The Eight-Point Circle Theorem tells us that
the midpoints of the sides and the feet of the perpendiculars
from these midpoints to the opposite respective sides of the
quadrilateral all lie on the same circle.
If you can’t prove the Eight-Point Circle Theorem now,
try coming back to it after you’ve tackled Problem 8.19. You
might find the result of Problem 7.6 useful, as well.

ZVe
8.2. TRAPEZOIDS

We'll now take a look at a special class of trapezoids.

Problem 8.7: In trapezoid ABCD, we have AD ||BC, 2B = ZC, and ZB < 90°. Prove thatAB = CD.
Solution for Problem 8.7: Once again, we introduce right triangles by drawing A D
altitudes from A and D. Then AX = DY because AD ||BC, so by AAS Congruence,
we have AABX = ADCY. Therefore, AB = CD.
Notice that we can easily deal with the case of ZB and ZC being obtuse by
notine that i 2B = ZC, then 4A\= 180° — 7B = 180° — ZC = ZD. If ZB and ZC are B xX Y C
obtuse, then ZA and ZD are acute, and we can use essentially the same proof as above to prove that the
legs of the trapezoid are equal in length. 0
Two angles of a trapezoid that share one of the bases of the trapezoid as a side are together called
base angles of the trapezoid. A trapezoid in which two base angles are equal is called an isosceles
trapezoid. As an Exercise, you'll also prove that the diagonals of an isosceles trapezoid are congruent.
Finally, in Section 8.7, you'll show that these relationships run ‘backwards’; i.e., that if the legs are equal
in length (and not parallel) or if the diagonals of a trapezoid are congruent, then both pairs of base
angles are equal.

Important: nan isosceles trapezoid:


V (a) The base angles come in two pairs of
equal angles as shown at right.

(b) The legs are equal.


(c) The diagonals are equal.

If any one of these items is true for a trapezoid with exactly one p. ir of
parallel sides, then all the others must be true for that trapezoid. _

Problem 8.8: Find HJ given that GHIJ is an isosceles trapezoid with GH = 8, HI = GJ = 6, and
IJ = 10.

Solution for Problem 8.8: We start by drawing altitudes GY and HX, hoping to
build useful right triangles. We note that since GY = HX and GJ = HI, we
have AGJY = AHIX by HL Congruence. Therefore, JY = XI. Next, we see that
YX = GH because GY ||HX and YX and GH are both the distance between GY and
HX. (Alternatively, we could note that GHXY is a rectangle.) Therefore, YX = 8,
so JY = Xl= (10 —8)/2 = 1.

Now we find HX from AHXI: HX = VHI2 - IX? = 35. Finally, we use right triangle HX] to get HJ:
H] = \/HX?2 + XJ? = V35 + 81 = V116 = 2 V29. Building right triangles to find lengths strikes again. O
If you found this section a little tough, review it after reading the rest of this chapter. You'll find that
the other quadrilateral types are much simpler to handle than trapezoids, primarily because our other

213
CHAPTER 8. QUADRILATERALS

types have restrictions on all the sides instead of just on one pair of opposite sides. Therefore, the other
quadrilateral types don’t offer as wide a variety of problems as trapezoids do.

|Exercises
8.2.1 In quadrilateral PQRS, PQ ||RS, PS is not parallel to OR, OR = PS, and ZP = 83°. Find the rest of
the angles of the trapezoid.

8.2.2 Find the area of a trapezoid with bases 44 and 24 and with height 18.

8.2.3 The area of trapezoid ABCD is 96. One base is 6 units longer than the other, and the height of the
trapezoid is 8. Find the length of the shorter base.

8.2.4 ABCD shown at right is a trapezoid with BC || AD, ZABD = 88°, B &
T= DP eeanGdi.G =126., Hing ZAC and Z CBD, ge

8.2.5 Prove that the diagonals of an isosceles trapezoid are congruent.


8.2.6 In this problem we prove that if ABCD is a trapezoid such that 79°
AB ||CD and AB < CD, then the median of ABCD is parallel to the bases 4 D
of ABCD and equals the average of the bases of the trapezoid. Use Problem 8.4 as your guide. (Note
that this proof covers any trapezoid in which the legs are not parallel.)

(a) Let Mand N be the midpoints of AD and BC respectively. Extend AD and BC to meet at X. Prove
AXDC ~ AXAB.
(b) Prove that AD/XA = BC/XB, and use this to prove that AM/XA = BN/XB.
(c) Prove that XM/XA = XN/XB.
(d) Prove that MN ||AB.
(e)x Prove that MN = (AB + CD)/2. Hints: 112, 153

8.2.7x The bases of a trapezoid have lengths 50 and 75. Its diagonals have lengths 35 and 120. Find
the area of the trapezoid. Hints: 339, 420

Extra! If we construct an equilateral triangle on the outside of each \7>~-~-~__


mia ~Side of any triangle, the triangle formed by connecting the \
centroids of these equilateral triangles is called the Outer
Napoleon Triangle. As the diagram strongly suggests, the
outer Napoleon triangle is always equilateral. As you might
guess, we can construct the equilateral triangles ‘inward’ and
connect their centers to build an Inner Napoleon Triangle. ce
Draw a triangle, then construct the Inner Napoleon Triangle. Siig
Does it look equilateral, too?
While the French emperor Napoleon Bonaparte iis generally given credit fordiscover
_ing these triangles, there’s no solid proof that he actually did so.

214
8.3. PARALLELOGRAMS

8.3. Parallelograms

D > C

Figure 8.5: A Parallelogram

A quadrilateral is a parallelogram if both pairs of opposite sides are parallel. ABCD in Figure 8.5 is a
parallelogram because AB || CD and AD || BC.

Problems

Problem 8.9: Given that ZA = x + y, ZB = 3x, and ZD = 30° in parallelogram


ABCD as shown, find x, y, and ZC.

Problem 8.10: In this problem we show that if AB = CD and AD = BC, then ABCD is a parallelogram.
(a) Draw a parallelogram ABCD with diagonal AC. Prove that ABCA = ADAC.
(b) Prove that AB ||CD.
(c) Prove that ABCD is a parallelogram.
(d) If ABCD is a parallelogram, must AD = BC and AB = CD? Make sure you see how this is a
different question from the one we have already tackled.

Problem 8.11:
(a) Prove that if EFGH is a parallelogram, then diagonals EG and FH bisect each other.
(b) Does this work in reverse? In other words, given that diagonals WY and XZ of quadrilateral
WXYZ bisect each other, is WXYZ a parallelogram?

Problem 8.12: Sides EU and NT of eae Bn are 5 units apart as


shown. EU = 12 and EN= 8.
(a) Find the area of ENTU. (Hint: Remember the dissection we used to get
the area of a trapezoid.)
(b) How far apart are sides EN and TU?

As we'll see, parallelograms are a little easier to handle than quadrilaterals that have only one pair
of parallel sides. Once again, our primary tools will be triangles and parallel lines.

25
CHAPTER 8. QUADRILATERALS

‘Problem 8.9: Given that the measures of the angles of parallelogram ABCD A B
_are as shown, find x, y, and ZC.
|

|
|

Solution for Problem 8.9: Since AD || BC, we have ZD + ZC = 180°, so ZC = 150°. Similarly, AB || CD
means ZB+ZC = 180°, so ZB = 30°. Therefore, 3x = 30°, sox = 10°. Finally, A+ZD = 180°, so ZA= 15:
Hence, x + y = 150°, so y = 140°.O
Notice that in our solution we found that ZB = ZD and ZA = ZC. At the end of the section, you'll be
asked to prove that the opposite angles of any parallelogram are equal.

|Problem 8.10: Show that if AB = CD and AD =


BC, then ABCD is a parallelogram.

Solution for Problem 8.10: Where did we go wrong here:

Bogus Solution: We cut ABCD into triangles by drawing Ac. A B


Since AD || BC, we have DAC = ZBCA.
Combining this with AD = BC and AC = D fe
AC, we have ADAC = ABCA by SAS Con- |
eruence. Therefore, BAC = ZACD, so AB ||CD. Similarly, we can
show that AD ||BC, so ABCD is a parallelogram. |

Our error is that we use AD ||BC to prove that AB ||CD. However, we don’t know that AD ||BC, so
we can’t use it yet. Fortunately, patching the solution is easy.
Still using our figure from the Bogus Solution above, we have ADAC = ABCA by SSS Congruence.
Therefore, ZBAC = ZACD, so AB || CD. Similarly, we have ZDAC = ZACB, so AD || BC as well. Thus,
ABCD is a parallelogram.O

We can also quickly show that in any parallelogram, the opposite sides A B
must be equal. We do so by again using diagonal AC of parallelogram ABCD.
We have AABC = ACDA by ASA Congruence. These congruent triangles give
us AB = CD and BC = AD. Make sure you see why this addresses a different D (c
question from the one initially asked in this problem!

Extra! In the spirit of David Hilbert’s famous 1900 lecture (see page 56), in 2000 the Clay
em Mathematics Institute offered $1,000,000 prizes for solutions to each of seven unsolved
Millennium Problems. Russian mathematician Grigori Perelman put forth a solution
to one of the Millennium problems, the Poincaré Conjecture, in 2003. His solution
was so complex that it took roughly three years to validate! As of June, 2009, the Clay
_ Mathematics Institute had not yet awarded the prize, since Perelman’s proof fe: not
been formally published in a peer-reviewed journal.

OG o#o° ae ee er
1) Le 8.3. PARALLELOGRAMS

We’ve looked at the sides and the angles of a parallelogram; let’s check out the diagonals.

| Problem 8.11:
(a) Prove that if EFGH is a parallelogram, then diagonals EG and FH bisect each other.
(b) Does this work in reverse? In other words, given that diagonals WY and XZ of quadrilateral |
isect och other, is3 WXYZ a Sadieweascitinal

Solution forProblem 8.11:


(a) Since EFGH is a parallelogram, we have EF = GH and EF || GH.
Therefore, ZFEG = ZEGH and ZEFH = ZFHG, so AEFO =~ AGHO
by ASA Congruence. Therefore, EO = OG and FO = OH, so O is
E
Se ie

the midpoint of both diagonals of EFGH. Thus, the diagonals of a


parallelogram bisect each other.

(b) Once again, we mark the information we’re given — the fact that the ae Deena Xx
diagonals bisect each other. We see that AWOZ = AYOX by SAS
Congruence, so ZZWY = ZWYX, which means WZ || XY. Similarly,
AWOX = AYOZ, which we can use to show WX || YZ. Hence, if the len
diagonals of a quadrilateral bisect each other, then the quadrilateral is Z s
a parallelogram.

We can combine the information from the last three problems to see parallelograms the way an
experienced geometer sees them.

"Important: In parallelogram AB CD, the opposite sidesarere equal,the opposite angles.


are equal, and the diagonals bisect each other.
A B A B

D S D C

Figure 8.6: Side and Angle Equalities in Parallelograms


Conversely, ABCD is a parallelogram if any one of the following are true:
(a) AB = Cand AD = BC.
(b) ZA = ¢C and
4B = 2D.
(c) Diagonals AC and BD bisect each other.
Proving oneee ee means as One two are true.

Finally, we tackle the area of a parallelogram. We could use the same tactic as we used with
trapezoids, but there is an even easier approach.

aa
dh! CCC tt~<Ci=i‘CSOSOSSCSCSCSCOCNNSS. CS,
CHAPTER 8. QUADRILATERALS

‘Problem 8.12: Sides EUand NT of parallelogram ENTU are 5 units apart E 8 N


'as shown. EU = 12 and EN = 8.
| (a) Find the area of parallelogram ENTU. 12
|
| ae en

| (b) How far apart are sides EN and TU?

Solution for Problem 8.12:


(a) We can approach the area of a parallelogram the same way as we did with a trapezoid. If we draw
diagonal ET, we cut the parallelogram into two triangles, AEUT and ANET, which have the same
base (EU = NT = 12) and the same height, 5. Hence, we have

[ENTU] = [EUT)+ [NET] =


(EU)(S)
5
, (NT)(S)
5
_ (EWN)
5
| (EW) = (EU)(5) = 60.
2
Notice that we can easily use this approach to show that the area of a parallelogram is its base
times its height to that base.
(b) We have the base, EN, and the area from the previous part. Since the area of ENTU is 60, the
height between EN and TU is
[ENG |S OUey lo
EN 8 ee

Important: The area of a parallelogram is its base times its


VY height.

[ABCD] = (AB)(XY) = (BC)(PQ).

8.3.1 TYUI isa parallelogram with TY = 6 and YU = 8.


(a) What is the perimeter of TYUI?
(b) Do we have enough information to find the area of TYUI?
8.3.2 WXYZ isa parallelogram. Prove that ZW = ZY.
8.3.3 In quadrilateral WORK, WO = RK and WO ||RK. Prove that WORK isa parallelogram.
8.3.4 Use a clever dissection of a parallelogram to turn it into a rectangle and prove that the area of the
parallelogram is its base times its height. Hints: 394
8.3.5 Is it possible for a parallelogram ENTU to have EN = 4, EU = 12, and for sides EU and NT to be
5 units apart? Why or why not? Hints: 528

BG
a ee ce eee
8.4. RHOMBI

8.3.6 In this Exercise, we give another way of deriving the length of the median and area of a trapezoid.
Let ABCD be a trapezoid with bases AB and CD.

(a) Extend AB past B to A’ such that BA’ = CD, and extend DC past C to D’ such that CD’ = AB.
Prove that AA’D’D is a parallelogram.
(b) Find the area of [AA’D’D] in terms of AB, CD, and h, the distance between AB and CD. Find the
relationship between [AA’D’D] and [ABCD], and then find [ABCD] itself. Hints: 341
(c) Let M, N, and M’ be the midpoints of AD, BC, and A’D’, respectively. Prove that MM’ = AB+CD,
and prove that MN = MM’/2. Conclude that MN = (AB + CD)/2.
8.3.7x The diagonals of convex quadrilateral ABCD meet at E. Prove that the centers of the circumcircles
of AABE, ABCE, ACDE, and ADAE are the vertices of a parallelogram. Hints: 80

8.4 Rhombi

D | Eg

Figure 8.7: A Rhombus

A quadrilateral is a rhombus if all of its sides are equal.

7
Problem 8.13: Prove that every rhombus is a parallelogram.

Problem 8.14: ABCD is a rhombus such that its diagonals AC and BD have lengths AC = 30 and
BD = 16. In this problem, we will find the area of ABCD and the side length of the rhombus.
(a) Let the diagonals intersect at X. Show that AAXB, ACXB, ACXD, and AAXD are congruent.
(b) Prove that AC 1 BD.
(c) Find the area of ABCD.
(d) Find AB.

We can prove a whole lot about rhombi in one simple step.

Problem 8.13: Prove that every rhombus is a parallelogram.

Solution for Problem 8.13: Let ABCD be a rhombus. Since AB = CD and AD = BC (because all the sides
of ABCD are equal), we know that ABCD is a parallelogram. 0

SSF
CHAPTER 8. QUADRILATERALS

‘Important: Every rhombus is a» parallelogram. Therefore, all that istrueabo


Y oe is true about rhombi.

(Bet:you saw this ¢one coming. )Every parallelogram iis nota rhomb
herefore, if we prove a property ofrhombi, this property is not
essarily true for all peal loptee ee have to provet! the
e prope yo
rparallelograms separately. — g PL

To see some of the properties that distinguish a rhombus from a parallelogram that is not a rhombus,
let’s try a problem.

Problem 8.14: ABCD is a rhombus such that its diagonals AC and BD have lengths AC = 30 and
BD= 16. Find AB and [ABCD].
Solution for Problem 8.14: We start with a diagram, in which we note that A B
all the sides of ABCD are equal, and that the diagonals bisect each other
since rhombus ABCD is also a parallelogram. All four little triangles in
the diagram are congruent by SSS Congruence. Since all four angles at
X must be equal and they add to 360°, they must be right angles. Hence,
the diagonals of a rhombus not only bisect each other, but they are also
perpendicular. Since each of these right triangles have legs of length 8 D G
and 15, their hypotenuses (which are the sides of the rhombus) each have length 17.
All those congruent right triangles give us a quick approach to finding the area.
AX)(BX
[ABCD] = 4[ABX] = 4 (Oe = 2(AX)(BX) = 240.

Notice that since AX and BX are each half a diagonal of ABCD, we have

[ABCD] = 2(AX)(BX) = 2 (AC)


a (EaVer
es :

Our solution gives us a few special rhombus facts to add to all the facts we know about rhombi
because they are parallelograms.

BOQ ae
8.5. RECTANGLES

RELI
8.4.1 PQRS isa rhombus with diagonals PR = 6 and QS = 12. Find the area and the perimeter of PQRS.
8.4.2 WXYZ isa rhombus with WX = 50 and WY = 96.

(a) Find XZ.


(b) Find [WXYZ].
(c) Find the distance between WX and YZ. Hints: 586
8.4.3. Diagonals AC and BD of quadrilateral ABCD are perpendicular. Prove that [ABCD] = (AC)(BD)/2.
Hints: 438

8.4.4 TUVW isarhombus with TU = 10 and ZTUV = 60°.

(a) Show that ZTUW = 30°.


(b) Find [TUVW].

8.5 Rectangles

We’ve looked at quadrilaterals in which all the sides are equal. Now we consider what happens if all
the angles are equal. Such a quadrilateral is called a rectangle.

>
Problem 8.15: What are the measures of the angles in a rectangle?

Problem 8.17: Prove that the diagonals of a rectangle are congruent.

Problem 8.18: Find the length of a diagonal of a rectangle given that its perimeter is 44 and one side
has length 10.

Problem 8.19: In this problem, we find two different proofs that the quadrilateral formed by connecting
the midpoints of a rhombus is a rectangle.
(a) Start with a diagram. Draw rhombus ABCD and let EFGH be the quadrilateral formed by
connecting the midpoints of its sides, with E on AB, F on BC, and so on.
(b) Solution 1: Show that ZAEH = ZAHE and that AAEH = ACGF. Use this as a starting point to
show that the angles of EFGH are all equal.
(c) Solution 2: Show that EF ||AC. Show that the angles of EFGH are right angles.

221
CHAPTER 8. QUADRILATERALS

We know the angles of a rectangle are equal, but to what?

‘Problem 8.15: What are the measures of the anglesinarectangle?


Solution for Problem 8.15: The angles must add to 360° and they’re all equal, so each of the four angles
must be a right angle. 0
As we did with rhombi, we can quickly get a wealth of information about rectangles.
Problem 8.16: Prove that all rectangles are also parallelograms. —
Solution for Problem 8.16: Let ABCD be our rectangle. Since ZA + 2B = 180°, we D €
know that AD ||BC. Similarly, ZB + ZC = 180°, so AB || CD. Therefore, ABCD is a
parallelogram. 0
Among the many useful aspects of knowing that every rectangle is a paral- ie 4
lelogram, the one that is probably most used is the fact that opposite sides are equal in length. When
talking about rectangles, these two dimensions are often called the length and width of the rectangle. We
have already seen that the area of a rectangle is the product of these two dimensions.
We know that the diagonals of a rhombus are perpendicular, so we wonder if there’s a similar special
property of the diagonals of a rectangle.

Solution for Problem 8.17: As we have in many other problems, we turn tocongruent 5S R
triangles for a solution. Drawing our two diagonals, we have APQR = AQPS by eee
SAS since PS = QR, PQ = PQ, and ZPQR = ZQPS. Therefore, PR = QS. 0 E Q
Problem 8.18: Find the length of a diagonal of a rectangle given that its perimeter is 44 and one side
has length 10.

Solution for Problem 8.18: Let WXYZ be our rectangle and let XY = 10. We know Z bs
the diagonals have the same length, so we don’t have to worry about which ee
diagonal to find. Since XY = 10, we have WZ = 10 as well. Since the perimeter is
44, we have
WX +XY+YZ+ZW = 44. is xX
Since WX = YZ and XY = WZ = 10, we have WX + 10 + WX #10 = 44, so WX = 12. Now we can use
the Pythagorean Theorem to find XZ = VWX? + WZ2 = V¥100 + 144 = 2 V61 0

Important: The diagonals of a rectangle are equal to each other


and equal to the square root of the sum of the
squares of the length and the width of the rectan- P ©
gle. :
QS = PR= \/QR?* + SR?

You shouldn’t have to memorize how to find the diagonals of a rectangle: the right angles of a

222
8.5. RECTANGLES

rectangle should make it clear to use the Pythagorean Theorem with the sides to get the diagonal.
Now we'll take a look at one way rectangles and rhombi are related, before moving on to a quadri-
lateral that is both a rhombus and a rectangle.

Problem 8.19: Prove that the quadrilateral formed by connecting the midpointsofthe sides ofa
rhombus is a rectangle.

Solution for Problem 8.19: We draw rhombus ABCD and let EFGH be the quadrilateral formed by
connecting the midpoints of the sides of ABCD, with E on AB, F on BC, and so on. We will take two
different approaches to solving the problem.

(a) Since we want to show that EFGH is a rectangle, we look for


a way to find the measures of each angle of EFGH. Since its
vertices are the midpoints of ABCD and ABCD is a rhombus,
the vertices of EFGH divide the sides of ABCD into 8 equal
segments as shown.
Since ABCD is a parallelogram, ZA = ZC and ZB = ZD.
Therefore, AAEH = ACGF and ABEF = ADGH. Furthermore,
each of these small triangles is isosceles, so we can let ZAEH =
x and ZDGH = yand identify equal angles as shown. Finally,
we see that each angle of EFGH has measure 180° — x — y. Thus, the angles of EFGH are equal and
EFGH must be a rectangle.
(b) For another approach, we might note that we want to prove
that the angles of EFGH are right angles. This gets us thinking
about where we can find right angles in ABCD. Since ABCD
is a rhombus, its diagonals are perpendicular. To relate the
sides of the rectangle to these diagonals, we find that AAEH ~
AABD by SAS Similarity since AE/AB = AH/AD = 1/2 and
ZEAH = ZBAD. Therefore, ZAEH = ZABD, so EH || BD.
Similarly, we can show that FG ||BD. Since AC is perpendic-
ular to BD, it must be perpendicular to EH and FG as well,
because both EH and FG are parallel to BD.
We can use exactly the same approach to show that EF ||AC || GH, from which we can conclude
that EF and GH are perpendicular to BD. Now we can tie all these parallel and perpendicular
lines together. Since EF 1 BD and FG || BD, we have EF 1 FG. In exactly the same way, we can
show that each of the other three angles of EFGH is a right angle. Thus, EFGH is a rectangle.

8.5.1 POST isa rectangle with PO = 8 and OS = 12.


(a) Find the perimeter of POST.
(b) Find PS.
ee
ae.ee ee
223
CHAPTER 8. QUADRILATERALS

(c) Find, [POST].

8.5.2 The length of a rectangle is one less than twice its width. If the perimeter of the rectangle is 36,
what is the area of the rectangle?
8.5.3. Diagonals WR and ET of rectangle WERT meet at Y. Given that ZWYE = x, find ZERY and ZYRT
in terms of x.

8.5.4 A semicircle with center O has a radius of 9cm. What is the number of aa
centimeters in the length of RQ, a diagonal of the rectangle shown? (Source:
MATHCOUNTS)
8.5.5 [havea 36 inch by 24 inch rectangular painting. I would like to place a frame RO
that is 2 inches wide around the painting. If the material for the frame costs $1.50 per square inch, how
much will the frame cost?
8.5.6 What kind of quadrilateral do we get when we connect the midpoints of the sides of a rectangle?
(Prove your answer!)

8.5.7x EFGH isa rectangle with area 48. If EGJI is a rectangle such that H is on JL, what is the area of
EGJI? Hints: 434

8.6 Squares

A B

Figure 8.8: A Square


We've studied quadrilaterals with all their sides equal, and quadrilaterals with all their angles equal.
What if a quadrilateral has both all sides equal and all angles equal? Such a quadrilateral is called a
square, an example of which is shown in Figure 8.8.
As we'll see, squares are the easiest quadrilaterals to work with since everything that is true about
rectangles, rhombi, and parallelograms is also true about squares.

“Problems _ >

Problem 8.20: Find the perimeter and area of a square with side length 9. |

Extra! The composer opens the cage door for arithmetic, the draftsman gives geometry its freedom.
i at st sn —Arthur Cayley

224
8.6. SQUARES

|Problem 8.21: Find the perimeter and area of a square that has a diagonal of length 8.

Problem 8.22: ABCD isa square. The circle with center A and radius AB meets A
the perpendicular bisector of AD in two points, of which O is the one inside the
square. Find ZAOC. Hints: 237 : /

We've already studied all the tools we need for squares. How we use these tools in the following
two problems should be unsurprising.

Problem 8.20: Find the perimeter and area of a square with side length 9.

Solution for Problem 8.20: Since all the sides of the square are equal, the perimeter of our square is
4x 9 = 36. Since a square is a rectangle, its area is its length times its width. These dimensions are the
same in a square, so the area is just the, um, ‘square’ of its side length, or 9* = 81. (Now we see why
multiplying a number by itself is called ‘squaring’ the number.) 0

Problem 8.21: Find the perimeter and area of a square that has a diagonal of length 8.

Solution for Problem 8.21: A square is a rectangle, so its diagonals are congruent. It is also a rhombus, so
its area is half the product of its diagonals, or (87) /2 = 32 in this case.

We can find a side of the square by noting that a diagonal of a square splits the square into two
45-45-90 triangles, so each side equals the diagonal divided by V2, or 8/2 = 4 y2. Alternatively, since
the area is just the square of the side length, we can take the square root of the area we already found:
32 = 4 V2. Thus, the perimeter is 4(4 2) = 16 V2. O
These two examples give us the most useful length and area relationships for a square.

Important: Let s be the side length of a square, P be the perimeter,

VY and A the area. We have

P=A4s Aes:
Ate
Drawing a diagonal creates two 45-45-90 triangles. Letting the length of:
# ee
a diagonal be d, we have
d2
d=sv2 P = d(2 V2)

We often encounter squares in problems together with circles, since both involve segments with
equal lengths (sides for the square, radii for the circle).

225
CHAPTER 8. QUADRILATERALS

[Problem 8.22: ABCD is a square. The circle with center A and radius AB A B
meets the perpendicular bisector of AD in two points, of which O is the one er
inside the square. Find ZAOC. L x
x

Solution for Problem 8.22: There doesn’t seem any easy way to get ZAOC, so we try A
splitting it into pieces by drawing OD. Since AAYO = ADYO by SAS, we have
AO = OD. Furthermore, AD = AO because they are radii of the circle. Thus, y
AAOD is equilateral, which implies ZAOD = 60°. Now we only have to find Deas
ZDOC.
Since OD equals the side length of the square, we have OD = DC. Therefore,
ADOC is isosceles, so ZDOC = ZDCO. Furthermore, we already know

LODG:=ZADC=IZADO = 90" = 60° =30*-

From ADOC we have


ZDOC + ZDCO + ZODC = 180°,
so 2ZDOC + 30° = 150°. Therefore, DOC = 75°. Finally, we have ZAOC = ZAOD + ZDOC = 135°.

Note that you might also have found ZAOD = 60° by seeing that AO = AD = 2AY and ZAYO = 90°,
so AAOY is a 30-60-90 triangle (and likewise for ADOY.) 0

Concept: Breaking a desired angle, length, or area into parts often makes it easier to
find. |

Exercises

8.6.1 The area of square EFGH is 80. Find EF and EG.


8.6.2. M is the midpoint of AB on square ABCD. If AC and BD meet at O, and MO = 4, what is the area
of square ABCD?
8.6.3 The diagonals of square TYUI meet at M. Point K is on side TY such that TK = TM. Find ZMTK
and ZTMK.
8.6.4 Show that a rectangle with perpendicular diagonals must be a square.
8.6.5 ABCD and ACFG are squares. Find [ACFG]/[ABCD]. Hints: 236
8.6.6 Point E is inside square ABCD such that AABE is equilateral. Given that AB = 4, find the following:
(a) AE. (d) ZDAE and ZDEA.
(Db) MABED): (e) The area inside ABCD but outside AABE.

(c) [ABE]. (ye:


a ETE
8.7. IF AND ONLY IF

8.7 If and Only If

In this section, we learn what ‘if and only if’ means by exploring what facts we need to classify a
quadrilateral, and what facts we know given a quadrilateral’s classification.

We already have a good idea of what ‘if’ means. For example, we can write ‘An animal has four legs
if itis a dog.’ However, ‘only if’ is a little trickier — for example, the statement ‘An animal has four legs
only if it is a dog’ is clearly false. Cats have four legs, too.

Many mathematical statements put ‘if’ and ‘only if’ together. Here’s what it looks like in a non-
mathematical context:

A month has less than thirty days if and only if the month is a February.

This statement is equivalent to saying both of the following statements at the same time:

Every month with less than thirty days is a February.


Every February has less than thirty days.

In other words, to prove an ‘if and only if’ statement, we must prove two different things. In our
example above, we would have to show that every month that has less than thirty days is a February,
and we would have to show that every February has less than thirty days. Make sure you understand
that these are two different statements to prove!

‘Important: - Proving ‘if and only if’ statements requires proving two differentstate-
Z aA ments.
©

a]
Problems ig
Problem 8.23: Prove that ABCD is a parallelogram if and only if ZA = ZC and ZB = ZD.

Problem 8.24: Soe


(a) Prove that if ABCD is a square, then its diagonals bisect its angles.
(b) Is it true to say that ABCD is a square if and only if its diagonals bisect its angles?

Extra! In the course of my law reading I constantly came upon the word demonstrate. I thought at
—s first that I understood its meaning, but soon became satisfied that I did not... Iconsulted all the
- dictionaries and works ofreference Icould find... At last Isaid, ‘Lincoln, you can never make a
lawyer ifyou do not understand what demonstrate means,’ and I left my situation in Springfield, |
went home to my father’s house and stayed there till I could give any proposition in the six books
aksEuclid at sight. I then liga out what demonstrate means and went back to my law studies. z
_ =Abraham Lincoln

22%
CHAPTER 8. QUADRILATERALS

Problem 8.25: Prove that the diagonals of a trapezoid are congruent if and only if the trapezoid is
isosceles. Hints: 216 |

Problem 8.26: In this problem, we prove that the circumcenter of a triangle is the same point as the
incenter if and only if the triangle is equilateral.
(a) First, we tackle the ‘if’ part: Show that in an equilateral triangle, the circumcenter and the
incenter are the same point.
(b) In the next three parts, we tackle the ‘only if’ part of the problem by showing that if the incenter
and the circumcenter are the same point, then the triangle is equilateral. Let X be this common
point and AABC be our triangle. What does X being the circumcenter tell us about X?
(c) What does X being the incenter tell us about X?
(d) Combine your answers from the previous two parts to find congruent triangles. Use these
triangles to deduce that AABC is equilateral.

Now we'll try our hand at a few proofs (or disproofs) of ‘if and only if’ statements.

Problem 8.23: Prove that ABCD is a parallelogram if and only if ZA = ZC and ZB = ZD. ;

Solution for Problem 8.23: What's missing from this solution:

Bogus Solution: Since ABCD iissaparallelogram, we have ADI


BC, so ZA + ZB= 180°. Similarly, AB || CD
means ZB + ZC=180°. Therefore, ZA= ZC.
Similarly, wecanshow ZB=ZD.

This solution only does half the problem. ‘If and only if’ problems are two-part problems! We must
prove two statements. First, we show:

ABCD is a parallelogram if ZA = ZC and ZB = ZD.

Since the angles of a quadrilateral add to 360°, we have ZA + ZB + ZC + ZD = 360°. Since ZC = ZA


and ZD = ZB, we have 2(ZA + ZB) = 360°, so ZA + ZB = 180°. Therefore, AD || BC. Since ZA = ZC, we
have ZB + ZC = 180° also. Therefore, AB || CD and ABCD is a parallelogram.
Next, we must show:

ABCD is a parallelogram only if ZA = ZC and ZB = ZD.

In other words, we must show that every parallelogram ABCD has ZA = ZC and ZB = ZD. This is
what we did in our Bogus Solution above, so we have already tackled this part.
Therefore, ABCD is a parallelogram if and only if ZA = ZC and ZB = ZD. 0
Sometimes one part of ‘if and only if’ is clearly true, but the other half isn’t so obvious. And
sometimes the other half isn’t even true!

a TT
8.7. IF AND ONLY IF

Sh a a a a a acs
pep
(a) Prove that if ABCD is a square, then its diagonals bisect its angles.
(b) Is it true to say that ABCD is a square if and only if its diagonals bisect its angles?

Solution for Problem 8.24:

(a) We know that each diagonal of a square cuts the square into two isosceles right triangles. In other
words, each diagonal cuts the right angles at the vertices into two 45° angles. So, if ABCD is a
square, its diagonals bisect its angles.
(b) We've already proved one direction of this statement. The other direction is

Prove that ABCD is a square if its diagonals bisect its angles.

Unfortunately, try as we may, we can’t prove this! For example, the A B


diagonals of any rhombus bisect the angles of the rhombus. We can
show this quickly by noting that AAED = ACED in rhombus ABCD at
right. Hence, ZADE = ZCDE and diagonal BD bisects ZADC. Similarly,
each of the four angles of rhombus ABCD is bisected by a diagonal of
ABCD. Therefore, if the diagonals of a quadrilateral bisect its angles, D C
we cannot deduce that the quadrilateral is a square, since the quadrilateral may be a rhombus that
is not a square.

To prove a statement is true, you must have a proof that covers all
VV possibilities. To disprove a statement, all you have to do is find one case
in which the statement is false. It’s much easier to break something than
to build it!

Let's try a couple more challenging ‘if and only if’ statements.

Problem 8.25: Prove that the diagonals of a trapezoid are congruent if and only if the trapezoid is
isosceles.

Solution for Problem 8.25: Our diagram shows trapezoid ABCD with AB || CD: A B
We'll assume that ZC < 90°, because if ZC > 90°, then we note ZB < 90° and
we can then use essentially the same proof as for ZC < 90°. (What happens if
ZC = 90°?) Since we have an if and only if statement, we must prove two items.
First, we must show: D a

The diagonals of a trapezoid are congruent if the trapezoid is isosceles.

We must show that AC = BD if ZADC = ZBCD. Back in Problem 8.7, we showed that ZADC = ZBCD
implies that AD = BC. Since we also have ADC = ZBCD and CD = CD, we have AADC = ABCD by
SAS. Therefore, AC = BD.

Next, we must show:


nnn TUE UEEIEIEEEEIEEEE ERE
cats)
CHAPTER 8. QUADRILATERALS

The diagonals of a trapezoid are congruent only if the trapezoid is isosceles.

A B In other words, we must show that if the diagonals are equal, the trapezoid
is isosceles. So, we must start from AC = BD and show that ZADC = ZBCD. 'We
would like to show that AACD = ABDC, so we draw altitudes AX and BY as
shown. Since AC = BD and AX = BY (because AB and CD are parallel), we have
D C AAXC = ABYD by HL Congruence. Hence, we have ACD = ZBDC. Since we
x ¥ also have AC = BD and CD = CD, we have AACD = ABDC by SAS Congruence.
Therefore, ZADC = ZBCD.

Therefore, the diagonals of a trapezoid are congruent if and only if the trapezoid is isosceles. O
You might be wondering, ‘How in the world would we think to draw the altitudes we drew in the
second part?” We think to add these because we have to somehow use the fact that AB ||CD. We could
also have used this fact by finding some equal angles (this path is a little longer), but we know that
drawing altitudes often simplifies trapezoid problems.

Concept: When stuck on a problem, think ‘What information haven’t I used yet?’ |
Then, try to find some way to use that information. |
Z Sa

Enough with quadrilaterals; let’s go back to triangles.

Problem 8.26: Prove that the circumcenter of a triangle is the same point as the incenter if and only
if the triangle is equilateral.

Solution for Problem 8.26: What's wrong with the following solution:

Bogus Solution: Let ABC be our triangle. We can easily take care of showing that |
- the circumcenter and incenter are the same point in an equilateral -
: triangle since the angle bisectors of an equilateral triangle are the -
same lines as the perpendicular bisectors of the sides of the triangle.
Going the other way is tougher. Let BE be the A
angle bisector of 2B. Since AB = BC, ZABE =
ZCBE, and BE = BE, we have AEBC = AEBA. So, E
AE = EC and ZAEB = ZCEB = 90°. Thus, angle
bisector BE is part of the perpendicular bisector (C B
of AC. Similarly, we can show that all the angle bisectors of AABC
are the same as the perpendicular bisectors, so the triangle must be
equilateral.
& Hence a WISTS NEIN Munvesna tte

This ‘solution’ has several missteps. First, we should be more explicit in the first part about showing
why the angle bisectors and perpendicular bisectors are the same lines in an equilateral triangle. Second,
and far more serious, is that our second part of the proof is logically flawed. We assume as a step in our
‘proof’ that AB = BC; however, we can’t assume this because we don’t know anything about the sides
of AABC in the second part yet — we are trying to prove that these are equal!
Ee a ee Se eT ee ee eae
230
8.7. IF AND ONLY IF

WARNING!! _ Be very careful that you do not assume what you are trying to prove
“S as part of a proof! |

Finally, our ‘proof’ ends by stating that since the angle bisectors and the perpendicular bisectors are
the same, the triangle must be equilateral. This statement, while true, is not proved in our solution — it’s
what we are asked to prove!
Let’s try again:
Let AABC be our triangle, O be the circumcenter, and I be the incenter.

First, we show that if a triangle is equilateral, then J and O are the same. We draw A
angle bisector AD. Since AABC is equilateral, we have ACAD = ABAD by SAS. So,
CD = DB and CDA = BDA = 90°: Therefore, AD is the perpendicular bisector of
BC. Similarly, each angle bisector of AABC is also a perpendicular bisector of one of
the sides. Thus, the intersection of the angle bisectors, the incenter, is the same point C D B
as the intersection of the perpendicular bisectors, the circumcenter.

Next, we turn to the second part, proving that if the incenter and the A
circumcenter are the same point, then the triangle must be equilateral. We
let this common point be X. Since X is the incenter, it is equidistant from
all three sides. Since it is the circumcenter, it is equidistant from all three uU V
vertices. These two sets of length equalities are shown in the diagram. We
quickly see that all six little right triangles are congruent by HL Congruence.
Hence, we have AV = VB = BT = TC =CU = UA, s0 AB = BC = CA‘and
the triangle is equilateral. (Note: There are plenty of other ways to do this C + B
part; perhaps you can find another way!) 0
Another phrase mathematicians will sometimes use to mean ‘if and only if’ is ‘necessary and suffi-
cient.’ For example:

It is necessary and sufficient for a quadrilateral’s diagonals to bisect each other in order for
the quadrilateral to be a parallelogram.

This is the same as saying:

The diagonals of a quadrilateral bisect each other if and only if the quadrilateral is a paral-
lelogram.

Unsurprisingly, we even have a symbol for ‘if and only if’, ©:

The diagonals of a quadrilateral bisect each other © the quadrilateral is a parallelogram.

Usually this symbol is only used in brief statements. For example: x7=4 x= +2. Finally, sometimes
the phrase ‘if and only if’ is shortened to ‘iff.’
on ee gy RSIS TE ear ea 231
CHAPTER 8. QUADRILATERALS

Pe
Prove or disprove each of the following statements.

8.7.1. EFGH isa rhombus if and only if its diagonals are perpendicular.
8.7.2 AXYZ is equilateral if and only if XY = YZ and 2X = 60°.
8.7.3 A quadrilateral is a parallelogram if and only if it is a rhombus.
8.7.4 A rhombus is a square if and only if it is also a rectangle.
8.7.5 PQRS isa parallelogram if and only if PQ = RS and SP = QR.
8.7.6 Trapezoid ABCD with AB ||CD is isosceles if and only if ZABD = 4BAC. Hints: 466
8.7.7. WXYZ isa rectangle if and only if WX = YZ, LWXY = ZXYZ, and WX ||YZ.
8.7.8 Let M be the midpoint of AB. AABC is isosceles with AC = BC if and only if CM bisects ZACB.
Hints: 4
8.7.9 ABCD isa square if and only if its diagonals are perpendicular and congruent.

8.8x Quadrilateral Problems

In this section we will tackle a few more challenging problems using the quadrilateral principles we
have learned.
Problems fa

Problem 8.27: Diagonal BD of square ABCD is drawn. Square EFGH is then A B


inscribed in ABDC with two vertices on BD as shown. If AB = 6, find the area of
EFGH.

D CKy. f

Problem 8.28: I have a rectangular painting that has a length 10 inches longer than its width. I frame it
with a rectangular frame that is two inches wide all the way around. Given that the area of the frame
is 152 square inches, what is the area of the painting?

Problem 8.29: Find the area of trapezoid ABCD given that AB || CD, AB = 16, BC = 15, CD = 30, and
Ald = 13,

Problem 8.30: Isosceles trapezoid ABCD with AB ||CD is inscribed in a circle with radius 10 such that
the center of the circle is inside ABCD. Isosceles trapezoid CDEF, with CD || EF, is inscribed in the
same circle, but the center of the circle is not inside CDEF. Given that AB = EF = 12 and CD = 16, find
the areas of ABCD and CDEF.

aei aR IE SINS Ne
8.8x. QUADRILATERAL PROBLEMS

Problem 8.31: Prove that the sum of the squares of the diagonals of a parallelogram equals the sum
of the squares of the sides of the parallelogram.

Problem 8.27: Diagonal BD of square ABCD is drawn. Square EFGH is then A


\inscribed in ABDC with two vertices on BD as shown. If AB = 6, find the area of
EFGH. 7 : x

Solution for Problem 8.27: We need the side length of EFGH to get its area, so we let the side length of
EFGH be x. Since AEBF is a 45-45-90 triangle, we have BF = EF V2 = x V2. We also have BE = EF = x
(and from ADHG, DH = x). Because BD ||FG, we have ZFGC = 45°, so AFCG is also a 45-45-90 triangle.
Therefore, FC = FG/V2 = x/V2 = x ¥2/2. We know BC = 6, so we have BF + FC = BC = 6. Substitution
gives

2
x V2
Hitaye te:

Therefore, x (337) = 6, s0 x = 2 Y2. So, the area of EFGH is EF* = x? = (2 V2)? = 8. Alternatively, we
could have noted that BD = 3x = 6 V2,sox =2 V2. 0

Concept: Don’t just stare at a diagram. Set A Bx


sides you seek equal to variables
and find other lengths in terms of
those variables. Label the diagram | x V2
when you find these new lengths. 6
Your final diagram for the last prob-
lem might look like the one at right. is

D ye
Wve Ca
C

Problem 8.28: [havea rectangular painting that has a length 10 inches longer than its width. I frame
it with a rectangular frame that is two inches wide all the way around. Given that the area of the
frame is 152 square inches, what is the area of the painting?

Solution for Problem 8.28: We start by making a sketch of our painting x+14
with its frame. We let the width of the painting without frame be x, so
the length is x + 10. Since the frame is two inches thick, we find that x +10
the painting with frame is x + 14 inches long and x + 4 inches wide. We x x+4
can then find the area of the frame in terms of x by subtracting the small pL
rectangle (the picture) from the large one (picture plus frame). The area

233
CHAPTER 8.a
QUADRILATERALS
et
a

of the picture plus frame is (x + 14)(x + 4) and the area of the picture alone is x(x + 10), so we have:

(x + 14)(x + 4) — x(x + 10) = 152.

Expanding the left side gives x? + 18x + 56 — x= 10x = 152) 5014 — 12.
Now we can find the area of the painting without the frame: x(x + 10) = 12(22) = 264 square inches.
CO

Concept: A picture is worth a thousand words — making a sketch can greatly help
©==2 ~=syou understand a geometric problem.

Problem 8.29: Find the area of trapezoid ABCD given that AB || CD, AB = 16, BC = 15, CD = 30,
and AD = 13.

Solution for Problem 8.29: We have the bases, so we just need the height. We draw
altitudes from A and B, thus forming a rectangle and two right triangles. (We like
right triangles!) Since XY = AB = 16 and CD = 30, we know that DX + CY = 14.
At this point, there are a couple approaches we could take to find the height.
Wishful thinking. We have the hypotenuses of AAXD and ABYC, and we know
they each have a leg of length h. The 15 and 13 make us think of the Pythagorean triples 5-12-13 and
9-12-15. These each have a leg of length 12, so perhaps our height is 12. This would make DX = 5 and
and CY = 9,so DX + CY = 14, as desired. Our wishful thinking has been fruitful! The height must be
12, so we can now find the area.

Algebra. Perhaps we don’t see the Pythagorean triples, or maybe they don’t work out (the answers
won't always be integers!); then, we have to do a little more work. We let DX = x, so CY = 14-— x. Now
we can use the Pythagorean Theorem on AADX and ABYC to build two equations in terms of h and x:

a Ph S13"
(4x) Sie 35°
We can subtract the first from the second to eliminate h?:

(VA Pee 15% 13%,


Factoring both sides as a difference of squares gives

(14-x —x)(14 =x + x) = (15 — 13)(154 13),

so (14 — 2x)(14) = 56, which gives x = 5. Now we can find h = 12 with either of our original equations.

Finally, we find our area: h(AB + CD)/2 = 276. o

Rectangles and right triangles are easier to work with th: rape
Ke
bpeh ©)
HHS GUNA
ea 4ean
,

Roni Na _ When stuck on a trapezoid problem, consider dropping altitudes,


234
8.8x. QUADRILATERAL PROBLEMS

Problem 8.30: Isosceles trapezoid ABCD with AB || CD is inscribed in a circle with radius 10 such
that the center of the circle is inside ABCD. Isosceles trapezoid CDEF, with CD || EF, is inscribed in
the same circle, but the center of the circle is not inside CDEF. Given that AB = EF = 12 and CD = 16,
find the areas of ABCD and CDEF.

Solution for Problem 8.30: We’ll tackle ABCD first. In our diagram, we connect
the center of the circle, O, to the vertices of the trapezoid, since we somehow
want to use the fact that the trapezoid is inscribed in a circle. We have OA =
OB = OC = OD = 10. We need a height, so we draw the height through O,
since this will build right triangles in which we already know the hypotenuses.
We let X and Y be the feet of the altitudes from O to AB and CD, respectively.

Since AO = BOand XO = XO, we have AAXO = ABXOby HL Congruence.


Therefore, X is the midpoint of AB. Similarly, Y is the midpoint of CD. Hence,
AX = 6and DY = 8. We can now use the Pythagorean Theorem on AXOA and ADOY to determine that
XO = 8 and YO = 6. Thus, the height is XO + YO = 14, and our area is (XY)(AB + CD)/2 = 196.

Now we find the area of CDEF. We could break the problem into right triangles as before, or we
could be a little more clever and apply what we learned about AAOB to AEOF. Since AO = EO, AB = EF,
and BO = FO, we have AAOB = AEOF. Therefore, the altitudes from O to AB and to EF are congruent.
Letting Z be the foot of the altitude from O to EF, we have OZ = OX = 8. Moreover, since CD ||EF and
OZ 1 EF, we have OZ 1 CD. Since Y is the foot of the altitude from O, OZ passes through Y. So, the
height of CDEF is YZ = OZ — OY = 2. Finally, the area of CDEF is (YZ)(CD + EF)/2 = 28. 0

We'll finish with a challenging proof.

Problem 8.31: Prove that the sum of the squares of the diagonals of a parallelogram equals the sum
of the squares of the sides of the parallelogram.

Solution for Problem 8.31: Let ABCD be our parallelogram. We wish to prove

AC’ + BDO = AB* + BC" + CD? DAZ.

Since AB = CD and AD = BC, we can simplify this to

AC* + BD* =2(AD* +. CD*).


Seeing the sums of squares of sides, we reach for the Pythagorean Theorem. We have no right triangles,
so we'll have to build them. We draw altitudes from A and B, thus building right triangles with our
diagonals as hypotenuses. From right triangles AAPC and ABQD, we have

AC2o= PC?.4-AP*
BD* = DQ*+BQ

We can add these to get an expression for the sum of the squares of the diagonals:

AC? + BD? = PC? + AP* + DQ? + BQ”.

235
CHAPTER 8.tt
QUADRILATERALS
hd al
pa

That looks like what we want, but we’re not quite there. We have to get AD and CD in the equation
somehow. We start by noting that DQ = CD + CQ and PC = CD - DP. We also note that AP = BQ and
AD = BC, so AADP = ABCQ by HL Congruence. Thus, DP = CQ. Now our equation becomes:

AC? BD# PEA APD Oar BOs


= (CD—-DP) + AP? +(CD + CQ)? +B’
= CD*—2(CD)(DP) + DP? + AP? + CD? + 2(CD)(CQ) + CQ’ + BQ?
= CD? -—2(CD)\(DP) + DP? + AP* + CD? + 2(CD)(DP) + DP? + AP?
= 2(CD*) + 2(DP? + AP”).
Since DP? + AP* = AD* from right triangle ADAP, we have the desired

ACA BLY = (Giae AD):


As we noted earlier, CD = AB and AD = BC, so we've shown that the sum of the squares of the sides of
a parallelogram equals the sum of the squares of its diagonals. 0

Exercises —

8.8.1 In rectangle ABCD, H is the midpoint of BC, E lies on AD, and F lies on AB. In rectangle CEFG,
H lies on FG and HG = 3. Given ZDEC = 45°, what is the positive difference between the areas of these
two rectangles?

8.8.2 Find the area of trapezoid ABCD shown at right. Boles


LJ

8.8.3 Quadrilateral ABCD is a trapezoid with AB || CD. We know AB = 20 and


CD = 12. What is the ratio of the area of AACB to the area of ABCD? (Source: =
MATHCOUNTS) Hints: 376 dp amai D
8.8.4 The diagonals of EFGH are perpendicular. Prove that EF* + GH? = FG? + EH. (A quadrilateral
with perpendicular diagonals is sometimes referred to as orthodiagonal.) Hints: 572
8.8.5% In AABC, AB = 6, BC = 7, and AC = 8. Given that M is the midpoint of AB, find CM. Hints:
240, 476, 117

8.9 Summary

| Definitions: A quadrilateral, such as ABCD at the right, has four segments as


| sides, four vertices, and four angles. The segments connecting opposite vertices are
called the diagonals of a quadrilateral. : D

: Important: The interior angles of any quadrilateral add to 360°.

236
8.9. SUMMARY

Definitions: A trapezoid is a quadrilateral with two parallel sides. The segment


connecting the midpoints of the non-parallel sides is the median of the trapezoid,
and the distance between the two parallel sides is the height of the trapezoid.

Important: e The median of a trapezoid is parallel to the bases of a trapezoid,


and equal in length to the average of the lengths of the bases.

e The area of a trapezoid equals the height of the trapezoid times


the length of the median of the trapezoid.

Important: In an isosceles trapezoid:

(a) The base angles come in two pairs of


equal angles as shown at right.
.
(b) The legs are equal.
: : De
equal.
(c) The diagonals are

If any one of these items is true foratrapezoid with exactly one pair of
parallel sides, then all the others must be true for thattrapezoid. .

Definitions: A parallelogram is a quadrilateral in which both pairs of opposite sides are parallel.

Important: e The area of a parallelogram is the product of a side length (the


Vv? base) and the distance between that side and the opposite side
of the parallelogram. This distance between opposite sides is
called a height of the parallelogram.
In parallelogram ABCD, the opposite sides are equal, the oppo-
site angles are equal, and the diagonals bisect each other.

A B A B

D C D G

Conversely, ABCD is a parallelogram if any one of the following}


are true:
(a) AB =CDand AD = BC.
(b) ZA= ZCand ZB = ZD.
(c) Diagonals AC and BD bisect each other.
Therefore, proving one of these means the other two are true.

237
CHAPTER 8. QUADRILATERALS

Definition: A quadrilateral is a rhombus if all of its sides are equal.

Important: e Every rhombus is a parallelogram. Therefore, everything that


VY is true about parallelograms is true about rhombi.

e The diagonals of a rhombus are perpendicular. The area of a


rhombus is half the product of its diagonals (and also equals its
base times its height).

Definition: A quadrilateral in which all angles are equal is a rectangle.

Important: e All rectangles are parallelograms, so all that is true of parallel-


vv? ograms is true of rectangles.
e Let two consecutive sides of a rectangle have lengths € and w.
The area of the rectangle is fw, and the diagonals of the rectangle
both have length vé? + w’.

Definition: A quadrilateral in which all sides are equal and all angles are equal is a square.

Important: e Each square isa parallelogram, a rectangle, and a rhombus so


Vv? all that is true of a parallelogram, a rectangle, or a rhombus is
true of a square.

e If the side length of a square is s and its diagonal is d, then


d = s V2 and the area of the square is s?, or d?/2.

In proving various facts about quadrilaterals, we encountered the phrase ‘if and only if.’ Proving ‘if
and only if’ statements requires proving two different statements.

Problem Solving Strategies

oea ee re
REVIEW PROBLEMS

Concepts: .. . continued from the previous page


O==
e Breaking a desired angle, length, or area into parts often makes it |
easier to find.

e When stuck on a problem, think ‘What information haven’t I used |


yet?’ Then, try to find some way to use that information.

e Don’t just stare at a diagram. Set sides you seek equal to variables |
and find other lengths in terms of those variables. Label the diagram |
when you find these new lengths.

e A picture is worth a thousand words — making a sketch can greatly |


help you understand a visual problem.

Things To Watch Out For!

WARNING! e Although every rhombus is a parallelogram, every parallelo-


“S gram is not a rhombus. Therefore, if we prove a property of |
rhombi, this property is not necessarily true for parallelograms. |
(The same is true of rectangles — rectangles are parallelograms, |
1

but not all parallelograms are rectangles, etc.) |


e Be very careful that you do not assume what you are trying to |
prove as part of a proof!

f& REVIEW PROBLEMS a

8.32 Fill in each of the boxes with the answer “Yes”, “Sometimes”, or “No”.

Fact“ Paralelogram |Rhombus |Rectangle |Square|


Opposite
sideshaveequallength | |
|
Allsideshaveequallength | |
[Opposite sidesareparallel | |
Opposite anglesareequal ||
|
Allanglesareequal | |
/Diagonalsbisecteach other | |
Diagonalshave equallength | |
[Areacan bedetermined fromsides |
|
239
CHAPTER 8. QUADRILATERALS

8.33 In quadrilateral ABCD, ZA = ZB = 128°, and ZD is 10° less than 5 times ZC. Find ZC and ZD.

8.34 In isosceles trapezoid ABCD, ZA = x and ZB = 2x — 45°. Find 4C and ZD in the following cases:

(a) AB|| CD.


(b) AD ||BC.

8.35 In which of the following quadrilaterals does a diagonal always divide the quadrilateral into two
regions of equal area: Rhombus, Square, Rectangle, Trapezoid, Parallelogram?
8.36 If the numbers denoting the perimeter and area of a square are equal, what is the length of its
diagonal?
8.37 If A,B,C, Dare midpoints of the sides of square EFGH, what is the ratio of the area of triangle ABC
to the area of square EFGH?
8.38 The bases of a trapezoid are 2x and 4x. The height is 2x. If the area is 48, what is x?
8.39 One of the angles of a rhombus is 120°. If the shorter diagonal is 2, what is the area?
8.40 What is the number of centimeters in the length of a longer side of a rectangle that has a perimeter
of 64 centimeters and an area of 192 square centimeters?
8.41 Quadrilateral WXYZ is a parallelogram. Given that WX = 2x —-3, XY =x +7, and YZ = 3x —8,
find the perimeter of WXYZ.
8.42 WXYZ isa rhombus with ZX = 90°. Prove that WXYZ is a square.

8.43 One less well-known type of quadrilateral is the kite. The four sides of a kite A
can be split into two pairs of consecutive equal sides. For example, the figure at right
shows kite ABCD with AB = BC and CD = DA. The diagonals of ABCD meet at E as
shown. Solve the following problems about kite ABCD. D B
(a) Prove that ZABD = ZCBD.
(b) Prove that AC 1 BD. Hints: 140, 249 C
8.44 Which of the following quadrilaterals is (are) a special case of a kite: parallelogram, trapezoid,
rhombus, rectangle, square? (More than one answer may be correct.)

8.45 Each side of an equilateral triangle is 8 inches long. An altitude of this triangle is used as the side
of a square. What is the number of square inches in the area of the square? (Source: MATHCOUNTS)

8.46 Shownat right is rectangle EFGH. Given that ZGHF = 31° and EY = HY,
solve the following problems:
(a) Find ZXEH.
(Bb) rinde2 EF
(c)x Prove that EY passes through G. Hints: 366, 214
8.47 EFGH isa rhombus. Given that EF = EG = 6, find the area of the rhombus. Hints: 404

240
CHALLENGE PROBLEMS

8.48 Prove or disprove this statement: ABCD is a square if and only if its sides are equal in length and
AC = BD.
8.49 The angles of quadrilateral WXYZ are such that ZW > ZX > ZY > ZZ. The angles are in an
arithmetic progression, meaning that ZW — ZX = £X — LY = LY - ZZ.
(a) If ZW is four times the measure of ZZ, what is ZW?
(b) If ZW is two times ZZ, what is ZW?

8.50 In quadrilateral ABCD, let E, F,G, and H be midpoints of AB, BC, CD, and DA, respectively. Prove
that EFGH is a parallelogram.

8.51 A square and an equilateral triangle have the same perimeter. What is the ratio of the area of the
square to the area of the equilateral triangle?

A B 8.52 Trapezoid ABCD is divided into four congruent trapezoids as shown. Given
AB = 4and CD = 8, find the sum of the lengths of all line segments in the figure.
a : (Source: ARML)

8.53 What is the ratio of the area of the shaded square to the area of the large square in the
figure at right? (The figure is drawn to scale.) (Source: AMC 8)

8.54 A street has parallel curbs 40 feet apart. A crosswalk bounded by two parallel stripes :
crosses the street at an angle. The length of the curb between the stripes is 15 feet and each stripe is 50
feet long. Find the distance between the stripes. (Source: AMC 10)

Challenge Problems

W Xx 8.55 In the diagram at left, AX = XY = 6, ZAXY = 70°, ZW = 125°,


LX ZAR 21° -andZxZw =34".

(a) Find ZZXA.


(b) Find WZ.
a A Mo) ey Uerbve that WY
2 XZ “Hintst 328" 558
8.56 In rectangle ABCD, AB = 16 and AD = 5. F is on AB and G on CD such that FG, BD, and AC are
concurrent at point O. Find [FOB] + [GOC]. (Source: MATHCOUNTS) Hints: 190

8.57 Each angle of rectangle ABCD is trisected by a pair of segments. The angle A NEw B
trisectors meet at P, Q, R, and S, as shown in the diagram at right. Prove that
PQRS isa rhombus. Hints: 243 “0

8.58 In rectangle ABCD, M is the midpoint of BC. Points P and Q lie on ABand D C
De respectively, such that PB = 4/3-BC and ZPMQ isa right angle. What is the ratio PM : MQ? (Source:
MATHCOUNTS) Hints: 306

241
CHAPTER 8. QUADRILATERALS

8.59 In the diagram, ABCD is a parallelogram and E is on the extension of BC


past C. AE and CD meet at F. Given [ADF]= 64 and [CEF]= 4, determine the
following:

(a) GE DE.
(b) [BFC]. Hints: 373
(c) [ABCD]. Hints: 213

(Source: ARML)

8.60 See Problem 8.43 for the definition of a kite.

(a) Find the area of a kite that has diagonals of length 10 and 12. Hints: 118, 299
(b)x Find the area of kite ABCD if AC = 6 V2, AB = 4 V3, and BC = 2 V3. Hints: 193, 18, 527

8.61 Consider a square of side length 1. We draw four lines that each connect a midpoint of a side with
a comer not on that side, such that each midpoint and each corner is touched by only one of these lines
as shown at left below. Find the area of the shaded region. (Source: HMMT) Hints: 303

Figure 8.9: Diagram for Problem 8.61 Figure 8.10: Diagram for Problem 8.62

8.62 Given MP ||NO, PO ||MN, MA = AP, and NC = CO as in the diagram at right above, prove that
[ABCD] = [MNOP]/4. Hints: 386

8.63 The diagonals of WXYZ meet at A. Prove that the quadrilateral formed by connecting the incenters
of AWXA, AXYA, AYZA, and AZWA has perpendicular diagonals. Hints: 368, 459

8.64 EFGH isa rectangle with EF = 12 and area 192. EGJI is a parallelogram such that H is on JI. What
are the possible values for the area of EGJI? Hints: 486

8.65 In rectangle ABCD, we have AB = 8, BC = 9,H ison BC with BH = 6,Eison G


AD with DE = 4, lineEC intersects line AH at G, and F is on line AD with GE . AE.
Find the length of GF. (Source: AMC 10) Hints: 521,379
8.66 In trapezoid ABCD, AD || BC. ZA = ZD = 45°, while ZB = ZC = 135°. If
AB = 6 and the area of ABCD is 30, find BC. (Source: HMMT) Hints: 552
8.67 A circle is drawn through the vertices of square ABCD, and point X is on
minor arc AB. Given that [XAB]= 1 and [XCD]= 993, find [XAD]+[XBC]. (Source: fi D4E A
Mandelbrot) Hints: 412

_ Extra! A pupil from whom nothing is1 ever demanded that he sites do, never does gilhecn,
- ossa ; - John Stuar 5

242
CHALLENGE PROBLEMS

8.68x The lengths indicated on the rectangle shown are in centimeters.


What is the number of square centimeters in the area of the shaded region?
(Source: MATHCOUNTS) Hints: 516

8.69x P is inside rectangle ABCD. PA = 2, PB = 3, and PC = 10. Find PD.


(Source: HMMT) Hints: 247, 381
9 5
E 8.70x In trapezoid ABCD shown, AD || BC, AB = 6, BC = 7, CD = 8,
ee es and AD = 17. The sides AB and CD are extended to meet at E. Prove
. & ae C that ZE = 90°. (Source: HMMT) Hints: 511

A D

Extra! One of the oldest known puzzles is the Chinese ch’ ch’iao t’u, which means ‘ingenious
Mm ttt SeVen-piece plan.’ In English, these are known as tangrams.

Rou
cr
Tangrams involve rearranging the seven pieces that together make the square shown
above into other given shapes. Some shapes that can be formed by these pieces are
shown below. See if you can figure out how!

Say Vey! er
Perhaps you'll notice a couple things tangrams have in common with math problems.
First, with experience, they get a lot easier. Second, often the simplest looking shapes
end up being the hardest ones to solve! Source: Martin Gardner’s Mathematical Puzzles &
Diversions

243
CHAPTER
He eee 9. POLYGONS
NN

Construction of a Regular Pentagon

Bees... .by virtue of a certain geometrical forethought. ..know that the hexagon is greater than the square and the
triangle, and will hold more honey for the same expenditure of material. —Pappus

CHAPTER

ee ee eos ee Polygons

9.1 Introduction to Polygons

We've tackled three sides, and we’ve handled four sides. Why stop there? Figure 9.1 shows several

Ta
different polygons, which are closed planar figures with line segments as boundaries.

Figure 9.1: Some Polygons

244
9.1. INTRODUCTION TO POLYGONS

All of the names of the parts of a general polygon are the same as in the polygons we’ve learned
about so far (triangles and quadrilaterals). The segments that form the boundary are sides, which meet
at vertices to form the interior angles. If we connect two vertices that are not adjacent on the polygon,
we form a diagonal, such as diagonal AC in Figure 9.2.

B ‘e

A
E D

Figure 9.2: Polygon Diagonal

Polygons are classified by the number of sides they have. Here are some names given to different
polygons.

Number of Sides Polygon Name


triangle
4 quadrilateral
4) pentagon
6 hexagon
7 heptagon
8 octagon
9 nonagon
10 decagon
12 dodecagon
n n-gon

A polygon in which all the sides are equal and all the angles are equal is called a regular polygon.
We’ve already seen two examples of regular polygons: equilateral triangles and squares. Figure 9.3
shows a few more regular polygons.

OOO Figure 9.3: Some Regular Polygons

As with quadrilaterals, we'll be focusing on convex polygons in this chapter unless otherwise stated,
which means that the interior angles of the polygons are all less than 180°.

Extra! The most distinct and heasile statement of any truth must at last take mathematical form.
Dn dundindind : ? _-Henry David Thoreau

245
CHAPTER 9. POLYGONS

9.2 Angles ina Polygon

In this section, we explore not only the interior angles of a polygon, but also the exterior angles. Exterior
angles are formed by extending the sides of the polygon. For example, BAX in the figure below is an
exterior angle of ABCDE.

Problem 9.1: Find the sum of the measures of the interior angles of pentagon
ABCDE shown without measuring the angles. (Hint: See how we did Problem 8.1.)

Problem 9.2: In this problem we find a formula for the sum of the interior angles of a convex polygon
with n sides. (By convex, we mean that all interior angles have measure less than 180°.)
(a) Draw all the diagonals from a single vertex of a polygon with n sides. In terms of n, how many
triangles are formed?
(b) Use your dissection from part (a) to find a formula for the sum of the measures of the interior
angles of a polygon.

Problem 9.3: The diagram shows the 10 exterior angles of a decagon. Find the sum of
these angles. (Note: You cannot assume all the exterior angles have the same measure!)
Hints: 517

Problem 9.4: Find the number of sides in a regular polygon in which each interior angle measures
1728

Problem 9.5: ABCDEFGHIJKLMNO is a regular 15-gon.


(a) Find ZACB.
(b) Find ZACD. Hints: 48
(c) Find ZADE. Hints: 250

OC Lee
9.2. ANGLES IN A POLYGON

We found the sum of the interior angles of a quadrilateral by using a diagonal to cut the quadrilateral
into triangles. This strategy works for all polygons.

Problem 9.1: Find the sum of the measures of the interior angles of pentagon D
ABCDE shown without measuring the angles.

Solution for Problem 9.1: We don’t know anything about pentagons, but we know D
plenty about triangles. Therefore, we use diagonals to cut the pentagon into
triangles. The sum of the angles in the pentagon is the same as the sum of the C
angles in the three triangles AABC, AACD, and AADE. Each triangle contributes E
180° to the total, so the sum of the interior angles in a pentagon is 3(180°) = 540°. A B
O

Problem 9.2: Find a formula for the sum ofthe interior angles of a convex polygon with n sides.

A Solution for Problem 9.2: As we did with the pentagon, we can break any polygon into
triangles by drawing all the diagonals from one of the vertices. The sum of the angles
in these triangles equals the sum of the angles in the polygon. Therefore, all we have to
do is figure out how many triangles we form when we draw the diagonals. We can use
our pentagon example for guidance. Let A be the common vertex of all our triangles.
Each side of the polygon, except the two sides connected to A, is the side opposite A
in exactly one of our triangles. So, there must be n — 2 triangles formed. Since there
are 180 degrees in each triangle the sum of the angles in an n-gon is 180(n — 2) degrees. (It’s a little more
complicated to prove, but this result also holds for concave polygons.) 0

Important: Thesum of the interior angles in an n-sided polygon is 180(n—2) degrees.


Therefore, the measure of each of the n interior angles in a regular n-gon
is 180(n — 2)/n degrees.

Now we turn our attention from the interior angles to the exterior angles.

Problem 9.3: The diagram shows the 10 exterior angles of a decagon. Find the
sum of these angles. (Note: You cannot assume all the exterior angles have the
a”
same measure!)

Solution for Problem 9.3: Each of the exterior angles is supplementary to one of the interior angles, so we
know that
Sum of interior angles + Sum of exterior angles = 10(180°) = 1800°,

247
CHAPTER 9. POLYGONS

since together, the interior and exterior angles make up 10 straight angles. Since the sum of the interior
angles of a decagon is (180°)(10 — 2) = 1440°, we have

Sum of exterior angles = 1800° — 1440° = 360°.

In the Exercises at the end of the section, you'll be asked to work through this problem for a general
n-gon. You'll thereby prove:

Important: The sum of the exterior angles in a convex polygon with n sides is 360°.
vv Therefore, the measure of each exterior angle in a regular n-gon is 360° /n.
=

Let’s try using our new knowledge about the angles of polygons on a couple problems.

Problem 9.4: Find the number of sides in a regular polygon in which each interior angle measures
i.

Solution for Problem 9.4: We let the polygon have n sides. There are two general approaches we can take
to this problem:
Focus on the interior angles. Since the measure of one angle is 172°, and the polygon is regular, the
sum of all angles is 172n degrees. Since we know the sum is also given by 180(n — 2) degrees, we can
solve the equation 172n = 180(n — 2) to find 8n = 360, so n = 45.

Focus on the exterior angles. Since the measure of each interior angle is 172°, each exterior angle is
180° — 172° = 8°. The sum of the exterior angles is 360°, so there must be 360/8 = 45 of them. O

Concept: Sometimes thinking about the exterior angles of a polygon offers a simpler
approach than thinking about the interior angles!

Problem 9.5: ABCDEFGHIJKLMNO is a regular 15-gon. Find ZACB, ZACD, and ZADE. E

Solution for Problem 9.5: We start with a diagram, then we find whatever B C D
angles we can determine. Since the polygon is a regular 15-gon, each exterior
angle is 360/15 = 24 degrees. Therefore, each interior angle is 180° - 24° = 4 E
156°. Since AABC is isosceles, BAC = ZACB. The sum of the angles of
AABC gives us ZABC + ZBAC + ZACB = 180°, so
156° + ZACB + ZACB = 180°.

Therefore, 2ZACB = 24°, so ZACB = 12°. We also have

ZACD = ZBCD=ZACB = 156" = 12° = 144°.

Turning to ZADE, we try the same strategy: find ZADC, then subtract it from ZCDE. We note that ABCD
is an isosceles trapezoid in which BC ||AD (make sure you see why), so ZADC = 180° — ZBCD = 24°. So,
ZADE= 1567— 24° = 132". 0

MS
ee
9.3. POLYGON AREA

||
__Exercises
9.2.1 Complete the table below for regular polygons.

Name # of Sides |Sum of Interior Zs |Int. Measure | Ext. 2 Measure


pentagon 5 540° 108° Wen
hexagon 6
heptagon 7
octagon 8
nonagon 9
decagon 10
dodecagon We
pentadecagon | 15
icosagon | 20
triacontagon 30 |

9.2.2 Does the formula for the sum of the interior angles in an n-gon still work
if the polygon is concave, as in the figure shown at right?
9.2.3. Each interior angle of a certain regular polygon has measure 160°. How
many sides does the polygon have?
9.2.4 The measures of the angles of a pentagon are in the ratio of 3: 3:3: 4:5. What is the number of
degrees in the measure of the largest angle? (Source: MATHCOUNTS) Hints: 587
9.2.5 The sum of the interior angles of a polygon is three times the sum of the exterior angles. How
many sides does the polygon have?

9.3 Polygon Area

[|
_Proviems
i
Problem 9.6: Find the area of pentagon ABCDE given the side lengths and right
angles shown. Hints: 538 a

Problem 9.7: In this problem we find the area of a regular hexagon with side length 8.
(a) Draw allthree long diagonals of the hexagon; what kind of triangles do you form?
(b) Find the area of the hexagon by finding the areas of the triangles from (a).

249
GHARTER 9.SPOLYGONS eee

Problem 9.8: In this problem we will find the area of regular octagon ABCDEFGH with side length 4.
(a) Extend sides AB, CD, EF, and GH to meet at points W, X, Y, Z. What type of quadrilateral is
WXYZ?
(b) Find the area of WXYZ.
(c) Use [WXYZ] to find the area of ABCDEFGH.

Problem 9.9: The center of a regular polygon is the point inside the regular polygon that is equidistant
from all of the vertices of the polygon. Let the distance from the center of a regular n-gon to one of its
sides be k and the length of each side of the n-gon be s. Break the polygon into triangles by connecting
the center of the polygon to each vertex. Find a formula for the area of the n-gon in terms of k, n, and
S.

We’ve seen all sorts of problems involving triangle and quadrilateral areas. You won't be at all
surprised that we rely heavily on triangles and quadrilaterals when we find the areas of polygons with
more sides.
Problem 9.6: Find the area of pentagon ABCDE given the side lengths and
right angles shown.

Solution for Problem 9.6: We start by drawing BE to separate right triangle AABE
from quadrilateral BCDE. We have [ABE] = (V¥3)(V6)/2 = 3 V¥2/2, so all we need
is [BCDE]. From the Pythagorean Theorem, we have BE = VAB? + AE? = 3, so
BCDE is arhombus. Since one of its angles is a right angle, it must also be a square.
Therefore, [BCDE] = 9 and [ABCDE] = [BCDE] + [ABE] = 9 +3 ¥2/2. 5

Let’s find the areas of a couple regular polygons.

Problem 9.7: Find the area of a regular hexagon with side length 8.

Solution for Problem 9.7: We start by breaking the hexagon into pieces we can handle.
Since the angles of a regular hexagon are each 120°, the long diagonals split each angle
into two 60° angles. Therefore, each of the little six triangles formed by drawing all
three long diagonals is equilateral. Thus, the area of the hexagon is 6 times the area of
an equilateral triangle with side length 8. As we saw in Problem 6.11 on page 143, the
area of such a triangle is 82 V3 /4 = 16 V3. So, the area of our regular hexagon is 6(16 v3) = 96 V3. 0

250
RS Ane LOCO Nae
|Concept: Many problems involving regular hexagons can be tackled by dissecting |
| C= J the hexagons into equilateral triangles.

Problem 9.8: Find the area of regular octagon ABCDEFGH with side length 4.

Solution for Problem 9.8: We can find the area with a dissection (try drawing AF,
BE, CH, and DG), but we can find a more elegant approach. We know each angle
of a regular octagon is 135°. Since these angles are supplements of 45° angles, we
start thinking about 45-45-90 triangles. This makes us think about squares, too. We
extend the sides of the octagon, and we form square WXYZ. Our octagon is just
WXYZ with the four corners (the four 45-45-90 triangles) lopped off.

We can handle squares and 45-45-90 triangles. Since AH = 4, we have WH = 4/ V2 = 2 V2. Similarly,
each of the legs of the little right triangles has length 2 V2. Therefore, the area of each little triangle is
(2 ¥2)(2 ¥2)/2 = 4. Since
[WXYZ] = WZ? = (WH +HG+GZ)* = (4+4 V2)? = 48 + 32 V2,
the area of our octagon is

[ABCDEFGH] = [WXYZ] — 4[WAH] = 32 + 32 V2.

“Sidenote: The figure below shows how an equilateral triangle can be dissected into
ay four pieces that can be reassembled to form a square.

Br Bo
Similarly, a regular dodecagon can be dissected into six pieces that can be |
reassembled to form a square.

|
||

|
||
ef ee |

|| ||
Bao Lee
| ma ||
||
| Po ||
|
|

|
Can you find more regular polygons that can be dissected to form other |
i

}
f 2

i
: 4

251
CHAPTER 9. POLYGONS

We now have another area-finding tool:

coe Sometimes vwe can view our desired 2area as the ‘leftover’ portion from
ee oes spe pices taken Sh EE from a ue Sarin figure.

Now we turn to the general problem of finding the areas of regular polygons.

Problem9.9: The center of a regular polygon is the point inside the regular polygon that is equidistant
from all of the vertices of the polygon. Let the distance from the center of a regular n-gon to one of
its sides be k and the length of each side of the n-gon be s. Find a formula for the area of the n-gon in
terms of k,n, and s.

Solution for Problem 9.9: Let O be the center of the polygon. (You will be asked Auk nt
to prove that such a point exists as a Challenge Problem.) We can cut the
regular polygon into n triangles by connecting the center to every vertex. One
such triangle is AOAZ, shown in the diagram. The altitude to side AZ of this
triangle is the distance from O to a side of the polygon, given in the problem
as k. Since AZ is also given as s, we have [AZO] = (AZ)(OP)/2 = ks/2. Finally,
the polygon consists of n of these triangles, so the area of the whole polygon
is nks/2. O

The distance from the center to the sides of a regular polygon is sometimes
given the obscure name of the apothem. Noting that the perimeter of the polygon is ns, we can write
our area formula as:

Important: The area of a regular polygon is half its perimeter times the distance
Vv from the center of the polygon to a side.

As we have seen in earlier problems, this isn’t always the fastest way to find the area. (Try it on the
dodecagon above if you want to torment yourself.)

‘Sidenote: One Darticulenly fascinating dissection is that of


a regular dodecagon shown at right. See if you
can figure out how to use the dissection to prove
that the area of the dodecagon is 3 times the ea ag
—__—

square of the radius of the dodecagon’s circum-


circle, which is shown in bold in the diagram.
This dissection of a regular dodecagon is used as
the logo for the Art of Problem Solving Founda-
tion, which supports problem solving activities aS<= |
for eager middle and high school students. You LSS can
e ers s
tion on the Foundation (and a graphical proof of the See c
above) at www. artofproblemsolving. org.

e202
9.3. POLYGON AREA

9.3.1 ABCDEF is a regular hexagon with side length 9.


(a) Find AD.
(b) Find [ABCDEF].
(c)x Find AC.
9.3.2 Find a formula for the area of a regular hexagon with side length s.

9.3.3 What’s wrong with pentagon ABCDE shown at right?


9.3.4 ABCDEFGH isa regular octagon with side length 8. Diagonals AE and CG
meet at X. Point M is the midpoint of AB.
(a) Find the area of ABCDEFGH.
(b) Find XM, the apothem of the octagon.
(c) Find XC. Hints: 22

9.3.5 Points A, B, C, D, E, and F are the vertices of a regular hexagon and also trisect
the sides of the large equilateral triangles shown. Given that the area of ABCDEF is 24,
what is the total area of the shaded regions? (Source: MATHCOUNTS)

9.3.6 We solved Problem 9.8 by extending the sides of a regular octagon to form a
square. We didn’t, however, prove that we form a square when we connect the points
where these extensions meet. Fix this oversight by providing the proof. Hints: 564
9.3.7x In this section we assumed that the long diagonals of a regular hexagon are concurrent. In this
problem we fix this oversight by proving that these diagonals are concurrent.

(a) Let the hexagon be ABCDEF and let point O be the intersection of the bisectors of ZA and ZB.
Prove that AAOB is equilateral.
(b) Draw OC. Prove that ABOC is equilateral.
(c) Prove that ACOD is equilateral and that AO goes through D.
(d) Prove that the long diagonals of ABCDEF all meet at the same point.

253
CHAPTER 9. POLYGONS

9.4 Polygon Problems

Probiems ia
Problem 9.10: Diagonals BD and CE of regular pentagon ABCDE meet at Q. A
Find ZBOE.

po Ne
Problem 9.11: In this problem we will find a formula for the number of diagonals in a polygon with
n sides.
(a) How many diagonals does a square have?
(b) How many diagonals does a pentagon have?
(c) How many diagonals does a hexagon have?
(d) Let A be a vertex of a polygon with n sides. How many diagonals of the polygon have A as a
vertex?
(e) Use your answer to the previous part to find a formula for the number of diagonals in a polygon
with n diagonals. Be sure to compare your result to your answers in the first 3 parts to check
your formula. :

Problem 9.12: ABCDEFGH is a regular octagon. Find the ratio AE/BH. Hints: 422

Problem 9.13: In the star diagram, find the sum ZA + 2C + ZE + ZG + ZI. (You
cannot assume the pentagon in the middle of the star is regular!)

Problem 9.14: ABCDEFGH is a regular octagon with side length 12. In this problem we will find the
area of ABDG.
(a) What kind of quadrilateral is ABDG?
(b) Consider FGDE. Draw altitudes from F and E to GD to find GD. (And see if you can find GD in
some other clever way!)
Use your work from the last part as a guide to find the distance from B to GD.
Find [ABDG].
9.4. POLYGON PROBLEMS

Problem 9.10: Diagonals BD and CE of regular pentagon ABCDE meet at Q. Find A


/BOE.

Solution for Problem 9.10: We start by finding the angles we can find. Each interior angle of a regular
pentagon is 108°. Since AEDC is isosceles, DCE = ZDEC = (180° — 108°)/2 = 36°. Similarly, we have
ZCDB = 36°. Therefore, ZCQD = 180° — ZQDC — ZQCD = 108°, so we have ZBQE = ZCQD = 108°. Oo

Problem 9.11: Deriveaformula for the number of diagonals in a polygon with n sides.

Solution for Problem 9.11: We consider a single vertex, A. A is connected to 2 other vertices by sides,
leaving n — 3 vertices to which A can be connected by diagonals. Since there are n vertices and each can
be connected to n — 3 others by diagonals, there appear to be n(n — 3) diagonals. However, this counts
each diagonal twice, once for each endpoint. Therefore, we must divide by two to count each diagonal
only once. This gives us n(n — 3)/2 diagonals in an n-gon. O

Problem 9.12: ABCDEFGH is a regular octagon. Find the ratio AE/BH.


Solution for Problem 9.12: It’s not immediately obvious how to compare BH to AE, but BAA
there are several other diagonals with the same length as BH. Considering these, we C H
see that four of them form square AGEC. Since AE is a diagonal of this square, we have
AE/BH = AE/AG = Y2. 0 a G
poked io Fee
Problem 9.13: In the star diagram, find the sum 4A + ZC + ZE+ ZG + ZI. (You A
cannot assume the pentagon in the middle of the star is regular!) C

Solution for Problem 9.13: All we have to work with is what we know about
the sums of the angles in pentagons and in triangles. We label the angles in
the pentagon as shown. We can find the non-star angles of the outer triangles
in terms of these (for example, ZIJH = 180° — a). However, if we use a little
ingenuity, we don’t even need the exterior angles of BDFHJ. Consider the
angles in each triangle like AAHE. These must sum to 180°, so we have:

ZA+e+ZE = 180°
ZC+a+ZG = 180°
ZE+b+ZI = 180°
ZG+c+ZA = 180°
Zl+d+ZC = 180°

259
CHAPTER 9. POLYGONS

We're looking for ZA + ZC + ZE + ZG + ZI, and we know thata+b+c+ d + e = (180)(5— 2)= 540°, since
these angles together make up the central pentagon. Therefore, our triangle equations beg to be added
together, giving:
2LA+lC+ZE+lG+ ZI) +(a+b+c+d+e) = 5(180°).
Therefore, 2(ZA + ZC + ZE + ZG + ZI) + 540° = 900°, so

LA ie Ee ae oe

|Problem 9.14: ABCDEFGH is a regular octagon with side length 12. Find the area of ABDG. ;

Solution for Problem 9.14: Our ABDG is a trapezoid (you'll prove this explicitly in
the Exercises). We'd like to dissect the octagon into pieces we can handle, and we
know altitudes of trapezoids can often be useful, so we draw diagonals AF and BE.
These are each perpendicular to GD and FE (you'll prove this fact in the Exercises,
too).

Since GD || FE, we have ZFGD = 180° — ZGFE = 45°, so AGXF is a 45-45-


90 triangle. Therefore, GX = GF/ V2 = 6v2. Similarly, YD = 6V2. XYEF isa
rectangle, so XY = FE = 12. Therefore, base GD of our trapezoid has length 12 + 12 v2. Before we go
through tricky computations to get AX, we can note that by symmetry we have AF = GD (alternatively,
we can use HL Congruence to show AAXG = ADXF), so AX = AF —- XF = DG-— GX = 12+ 6 v2.
Therefore, our area is:

(AB + DG)(AX) _ (24+ 12 V2)(12 + 6 v2)


[ABDG] =
5) y 2
_ 72(2 + ¥2)(2 + V2) = 36(6
+ 4 V2) = 216 + 144 v2.
vu 2

lygon problems are really quadrilateral or triangle problems.


Nt 1 stuck with a1 polygon, wydissecting it as Staller pes :

In our last solution, we used symmetry to note that AF = DG. By symmetry, we essentially mean
that the general properties of all the vertices are the same. For example, the distance from one vertex
to a vertex three vertices away on the regular polygon (such as AF or DG) is the same no matter which
initial vertex we are talking about. However, we couldn’t say that ‘AC = BE by symmetry’ in the last
problem, because AC is the distance between two vertices with one vertex between them, while BE is
the distance between two vertices with two vertices (C and D) between them.

ry canbe avery useful tool in problems involving regul r Pp

256
9.5. CONSTRUCTION: REGULAR POLYGONS

In the Exercises that follow, you'll be asked to solve several problems that could be solved using
symmetry. In addition to solutions involving symmetry, try to find explicit solutions using tools such
as congruent and similar triangles.

ia >
9.4.1 ABCDE at right is a regular pentagon. Show that AEQB is a rhombus. A
9.4.2 UVWXYZ is a regular hexagon with center O. UOWABC is also a regular E B
hexagon with AB = 12. Ne]
(a) Find [UOWABC]. r/c
(b) Find [|UVWO].
9.4.3 All of the interior angles of octagon EFGHIJKL are 135°, EF = GH = IJ = KL = 2 and FG = HI =
Tks LE =6;
(a) Find the perimeter of the octagon.
(b)* Find the area of the octagon. Hints: 277
(c)x Find EI. Hints: 406

9.4.4 Let ABCDEF be a regular hexagon. Find the ratio of the area of ABCE to the area of the hexagon.
(Source: ARML) Hints: 74

9.4.5 ABCDEFGH is a regular octagon. Prove that ACEG is a square.


9.4.6 Explicitly prove that ABDG of Problem 9.14 is a trapezoid.
9.4.7 Explicitly prove that ZFXG is a right angle in Problem 9.14.
9.4.8 The midpoints of the sides of a regular hexagon ABCDEF are joined in order to form a smaller
regular hexagon. What fraction of the area of ABCDEF is enclosed by the smaller hexagon? (Source:
AMC 12)

9.5 Construction: Regular Polygons

Problems

25 |
a
CHAPTER 9. POLYGONS

We already know how to construct a regular triangle, so we'll move on to a regular quadrilateral.
‘Problem 9.15: Construct asquare.
Solution for Problem 9.15: Solution 1: A natural solution is to start with one side,
AB, then construct the others. The other two vertices must be on lines through A
and B that are perpendicular to AB, so we construct lines m and n perpendicular
to AB as shown. We can then find C on m such that AB = BC by constructing
a circle with center B and radius AB. Where this circle meets m gives us C.
Similarly, we construct a circle with center A and radius AB to find D on n such
that AD = AB.
We'll leave the proof that this construction does indeed result in ABCD being a square for an Exercise.

Solution 2: We can find an even slicker construction by thinking a little more


about squares. For example, we know that the diagonals of a square are perpen-
dicular and they meet at a point that is equidistant from all four vertices. In other
words, the intersection of the diagonals of a square is the center of the circumcircle
of the square.
We can use this observation to start with a circle to make our square. We draw
a circle with center O. We then draw two perpendicular lines through O and label
the points where these lines hit the circle A, B, C, and D. Since the four triangles that meet at O are
congruent 45-45-90 triangles, we have AB = BC = CD = DA. Moreover, each angle of ABCD equals
45° + 45° = 90°. Therefore, ABCD is a square. 0

Let’s try using this “circumcircle of a regular polygon” idea on another polygon.

|Problem 9.16: Construct a regular hexagon. = j

Solution for Problem 9.16: Solution 1: We already know how to make an equilat-
eral triangle, and a regular hexagon can be built from six equilateral triangles.
Therefore, we can start with equilateral AAOB, then construct equilateral AOBC
on OB, then construct equilateral AOCD on OC, and so on. After making 6equi- fF
lateral triangles, we'll have regular hexagon ABCDEF.
But that’s an awful lot of work. There must be a faster Way.

Solution 2: Since the circumcircle worked so well with the square, we'll try
it with the hexagon. Since the long diagonal of a regular hexagon is equal both
to twice the side length of the hexagon and to the diameter of the circumcircle
of the hexagon, the radius of the circumcircle equals the length of a side of the
regular hexagon. Therefore, if we draw a circle with radius OA, the vertices
of a regular hexagon inscribed in the circle will be at intervals of length OA
around the circle.
So, we draw an arc with center A and radius OA to find point B on the
circle. Then we draw an arc with center B and radius OA to get C, and so on around the circle. The
resulting ABCDEF is a regular hexagon.

258
9.5. CONSTRUCTION: REGULAR POLYGONS

We can prove ABCDEF is regular by noting that each of the triangles formed by connecting con-
secutive vertices to the center is equilateral. For example, AO = BO = AB means AAOB is equilateral.
Similarly, so is ABOC, so ZABC = 2(60°) = 120°. In the same way, we see that each of the angles ABCDEF
is 120°, and each of the sides is equal (to the radius of the circle). Therefore, ABCDEF is indeed regular.
a)

Concept: The circumcircle ofa regular polygon can be extremely useful in working
with the polygon.

We've already solved a few regular octagon problems, so you should be ready to construct one now.

Problem 9.17: Construct a regular octagon.

Solution for Problem 9.17: Solution 1: Looking at two consecutive sides of


a regular octagon, we see that if we can construct a 135° angle, we can
construct a regular octagon. Since 135° = 90° + 45° and we know how
to make both a 90° and 45° angle, we can make a 135° angle.
Specifically, we start with AB, which we continue to point P. We
then draw a line through B perpendicular to AB and let point X be on
this line inside the octagon as shown. We bisect ZXBP and let C be the
intersection of this bisector and the circle centered at B with radius BA.
Therefore, AB = BC and ZABC = ZABX + ZXBC = 135°, so C is another
vertex of our octagon. We can then copy ZABC to make ZBCD, and so
5 E on, completing the regular octagon.

But again, that’s a lot of work!

Solution 2: Rather than look at the octagon from the point of view of
the sides, let’s look at it from the center’s perspective. Again, we can
use the circumcircle. We start by creating square WXYZ by constructing
perpendicular lines that meet at the center of the circle. Where these
lines meet the circle give us the vertices of square WXYZ. The other
four vertices of our regular octagon must also form a square, with its
vertices at the midpoints of the arcs connecting the vertices of WXYZ.
To construct these points, we just bisect the right angles formed by WY
and XZ. Where these bisectors meet the circle give us the remaining
vertices of our regular octagon, WPXQYRZS. O

9.5.1 Prove that our first solution to Problem 9.15 does produce a square.
9.5.2 Prove that our second solution to Problem 9.17 does produce a regular octagon.
9.5.3 Construct a regular dodecagon.

es
é- 869
CHAPTER 9. POLYGONS

9.6 Summary

Definitions: A polygon is a closed planar figure with line segments as boundaries. As with |
triangles and quadrilaterals, the segments that form the boundaries are the sides, which meet at the |
vertices of the polygon. A diagonal is a segment that is not a side, but connects two vertices of a |
| polygon. A regular polygon is a polygon in which all the angles have the same measure and all the
sides have the same length. he

Important: e The area of a regular polygon is half its perimeter times the |
WW distance from the center of the polygon to a side.
e In a polygon with n sides, the measures of the interior angles |
have a sum of (n — 2)(180°) and the measures of the exterior |
angles have a sum of 360°.

The names of common polygons and the interior and exterior angle measures for common regular
polygons are below:

Name # of Sides | Int. Z Measure | Ext. Z Measure


triangle 3) 60° 120°
quadrilateral 4 90° 90°
pentagon 5) 108° pg
hexagon 6 £205 60°
octagon 8 eae 45°
nonagon ) 140° 40°
decagon 10 144° 36°
dodecagon 12 Mh Oe 30°

Problem Solving Strategies

B00.
en
REVIEW PROBLEMS

Concepts: ... continued from the previous page

> e Most polygon problems are essentially quadrilateral and triangle |


problems. When stuck with a polygon, try dissecting it into quadri-
laterals and triangles.

e The circumcircle of a regular polygon can be extremely useful in


working with the polygon.

REVIEW PROBLEMS

9.18 How many sides does a regular polygon with interior angles that measure 140° have?
9.19 What is the measure of each interior angle of a regular polygon that has 36 sides?
9.20 Find the area of a regular hexagon with side length 4.
9.21 How many sides does a regular polygon with exterior angles that each measure 6° have?
9.22 Find the area of a regular octagon with side length 6.
9.23 The measure of each exterior angle of a regular polygon is 1/8 the measure of an interior angle.
How many sides does the polygon have?
9.24 EFGHIJ isa regular hexagon. Find EH/EG. Hints: 207
9.25 A square and a regular hexagon are drawn with the same side length. If the area of the square is
¥3, what is the area of the hexagon? (Source: HMMT)

9.26 Ina given octagon, four angles have measure x degrees and two have measure 2x degrees. The
other two angles are right angles. Is the octagon concave or convex?
9.27 A given polygon has 27 diagonals. How many sides does the polygon have?
9.28 ABCDEFGH isa regular octagon with AB = 8. Find AC, AD, and AE.

9.29 Aregular pentagon anda regular hexagon share a common side AD, as shown.
What is the degree measure of ZBAC? (Source: MATHCOUNTS)

9.30 The number of diagonals in a regular polygon is equal to the number of sides.
What is the number of degrees in the sum of all the interior angles of the polygon?
(Source: MATHCOUNTS) A D
9.31 ABCDEFGHIJ is a regular decagon. Find appropriate congruent triangles (and prove they’re
congruent) to prove each of the following:

(a) AC =Al.
(b) AD = AH.
(c) AE =CG.

261
CHAPTER 9. POLYGONS

9.32 Let ABCD be a square and let ABEFG be a regular pentagon in the diagram at F
right. Find ZBCE. re: ie

9.33 ABCDEFGH isa regular octagon. Diagonals AE and CG meet at X. Point M is the
midpoint of AB. A

(a) Find ZXAB. ee


(b) Find ZMXA. D C

(c) Find ZMXD.

Challenge Problems

9.34 A,A2A3---An is a regular n-gon.

(a) Prove that AjA2A3Az is a trapezoid.


(b) Prove that ZA7A;Aq4 = 360°/n. (Source: ARML)

9.35 In our discussion of regular polygons, we assumed that every regular polygon has a point that is
equidistant from all the vertices. We called this point the ‘center’ of the polygon.

(a) Prove that inside every regular polygon Ff there is a point O that is equidistant from all of the
vertices of P. Hints: 120
(b) Let O be the center of regular polygon P, and A be one of the vertices of P. Prove that OA bisects
an interior angle of P.

9.36 The pentagon ABCDE at right has a right angle at A, AB = AE, and ED = A
DC = CB = 1. If BE = 2 and BE || CD, what is the area of the pentagon? Hints: 191
9.37 The total number of degrees in the sum of the interior angles of two regular p E
polygons is 1980. The sum of the number of diagonals in the two polygons is 34.
What is the positive difference between the numbers of sides of the two polygons? C D
(Source: MATHCOUNTS) Hints: 245

9.38 Find the sum of the angles at the points of the 7-pointed star ABCDEFG shown. Do not assume
that the heptagon in the center is regular. Hints: 408, 107
CHALLENGE PROBLEMS

9.39 A regular hexagon and an equilateral triangle have the same perimeter. What is the ratio of the
area of the hexagon to the area of the triangle?

9.40 Ina patio, a pattern is determined by regular hexagonal tiles, square tiles and
equilateral triangular tiles as shown in the diagram. If the area of each hexagonal tile is
96 in*, what is the number of square inches in the area of each square tile? What is the
area of the whole tiled region shown? (Source: MATHCOUNTS) Hints: 362

9.41 Given that ABCDEF is a regular hexagon with side length 12, find the area of
AGE

9.42 The interior angles of a convex polygon form an arithmetic progression with a common difference
of 4°. Determine the number of sides of the polygon if its largest interior angle is 172°. (Source:
USAMTS) Hints: 123

9.43 Opposite sides of hexagon ABCDEF shown are parallel. Given that AD = 7a ae
25 and the side lengths of the hexagon are as shown, find [ABCDEF]. Hints: 15 20

9.44 Determine with proof the number of positive integers n such thataconvex F
regular polygon with n sides has interior angles whose measures, in degrees, are D C
integers. (Source: USAMTS & AHSME) Hints: 571 50
9.45 : oe

(a) What is the largest possible number of interior angles of a convex pentagon that can have measure
90°? Hints: 225
(b) What is the largest possible number of interior angles of a convex decagon that can have measure
90°?
(c) Find a formula in terms of n for the largest number of interior angles of a convex n-gon that can
have measure 90°.

9.46x It is possible to surround a point with three regular hexagons with side length 1 as shown at left
below such that each hexagon shares a side with the other two hexagons. At right below, we see that
it is also possible to surround a point with a regular hexagon, a square, and a regular dodecagon such
that each has side length 1 and each of the polygons shares a side with the other two.

Find all other groups of three regular polygons with side length 1 that can surround a point such
that each polygon shares a side with the other two. Prove that you have found all possible groups of
three polygons. Hints: 68, 259

263
CHAPTER 9. POLYGONS

9.47x Let ABC be a right triangle, with ZACB = 90°. Let Pa, Pp, and Pc be regular pentagons, with
side lengths BC, CA, and AB, respectively. Prove that [P,] + [Pz] = [Pc]. Hints: 28, 414

9.48x The two squares shown share the same center O and have sides of B
length 1. The length of AB is 43/99. Find the area of octagon ABCDEFGH
formed by connecting the 8 points where the two squares intersect as shown. *
(Source: AIME) Hints: 56, 442, 233

Extra!
SR Have you ever seen a bathroom floor tiled with regular pentagons? How about with
regular hexagons? Heptagons? Why do squares, triangles, and hexagons seem to show
up everywhere, while the other regular polygons don’t get much use?
Like lots of other things in geometry, it all boils down to angles. Now that you know
how to find the angle in a regular polygon, you can answer this problem for yourself!
Take a look at a tiling that does work, such as covering the plane with squares (like a
piece of graph paper). At each vertex, exactly four squares meet. The angle of each
square is 90°, so the four angles at each vertex add up to 360°.
If you solved Problem 9.46, you should be able to quickly identify which regular
polygons can be used to tile a plane all by themselves. We call such a regular tiling of
the plane a tessellation. The three tessellations that use only a single regular polygon
over and over are shown below. These are called regular tessellations.

As we saw in Problem 9.46, we have many more possibilities if we allow ourselves to


use more than one type of polygon. Shown below are two semiregular tessellations, in
which the same set of regular polygons surround each vertex in the tessellation. There
are eight such tessellations (not including the regular tessellations); see if you can find
the other six.

264
CHALLENGE PROBLEMS

Extra! If we no longer restrict ourselves to having the same polygons around each point, we
mpi have demiregular tessellations, one of which is shown below.

26) ES ay,
as

Of course, we need not only use regular polygons in building tilings. Moreover,
tilings need not be periodic (meaning having the same pattern over and over). Penrose
tilings consist of two quadrilaterals that are used as tiles to make fascinating non-periodic
tilings. An example is shown below.

We don’t have to restrict ourselves to simple geometric figures to tile a whole plane.
The Dutch artist M. C. Escher produced thousands of tessellations using various animal
shapes and other whimsical designs. Rather than being satisfied with just a tessellation,
Escher often wove his tessellations into very engaging pieces of art. You'll find links to
some of Escher’s art on the links list described on page viii.
Tessellations have even made their way into puzzles and games, as the two pictures
of the tessellation game Bats and Lizards below show. Perhaps you can see how these
patterns were created from the regular tessellations on the previous page!

265
CHAPTER 10. GEOMETRIC INEQUALITIES

Morley’s First Triangle

The worst form of inequality is to try to make unequal things equal. — Aristotle

CHAPTER T O a) as en

Hee ie is Te Inequalities

In this chapter we will explore three geometric inequalities involving triangles. First, we will examine
how the order of the lengths of the sides is related to the order of the measures of the angles in a triangle.
Then, we will learn how to tell if a triangle is acute or obtuse just by considering its side lengths. Finally,
we will explore the most widely used geometric inequality of all — the Triangle Inequality. Like many of
the most useful mathematical tools, the Triangle Inequality is so simple it’s almost obvious, but it can
be used to develop many non-obvious solutions to complex problems.

10.1 Sides and Angles of a Triangle

: Problems B-

Problem 10.1: Draw a few triangles. Label each triangle AABC, and compare the order of the lengths
AB, BC, and AC (from shortest to longest) to the order of the measures of the angles 2A, 2B, and ZC.
Make a guess about how the order of the lengths of the sides is related to the order of the measures of
the angles in a triangle. Try to prove your guess always works before continuing this section! (Don’t
forget, you have to consider acute, right, and obtuse triangles.)

266
1. SIDES AND ANGLES OF A TRIANGLE

Problem 10.2: Prove that the hypotenuse of a right triangle is longer than each of the other two sides
of the triangle.

Problem 10.3: In this problem, we will prove that in an obtuse triangle, the side A
opposite the obtuse angle is the longest side of the triangle. We start with AABC
with obtuse ZC as shown. c B
(a) We already have information about the longest side in a right triangle. Try building a right
triangle with AB as a hypotenuse to show that AB > BC. Can you use a similar approach to
show AB > AC?
(b) Try finding a second solution by drawing a right triangle with BC as a leg.

Problem 10.4: Let AABC be acute with ZB > ZC. We will show that AC > AB.
(a) Draw altitude AX and find the point D on BC such that AD =AC (C and D are different
di points).
Show that ZDAX > ZBAX. Use this inequality to show that D is beyond B on XB.
(b) Prove that AC > AB.

Problem 10.5: Let AABC be acute. Show that if AC > AB, then ZB > ZC.

Problem 10.6: Given the angles shown in the diagram,


order the lengths AB, BD, CD, BC, and AC from least to
greatest. Note that the figure is not drawn to scale!

Problem 10.7: In AABC, the median AM is longer than BC/2. Prove that ZBAC is acute.

After looking at hundreds, if not thousands, of triangles while working to this point in the book, you
probably think that the largest side of a triangle is opposite the triangle’s largest angle, and the shortest
side is opposite the smallest angle. Good instincts! We’ll have to work through a few cases to prove it,
though.
Problem 10.2: Prove that the hypotenuse of a right triangle is longer than each of the other two sides
of the triangle.

Solution for Problem 10.2: Let AC be the hypotenuse of AABC. The Pythagorean Theorem tells us
AC? = AB? + BC. Since AB* and BC? are both positive, we must have AC > AB and AC > BC. Hence,
the hypotenuse of a right triangle is longer than each of the other two sides of the triangle. 0
Taking care of the longest side in a right triangle was pretty easy. Now let’s take a look at obtuse
triangles.

267
CHAPTER 10. GEOMETRIC INEQUALITIES

Problem 10.3: Prove that in an obtuse triangle, the side opposite the obtuse angle is the longest side
of the triangle.

Solution for Problem 10.3: Let ZC in AABC be obtuse. We already know


that the hypotenuse of a right triangle is the longest side of the triangle,
so we try building a right triangle we can use. We want to show that
AB is the longest side of our triangle, so we make it the hypotenuse of a = = 5
right triangle by drawing altitude AD from A as shown. Since AB is the
hypotenuse of AABD, we have AB > BD. Since BD > BC, we have AB > BD > BC. Similarly, we can
draw an altitude from B to show that AB > AC.

a E We could also have built a right triangle with BC asa leg by drawing
a line through C perpendicular to BC as shown. Since AECB is a right
triangle, we have EB > BC. Since AB > EB, we have AB > EB > BC.
C 5 Similarly, we can show that AB > AC.
Therefore, in an obtuse triangle, the side opposite the obtuse angle is the longest side of the triangle.
O

Now we’te ready to deal with acute angles.

Problem 10.4: Let AABC be acute with ZB > ZC. Show that AC > AB.

Solution for Problem 10.4: See if you can find what’s missing in this ‘proof’:

Bogus Solution: We start by drawing altitude AX. Since ZB > LG,


0 we have <BAX < <CAX. Therefore, BX < CX.
= Since AB* =AX? + BK? and AC’ = AX? + CX?
our BX < CX tells us that AB < AC. .

The problem here is that we haven't justified that BX < CX claim. Why does ZBAX < ZCAX mean
that BX < CX (and no, you can’t just say ‘It’s obvious!’)?

It’s not clear how we can build a right triangle to directly compare AB and AC, A
but we can build an obtuse one. We draw altitude AX and locate point D on BC
such that AD = AC. Since AD = AC, we have ZADX= LC. Therefore, ZD < ZABC.
Since 2D= 90°—ZDAX and ZABC= 90°—ZBAX, we have 90°—ZDAX < 90°-ZBAX,
so BAX < £DAX. Therefore, D is beyond B on CB as shown at right.
DisB XxX Ss
Since ABC is acute, ABD is obtuse. This tells us AD > AB. Since AD = AC, we have the desired
ACAD aie
Problem 10.5: Let AABC be acute. Show that if AC > AB, then ZB > aCe

Solution for Problem 10.5: What's wrong with this solution:

268
10.1. SIDES AND ANGLES OF A TRIANGLE

De ee: |
|
|

The problem with that ‘proof’ is that it is exactly the same as saying ‘If I live in California, I live in
America. Since I live in America, I live in California.’ As we’ve noted before, we cannot assume that the
converse of a true statement is true! We must prove a statement and its converse separately. Here, we
will present two solutions.
Solution 1: Although our Bogus Solution was indeed quite bogus, we can still use what we already
know about angles and sides. Specifically, consider three cases:

Case 1: ZB > ZC. As we have seen, if ZB > ZC, then AC > AB.
byse 7s (Re Calt.2B =7C then. AC = AB.
Case 3: ZB < ZC. As we have seen, if ZB < ZC, then AC < AB.

We are given that AC > AB. The only one of these cases that leads to AC > AB is Case 1, so we can
deduce that 2B > ZC. Make sure you see why this solution works, but the bogus one doesn’t!

Solution 2: We can backtrack through our solution to Problem 10.4. We find A


point D on CB such that AD = AC. To show that D is beyond B on CB, we first
draw altitude AX. Since we are given AC > AB and we have both AC’ = AD? =
AX2 + XD* and AB? = AX2 + XB2, we must have XD > XB.

Now we can use AABD to show that ZC < ZABC. From AABD, we have D B > E
ZADB + ZABD = 180° — ZDAB. Therefore, ZADB + ZABD < 180°. Since ZADB = ZC and ZABD =
180° — ZABC, substitution into ZADB + ZABD < 180° gives us

ZC + (180° — ZABC) < 180°,


so ZC < ZABC as desired. 0

We can summarize all of our discoveries thus far in this section very simply:

Important: In any triangle, the longest side is opposite the largest angle and the
WW shortest side is opposite the smallest angle. The middle side, of course,
is therefore opposite the middle angle.
In other words, in AABC, AB > AC = BC if and only if ZC > ZB = ZA.

Let’s try using these facts on a couple problems.

Problem 10.6: Given the angles as shown, order the lengths AB, BD,
CD, BC, and AC from least to greatest. (Note: The diagram is not drawn
to scale!)

269
CHAPTER 10. GEOMETRIC INEQUALITIES

Solution for Problem 10.6: First we focus on AABC. Since 4A > 2B > ZC, we have BC > AG ean nen,
we turn to ABDC. Since ZB > ZC > ZD, we have CD > BD > BC. We got lucky! BC is the smallest in one
inequality string and the largest in the other, so we can put the inequalities together:
CD > BD be ACs AL:

ia

Problem 10.7: In AABC the median AM is longer than BC /2. Prove that BAC is acute.

A Solution for Problem 10.7: We start with a diagram. We'd like to prove something about
an angle, but all we are given is an inequality regarding lengths. So, we use the length
inequality to get some angle inequalities to work with. Specifically, since AM > BM
in AABM and AM > MC in AACM (because AM > BC/2 and BM = MC = BC/2), we
have
/B > LBAM
EC <>-31CAM,
We want to prove something about ZBAC, which equals ZBAM + ZCAM, so we add these two inequalities
to give ZB + ZC > ZBAC. Since ZB + ZC + ZBAC = 180°, we can write ZB + ZC > ZBAC as

180° — ZBAC > ZBAC.

This gives us 180° > 22BAC, so 90° > ZBAC. Therefore, ZBAC is acute. 0

Exercises b>

10.1.1 In each of the parts below, order all the segments in the diagram from longest to shortest. (The
diagrams are not drawn to scale!)

ee. (b) P p &) W

Bu C O ©
Q ‘a Y

10.1.2 In isosceles triangle APQR, we have ZP = 54° and PQ = PR. Which is longer, PQ or QR?
10.1.3 Point X is on AB. Point C is given such that ZAXC = ZACB= 100°. Show that X is on segment
AB, rather than beyond B on AB or beyond A on BA. Hints: 579, 483

10.2 Pythagoras — Not Just For Right Triangles?

Back in Section 6.1, we learned that if AC* + BC? =AB?, then ZACB isa right angle. But what if AC? + BC2
doesn’t equal AB*?

AD
10.2. PYTHAGORAS — NOT JUST FOR RIGHT TRIANGLES?

J)Problems la
Problem 10.8: Draw lots of different triangles. Make a conjecture about what must be true if AB?+BC? >
AC’, and what must be true if AB* + BC? < AC?. Try to prove your conjectures before continuing!

Problem 10.9: Let 4C in AABC be acute as shown. We will prove that A

AC? + BC? > AB?.


(a) Draw altitude AX. Find an expression for AB?.
(b) Show that AC > AX. B C
(c) Show that AB* < AC? + BC?.
(d) Our diagram assumes that ZB is also acute. What if it isn’t?

Problem 10.10: Let ZC in AABC be obtuse as shown. We will prove that

AB’ > AC* + BC*.


(a) Draw altitude AD to BC.
(b) Find an expression for AB.
(c) Find AD in terms of AC and CD, and find BD in terms of BC and CD.
(d) Use the previous two parts to show that AB? > AC? + BC?.

Problem 10.11: Prove that the converses of the facts we proved in Problems 10.9 and 10.10 are also true.
In other words, prove that if AB? > AC + BC?, then ZC is obtuse, and prove that if AB? < AC? + BC’,
then ZC is acute.

Problem 10.12: In AXYZ, we have XY = 11, YZ = 14. For how many integer values of XZ is AXYZ
acute?

We know that if AABC is a right triangle with hypotenuse AB, we have AC*+BC? = AB?. Intuitively, it
seems that if AABC is acute, then AC2+BC? > AB?, and if itis obtuse with ZC > 90°, then AC?+BC? < AB?.
But intuitively isn’t good enough for us, is it?

Problem 10.9: Let ZC in AABC be acute. Prove that AC? + BC? > AB?.

Solution for Problem 10.9: Those sums of squares of sides send us hunting for right A
triangles. Drawing altitude AX gives us a couple. From right triangle AABX, we have
AB? = AX? + BX?. Clearly BX < BC, and right triangle AACX gives us AX < AC.
Therefore, we have
AB? = AX* + BX? < AC? + BC’. Boy C
fact that ZC isacute?’ Look closely — why must X be on
You might be wondering ‘Where did we use the
BC? If ZB and ZC are both acute, then altitude AX must meet BC. Our proof, therefore, doesn’t address

271
CHAPTER 10. GEOMETRIC INEQUALITIES

the case where ZB is obtuse. However, in this case we have AC > AB, so obviously AC? +BC2 > AB?. oO

Eee age Pe

Concept: One of the most useful approaches to solving new problems is to try to |
O==3 J think of similar-looking problems you know how to handle. For example, —
our main step in the solution to Problem 10.9 was thinking to build right |
triangles once we saw the sum of squares of sides.

Solution for Problem 10.10: Once again, we start by drawing an altitude


(AD) to build right triangles. From right triangle AABD, we have AB? =
AD? + BD. In order to get a BC term and an AC term in our equation, we
note that AD? = AC? — CD? from AADC, and BD = BC + CD. Therefore,
D C B we have

AB = AD BD
= MAC 46D2+ (GGE+ CR)
SAC CD? + hC ABC (GD ae
= AC? + BC? + 2(BC)(CD)
Since 2(BC)(CD) must be positive, we have AB? > AC? + BC?. O
Make sure you see where in our solution we used the fact that ZACB is obtuse.

Concept: If you don’t use all the given information in a solution, proofread it closely -
: to make sure you haven’t made a mistake! Sometimes you won’t need |
all the information you're given, but often if you haven’t used it all, it’s |
because you made a mistake somewhere. |
Furthermore, when you're stuck on a problem, read it again and see if |
there is any information you haven't used yet! |

In Problem 6.7 on page 138 we proved the converse of the Pythagorean Theorem; namely, that if the
sides of a triangle satisfy a? + b* = c*, then the triangle is right. Now we investigate if the converses of
the inequalities we have just discovered are true.
Problem 10.11: Prove that the converses of the facts we proved in Problems 10.9 and 10.10 are
also true. In other words, prove that if AB? > AC? + BC?, then ZC is obtuse, and prove that if
AB? < AC? + BC“, then ZC is acute.

Solution for Problem 10.11: We can mirror our first solution to Problem 10.5 by considering three different
cases:

Case 1: ZC is acute. If ZC is acute, then AB* < AC? + BC?.


Case 2: ZC is right. If ZC is right, then AB* = AC? + BC?.
Case 3: ZC is obtuse. If ZC is obtuse, then AB? > AC2 + BC?.

Ora a a
10.2. PYTHAGORAS — NOT JUST FOR RIGHT TRIANGLES?

Each possible triangle AABC must fall under exactly one of these cases. Only in the first case is
AB* < AC? + BC2, so if AB? < AC? + BC?, we can conclude that ZC is acute. Similarly, only in the third
case is AB* > AC* + BC?, so we can conclude that ZC is obtuse whenever AB2 > AC2 + BC2.
In the Exercises, you'll have a chance to provide a second ‘geometric’ argument, as we did in Solution
2 to Problem 10.5. 0
We can now relate the squares of the sides of a triangle to the angles of the triangle:

Important: ZC of AABC is acute if and only if AB* < AC? + BC”.


WW ZC of AABC is right if and only if AB* = AC? + BC?.
ZC of AABC is obtuse if and only if AB* > AC + BC?.

Let’s try using some of this information on a problem.

Problem 10.12: In AXYZ, we have


XY = ll and YZ = 14. For how many integer values of XZ is
AXYZ acute?

Solution for Problem 10.12: In order for AXYZ to be acute, all three of its angles must be acute. Using the
given side lengths and our inequalities above, we see that we must have

121+196 > XZ?


1964 KZA0s 7191
121-2 e196

Simplifying these three yields:

ole
A a ha75
OE
The middle inequality is clearly always true. (We really didn’t even have to include it — clearly 11
couldn’t ever be the largest side!) Combining the other two and noting that we seek integer values of
XZ, we have 9 < XZ < 17 (since 8 < 75 < 9? and 18* > 317 > 17°). So, there are 9 integer values of XZ
such that AXYZ is acute. 0

Bi)
CiEXercises
10.2.1 Each of the following groups of three numbers are the lengths of the sides of a triangle. Identify
each triangle as acute, right, or obtuse.
(a) 6,8, 10
(b) 6,8, 11
(Ger Onr03
(d) v13,3 v3, V15

273
CHAPTER 10. GEOMETRIC INEQUALITIES

(e)ie2 1 11.873.1
(f) 5/7,9/14, 1
10.2.2 In AABC, AB = 17 and BC = 27. For how many integer values of AC is AABC acute?
10.2.3x In this problem we will find a geometric proof that ZACB is obtuse if AB? > AC? + BC?.

(a) Let X be the point on BC such that AX is an altitude of AABC. Find an expression for AB? in terms
of other segment lengths in the resulting diagram. (Note that at this point, we do not know if X is
on BC or not. It might be on the extension of the segment instead.)
(b) Why must AABC be obtuse if X is not on BC, but is instead on BC such that C is between B and X?
(c) If Xison BC, then how are BX, CX, and BC related?
(d) Solve your expression from part (c) for BX and substitute the result into your equation for part
(a). Is it possible for AB* to be greater than AC? + BC?? Hints: 488
(e) Complete the proof by concluding that X must be on BC such that C is between X and B, so ZACB
must be obtuse. Hints: 169

10.3 The Triangle Inequality

The Triangle Inequality answers the question ‘When can three given segment lengths be the side lengths
of a triangle?’ The answer to this question is both more obvious and more powerful than the other
inequalities we have explored in this chapter. We'll start with a proof of this simple inequality, then
show why the inequality is so powerful by using it to solve a variety of problems.

“Problems
Problem 10.13: In this problem we will prove the Triangle Inequality. Don’t skip over this problem
because it looks obvious! Try to find a mathematical proof for each part.
(a) Use the inequalities from the previous sections to prove that in any triangle, the sum of two
sides is greater than the third side. This is the Triangle Inequality.
(b) The Triangle Inequality is often written as ‘the sum of two sides is greater than or equal to the
third side.’ Why — where does the ‘or equal to’ come from?
(c) Ifa, b, and c are positive numbers such that a+b > c,a+c > b,andb+c >a, then show ie
there exists a triangle with side lengths a, b, and c.

| Problem 10.14: In how many ways can we choose three different numbers from the set {1,2,3,4,5, 6}
such that the three could be the sides of a triangle? (Note: The order of the chosen numbers doesn’t
matter; we consider {3, 4,5} to be the same as {4, 3, 5}.)

Problem 10.15: Can the lengths of the altitudes of a triangle be in the ratio 2 : 5 : 6? Why or why not?

Problem 10.16: Let AM be a median of AABC. Prove that AM > (AB + AC = BC)/2.

a
ee:C*~C~;7«CS~S;7CS~S
10.3. THE TRIANGLE INEQUALITY

Problem 10.17: Circle O and circle P are tangent at point T such that neither circle passes through the
interior of the other. In this problem we will prove that O, P, and T are collinear (i.e., that a line passes
through all three).
(a) Prove that OT + TP > OP.
(b) Show that if OT + TP > OP, then ©O and ©P meet at a second point besides T.
(c) Show that O, P, and T must be collinear.

We'll start by ‘proving the obvious.’

Problem 10.13: = ae se
(a) Prove that in any triangle, the sum of two sides is greater than the third side. This is the
Triangle Inequality.
(b) The Triangle Inequality is often written as ‘the sum of two sides is greater than or equal to the
third side.” Why — where does the ‘or equal to’ come from?
(c) Ifa, b, and c are positive numbers such thata +b >c,a+c > b,andb+c >a, then show that
there exists a triangle with side lengths a, b, and c.

Solution for Problem 10.13:


(a) What’s wrong with this solution:

Bogus Solution: Suppose we draw AB first. Clearly the farthest C could be from A is
| if we just draw BC inthe same direction, so that AB + BC = AC. Any
other way we draw BC will result in C being closer to A. Therefore, |
| AB + BC > AC if AABC is a real triangle.
Hiss :
The second sentence is true, but we haven’t proved it. Since we’re dealing with an inequality,
we reach for the inequalities we’ve already proved to tackle the Triangle Inequality.
We’ve had so much success building right triangles, we do so again here, B
drawing altitude BD as shown. We have two cases to consider:
Case 1: AABC is acute. We draw altitude BD, forming two right triangles.
We thus have AB > AD and BC > CD. Adding these gives the desired
AB + BC > AD + DC = AC. We can do the same for each angle, showing that A Ae
each side is less than the sum of the other two sides.
Case 2: AABC is right or obtuse. If ZB > 90° as shown, then clearly AC > AB
and AC > BC, so we definitely have AB < AC + BC and BC < AC + AB. All E
we have left to prove is AC < AB + BC. Once again, right triangles come
to the rescue. Drawing altitude BE gives us AB > AE and BC > EC, so 4 B
AB + BC > AE+EC = AC.
(b) We have seen above that if A, B, and C are not collinear, then there cannot be equality. However,
when A, B, and C are collinear such that B is between A and C, we have AB+ BC = AC. If A, B, and
C are collinear, we call the resulting ‘triangle’ AABC a degenerate triangle. (Typically the term
‘triangle’ only refers to nondegenerate triangles, i.e. those in which the vertices are not collinear.)

Zi
CHAPTER 10. GEOMETRIC INEQUALITIES

Note that when B is on AC, we still have AC + BC > AB and AC + AB > BC in addition to
AB + BC = AC, so the Triangle Inequality is still satisfied with a small modification:
‘Important The Triangle Inequality states that for any three points, A, B, and C, we
WW, have
AB + BC SAG,
where equality holds if and only if B is on AC. Therefore, for nondegen-
erate triangles, AB + BC > AC.
(c) Suppose a > b > c, and let our triangle be AABC with AB = c,
AC = b, and BC = a. We can start building our triangle by
drawing BC with length a. We know that A must be on the A
circle with center B and radius AB = c, and on the circle with B 5
center C and radius AC = b. We draw both circles. Because \/
a > b >c, we know that BC > AC > AB. Therefore, the radius
of ©B is less than BC, so ©B intersects BC. We call this point of
intersection E. Similarly, ©C meets BC at point D as shown.
Since b +c > a, we know that BE + DC = b+c >a = BC. Since BE + DC > BC, we know that
D cannot be on EC. In other words, our two circles must meet! Where these circles meet gives us
our final vertex of AABC.

Note that we started off our proof to the last part with ‘Suppose a > b > c.’ But what if that’s not the
case? Does our whole proof fall apart?
No, it doesn’t. The whole proof still works; we just have to rearrange the letters a little. All the same
logic still works.

Important: Mathematicians have a special way of saying ‘All the different cases are
essentially the same, so proving it for this one proves it for all of them.’
To say this in our solution to the last part of the previous problem,
a mathematician would have written, ‘Without loss of generality, let
a2b2c....’ Then, the mathematician doesn’t need a whole separate
proof for essentially equivalent cases such as b > a > c. Sometimes
‘without loss of generality’ is shortened to ‘WLOG’.

Now we'll use the Triangle Inequality to determine sets of specific side lengths that can be the sides
of a triangle.

Problem 10.14: In how many ways can we choose three different numbers from the set {1, 2, 3, 4, 5,6}
such that the three could be the sides of a nondegenerate triangle? (Note: The order of the chosen
numbers doesn’t matter; we consider {3, 4,5} to be the same as {4,3, 5}.)

Solution for Problem 10.14: We first notice that if we have three numbers to consider as possible side
lengths of a triangle, we only need to make sure that the sum of the smallest two is greater than the
third. (Make sure you see why!) We could just start listing all the ones we see that work, but we should
a
tci RR es
276
10.3. THE TRIANGLE INEQUALITY

take an organized approach to make sure we don’t miss any. We can do so by classifying sets of three
numbers by the smallest number.
Case 1: Smallest side has length 1. No triangles can be made with three different lengths from our set if
we include one of length 1.
Case 2: Smallest side has length 2. The other two sides must be 1 apart, giving the sets {2,3, 4}, {2,4, 5},
and {2,5, 6}.

Case 3: Smallest side has length 3. There are only three possibilities and they all work: {3, 4,5}, {3,4, 6},
1979, 6}:
Case 4: Smallest side has length 4. The only possibility is {4,5,6}, which works.
Adding them all up, we have 7 possibilities. 0

We sometimes have to use some other tools in addition to the Triangle Inequality.

‘Problem 10.15: Can the lengths of the altitudes


of a triangle be in the ratio 2: 5: 6? Why or why
not?

Solution for Problem 10.15: We don’t know anything about how the lengths of the altitudes of a triangle
are related to each other. We do, however, know a whole lot about how the lengths of the sides of a
triangle are related to each other. Therefore, we turn the problem from one involving altitudes into one
involving side lengths. We let the area be K, let the side lengths be a, b, c, and the lengths of the altitudes
to these sides be h,, hy, hc, respectively. Therefore, we have K = ah,/2 = bh,/2 = ch,/2, so the sides of the
triangle have lengths
2K 2K 2K
WERE
If our heights are in the ratio 2 : 5 : 6, then for some x, our heights are 2x, 5x, and 6x. Then, our sides are

2K22 2KAx 2K16% |


However, the sum of the smallest two sides is then

2KBx a| 2K6x _hoe


12K | 10K
30
_ 22K
"304"
which is definitely less than the largest side, which is 2K/2x = K/x. Therefore, the sides don’t satisfy the
Triangle Inequality, which means it is impossible to have a triangle with heights in the ratio 2:5: 6.0

Concept: When facing problems involving lengths of altitudes of a triangle, consider |


(==> ~~ iusing area as a tool.

The Triangle Inequality is probably the most commonly used tool in geometric inequality proofs;
here’s an example of the Triangle Inequality in action.

Problem 10.16: Let AM be a median of AABC. Prove that AM > (AB + AC - BC)/2.

277
CHAPTER 10. GEOMETRIC INEQUALITIES

Solution for Problem 10.16: We can apply the Triangle Inequality to both AABM and A
AACM, giving:

AM+BM > AB
AM+CM > AC B M C

Noting that BM + CM = BC, we think to add these inequalities, which gives 2AM + BC > AB + AC.
Subtracting BC from both sides and dividing by 2 yields
AB + AC — BC
AM e 2

Concept: If you don’t see the path to the solution immediately, don’t just sit and —
stare at the problem! Make some observations. Write down statements |
you can prove that might be helpful. Perhaps you'll be able to combine ©
these observations to complete your proof. |

In Problem 10.16, even if we didn’t see the solution immediately, the problem involves an inequality
that has sums of lengths. This makes us think of using the Triangle Inequality to make observations.
However, we don’t simply write Triangle Inequality relationships blindly. We use the problem as a
guide to make the observations. Specifically:

Concept: When trying to solve geometric inequalities, pay attention to which side >
Sey of the inequality you want to prove each length is on. For example, we |
wouldn’t want to start Problem 10.16 by observing AM < AB + BM, since |
this inequality has AM on the smaller side, but what we want to prove has —
AM on the larger side. |
et

We end the chapter with an important application of the Triangle Inequality.

Problem 10.17: Circle O and circle P are tangent at point T such that neither circle passes through
the interior of the other. Such circles are said to be externally tangent. Prove that O, P, and T are
collinear.

Solution for Problem 10.17: We don’t know a whole lot about


tangent lines at this point. However, in this section we [,
have discovered a way to show that three points are on a
line. Specifically, if OT + TP = OP, then T is on OP, because AN
otherwise the Triangle Inequality guarantees OT+TP > OP. D
There doesn’t seem to be any easy way to approach I
showing that OT + TP = OP. Instead, we try to show that
it’s impossible to have OT + TP > OP and still have tangent R
circles. If we have OT + TP > OP, then T cannot be on OP.
Therefore, there is some point R such that OP is the perpendicular bisector of TR. To find point R, we

278
10.3. THE TRIANGLE INEQUALITY

draw altitude TX from T to OP, then we draw RX such that RX = TX and RX 1 OP. Since OX = OX,
LTXO = ZRXO, and RX = TX, we have ATOX = AROX. Therefore, TO = RO, so R is on OO. Similarly,
we have APTX = APRX, so PT = PR and R is on ©P. Therefore, if OT + TP > OP, it is impossible for the
circles to be tangent because there is a second point at which the circles meet.

However, our circles must be tangent, so we can’t have OT + TP > OP. The Triangle Inequality tells
us we can’t have OT + TP < OP. Therefore, we must have OT + TP = OP, which means O, T, and P are
collinear. O

Important: If ©O and ©P are tangent at point T, then O, P, and T are collinear. We'll |
| often use this fact in problems with tangent circles by connecting the —
| centers of the circles, knowing this line also goes through the point of |
tangency. |
Sek — Ee ee ps aha a ate j

Notice that Problem 10.17 only tackles the case in which the circles
are externally tangent. It’s also true for circles that are internally
tangent, i.e., when one circle is wholly inside the other except at the
one point at which they are tangent. In the diagram at right, ©B and
OC are internally tangent, while ©D is externally tangent to both of
the other circles.

cases =
10.3.1 Which of the following sets of three numbers could be the side lengths of a triangle?
(a) 4,5,6
(b) 7,20,9
(2) 2, 1/6, 15
(d) 3.4, 11.3, 9.8
(e) v5, ¥14, V19
10.3.2 The lengths of two sides of a triangle are 7cm and 3cm. If the number of centimeters in the
perimeter is a whole number, what is the number of centimeters in the positive difference between the
greatest and least possible perimeters? (Source: MATHCOUNTS)
10.3.3. Prove that the sum of the diagonals of a quadrilateral is less than the quadrilateral’s perimeter.
10.3.4* ABCD isa square and O is a point. Prove that the distance from O to A is not greater than the
sum of the distances from O to the other three vertices, no matter which point we take to be point O.
Hints: 97
10.3.5x Prove that if a, b, and c are the sides of a triangle, then so are va, Vb, and vc. What about a’,
b?, and c?? Hints: 453, 36

Extra! Calculating replaces thinking, while geometry stimulates it. a oe


ac ad

279
CHAPTER 10. GEOMETRIC INEQUALITIES

10.4 Summary

Nebel ee fides ;
|‘Important: In any triangle, ‘the longest side iiss opposite the largest angle and the
Vv? shortest side is opposite the smallest angle. The middle side, of course, |
is therefore opposite the middle angle.
i One vo in AABC, oe = So = BCit and only if ZC S 7B > fA
|

Important: ZC of SABC is aeutecian pale if AB? < AC* + BC?. |


| ZC of AABC is right if and only if AB? =AC? + BC?. |
a ofa iss Ope if ae only |if AB* > AC? + BC?. |

| Important: The Triangle Inequality states that for any three points, A, B, and C, we
have
AB + BC 2 AC,
where equality holds if and only if B is on AC. Therefore, for nonde- |
generate triangles (i.e., those in which the vertices are not collinear), —
|
AB+BC> AC. ZA

Important: If ©O and ©P are tangent at point T, then O, P, and T are collinear. We'll
often use this fact in problems with tangent circles by connecting the _
centers of the circles, knowing this line also goes through the point of |
tangency. |

Problem Solving Strategies

Concepts: e One of the most useful approaches to solving new problems is to try
O=s to think of similar-looking problems you know how to handle.

e If you don’t use all the given information in a solution, proofread it


closely to make sure you haven’t made a mistake! Sometimes you
won't need all the information you're given, but often if you haven't
used it all, it’s because you made a mistake somewhere.
Furthermore, when you're stuck on a problem, read it again and
see if there is any information you haven't used yet!
e When facing problems involving lengths of altitudes of a triangle,
consider using area as a tool.

Continued on the next page. . 2

280
REVIEW PROBLEMS

Concepts: . . . continued from the previous page

e If you don’t see the path to the solution immediately, don’t just sit.
and stare at the problem! Make some observations — write down |
statements you can prove that might be helpful. Perhaps you'll be |
able to combine these observations to complete your proof.

e When trying to solve geometric inequalities, pay attention to which |


side of the inequality you want to prove each length is on. For
example, we wouldn’t want to start Problem 10.16 by observing ©
AM < AB + BM, since this inequality has AM on the smaller side, but |
penal we want tog Brave LGAM on nats ae a
ee. a zins RS AR Gab oar wed SS ia 3

REVIEW PROBLEMS a

10.18 For each of the groups of three numbers below, state whether the numbers could be the side
lengths of a triangle or not. If they can be, identify whether or not the triangle is acute, obtuse, or right.
(a) 2,3,4
(b) e221 a Ow
(on V5) 2an3
(d) 199, 401, 297 (See if you can do it without squaring those numbers!)
ceva 1/2,1/3, 1
(f) 60,24, 48
10.19 Ariis solving a problem involving a right triangle with legs 119 and 120. He uses the Pythagorean
Theorem and gets 261 as the hypotenuse. He immediately shakes his head and starts over. How did he
know so quickly that he made a mistake?
10.20 In AABC, AB = 5 and BC = 11. For which integer values of AC is AABC an obtuse triangle?
10.21 A,A2A3--- A, is a regular polygon with n > 3. Prove that A;A3 > A,Az2.

10.22 Prove that it is impossible for the length of a side of a triangle to be greater than half the triangle’s
perimeter.
10.23 The perimeter of an isosceles triangle is 38 centimeters and two sides of the triangle are whole
numbers in the ratio 3 : 8. What is the number of centimeters in the length of the shortest side? (Source:
MATHCOUNTS)
10.24 Find all positive integers x for which it is possible for 2x + 3, 3x + 8, and 6x + 7 to be the side
lengths of a nondegenerate triangle.
10.25 How many different obtuse triangles with integer side lengths and a perimeter of 20 can we draw
such that no two of them are congruent?

281
CHAPTER 10. GEOMETRIC INEQUALITIES

10.26 The length of each leg of an isosceles triangle is x + 1 and the length of the base is 3x—2. Determine
all possible values of x. (Your answer should be an inequality expressing the possible values of x.)
10.27 YZ is the base of isosceles triangle AXYZ. Given that YZ > XY, show that ZX > 60°. Hints:
551, 209

Challenge Problems >

10.28 Ihave four stakes in my yard arranged like points W, X, Y, and Z shown. I wish e Y
to connect each stake to the same point in my yard with a string. What point should
I choose to minimize the amount of string I must use? (Make sure to prove that your
choice is the best possible!) °

10.29 Orion chooses a positive number and Michelle chooses a positive number. X
Joshua chooses a positive number that is smaller than Orion’s number. No matter
what number Joshua chooses, we can always form a triangle with the three chosen numbers as side
lengths. Show that Orion’s and Michelle’s numbers must be the same. Hints: 493

10.30 Show that it is impossible for any 5-pointed star like ABCDEFGHI] at right
to have AB > BC, CD > DE, EF > FG, GH > HI, and IJ > JA all be true.

10.31 Asshown below, Q is on PR and circles ©P, OQ, and OR are drawn such that
no two circles intersect and no one circle contains the other two. Suppose a fourth
circle can be constructed that is externally tangent to all three of these circles.

(a) Use the Triangle Inequality to prove that the radius of our fourth circle is greater than the radius
of OQ. Hints: 270
(b) Use the inequalities relating the order of sides in a triangle to the order of angles in a triangle to
prove that the radius of our fourth circle is greater than the radius of OQ. Hints: 351

10.32 In convex quadrilateral WXYZ, we have WX > WY. Show that we must also have XZ > YZ.
Hints: 447,85

10.33 Let a, b, and c be three positive real numbers such that a? + b? > c2,a2+c2> band lb? +2 > a.
Prove that a, b, and c can be the lengths of the sides of a triangle. Hints: 160
10.34 Prove that the sum of the lengths of the diagonals of a quadrilateral is greater than half the
perimeter of the quadrilateral.

ia
CHALLENGE PROBLEMS

10.35* In AXYZ, we have ZX = 20° and XY = XZ. Prove that 2YZ < XY < 3YZ. Hints: 533, 84, 12,
395, 498

10.36 In AABC and AA’B’C’, we have AB = A’B’, AC = A’C’, and ZBAC > ZB’A’C’. Prove that
BC > B’C’. Hints: 266, 578

10.37x Prove that the distance between any two points inside AABC is not greater than half the
perimeter of AABC. Hints: 430, 525, 437

Extra! Proof That Every Triangle Is Isosceles?


LL 1 | |

In triangle ABC below, let D be the midpoint of BC. Let P be the intersection of the angle
bisector of A and the perpendicular bisector of BC. Let M be the foot of the perpendicular
from P to AB and N be the foot of the perpendicular from P to AC, as shown.

Since P lies on the angle bisector of A, ZPAM = ZPAN. Also, ZPMA = £PNA = 90°,
so by AAS, triangles PAM and PAN are congruent. Therefore, PM = PN and AM = AN.
P lies on the perpendicular bisector of BC, so PB = PC. Also, ZPMB = ZPNC = 90°,
so triangles PMB and PNC are congruent by HL Congruence. Therefore, MB = NC. But
AM = AB + BM, AN = AC +CN, and AM = AN, so AB = AC. Therefore, triangle ABC is
isosceles.
It is obviously not true that every triangle is isosceles, so something is going wrong
in the proof. But what?

283
CHAPTER 11. CIRCLES

Monge’s Theorem

Probably no symbol in mathematics has evoked as much mystery, romanticism, misconception and human interest
as the number pi. — William L. Schaaf

CHAPTER ee |

Circles

11.1 Arc Measure, Arc Length, and Circumference

The portion of a circle connecting two points on the circle’s circumference is called an are. We find the
measure of an arc of a circle by considering what fraction of the circle the arc is. For example, a whole
circle is 360°, so an arc that is 1/6 of the circle has measure (1/6)(360°) = 60°.

However, not all 60° arcs are the same. Some are much longer than others,as shown 2S
in the figure to the right. Thus, we need more than just angle measure to classify arcs. ————
We need a way to measure of the lengths of arcs. Paar

We start with the length of an entire circle, which would be the perimeter of a circle. The perimeter
of a circle is called the circle’s circumference.

Before reading the rest of this chapter, put yourself in the sandals of the ancient Greeks and try a little
experiment. Get a string, and find numerous circular objects. Measure the distance around each object
by wrapping the string around it. Then measure the diameter of the object. Finally, for each object find
the quotient
Distance Around the Object
Diameter of Object
You should find that in each case the quotient is around 3.14. (If you get anything different, try measuring
and dividing again!)

284
11.1. ARC MEASURE, ARC LENGTH, AND CIRCUMFERENCE

In every single circle, the ratio of circumference to diameter is the same. This ratio is called pi, and
is given the symbol 7. Its value is approximately 3.14. (If you ask a bunch of your math friends, you’re
likely to find someone who knows dozens more digits.)

Sidenote: For some reason, memorizing huge portions of pi is a bit of a sport among
| ay math-lovers. This should help you get started:
| 3. 141592653589793238462643383279502884197169399375 105820974944
5923078164062862089986280348253421170679821480865 1328230664709
38446095505822317253594081284811174502841027019385211055596446
22948954930381964428810975665933446128475648233786783165271201
90914564856692346034861045432664821339360726024914127372458700
66063155881748815209209628292540917153643678925903600113305305
| 48820466521384146951941511609433057270365759591953092186117381
93261179310511854807446237996274956735188575272489122793818301

iFlease dontttell oe ete you got peeUe es ass ee us.


ae |

Pi is an irrational number, which means that it cannot be expressed as a ratio of integers. Because
pi is irrational, its decimal expansion does not terminate and does not become periodic. In other words,
it does not get to a point where the same set of numbers is repeated over and over. So, there are no
shortcuts to memorizing digits of pi!

i Problems >

Problem 11.1: The circumference of a circle is 327. Find the circle’s radius.

Problem 11.2: The length of a 72° arc of a circle is 15. What is the circumference of the circle?

Problem 11.3: Chord YZ of a circle with center O has length 12. The circumference of the circle is ar
(a) What kind of triangle is AYOZ?
(b) Find the radius of the circle.
(c) Find the length of YZ.

_ Extra! the Pwelve pentominoes (see page 132) fit inside a 6 x 10 Pee asa seine

ey
i They can also fit inside a5 x 12,a 4x 15, and evena 3 x 20 rectangle. Can you find how?

285
CHAPTER 11. CIRCLES

Problem 11.4: Chords AB and CD of ©O have the same length. We will prove that AB = CD.
(a) Connect A, B, C, and D to the center of the circle. Find some congruent triangles.
(b) Use your congruent triangles to prove that AB = CD.

Problem 11.5:
(a) Anna and Betty are 180 yards apart, at points A and B in the diagram, respec- ©
tively. There is a semi-circular path from Anna to Betty. Chuck is at point C,
the midpoint of this path. Anna must walk to Chuck and then to Betty. How 4 =
much farther must Anna walk if she follows the path than if she ‘cuts across,’
walking straight to C then straight to B?

What if the path is a 120° arc of a circle instead of a semi-circular arc? (Chuck is on the midpoint
of this new arc, and we wish to see how much farther Anna must walk by staying on the path.)
Hints: 20, 569

Circumference is a pretty simple concept, once you've finished memorizing all the digits of pi.

‘Problem 11.1: The circumference of a circle is 327. Find the circle’s radius.

Solution for Problem 11.1: Let C be the circumference and d be the diameter. We know that C/d = m and
C = 32x. Therefore, d = 32. The radius is half the diameter, or 32/2 = 16. 0

Important: In a circle, let C be the circumference, d be the diameter, and r be the


Vv? radius.
C=70d=277

Problem 11.2: The length of a 72° arc of a circle is 15. What is the circumference of the circle?

Solution for Problem 11.2: A 72° arc is 72°/360° = 1/5 of an entire circle. Since this arc has length 15, the
entire circumference of the circle has length 5(15) = 75. o

Problem 11.3: Chord YZ of a circle with center O has length 12. The circumference of the circle is
24n. Find the length YZ.

Solution for Problem 11.3: To find the length of the arc, we must find ZYOZ. We don’t have if
any information about angles, however, so we start by figuring out some lengths. Since Y
the circumference is 2471, the diameter is 24 and the radius is 12. Hence, OY = YZ = ZO,
so AYOZ is equilateral. Therefore, ZYOZ is 60°, and our arc is 1/6 of the circle. So, the
length of YZ is (1/6)(24n) = 47. 0
A central angle of a circle is an angle with the center of the circle as its vertex. Now that we know how
to relate arcs to central angles, we can find a useful relationship between the arcs cut off by congruent
chords of a circle.

286
11.1. ARC MEASURE, ARC LENGTH, AND CIRCUMFERENCE

‘Problem 11.4: Chords AB and CD of ©O have the same length. Prove that AB = CD.

Solution for Problem 11.4: We form two triangles by connecting A, B, C, and D to


O. Since AB = CD, AO = CO, and BO = DO, we have
hi AABO = ACDO by $88 _, ae,
B
\
Congruence. Therefore, ZAOB = ZCOD, so arcs AB and CD are congruent. 0 wy A

As you might suspect (and will prove as an Exercise), this relationship between =C
congruent chords and congruent arcs works in reverse, too.

Important: If two chords of a circle are congruent, then the arcs they subtend (cut
off) are congruent. Conversely, if two arcs of a circle are congruent, then _
the chords that subtend them are congruent.

‘Problem 11.5:
(a) Anna and Betty are 180 yards apart, at points A and B in the diagram, respec- &
tively. There is a semi-circular path from Anna to Betty. Chuck is at point C, We~
the midpoint of this path. Anna must walk to Chuck and then to Betty. How
much farther must Anna walk if she follows the path than if she ‘cuts across,’ A B
walking straight to C then straight to B?

(b) What if the path is a 120° arc of a circle instead of a semi-circular arc? (Chuck is on the midpoint
of this new arc, and we wish to see how much farther Anna must walk by staying on the path.)

Solution for Problem 11.5:


(a) We might immediately see that AACB is a 45-45-90 triangle, but if we don’t, C
we can start by connecting C to the center of the circle, O, as shown in the
diagram. Since AC = CB = 90°, we have ZAOC = ZCOB = 90°. OA, OB, a B
and OC are all radii of the circle, so AAOC and ABOC are 45-45-90 triangles. O
Since AB = 180 yards, we have AO = OB = OC = 180/2 = 90 yards and
AC = BC = COY2 = 90 ¥2 yards. Since arc AB is one-half a circle, its length is (AB)n/2 = 907
yards. So, Anna must walk 907 — (AC + BC) = 907 — 180 Y2 yards farther if she follows the path.
(b) We start by connecting A, B, and C to the center of the circle of which the path C B
is a part, since we know more about radii than we do about arcs. Since AB is
120°, and C is its midpoint, AC = BC = 60°. Therefore, ZAOC = ZCOB = 60°.
OA = OC because they are radii of the same circle. Therefore, AAOC is A O
isosceles with ZAOC = 60° and AO = OC, so AAOC is equilateral. Similarly,
ABOC is also equilateral.
We need to find the radius of this circle to finish the problem. Let AB meet CO at P. Since
AAOP = ABOPby SAS, ZAPO = ZBPO. These angles together make up a straight angle, so ZAPO =
ZBPO = 90°. So, AAPO and ABPO are 30-60-90 triangles. Since AP = AB/2 = 90 yards, we have
AO = AP(2/V3) = 60 V3 yards. Since AC = AO = BO = BC, we have AC+ CB = AO+BO = 120 3.
The length of AB is (120° /360°)(2AOn) = 407 V3. Therefore, by following the path, Anna walks
AB — (AC + BC) = 407 V3 — 120 V3 farther. 0
287
CHAPTER 11. CIRCLES

The key step in the second part of this problem is connecting the points on the path to the center of
the circle. This is often an important step in challenging geometry problems.

|Concept: “In complicated problems involving a circle, try connecting the center of a |
C= circle to meat pone on Ne circle. |

a —— ee = 7

|Sidenote: We defined 1rt as the ratio of the circumference of a circle to its diameter.
But how do we know what x is numerically? In other words, how do we
know that 7 = 3.1415926535..., and so on? |
Since 7 is one of the most eer mathematical constants, much time -
and effort have been devoted to calculating 7. At the present time, 7 is
known to over one trillion decimal places. Why bother with so many digits?
Certainly not for the sake of accuracy in everyday use — even 39 decimal
places is sufficient to calculate the circumference of the known universe to
within the radius of a hydrogen atom.
The answer, in part, is for the prestige of having set anew record number |
of places. The digits of 7: can also be used to check the speed and accuracy |
of computers. Finally, mathematicians are interested in seeing if they can ||
find any patterns among these digits, such as whether the ten digits 0, 1, 2,
3,...,9 appear equally often. However, despite all this effort, no patterns
of noe have been found.
So how do we calculate the value of 7? One simple method is as follows:
Suppose we take a circle of radius 1, and circumscribe it, and inscribe it, |
with a regular n-gon. The figure below shows the n-gons for n = 6 and |

——- The perimeter of the polygons approximate the circumference of the |


circle, which is 2x. The greater the number of sides in the polygon, the |
better the approximation. Let a, denote the perimeter of the circumscribed
n-gon (the outer polygon), and let b, denote the perimeter of the inscribed
n-gon (the inner polygon). Then it turns out that

and bon = V A2nbn.

Continued on the next page. . .

288
11.1. ARC MEASURE, ARC LENGTH, AND CIRCUMFERENCE

Sidenote: .. . continued from the previous page |


ay Thus, if we start with a¢ and be, we can calculate a}2 and bj, which can
be used to calculate a24 and b24, and so on. This was the observation of |
the ancient Greek mathematician Archimedes, after whom this algorithm |
is named. The following table shows some results of this algorithm. We.
begin with the values ag = 4 V3 and be = 6. (Make sure you see why
ag = 4 V3 and be = 6; remember, the radius of the circle is 1.) |

b,/2
6.92820323 6.00000000 3.46410162 3.00000000
12 | 6.43078062 6.21165708 3.21539031 3.10582854
24 | 6.31931988 6.26525723 | 3.15965994 3.13262861
48 | 6.29217243 6.27870041 | 3.14608622 3.13935020 |
96 | 6.28542920 6.28206390 | 3.14271460 3.14103195 |
192 | 6.28374610 6.28290494 | 3.14187305 3.14145247 |
384 | 6.28332549 6.28311522 | 3.14166275 3.14155761 |
768 | 6.28322035 6.28316778 | 3.14161018 3.14158389 |
As we can see, the sequences (a;,/2) and (b,/2) both converge to 7. This _
is not the best algorithm for calculating the digits of 7, but it a how |
we can use a simple Boome idea to approximate 77.
bee ES

|_Exercises
ig
11.1.1 Find the circumference of a circle with radius 4.

11.1.2 Find the radius of a circle with circumference 127.

11.1.3 AC of OO has length 127, and the circle has radius 18.

(a) Find OC.


(b) Find ZAOC.
(c)x Find AC. Hints: 471

11.1.4 Arcs WX and YZ of OQ are congruent. Prove that WX = YZ.

Extra! Degrees are not the only way to measure angles! Just as there are different units for
> imme measuring length, like inch and centimeter, there are different units for angle measures.
Besides the degree, the most commonly used unit for angle measure is the radian. Just
as there are 360 degrees in a circle, there are 27 radians in a circle. Therefore, the measure
in radians of a quarter-circle is (27)/4 = 7/2 radians, and of a semicircle is (27)/2 = 7
radians.
As you might guess, the usage of radians is inspired by circumference. You might
also notice that every arc of a circle with radius 1 has its length in units equal to its
measure in radians. However, radians are not very useful until our study of advanced
trigonometry and calculus, so we'll stick with degrees for now.

289
CHAPTER 11. CIRCLES

11.2 Area

Problems ila
Problem 11.6: In this problem we will develop a formula for the area of a circle with radius r.
(a) The circle in the diagram has radius r and is divided into 16 equal
pieces by drawing 8 diameters. These pieces are called sectors.
Rearrange the sectors to look very similar to a quadrilateral whose
area we can find.
(b) What is the length of the base of this ‘quadrilateral’ in terms of r?
(c) What is the height of this ‘quadrilateral’ in terms of r?
(d) What is the area of this ‘quadrilateral’ in terms of r?

Problem 11.7: Find the area of a circle with diameter 12.

Problem 11.8: AXYZ is equilateral with side length 10. A circle is constructed with center X and
radius 10, thus passing through Y and Z. Find the area of sector YXZ of the circle. (In other words,
find the area of the portion of the circle bounded by radii XY and XZ and by arc YZ.)

Problem 11.9: Farmer Tim has 50 feet of fence. He wants to enclose a semicircular area adjacent to his
barn, thus using his barn as one side of the enclosure. What is the area of the space Farmer Tim can
enclose? 7

Having tackled the ‘perimeter’ of a circle, we now turn to its area. While proving the formula for the
area of a circle would require far more advanced tools than we have now, we can develop an intuitive
explanation for the formula.
a

Problem 11.6: Find a formula for the area of a circle by cutting upa circle and rearranging the pieces
to look a lot like a rectangle.

_ Extra! GggXyZ the Martian has been told by the Martian King to make a rope to lasso the Earth.
tity GeeXyZ makes a rope that will fit snugly around the equator of the Earth. The Martian
King decides he wants a little slack so he can put the rope around the Earth more easily.
He wants the rope to fit around the equator such that there is one inch between the Earth
and the rope all the way around the equator.
GggXyZ says this is no problem. She pulls out a short piece of rope and says she can
just add this little piece to her rope to make the rope long enough. The Martian King
laughs at her, saying she'll need miles of rope because the diameter of the Earth is almost
8,000 miles. Who is right? How much rope does she need?

290
11.2. AREA

Solution for Problem 11.6: We start by cutting the circle into equal slivers, which we call sectors, by
drawing equally spaced radii as shown below. We then rearrange the sectors as shown in the second
figure, thus forming a figure that resembles a parallelogram. Finally, we can take half of one of the end
sectors and slide it to the other end as shown in the final diagram. Now we have a figure that strongly
resembles a rectangle.

6 - TM - aa
The ‘width’ of this ‘rectangle’ is clearly the radius of the circle, which we'll call r. The ‘length’ of the
‘rectangle’ is half the circumference of the circle, since the circumference of the circle is equally divided
among the top and bottom of our ‘rectangle.’ Hence, the ‘length’ is (27r)/2 = mr. Since the area of a
rectangle is its length times its width, the area of our ‘rectangle’ is (77r)(r) = mr.
This is not a rigorous proof, but it should be clear that if we cut the circle into more and more sectors
and do the described rearrangement, the ‘rectangle’ looks more and more like a real rectangle. 0
We now know how to find the area of a circle.
|
20m The area of a circle with radius r is 777.

Let’s use our newfound knowledge of how to find the area of a circle on a few problems.

Problem 11.7: Find the area of a circle with diameter 12.

Solution for Problem 11.7: Since the diameter is 12, the radius is 6. Therefore, the area is (6*)n = 36n. O

Problem 11.8: AXYZis equilateral with side length 10. A circle is constructed with center X and
radius 10, thus passing through Y and Z. Find the area of sector YXZ of the circle.

Solution for Problem 11.8: Our sector is shaded in the diagram. To find its area, we must
determine what portion the sector is of the whole circle. Since AXYZ is equilateral,
LYXZ = 60°. Since the whole circle is 360°, this means that our sector is 60°/360° = 1/6
of the entire circle. The whole circle has area m1r* ="71(107) = 1007, so the sector has area
(1007z)(1/6) = 5077/3. O

Important: Following our solution to Problem 11.8, we can find the


eu area of a sector of a circle of radius r given the central
angle of the sector. In the figure to the right, ZAOB is
(ZAOB)/360° of the whole circle. Therefore, we have:

(ee 2.
~ Area of sector AOB =
360° :

201
2s ee eee
CHAPTER (1. CIRCLES

You should not have to memorize this formula. Instead, understand that the area of a sector is simply
related to the ratio of the sector’s angle to the number of degrees in a whole circle.

Problem 11.9: Farmer Tim has 50feetoffence. He wants to enclose a semicircular area adjacent to |
|his barn, using his barn as one side of the enclosure. What is the area of the space Farmer Tim can
enclose?

Solution for Problem 11.9: We need to find the radius of the semicircle in order to find
its area. We start with a sketch of the problem. Our fence forms a semicircular arc, so
the arc has length 50. Therefore, the semicircle is half of a circle that has circumference 4 B
100. The diameter of such a circle is the circumference divided by 7, or 100/7. So, the radius of our
semicircle is (100/7)/2 = (50/7).
The semicircle therefore has an area that is 1/2 the area of a circle with radius (50/72), or (1/2)(50/7)?(72) =
1250/7 square feet. O

Fee Start geometric word problems with a sketch. |


|

11.2.1 Find the area of a circle with diameter 18.

11.2.2. Find the area of a circle that has circumference 127.

11.2.3. Find the radius of a circle if its circumference is numerically equal to its area.

11.2.4 Ifa pizza that is 12 inches in diameter provides four full meals, how many meals are provided
by a pizza that is 20 inches in diameter?
11.2.5 Points A and B are on the circumference of ©O such that ZAOB = 120° and OA = 12.

(a) Find the area of OO.


(b) Find the area of sector AOB.

11.2.6 Sector XQZ of circle Q has area 307. Given that the whole circle has area 1007, find 2XQZ and
LXZQ. Hints: 342
11.2.7x A man standing on a lawn is wearing a circular sombrero of radius 3 feet. Unfortunately, the
hat blocks the sunlight so effectively that the grass directly under it dies instantly. If the man walks in a
circle of radius 5 feet, what area of dead grass will result? (Source: HMMT) Hints: 81

Extra! In 1897, the Indiana state legislature almost passed a bill that set the value of 7 to
impimeimeime exactly 3.2. The House voted unanimously for it and it passed a first reading in the
Senate. Fortunately, a math professor at Purdue University happened to be visiting the
legislature at the same time and advised that the bill be postponed indefinitely, effectively
killing it. If he hadn’t, manholes in Indiana would look very strange.

TTFTS
SS
eS Sc sn SSS SSS SSS Ss SSS SSS
11.2. AREA
A SS

' Sidenote: The ancient Greeks didn’t have calculus, so how did they find the for-
| ay mula for the area of a circle?) While they didn’t have calculus, they did
have Archimedes, some of whose proofs came so close to calculus (which |
wouldn’t be developed until nearly 2000 years after his death) that some >
suggest he might be called the father of calculus. Archimedes used two
favorite tactics of the Greeks — contradiction (see page 41) and comparing |
a desired area to that of a figure whose area is easy to find.
| Archimedes claimed that a circle has the same area as that of a right |
triangle with the circle’s radius as one leg and its circumference as the
other leg. He did so by proving that the area of the circle could be neither |
more nor less than that of the triangle. We’ll show you his proof that the -
circle’s area could not be greater than that of the triangle, and leave you to
walk in his footsteps and supply the proof for the other half.

2Ttr

Let the radius of the circle be r, the area of the circle be A, and the area of
the triangle be T. Then, we assume that A > T. As we saw in our discussion
of calculating 7, we can use regular polygons with more and more sides
to approximate a circle. Specifically, at some point, the difference between
the area of such a polygon, which we'll call P, and the area of the circle is
less than A — T. Therefore, we have A— P< A-T,soT <P.

However, we showed on page 252 that the area of sucha regular polygon
is one-half the product of its perimeter, p, and its apothem, a. Since the
perimeter of the polygon is smaller than the perimeter of the circle, we
have p < 2mr. Moreover, the apothem of the polygon is clearly less than
the radius of the circle. Therefore, we can use p < 27r and a < r to write
P = ap/2 < (r)(2nr)/2 = T. So, we have both T < P and P < T. This is
impossible, so we have reached a contradiction. We can thus conclude that
our assumption A < T was false.
See if you can complete the proof by showing that it is impossible to
have A > T. As you might guess, you'll have to consider a polygon
circumscribed about the circle!
Source: Journey Through Genius by William Dunham

293
CHARTRER 1, CIRGEES

11.3 Funky Areas

We now know how to tackle the area of sectors and of triangles. We can put these two basic tools
together to find the areas of all sorts of funky figures.

Problems ig
Problem 11.10: O is the center of the shown circle and OA = 8. The shaded region
between chord AB and the circle is called a circular segment.
(a) Find the area of sector AOB.
(b) Find the area of AAOB.
(c) Find the area of the shaded region.

Problem 11.11: I have a barn that is a regular hexagon, as shown. Each side of
the barn is 100 feet long. I tether my burro to point A with a 150 foot rope. In
this problem we will find the area of the region in which my burro can graze.

(a) Imagine youare the burro. Sketch out the area in which you can graze. Pay
close attention to what happens when you try to go past point B towards
point C.
(b) Break your grazing region into sectors whose areas you can find, then find
the area of the grazing region.

Problem 11.12: The shaded portion of the figure is called a lune. Given that
AB = 1, CD = Y2, and that AB and CD are diameters of the respective semicircles
shown, we wish to find the area of the lune. (Source: AMC 10)
(a) Let O be the center of the larger semicircle. Draw OA and OB. How long are v2
these segments? What kind of triangle is AAOB?
(b) Find the area of circular segment AB (i.e., the unshaded part of the small semicircle).
(c) Find the area of the lune.

Problem 11.13: Circle A passes through B and circle B passes through A. Given
that AB = 6, we wish to find the area of the shaded region common to both circles.

(a) Let the two points where the circles intersect be X and Y. Connect A and B to
each other and to X and Y.
(b) What kind of triangles are AXAB and AYAB?
We know how to find the areas of circles, triangles, and circular segments. Find an expression
for the shaded area in terms of pieces of the diagram whose areas you can find.
Find the shaded area.
11.3. FUNKY AREAS

We know how to find the areas of circles, sectors of circles, triangles, and lots of other polygons. In
this section we'll use that knowledge to find the area of even funkier shapes.
——EE

Biocon 11.10: oe is the center of the shown circle and OA = 8. The shaded region
between chord AB and the circle is called a circular segment. Find the area of this
circular segment.

Solution for Problem 11.10: We view the circular segment in terms of pieces whose areas we can find.
The circular segment is what’s left when we cut AAOB out of sector AOB. These areas are easy to find,
so we have
Area of circular segment AB = Area of sector AOB — Area of AAOB
207 2, (AO)(BO)
(sa08 )ns" 2
1G ao!)

Concept: Nearly all funky area problems are solved by expressing the funky area as |
O== sums and/or differences of pieces whose areas we can easily find. The first.
step should be clearly expressing the funky area in terms of simple areas,
as we did in Problem 11.10 when we wrote

Area of circular segment AB = Area of sector AOB — Area of AAOB.

Sometimes our region isn’t already drawn for us.

Problem 11.11: I have a barn that is a regular hexagon, as shown. soe side of the
barn is 100 feet long. I tether my burro to point A with a 150 foot rope. Find the area
of the region in which my burro can graze.

Solution for Problem 11.11: First we have to figure out what the burro’s region
looks like. Say the burro is tethered at A and roams so far that the rope pulls taut.
Suppose the burro keeps the rope taut and vwalks until part of the rope coincides
with AB. The burro then is at point G on AB. Since AB = 100 feet and the rope is
only 150 feet long, when the burro moves beyond G towards C, there’s only 50 feet
of rope for the burro to use. Hence, once the burro moves past G, the region he
can reach is just sector JBG. We have the same situation on the other side, leading
to sector HFI. Thus, our desired area is

Area of major sector HAG + Area of sector JBG + Area of sector HFI.
CHAPTER 11. CIRCLES

Since ZABC = 120° (because ABCDEF is a regular hexagon), we have ZJBG = 180° — 120° = 60°.
Therefore, sector JBG is 60°/360° = 1/6 of a circle with radius 50 feet. Hence, its area is (1/6)7(507) =
12507:/3 square feet.
On the other end of the burro’s range is sector HFI, which is exactly the same as sector JBG, so it
contributes 12507:/3 ft? to the burro’s area. Finally we have the sector between major arc GH and radii
AG and AH. Since ZFAB = 120°, the sector is (360° — 120°)/360° = 2/3 of a circle. The radius of this
sector is 150 feet, so its area is (2/3)7(1507) = 150007.
Combining these three parts gives us a grazing area of
=") S 4750071
150007 + 2 ( square feet.
3 Be ck:

Once again, notice that one of our key steps is to express our desired region in terms of not-so-funky
pieces we know how to tackle.

Problem 11.12: The shaded portion of the figure is called a lune. Given that
AB = 1, CD = Y2, and that AB and CD are diameters of the respective semicircles
shown, find the area of the lune. (Source: AMC 10)

Solution for Problem 11.12: We start by noting that if we can find the area of circular a B
segment AB, then we can find the area of the lune:

Area of lune = Area of small semicircle — Area of circular segment AB. C x 8


2

To get the area of the circular segment, we start by connecting its endpoints to the center, O, of its circle.
Since
Area of circular segment AB = Area of sector AOB — Area of AAOB,
we have

Area oflune = Area of small semicircle — Area of circular segment AB


= Area of small semicircle — (Area of sector AOB — Area of AAOB)
= Area of small semicircle — Area of sector AOB + Area of AAOB.

To find the area of sector AOB, we need ZAOB. Since CD = V2, we have OC = OA = OB = OD = v2/2.
Seeing those ¥2s makes us think of 45-45-90 triangles. We check and see that indeed AO? + BO? =
1/2+1/2 =1 = AB*, so ZAOB is a right angle since the sides of AAOB satisfy the Pythagorean Theorem.
We can now find the areas of all of our pieces. The radius of the small semicircle is 1/2, so its
area is 7(1/2)*/2 = m/8. Sector AOB is 90° /360° = 1/4 of a circle with radius ¥2/2, so its area is
(1/4)n( V¥2/2)2 = 71/8. Finally, [AOB] = (AO)(OB)/2 = 1/4, so we have

A pea Ot
Timea i
f] Uune =—-—
ae aa : 1
4° 4
O

i TEES GE
11.3. FUNKY AREAS

Again, look at our key step — expressing our funky area (the lune) in terms of not-so-funky areas
(semicircle, sector, triangle).

Sidenote: You might have noticed that the area of the lune in Prob-
| ay lem 11.12 equals the area of AAOB. This isn’t a coincidence!
| Moreover, the discovery that the area of the lune described
in Problem 11.12 equals the area of AAOB led to one of the
earliest notable proofs in math history.
| As we'll discuss on page 338, mathematicians tried in vain to square |
| the circle (to ‘square the circle’ means construct a square with the same _
area as a given circle). However, the Greek mathematician Hippocrates |
succeeded in squaring the lune using the insight that the area of the lune
of Problem 11.12 equals the area of the triangle with the lune’s diameter |
and the center of the larger semicircle as vertices. See if you can walk in his _
footsteps by both re-creating the proof, and by performing the construction. -
Source: Journey Through Genius by William Dunham |

_that AB = 6, find the area of the shaded region common to both circles.
|

Solution for Problem 11.13: We have a funky circular area, so we start by drawing x
some segments to cut the region into pieces we can handle. Some obvious candidates
are AB connecting the centers, and the segments from the centers to the points where
the circles meet. We thus form some triangles, sectors, and circular segments. We
know how to handle these — we start by finding angles. Ne
Since each of our segments is a radius of one (or both) of the circles, all of them are equal in length.
Specifically, AX = XB = AB = AY = YB = 6. Therefore our two triangles are equilateral, so all of their
angles are equal. We’re now ready to express the shaded region in terms of pieces we can handle. We
can do so ina few different ways. Here’s one way:
Shaded Area = Area of sector XBY + Area of sector XAY — Area of XAYB.

We subtract [XA YB] because when we add the sectors, we include the overlap, [XAYB] twice. We could
also have written .
Shaded Area = [XAYB] + 4(Area of circular segment XB),
since all the circular segments are congruent. We’ll take the first approach here since sectors are easier
to deal with than circular segments. Since ZXBY = ZXBA + ZYBA = 120°, the area of sector XBY
is (120°/360°)7(67) = 127. Similarly, the area of sector XAY is 127. Finally, XAYB consists of two
equilateral triangles with side length 6. Each has area 6* 3/4 = 9 V3, so [XAYB] = 2(9 Y3) = 18 V3.
Finally, we have
Shaded Area = 247 — 18 V3.
O

Zo
CHAPTER 11. CIRCLES

a PM oe é :
“Concept: In problems involving multiple circles, connecting the centers can be help-
| O=ze ful. In problems involving intersecting circles, connecting the intersection
pone toe the centers (and toee other) is often useful.

11.3.1 Find the area of the shaded region given that O is the center of the circle, ZAOB= 8B
120°, and the radius of the circle is 6. Hints: 78

11.3.2 Each side of equilateral AXYZ has length 9. Find the area of the region inside
the circumcircle of the triangle, but outside the triangle.
11.3.3 In the diagram at left below, AABC is an equilateral triangle with side length 6. Arcs are drawn
centered at the vertices connecting midpoints of consecutive sides, as shown. Find the area of the shaded
region.

C D C

DX, 4 'B

Figure 11.1: Diagram for Problem 11.3.3 Figure 11.2: Diagram for Problem 11.3.4

11.3.4 In the diagram at right above, quarter-circles have been drawn centered at vertices A and C of
square ABCD. Given that AB = 6, find the shaded area.

11.3.5x Inthe diagram at left below, XOY is a quarter-circle. Semicircles are drawn with diameters OX
and OY as shown. Find the area of the shaded region given that XO = 4. Hints: 405, 458

Figure 11.3: Diagram for Problem 11.3.5 Figure 11.4: Diagram for Problem 11.3.6

11.3.6* In the diagram at right above, ABCD is a square of side length 4. Two quarter-circles and a
diagonal are drawn as shown. Find the area of the shaded region. Hints: 121, 485, 450

Extra! 22/7 is a common approximation of 7. 355/113 is an even better approximation. In 1914,


mii the great Indian mathematician Ramanujan provided the uncanny approximation

How close is this to 7? (And how in the world did he find this?)

298
11.4. SUMMARY

11.4 Summary

Definitions:

e The set of all points that are the same distance from a given point is a circle. The given point
is the center of the circle, and the fixed distance is the radius.

We often refer to a cirele by its center using the symbol ©: ©O refers to a circle with center O.

A line that touches a circle in a single point is tangent to the circle, while a line that hits a circle
in two points is a secant. A segment connecting two points on a circle is a chord, and a chord
that passes through the center of its circle is a diameter.

The portion of a circle that connects two points on the circle is an arc, which we denote with
the endpoints of the arc: MN is the shorter arc that connects M and N.

The perimeter of a circle is called the circle’s circumference.

Important: Ifa circle has diameter d and radius r, then:

WW e d = ts

e The circumference of the circle is 7d, or 27r.

e The area of the circle is m7.

Definitions: A portion of a circle cut out by drawing two radii of the circle is called a sector of the
circle. A portion of a circle between a chord and the arc of the circle connecting the endpoints of the
chord is a circular segment of the circle.

Important: We can find the area of a sector of a circle of radius


V? r given the central angle of the sector. In the figure
to the right, since ZAOB is (ZAOB)/360° of the whole
circle, we have:
aa |i
Area of sector AOB = (
360°
We can find the area of a circular segment by first finding the area of
the sector formed by drawing radii to the endpoints of the segment’s
chord, then subtracting the area of the triangle with these radii and
the chord as sides.

Extra! us
00> 01 0- AROS |

209
CHAPTER 11. CIRCLES

Problem ay Strategies
—— oe

Concepts: e In complicated problems involving a, circle, tryconnecting the center |


O== of the circle to important points on the circle. |

e Start geometric word problems with a sketch. |

e Nearly all funky area problems are solved by expressing the funky
area as sums and/or differences of pieces whose areas we can easily
find. The first step should be clearly expressing the funky area in
terms of simple areas. |
/
e In problems involving multiple circles, connecting the centers can be -|
helpful. In problems involving intersecting circles, connecting the -
intersection points to the centers (and to each other) is often useful. —
b ess a oe — =

REVIEW PROBLEMS >

11.14 Circle O has radius 3 V3. Points A and Z are on the circumference of the circle such that
ZAOZ = 90°.

(a) Find the area of OO.


(b) Find the circumference of OO.
(c) Find the length of AZ.
(d) Find the area of sector AOZ.
(e) Find the area of circular segment AZ.

11.15 A giant earth-mover has rubber circular tires 11.5 feet in diameter. Given that there are 5280 feet
in a mile, how many revolutions does each tire make during a 6-mile trip? (Answer to the nearest full
revolution.) (Source: MATHCOUNTS)

11.16 Find the length of a 78° arc of a circle that has radius 14.
11.17 Find a formula that expresses the area, A, of a circle in terms of its circumference, C.

11.18 Radius OA of OO is a diameter of OB. Radius OB of ©B is a diameter of OC.


(a) Find the ratio of the area of OC to the area OO.
(b) Find CA/OA:
11.19 A 36° arc of OC is 247 units long. Find the circumference and the area of the circle.
11.20 Regular hexagon ABCDFF is inscribed in OO with radius 6. What is the ratio of the circumference
of the circle to the perimeter of the hexagon? Hints: 106

B00 °° eee
CHALLENGE PROBLEMS

11.21 On each side of a right triangle a semicircle is constructed using that side as a diameter. How
many square centimeters are in the area of the semicircle on the hypotenuse of a right triangle if the areas
of the semicircles on the legs of the triangle are 36 and 64 square centimeters? (Source: MATHCOUNTS)

11.22 What is the number of square centimeters in the area that is not shaded in
the diagram at right? The radius of the large semicircle is 1 centimeter, the radius
of the small circle is 0.5 centimeters, and the length of the longer leg on the right
triangle is 3 centimeters. (Source: MATHCOUNTS)

11.23. The larger circle at right has radius 1.5 times the smaller circle. Compute
the ratio of the partial ring ABCD to the area of sector BOC. (Source: ARML)
11.24 A rectangle that is 12 inches long and 5 inches wide is inscribed in a circle.
What is the area of the region that is inside the circle but outside the rectangle?
(Source: MATHCOUNTS)

11.25 Charlyn walks completely around the boundary of a square whose sides
are exactly 5 km long. From any point on her path she can see exactly 1 km horizontally in all directions.
What is the area of the region consisting of all points Charlyn can see during her walk? (Source: AMC
10)

Challenge Problems

11.26 Three circles of radius 12 lie in a plane such that each passes through the
center of the other two. Find the area common to all three circles. Hints: 479

11.27 When my car has wheels with a diameter of 24 inches, its speedometer
reports the correct speed of my car. I recently replaced my 24-inch wheels with
28-inch diameter wheels. I didn’t change my speedometer, however. When the
speedometer tells me the car is going 40 miles per hour (and I’m driving with my
28-inch wheels), how fast is my car really going? Hints: 515
11.28 Two congruent circular coins, A and Z are touching at point P. A is held stationary while Z is
rolled around it one time in such a way that the two coins remain tangent at all times. How many times
will Z revolve around its center? (Source: MATHCOUNTS) Hints: 568

11.29 The number of centimeters in the perimeter of a semicircle is numerically the same as the number
of square centimeters in its area. What is the number of centimeters in the radius of the semicircle?
(Source: MATHCOUNTS) Hints: 457

11.30 A cross-section of a river is a trapezoid with bases 10 ft and 16 ft and slanted sides of length 5 ft.
At this section the water is flowing at 7 mph. A little ways downstream is a dam where the water flows
through 4 identical circular holes at 16 mph. What is the radius of the holes? Hints: 383

"= 3
Extra! March14(3/14)isknownasPiday.
CHAPTER 11. CIRCLES

11.31. Given that AABD in the diagram at right is a right triangle with BD = 8 and
AB = 8 V3, find the total area of the shaded regions. Hints: 559

11.32 A circle of radius 3 passes through the center of a square with side length 2.
Find the positive difference between the areas of the nonoverlapping portions of the
figures. (Source: HMMT) Hints: 126

11.33 Six 120° arcs are drawncentered at the vertices of regular hexagon ABCDEF.
Each arc connects two vertices of the hexagon as shown in the diagram at right.
Given that AF = 6, find the total area of the shaded regions. Hints: 114

11.34 The number of inches in the perimeter of an equilateral triangle equals the
number of square inches in the area of its circumscribed circle. What is the radius
of the circle? (Source: AMC 10)
11.35 A circle has two parallel chords of length x that are x units apart. If the
part of the circle included between the chords has area 2 + 71, find x. (Source: HMM 8)

11.36x Quarter circles are drawn centered at each vertex of square ABCD as shown at left below. Given
that AB = 12, find the area of the shaded region. Hints: 217, 272,51

O xX D

Figure 11.5: Diagram for Problem 11.36 Figure 11.6: Diagram for Problem 11.37

11.37x The figure at right above shows a quarter-circle of radius 1, with A on BD such that ZAOD = 30°.
What must the distance OX be such that the region bounded by AX, BX, and AB occupies half the area
of the quarter circle? Hints: 514, 174, 304

Extra! In the third century B.C., the Greek scholar Eratosthenes (the same man who invented
‘wim the prime number sieve) was the director of the great library at Alexandria. At the
time, other Greek scholars knew that the Earth was round, but had no idea how large
it was. Eratosthenes learned of a deep well in Syene, a city south of Alexandria, whose
bottom was sunlit at noon every year on the summer solstice. This meant that the sun
was directly overhead. This gave Eratosthenes an ingenious idea: He could use this
observation to measure the size of the Earth.
He assumed that since the sun was so far away, the rays coming from the sun were
virtually parallel. Eratosthenes then measured a shadow (some say of a stick, some say
of a tower) in Alexandria on the summer solstice, and found that the rays made an angle
of approximately 7.2° with the object. Eratosthenes then required one more piece of data:
the distance between Alexandria and Syene, which he estimated at 800 km.
Continued on the next page. . .
ONEiene ee ee
302
CHALLENGE PROBLEMS

Extra! .. . continued from the previous page


|| |>

Sun’s rays

Oe 72°]
Alexandria

/; /Well at Syene

Gs / 2

Since there are 360° in a circle, Eratosthenes estimated the circumference of the Earth
to be
360°
800 km x = 40,000 km.

This is remarkably close to the actual figure, which is approximately 40,075.16 km.
Eratosthenes used an ancient measure of distance called a “stadium,” whose actual
length in today’s units is uncertain. Also, Syene is 729 km, not 800 km, from Alexandria
and slightly southwest instead of due south. Furthermore, Syene is not exactly on the
Tropic of Cancer (where the sun is directly overhead on the summer solstice), but 55 km
north. Soa number of errors seem to have fortuitously cancelled out to give Eratosthenes
such an accurate figure.
What is remarkable about this experiment is not the accuracy of Eratosthenes’ esti-
mate, but rather the conception of the experiment itself. As Dave Hanes, professor of
astronomy at Queen’s University, puts it on his website,

The critical point is that Eratosthenes recognized the nature of the problem,
found a method, and was able to derive an answer that was correct in spirit
in the sense that he correctly deduced that the Earth was an immense body
that was very much larger in extent than the then-known lands of the
Mediterranean basin, the home of Greek civilization at the time. The
sense of the discovery is the wonderful thing, not the mere accident that the
numerical value was also correct.
Even if Eratosthenes had been off by a factor of two or more in his calculations, it
would not have taken away from his brilliant insight into taking a simple observation of
the sunlit well and deducing the circumference of the Earth, demonstrating the power
_of a simple idea.

303
CHAPTER 12. CIRCLES AND ANGLES

The Simson Line

At a round table there is no dispute about place. — Italian Proverb

CHAPTER

Bs ins and Angles

In this chapter we will explore the many ways in which angles and arc measures are related.

12.1 Inscribed Angles

Q
Figure 12.1: An Inscribed Angle

We say that ZPQR is inscribed in PR in the diagram above because its vertex is on the circumference
of the circle and its sides hit the circle at P and R. Inscribed angles are enormously useful tools in
geometry; in this section we explore how inscribed angles are related to the arcs they cut off.

304
12.1. INSCRIBED ANGLES

|Problems
_ i
Problem 12.1: On page 175, we learned that the circumcenter of a right triangle C
is the midpoint of its hypotenuse. Thus, when we draw the circumcircle of a
right triangle, the right angle is inscribed in a semicircle. In this problem, we
investigate if it is true that any angle inscribed in a semicircle, such as ZACB in A B
the diagram at right, must be a right angle. Let O be the center of the circle. he
(a) Draw OC. What do we know about AAOC and ABOC?
(b) Let ZA = x and ZB = y. What other angle measures x degrees? What other angle measures y
degrees?
(c) Consider the sum of the angles in AABC to show that x + y = 90°.
(d) Must ZACB be a right angle?

Problem 12.2: In this problem we will find inscribed ZABC shown at right, given
that AC = 80° and ACB = 130°.
(a) Draw OA, OB, and OC. Find ZAOC and ZCOB.
B
(b) Find ZOBC and ZOBA. A —<o.
(c) Use your answer to the previous part to find ZABC. S
(d) Redo the problem with BC = 64°.
(e) Make a guess about how we can figure out ZB from the arcs without going through all the steps
above. Can you prove your guess?

Problem 12.3: In Problem 12.1, we showed that if an angle is inscribed in a 180° arc Z.
(a semicircle), the measure of the angle is 180°/2 = 90°. In this problem we will prove X
that when ZXYZ is inscribed in XZ as shown in the diagram (with O, the center of
the circle, inside ZXYZ), then ZY = XZ/2.

(a) Draw OX, OY, and OZ. Let ZOXY = x and ZOZY = z. Find ZXYZ in terms of
x and z.
(b) Find 2XOY, ZYOZ, and ZXOZ in terms of x and z.
(c) Find XZ in terms of x and z, then show that ZXYZ = xe 2

Problem 12.4: Given that AABC is inscribed in the circle as shown, ZA = 70°,
and AC = 130°, find ZC.

Extra! The primary question is not what do we know, but how do we know it.
11> {1111p —Aristotle

305
CHAPTER 12. CIRCLES AND ANGLES

Problem 12.5: Find x given that ZAPB = 2x, ZACD = x and BC =x.

Problem 12.6: Chords AC and BD of a circle meet at P as shown. Given AP = PB,


show that AB ||CD.

On page 175, we learned that when we draw the circumcircle of a right triangle, the right angle of
the triangle cuts off a semicircle. We start this section by exploring whether the converse is true.

Problem 12.1: When an inscribed angle cuts off a 180° arc,


we say that it is inscribed in a semicircle.
Is such an angle necessarily a right angle?

Solution for Problem 12.1: Let ZACB be inscribed in a semicircle. Since AB = &
180°, AB is a diameter of the circle. Therefore, the midpoint of AB, point O,
is the center of the circle. Drawing OC forms two isosceles triangles since
AO = BO = CO.
We suspect ZACB is 90°, so we let ZACO = x and ZBCO = y and try
to show that x + y = 90°. Since AAOC and ABOC are isosceles, we have
ZOAC = ZOCA = x and ZOBC = ZOCB = y. From AABC, we have ZA-+
ZACB + ZB = 180°, so
x+(x+y)+y= 180°.
Therefore, x + y = 90°, which means ZACB is a right angle. 0

Sidenote: That any angle inscribed in a semicircle is-a right angle is sometimes called
Thales Theorem, after the mathematician Thales of Miletus, who is re-
garded by many as the father of the idea of ‘proof’. He is credited by many
with the first proofs that vertical angles are equal, that the sum of the angles
ina triangle equals two right angles, and that the base angles of an isosceles
triangle are equal.
Thales was an outstanding problem-solver. Legend has it that a farmer
once complained to Thales, saying the farmer’s donkey would lighten its
load of salt on trips to the market by rolling in a river to dissolve the
salt. Thales suggested the farmer load the donkey with sponges next time.
Source: Journey Through Genius by William Dunham

306
12.1. INSCRIBED ANGLES

Important: Any angle inscribed in a semicircle is a right angle.

Having shown that any angle inscribed in a 180° arc has measure 180°/2 = 90°, we should wonder
if there’s a general rule relating the measure of an inscribed angle to arcs in the circle.
ecient
Concept: When trying to prove something or find some general rule, investigating
a fewasus cases can be an excellenteG:

We'llee thisBoe here.

Problem 12.2: Points A, B, and C are on ©O such that AC = 80° and ACB= 130°. Find ZABC.

Solution for Problem 12.2: We know that the measure of an arc equals the angle
formed by the radii that cut off the arc (we call such an angle a central angle).
Therefore, we draw radii to A, B, and C, thus forming some isosceles triangles.
Since BOC = BC = AB -— AC = 50°, we have ae
180° — 50° A z
LOBC = LOCB = —,— = 68°. <7
Similarly, ZAOB = AB = 130°, so &
ZOAB = ZOBA = {aS = 25°,
Therefore, ZABC = ZOBC — ZOBA = 40°.

We see that ZABC = AC/2 and we wonder if this is always the case. We can try changing BC to see if
that matters. If we let BC = 64°, we can go through the same series of calculations as above to find that,
indeed, ZABC is still 40°. 0

Now that we have a specific case as a guideline, we'll try to prove that an inscribed angle is always
half the arc it intercepts. Unfortunately, to completely prove this, we’ll need a number of cases. We'll
try one of these cases here, then leave the others for Exercises.

Problem 12.3: Prove that if ZX YZ is inscribed in a circle such that the center of the circle, O, is inside
AXYZ, then XYZ = XZ/2.

Solution for Problem 12.3: We use our specific case as a guide, and we connect the Z,
radii to X, Y, and Z to form isosceles triangles. We don’t know any angles this time, y a
so we'll have to use variables. Since we know we want to use our isosceles triangles,
we let ZOYX = x and ZOYZ =z, so that ZXYZ = x +z. Isosceles triangles AXOY and
AZOY tell us that ZOXY = x and ZOZY = z. Therefore, we have ZXOY = 180° — 2x
from AOXY, and ZZOY = 180° — 2z from AOZY. We can now find ZXOZ in terms of
x and z:
~ZXOZ= 360 =ZXOY —Z2ZYOZ = 2x +22 = 2(x-+ 2).
Therefore, XZ = 2(x + z) = 2ZXYZ, so LZXYZ= Nf 2

307
CHAPTER 12. CIRCLES AND ANGLES

Same = 9
|

“Important: — The measureof ofann inscribed angle isis one-half themea- P |


V? sure of the arc it intercepts. For example, R |

ZPOR =.
PR ||
Q a
|
|

Note that our proving that an angle inscribed in a semicircle is right is just a special case of this
result. However, we draw special attention to the right angle case because right angles are so important
in geometry.
Let’s try peule our newfound knowledge.

Problem 12,1 Given that AABC is peebed inthe circle as shown, ZA= 70°, A eS
|and AC = 430" tind: 2c. B

Solution for Problem 12.4: Solution 1: Since ZB is inscribed in AG we have ZB = AC /2 = 65°. Therefore,
ZC = 180° — ZA — ZB = 45°.

Solution Ie Since ZA is inscribed in BC, we |have BC = 2LA_= 140°. Therefore, we have AB =


360° — AC — BC = 90°. Since ZC is inscribed in AB, we have ZC = AB/2 = 45°. O
= a ————

Problem 12.5: Find x given that ZAPB = 2x, ZACD = x, and BC =x. A D

Solution for Problem 12.5: Since ZB and ZC are inscribed in the same arc, they must be equal (s
(since each
equals half the arc). Therefore, ZB = ZC= x. Since ZA is inscribed in BC, we have ZA = BC/2 = 2
Now we can use AAPB to write an equation for x. Since

ZA + ZAPB + ZB = 180°,

we have
96
5 + 2x +x = 180°.

Solving this equation gives x = 51 3°. Oo


We could have solved the last problem in many different ways, but our solution above illustrates a

oo 8C08”C*C*~C“<;<;< (C=«R:S:*~<C=C CO ee
12.1. INSCRIBED ANGLES

powerful principle that will be a crucial step in many problems when you move on to more advanced
geometry.

Important: Any two angles that are inscribed in the same arc are equal.
|
|

We'll now use this important fact in a proof.

Problem 12.6: Chords AC and BD of a circle meet at P as shown. Given AP = PB, A B


show that AB || CD.

Solution for Problem 12.6: Since AP = PB, we have ZPBA = ZPAB. Since ZCAB and ZCDB are inscribed
in the same arc, we have ZCDB = ZCAB. ZCAB is the same angle as ZPAB, so we have ZCDB = ZPAB =
ZPBA = ZDBA. Since ZCDB = ZDBA, we have AB ||CD. o

I)
Exercisesi
In Exercises 12.1.5 and 12.1.6 below, do not assume that an inscribed angle equals half the arc it
intercepts. You are asked to prove this fact for various cases in these two problems.

12.1.1 Points P, Q, and R are on ©O such that PO =37 35, OR = 1235 vand POR =)".

(a) Find ZQPR.


(b) Find ZPQR.
(c) Find ZPRQ.
(d) Find ZPOQ.
(e) Find ZPQO.
(f) Find ZPOR. (Be careful on this one!)

12.1.2 Points R and S are on ©E such that RS = 50°. Point T is also on OE. Find all possible values of
LRTS.
12.1.3. Points E, F and G are on ©O such that ZEFG = 48° and ZGEF = 78°.

(a) Find ZEGE.


(b) Find EG.
(c) Find EFG.
a SSS SSS

Extra! The advancement and perfection of mathematics are intimately connected with the prosperity of
a> i> > §—¢TE State.
(se —Napoleon Bonaparte

309
CHAPTER 12. CIRCLES AND ANGLES

12.1.4 Inthe diagram, O is the center of the circle and ZACB = 36°.
(a) Find ZD.
(b) Find AB.
(c) Find ZABF. (Do not assume EF ||AB at this point.) : -
(d) Find ZEFB. (Do not assume EF ||AB at this point.)
(e) Find CD.
(f) Prove that EF ||AB.
(g) Prove that AD = BC.

12.1.5 In Problem 12.2, we computed ZABC in the diagram at right for specific mea-
sures of AC and BC. We found that ZABC = AC/2:

(a) Prove that ZABC = AC/2 for all inscribed angles for which the center of the circle A 3 B
is outside the angle as shown in the figure at right. (Make sure you see how this C
problem is a little different from Problem 12.3.)
(b) Prove that ZACB has measure equal to one-half major arc AB.

G 12.1.6 In Problem 12.1, we showed that an angle inscribed in a semicircle is a


right angle. In this problem, we tackle the other angles that are formed when we
connect the endpoints of a diameter of a circle to a point on the circle. AB is a
diameter of ©O as shown. Prove that ZCAB = BC /2. Hints: 83

12.1.7 Arcs WX and YZ of ©Q are congruent. Prove that either WY || XZ or


; Lae WZ ||XY.
12.1.8 Back in Problem 9.5 of Chapter 9, we considered the regular 15-gon ABCDEFGHILKLMNO, and
found ZACD and ZADE. Find another solution with your new knowledge about angles and circles, by
considering the circumcircle of the polygon to find ZACD and ZADE.

12.2 Angles Inside and Outside Circles

A line that intersects a circle in two points is called a secant. In this chapter we'll explore relationships
between the angles formed by intersecting secants and the arcs of circles these angles cut off.

“Problems = |

Problem 12.7: Our goal in this problem is to find ZAEB given that AB = 40°
and DC = 80°.
(a) Draw AD to create inscribed angles. Find ZDAE and ZADE.
(b) Find ZAED and ZAEB.
(c) How is ZAEB related to AB and CD?

310
12.2. ANGLES INSIDE AND OUTSIDE CIRCLES

Problem 12.8: In this problem we will find a relationship between an angle


inside a circle and the arcs the angle intercepts. In the diagram, let QR = x and
PS=y.
(a) Draw PQ and find ZPQT and ZQPT in terms of x and/or y.
(b) Find, with proof, ZQTR in terms of x and y.

Problem 12.9: In this problem we will find ZDAE given DE = 110° and D
BC 20" B
(a) Draw BE to create inscribed angles. Find ZBEC and ZDBE. A
(b) Find ZDAE. C
(c) Can you find a general relationship that must hold among ZA, BC,
and DE? (In other words, what if we replace 110° and 20° with x
and y?)

Problem 12.10: Given that XV = band YW =a (these are arc measures, x


not lengths), prove that 2Z = (b — a)/2.

Problem 12.11: Given that AD = 40°, AB = 120°, and DC = 100°, find


¥BPC and 7 BEC.

Problem 12.12: In the figure, ZY = 58°, ZW = 20°, and XZ 1 WX.


(a) Find ZWOV.
(b) Find /ZXV.

Extra! You’ll sometimes see the letters ‘Q.E.D.’ at the end of a mathematical proof. These letters
1m
im Are an abbreviation for the Latin phrase quod erat demonstratum, which means ‘which was
to be proved.’

311
CHAPTER 12. CIRCLES AND ANGLES

Problem 12.13: P meets the circle at four points as shown. The arcs WX,
WZ, XY, and YZ are in order from smallest to largest, and each of the last
three is 42° larger than the arc listed before it. Find 2P. Hints: 23 (Source: P
Mandelbrot)

Having found a general formula for inscribed angles, we wonder how angles formed by secants are
related to the arcs they intercept. We start, of course, with a specific example.
2
Problem 12.7: Find ZAEB given that AB = 40° and DC = 80°.
A
40° 80°
B
‘€

Solution for Problem 12.7: We try to put what we just learned about inscribed D
angles to work by drawing AD to create some inscribed angles. We have ZDAC = A
Gere = 40° and ZADB = AB /2 = 20°. Therefore, 40° 80°

ZAED = 180° — ZEAD — ZEDA = 120°, B


i:
so, ZAEB = 180° — ZAED = 60°. (Note that we could have seen that ZAEB = 20°+ 40° since it is an
exterior angle of AAED.) 0
With that basic case as a guide, we’re ready to tackle finding a general formula.

Problem 12.8: Chords OS and PR intersect at T. Given that OR = x and PS = y, find a formula for
ZQTR in terms of x and y.

Solution for Problem 12.8: We form inscribed angles by adding PQ to our diagram.
Since ZQPR is inscribed in QR, ZQPR = x/2. Similarly, ZPQS = y/2. Since ZQTR is
an exterior angle of APQT, we have
x+y
LQTR = <PQT + ZQPT =

Important: The measure of the angle formed by two intersect-


! ing chords is the average of the measures of the
arcs intersected by the chords. For example, in the
diagram at right, we have

LC || = OTR = PS+QR x+y


PTS
27
cae ie BRE OR

312
12.2. ANGLES INSIDE AND OUTSIDE CIRCLES

We've tackled angles on a circle and angles inside a circle. You know where we’re headed now:
outside the circle.

P
Problem 12.9: Find /DAE given DE= 110° and BC=20°. D

Solution for Problem 12.9: We succeeded with inscribed angles before, , so D


we try using them again here by drawing BE. We have ZBEC = 20°/2 = 10° B
and ZDBE = 110°/2 = 55°. Thus, ZABE = 180° — ZDBE = 125°, and we have %

ZDAE = 180" — ZABE — ZBEC = 180° — 125° — 10° = 45”. =


:
Note that we could have seen that ZDBE is an exterior angle of AABE, so
ZDBE = ZBAE + ZBEC. As before, this gives us ZBAE = ZDBE — ZBEC = 45°. O

Now we have a clear path to prove the anes formula.

Eahion 12.10: Wereak that XV =


= 5} ae YW ==A ee are arc
measures, not lengths) in the diagram, prove that ee

b-a
LZ= cate

Solution for Problem 12.10: We draw YV and have ZYVW = a/2


and ZXYV = b/2. Since ZXYV is an exterior angle of AVYZ, we
have ZXYV = £Z + LYVZ. Therefore,

FeV =Vz =psae


0

Important: Twosecants that meet ata point outside x


a circle form an angle equal to half the x ae
difference of the arcs they intercept. For
example, in the diagram we have Z b

b-@ W ee V
i= Gn |

Let’s solve a few problems with our new relationships.

313
CHAPTER 12. CIRCLES AND ANGLES

Problem 12.11: “ Civen that 4D = 40° AB = 120° and


DC = 100° Gnd /BPC and /BEC.

Solution for Problem 12.11: Since all the arcs together in a circle must sum to 360°, we have BC =
360° — 120° — 40° — 100° = 100°. Therefore, we have

Bey ADI,
AVE
Bi
BE=aAD
ZBPG =a
2

Not all problems can be solved in one or two steps. Here’s an example in which we have to do a
little more work.

Problem 12.12: In the figure, ZY = 58°, ZW = 20°, and XZ 1 WX.


(a) Find <WOV. oS

Solution for Problem 12.12: We can’t find the angles we want right
away, but we have a circle with our angles, so we go hunting for
measures of arcs. Since ZXWZ = 20°, we know that XZ = 40°.
Since 2e= (WV — SAYS we have

Dox
_=.
WV
- 40°

Solving for WV gives WV = 156°. Now we can find ZWOV:

Se Wise _ 156 a ae
156°
We can use VZ to find ZZXV,s0 we focus on VZ. Since WVZ = 2ZZXW = 180°, we have VZ =
180° — WV = 24°. Therefore, ZZXV = 12°. oO

cs:
=O ° 8 ee
12.2. ANGLES INSIDE AND OUTSIDE CIRCLES

Concept: Exploration can help unravel problems whose solutions require several |
steps. In geometry problems, effective exploration often requires drawing |
an accurate diagram and finding any angle measures or segment lengths |
you can. When you find any new information, you should label your
diagram with your new information, as we did in the solution to Prob- |
lem 12.12. This helps you see when you have the information you need to
solve the problem.
bs

Problem 12.13: ZP meets the circle at four points as shown. The arcs WX, Xx
WZ, XY, and YZ are in order from smallest to largest, and each of the last
three is 42° larger than the arc listed before it. Find ZP. (Source: Mandelbrot) p

Solution for Problem 12.13: We start by labeling our diagram with the
given information. We let our smallest arc have measure x, and we
are given that the other three arcs are x + 42°, x + 84°, and x + 126°.
We could find x if we wanted to, but we don’t have to, because

XY-WZ (x +84°)
—(x+ 42°)
YP oe lea
2

Just because we use a variable in a problem doesn’t mean we have to find its value. However, the
variable did make finding the solution easier.

Concept: When you can’t find any more lengths or angles in a problem but you still
O=== _ haven't solved it, try assigning a variable to one of the lengths or angles.
Then, find other lengths or angles in terms of that variable. Finally, label |
your diagram with everything you find — this will make it easier to see
when you have enough information to solve the problem.

|
Exercises
12.2.1 Inthe diagram at left below, AB = 40°, BC = 103°, and CD = 83°. Find ZAED.
BA
w—4 ¥
V
C
D Z

Figure 12.2: Diagram for Problem 12.2.1 Figure 12.3: Diagram for Problem 12.2.2

12.2.2 Find YZ and VZ in the diagram at right above if XY = 78°, XV = 32°, and ZYWZ = 30.5°.
315
CHAPTER 12. CIRCLES AND ANGLES

12.2.3. Points P, Q, R, and S are in that order on OO. PR and QS meet at Z. Given that PQ = 2QR =
3RS = 4SP, find ZOZR. Hints: 336
12.2.4 Given that XY = yo XYZ = 117°, and YZW = 173° in the diagram at left below, find the
following:

(a) eAYRZ
(b)eZ.OZ
(c) ZXYW
(d) LOWZ

Figure 12.4: Diagram for Problem 12.2.4 Figure 12.5: Diagram for Problem 12.2.5
OR ae

12.2.5 Prove that ZPOR = onaet in the diagram at right above. Hints: 433

12.3 Tangents

Figure 12.6: A Tangent Line

A line is tangent to a circle when it touches the circle in only one point. For example, line m is tangent
to the circle at point A in the diagram above.
In this section we explore how a tangent line is related to the circle it touches.

Extra! President James A. Garfield used the diagram at right to prove


mini the Pythagorean Theorem. How did he do it? Hints: 5

316
12.3. TANGENTS

Problems >

Problem 12.14: Line m passes through point B on a circle with center O. In this problem, we will first
show that if m is tangent to the circle, then m 1 OB. We do so by showing that if ZOBC is not 90°, then
m must hit the circle a second time. Then, we will show that if m 1 OB, then m must be tangent to the
circle.
(a) Suppose ZOBC < 90° as shown at right. If so, there must
C
be some other line through O besides OB that is perpen- Ec Be
dicular to m. Let this line hit m at point D. What kind of “
triangle is AOBD?
(b) Show that OD < OB. Why does this mean that m must hit
the circle a second time?
(c) Show that if ZOBC > 90°, then m must hit the circle a second time. Conclude that if m is tangent
to the circle at B, then m 1 OB.
(d) Our first three parts show that a tangent line must be perpendicular owe €
to the radius drawn to the point of tangency. Now, we show that if LJ

a line m passes through a point B on a ©O such that m 1 OB, then m


must be tangent to the circle.
Suppose line m is meets ©O at B such that m 1 OB. Let X be any
point on m besides B. Consider AOBX and show that OX > OB. Does
this prove that m can’t hit the circle a second time if m 1 OB?

Problem 12.15: Our goal in this problem is to find ZZAB given that line
m
m is tangent to the circle at A and AB = 80°.
(a) Draw diameter AC and find BC.
(b) Find ZBAC, ZZAC, and ZZAB.
(c) What relationship holds between ZZAB and AB?

Problem 12.16: In this problem we will explore how an angle between a


secant and a tangent is related to the arcs intercepted by the angle. In the
diagram, let BD = 110° and BC = 40°. AB is tangent to the circle at B. A
(a) Draw BD and find BDC and ZDBA.
(b) Find ZA.

Problem 12.17: In this problem we investigate the relation- a


ship between the lengths of two tangent segments to a circle
from the same point, as well as how to find the angle between
recente —— i 3
them. PA and PB are tangent to the circle, which has center O. ><

(a) Prove that APOB = APOA.


AXB— AYB
(b) Prove {P= eS a

317
CHAPTER 12. CIRCLES AND ANGLES

Problem 12.18: Point P is 25 centimeters from the center, O, of a circle with radius 10 centimeters.
Point Q is on the circle such that PQ is tangent to the circle. Find PQ.

We start this section Slane a highly useful fact about tangents.

Problem 12.1: Given that line m is; tangent to ce) ati ‘prove that radius OB is B m
perpendicular to m. Conversely, prove that if a line m passes through point B on
©O such that OB 1 m, then m is tangent to ©O at B.

Solution for Problem 12.14: This is one of those ‘obvious’ facts in geometry that isn’t so easy to prove
rigorously. Here’s one ‘obvious’ solution that doesn’t quite work:

| Bogus Solcnon: ae the diagram is heesame on both sides of OB, the angles on
aL both sides of B must be the same. These angles must also add to
180°, sooy must be right angles. | i
=]

The problem with this solution is that the statement ‘the diagram is the same on both sides of OB’
essentially assumes what we are trying to prove. Moreover, the solution is not clearly written — what do
we mean by ‘the angles on both sides of B’? We'll have to find another way.

We let C be a point on m besides B. After trying and failing to rc ae


directly show that ZOBC = 90°, we decide to try proving it can’t be EO 82
anything else. First, we consider ZOBC < 90°. Since ZOBC is not \\
90°, there must be some other point D on line m such that OD 1 m.
Hypotenuse OB of right triangle AOBD must be the largest side of
AOBD, so OD < OB. Since OB is a radius, this means that line m
must go inside the circle if ZOBC < 90°. Therefore, the line will hit
the circle a second time. Specifically, consider the point F on m such that BD = DF (i.e., D is the midpoint
of BF). By SAS Congruence we have ABDO = AFDO, so OB= OF. Therefore, point F is on @O, so if
ZOBC < 90°, then line m cannot be tangent to the circle since it will hit the circle twice.
Similarly, if ZOBC > 90°, then ZOBE < 90°, and we can use exactly the same logic to show that m
cannot be a tangent line in this case either. Therefore, if m does not form a 90° angle with OB, it cannot
be a tangent line.

Extra! The feet of the perpendiculars from any point on the circumcircle of
“im a triangle to the sides of the triangle are collinear, as shown at right.
The line through these points is called the Simson line. See if you
can prove that these three points are always collinear! Hints: 21

318
12.3. TANGENTS

Unfortunately, we are not finished yet! We have shown that a line that
makes any angle besides 90° can’t be a tangent line, but we haven’t shown
that a line that makes a 90° angle is always a tangent. It might even be true
that it’s impossible for there to ever be a tangent line. Fortunately, it’s not,
and we'll prove it by considering line m at left, which hits the circle at B such
that m 1 OB. To show that mis a tangent line, we must show that it does not
hit the circle again.
We consider some point on line m besides point B, and call this point X. Since OX is the hypotenuse
of AOBX, we must have OX > OB. Hence, point X must be outside the circle, since its distance from O
is greater than the circle’s radius. This is true for any point on line m besides point B, so all points on m
besides B are outside the circle. Thus, line m is indeed tangent to the circle at point B. O
We therefore have a very non-obvious proof for our ‘obvious’ fact that:

|Important: A line tangent toa circle is perpendicular to Cae!


| the radius drawn to the point of tangency.
| Conversely, a line drawn through a point
| on a circle that is perpendicular to the ra-
dius drawn to that point must be tangent |
| to the circle. For example, for the diagram |
at right, we can write: |

| BC is perpendicular to radius OB if and only if BC is tangent to circle O.


Sats Wee Siete ee IEPA

You'll be seeing this ‘radius perpendicular to a tangent at the point of tangency’ again and again.

Problem 12.15: Line m is tangent to a circle at point A. Given point B on the circle such that AB = 80°,
find the acute angle formed by chord AB and line m.

Solution for Problem 12.15: We start by drawing diameter AC, since 7! A Z


we know this line is perpendicular to m at point A. Since AC is a
semicircle, BC = 180° — AB = 100°. Thus, ZBAC = BC/2 = 50°. Finally, B
/ZAB = LZAC — LBAC = 90° — 50° = 40°.
Note that ZZAB = AB/2. o
As an Exercise, you can follow these same steps to show:

Important: An angle formed by a tangent and a A


chord with the point of tangency as any, |
endpoint equals one-half the arc inter-
cepted by the angle. For example, in the
_ figure at right, line m is tangent to the
circle at A, so Che
ZZAB = ns
>
CHAPTER 12. CIRCLES AND ANGLES

An angle between a tangent and a chord is also often referred to as an inscribed angle. Yes, this is
the same name we give to an angle between two chords that share an endpoint, the measures of both
types of angles equal half the measures of the arcs they cut off.
We now turn to angles formed by a tangent and a secant.

Problem 12.16: In the diagram, BD = 110° and BC = 40°. AB is tangent to B


the circle at B. Find ZBAD.

Solution for Problem 12.16: Since this looks so much like problems x
involving an angle between two secants (such as Problem 12.9), we
try the same approach by drawing BD. We have ZBDC = BC/2 = 20°
and ZXBD = BD/2 = 55°.
Finally, since 2XBD is an exterior angle of AABD, we have ZXBD = G D
ZBAD “ZBDCso ZBAD = ZXBD— ZBDG = 552-20 = 357n

Unsurprisingly, the angle formed by a tangent and a secant has the same relationship to the arcs the
angle intercepts as an angle formed by two secants does.

Important: The angle formed by a tangent and a secant is B |


Vv? half the difference of the intercepted arcs. For |
example, A |
i ee
BD — BC C D |

Next we consider two tangents from the same point to the same circle.

Problem 12.17: Segments PA and PB are tangent to the same circle at A and B, respectively. Prove |
that PA = PB, and that ZAPB equals half the difference of major arc AB and minor arc AB.

Solution for Problem 12.17: We start by drawing the radii to the points A
of tangency, thus forming right angles. We also draw PO to complete a \
pair of right triangles. Since AO = OB, we have AOAP = AOBP by HL fon xX
Congruence. Therefore, PA = PB. P IL ORE
We can find ZAPB is a variety of ways. One is to use quadrilateral -
PAOB to note that "
ZAPB = 360° — ZPAO — ZAOB — ZOBP = 180° — ZAOB = 180° — AYB.

Since AXB = 360° — AYB, we also have

AXB-AYB
a 360° —2AYB
ee 4
eee

320
12.3. TANGENTS

The information about the angle between the tangents is unsurprising, as it’s essentially the same as
the angle between two secants. However, the information about the two tangents from the same point
is new, and is one of the most useful tangent facts:

Important: As shown in the diagram at right, we can draw A


vv? two tangent segments to a circle from a point
outside the circle. These tangents are always
equal to each other in length. ¥

We'll finish with a very common problem involving tangents.

|Problem 12.18: Point Pis 25 centimeters from the center, O, of a circle with radius 10 centimeters.
Point Q is on the circle such that PQ is tangent to the circle. Find PQ.

Solution for Problem 12.18: We start by drawing a radius to the point of tangency P Q
as shown. Since PQ is tangent to the circle, APOQ is a right triangle. Thus, ere 7
OP? = OQ? + PQ?, so

PQ = OP? — OQ? = V252 — 102 = 1/52(52 — 22) = 5 V52 — 22 = 5 V21 centimeters.


Notice how a little factoring makes our arithmetic easier. 0

Exercises i
12.3.1 Points U and | are on ©O such that major arc UI is twice as long as minor arc UI. P is outside
the circle such that PU and PI are tangent to ©O.
(a) Find minor arc UI.
(b) Find ZUPI.
(c) Find ZPIO.
(d) Find zJOU.
(e) ‘Find ZPIU.
12.3.2 Point T is outside ©O. X is on OO such that TX is tangent to ©O. The radius of OO is 6 and
PX 2 kind: FO.
12.3.3 Generalize Problem 12.15 by proving that the angle formed by a tangent and a chord that has
the point of tangency as an endpoint is half the arc intercepted by the angle.
12.3.4 Y and Z are on OG and X is outside OG such that XY and XZ are tangent to OG. Given that
ZYXZ = 51°, find the measure of minor are YZ.

12.3.5 Givena circle and a point outside the circle, it is intuitively clear that there are exactly two lines
through the point that are tangent to the circle. However, intuitively isn’t good enough — we're going to
prove it. Let the point be P and the circle be 00.
nn
321
CHAPTER 12. CIRCLES AND ANGLES

(a) Consider the circle with diameter OP. Call this circleC. Why must C hit ©O in at least two different
points?
(b) Why is it impossible for C to hit OO in three different points? Hints: 530
(c) Let the points where C hits ©O be A and B. Prove that ZPAO = ZPBO = 90°.
(d) Prove that PA and PB are tangent to ©O.
(e)x Now for the tricky part — proving that these are the only two tangents. Suppose D is on ©O such
that PD is tangent to ©O. Why must D be on C? Hints: 478
(f) Why does the previous part tell us that PA and PB are the only lines through P tangent to OO?

12.3.6x ©O is tangent to all four sides of rhombus ABCD, AC = 24, and AB = 15.

(a) Prove that AC and BD meet at O (i.e., prove that the intersection of the diagonals of ABCD is the
center of the circle.) Hints: 439
(b) What is the area of ©O? Hints: 508

12.4 Problems

Problems >

Problem 12.19: A quadrilateral is said to be a cyclic quadrilateral if a circle can be drawn that passes
through all four of its vertices. Prove that if ABCD is a cyclic quadrilateral, then ZA + ZC = 180°.

Problem 12.20: In the figure, PC is tangent to the circle and PD


'bisects ZCPE. Furthermore, CD = 70°, DE = 50°, and ZDQE =
40°. In this problem we determine the measure of the arc from A Se.
to E that does not include point C. Ng 50°
(a) Find BC. SG
(b) Find ZCPB.
- (c) Find AB.
(d) Finish the problem.

/Problem 12.21: Three congruent circles with radius 1 are drawn inside equilateral
AABC such that each circle is tangent to the other two and to two sides of the triangle.
| Find the length of a side of AABC. Hints: 428

Problem 12.22: XY is tangent to both circle O and circle P. Given that


OP = 40, and the radii of circles O and P are 31 and 7, respectively, find
XY. Hints: 548, 267

322
12.4. PROBLEMS

Problem 12.23: Median AM of AABC has length 8. Given that BC = 16 and AB = 9, find the area of
AABC. Hints: 25, 187

Problem 12.24: The diagram shows the incircle of AABC. X, Y, and Z are the
points of tangency where the incircle touches the triangle. In this problem
we will find an expression for AZ in terms of the sides of the triangle. Z Y

(a) Find equal segments in the diagram and assign them variables.
(b) Let AB =c, AC = b, and BC = a. Use your variables from the first part B C
to write equations that include these lengths.
(c) Solve your resulting equations for AZ. Hints: 157

We'll now apply our knowledge of circles, angles, and tangents to more challenging problems and
develop some useful geometric concepts.

|Problem 12.19: A quadrilateral is said to bea cyclic quadrilateral if a circle can be drawn that passes |
through all four of its vertices. Prove that if ABCD is a cyclic quadrilateral, then ZA + ZC = 180°.

Solution for Problem 12.19: Since ZA and ZC are inscribed angles, we have A D

BCO®? EAL BCD + DAD p00


a ees 5 5 1
180 foe

Oo B
g

A quadrilateral is a cyclic quadrilateral if a circle can be drawn that |


passes through all four of its vertices. Such a quadrilateral is said to
inscribed in the circle. The opposite angles of any cyclic quadrilateral
sum to 180°.

You'll be seeing a lot more of cyclic quadrilaterals when you move into more advanced geometry.

Problem 12.20: In the figure, PC is tangent to the circle and C79


PD bisects CPE.) If.CD = 70°, DE = 50°, and ZDQE = 40°, P er
then determine the measure of the arc from A to E that does not Pome ered
include point C.

Solution for Problem 12.20: We can’t directly find the desired arc, so we try finding whatever we can.
First, we note that since ZDQE is the average of DE and CB, we have CB = 30°. Now that we have CB,
we can find ZCPD: oe aa
ZCPD = (CD — CB)/2 = 20°.

Since DP bisects ZCPE, we know LDPE = LCPD = 20°. Because DPE = (DE — AB)/2, we find AB = 10°.
i eiicn ( °° = = 689
CHAPTER 12. CIRCLES AND ANGLES

Now that we have all the other arcs of the circle, we can find our desired arc by subtracting from
360°: 360° — 50° — 70° — 30° — 10° = 200°. Oo

“Concept: Label all the given information in the problem, and all the information
| O==2 ~syou find as you find it. This will help you discover new facts about the

like the figure below. Notice that our various arc measures and angles are
marked, and that the equal angles at P are marked equal. |

kere ee

each circle is tangent to the other two and to two sides of the triangle. Find the length of a side of
AABC.

Solution for Problem 12.21: We need to create simple figures to work


with, so we start by connecting the centers of our circles and drawing
radii to tangent points. (Note that connecting the centers of the circles
is the same as drawing radii to where the circles are tangent.) Since
WXYZ is a rectangle (because XW = YZ, XW ||YZ, and ZXWZ = 90°),
we have WZ = XY = 2. Hence, we need only find CW to finish, since
BZ is the same as CW.
We draw CX to build a right triangle and note that this segment
bisects ZACB because circle X is tangent to both AC and BC (and hence
its center is equidistant from them). Since ZXCW = (ZACB)/2 = 30°,
ACXW is a 30-60-90 triangle. Thus, CW = XW V3 = V3. Finally, we have BC = BZ+ZW+WC = 242 V3.
I

oo
Concept: When you have tangents in a problem, it’s often very helpful to draw radii
to points of tangency to build right triangles. When you have tangent
circles, connect the centers. (In fact, if you have multiple circles in a
problem, connecting the centers will sometimes help even when the circles
aren't tangent.)

Problem 12.22: X is on circle O and Y on circle P such that XY is tangent to both circles. Given that
OP = 40, and the radii of circles O and P are 31 and 7, respectively, find XY.

324
12.4. PROBLEMS

Solution
for Problem 12.22: We start with the usual segments to
draw: XY, the radii to points of tangency, and the segment con-
necting the centers. We still don’t have a right triangle to work
with, but we do know that radii OX and PY are both perpendicular
to tangent XY as shown. Since both OX and PY are perpendicular
to the same line, they are parallel. We make a right triangle and
a rectangle by drawing a line through P parallel to XY. ZX = PY = 7,so OZ = OX — ZX = 24. Since
OP = 40, we have PZ = 32 from right triangle AOZP. Since XYPZ is a rectangle, we have XY = ZP = 32,
so the length of the common tangent is 32.

XY is called a common external tangent of the two circles. As an Exercise, you'll find the length of
the common internal tangent, too. 0
=a So aes, Se ko Sees Med Gea a GAD ret amen amatali aaa: Ree cae
|Problem 12.23: Median AM of AABC has length 8. Given that BC = 16 and AB = 9, find the area of
AABC.

Solution for Problem 12.23: When we draw the figure and label all our lengths, A
we see that BM = AM = CM = 8. Therefore, a circle centered at M with
radius 8 goes through all three vertices of AABC. Since BC is a diameter of this
circle, ZBAC is inscribed in a semicircle and therefore must be a right angle. p G
So, AC = VBC? — AB? = 5 V7. AABC is a right triangle, so its area is half the
product of its legs:
_ (AB)(AC) _ (9)(5V7) _ 45 v7
ce Da 2 Z.

Using this same reasoning, we can also prove this important fact:

Important: _ Ifthe length of a median ofa triangle is half the length of the side to which |
it is drawn, the triangle must be a right triangle. Moreover, the side to
which this median is drawn is the hypotenuse of the right triangle.

We can also look to this problem for some important problem solving techniques:

Concept: When stuck on a problem, always ask yourself ‘Where have I seen some-
}=s5 ‘thing like this before?’ In Problem 12.23, we have a median that is half
the side to which it is drawn. This should make us think of right tri-
angles, since the median to the hypotenuse of a right triangle is half the
hypotenuse. Then we go looking for right triangles.

4Concept: : Always be on the lookout for right triangles.

Problem 12.24: Let Z be the point where the incircle of AABC meets AB. Find AZ in terms of the
sides of AABC.

325
CHAPTER 12. CIRCLES AND ANGLES

Solution for Problem 12.24: We start by labeling the equal tangents


from the vertices as shown in the diagram. We want to relate x to the
sides of the triangle, so we write the sides of the triangle in terms of
x, y, and z. We let the sides of the triangle be AB = c, AC = b, and
BC =a and we have:

ABA se "= XY
ACTS VUES ox ez
BC a= ytz

We want x in terms of a, b, and c. Adding the three equations will give us x + y + z, which we can
use with y + z = a to find x:
ot Die
avbote=2(x+y + Z) SO xty+z=" 5 ‘

We can then subtract the equation


y + z =a from
x + y+z =(a+b+c)/2 to find

gaye SIDE copecee et


ee = ;

where s is the semiperimeter (half the perimeter) of the triangle. Similarly, y=s-—bandz=s-—c. 0

Concept: Symmetric systems of equations can often be easily solved by either mul- |
co tiplying all the equations or adding them. |
|
-

Important: The lengths ‘from the vertices of —


AABC to the points of tangency of
its incircle are given as follows:

AL = AY = sa
BZ. = BX == ¢—h
CX = CY. = sc s—b

where AB = c, AC = b, and BC =a, B


and the semiperimeter of AABC is s.

Exercises

Problems 12.4.3, 12.4.4, 12.4.5, and 12.4.8 are very important relationships that you'll be seeing again
in your study of more advanced geometry. Be sure to pay special attention to them.
12.4.1 Is every quadrilateral cyclic?
12.4.2 Prove the following about cyclic quadrilaterals:
(a) Acyclic parallelogram must be a rectangle.
(b) Acyclic rhombus must be a square.
(c) Acyclic trapezoid must be isosceles.

Cee
ee ae eee
12.5. CONSTRUCTION: TANGENTS

12.4.3 Prove that if median XM of AXYZ has half the length of YZ, then AXYZ is a right triangle with
hypotenuse YZ.
12.4.4 ABCD isa cyclic quadrilateral. Prove that ZACB = ZADB.

A 12.4.5 Quadrilateral ABCD in the diagram at left is a circumscribed quadrilat-


B eral, meaning that it is circumscribed about a circle, so the circle is tangent to all
four sides ofABCD. Show that AB + CD = BC + AD. Hints: 570

c 12.4.6 Does every quadrilateral have an inscribed circle (a circle tangent to all
four sides), as ABCD does in the previous problem?
D
12.4.7 In the figure at right, AB is tangent to both ©O and ©P. The radius of OO A
is 8, the radius of OP is 4, and OP = 36. find AB. (A common tangent like AB is (+P)
sometimes called the common internal tangent of two circles.) Hints: 591 D
12.4.8 Prove that the inradius of a right triangle with legs of length a and b and
hypotenuse c is (a + b — c)/2.

A 12.4.9x The five circles in the diagram are congruent and ABCD is a square with
side length 4. The four outer circles are each tangent to the middle circle and to the
square on two sides as shown. Find the radius of each of the circles. Hints: 543, 416

12.5 Construction: Tangents

Now you'll use your newfound tangent knowledge to construct tangents to circles.

[___Proviems
i
Problem 12.25: Given a circle with center O and point A on the circle, construct a line through A that
is tangent to the circle.

Problem 12.26: Given a circle with center O and point P outside the circle, construct a line mrougP
thatis tangent to the circle. oo

Problem 12.27: -Nonintersecting circles with centers O and P are shown.


Constructa line that is tangent to both circles.

Extra! Seek simplicity, and distrust it.


ledindindina —Alfred North Whitehead
327
CHAPTER 12. CIRCLES AND ANGLES

We'll start with constructing a tangent through a point on a circle.

25: Give a circlenwith center O and point A on the circle, construct a line through A
|
that is tangent to the circle.

Solution for Problem 12.25: Make sure you see why this is not a correct solution:

Bogus Solution: We place our straightedge on the paper so that it goes through the |
AL point A, then we slowly turn it, keeping it through A, until it only _
| touches the circle in one point. |
ee
This Bogus Solution is not a Euclidean construction — it’s just a sketch. We need to be able to prove
our construction works. Here, the proof would essentially be ‘Our line touches the circle once because
we say so.’ That’s not good enough. We need to use geometric principles to prove our line is tangent,
not just eyeball judgement.

We know how to construct perpendiculars, and we know that a tangent


is perpendicular to the radius drawn to the point of tangency. Putting these
together, we have a pretty straightforward construction. We draw OA, which
includes the radius to point A, then construct the line line through A perpen-
dicular to OA. This line is our tangent line.
Our construction of this tangent line is indicated in the diagram at right.
We draw a circle centered at A, which hits OA at B and C. We then draw two
pairs of intersecting arcs with the same radius centered at B and C to find X
and Y. XY is the perpendicular bisector of BC, and is therefore our tangent
line. O
So we can handle a tangent through a point on a circle; how about one through a point outside the
circle?
Problem 12.26: Given a circle with center O and point P outside the circle, construct a line through
P that is tangent to the circle.

Solution for Problem 12.26: Unfortunately, we can’t simply construct a radius, then draw a perpendicular
line, because we don’t know which radius to draw! So, feeling a little stuck, we look for a simpler
problem to solve. But that doesn’t get us anywhere, either — there’s not any obvious simpler problem
that will help us with this one. Therefore, we draw the completed diagram and look for clues how we
might possibly construct it given only the circle and point P.

We go ahead and include the radius to the point of tangency because A


right angles are very useful, and we know how to construct perpendicular
lines. If we can construct point A, where the tangent meets the circle, we’ll
be set. The only seemingly useful information we have is that ZOAP is a ie
right angle, i.e, AOAP is a right triangle. So, we consider what we know
about right triangles.

O28F
st ee ae ees
12.5. CONSTRUCTION: TANGENTS

The circumcenter of any right triangle is the midpoint of its hypotenuse.


Since A must be on the circumcircle of AOAP, and AOAP must be a right Aa
triangle with hypotenuse OP, we know that A is on the circle that has OP as
a diameter. Therefore, to find A, we construct M, the midpoint of OP. We
then draw a circle with center M and radius MO. Where this circle hits ©O
is the point A where our tangent touches the circle. PA is the tangent. (And
the other point where the new circle hits ©O gives us the other tangent to OO A ie
through P.) O x

Concept: When tackling challenging construction problems, ‘trydrawing aa com-


pleted diagram (without straightedge & compass construction). Then use -
observations about your completed diagram to figure out how to construct |
it with straightedge & compass alone. |

Now that we’ve mastered tangent constructions, and have an intriguing new construction-finding
strategy, let’s try an even more challenging problem.

Problem 12.27: Given two nonintersecting circles with different radii, construct aline that is tangent
to both.

Solution for Problem 12.27: This construction looks pretty compli-


cated, so we start from the completed diagram. We should prob-
ably add radii to the tangent points to give us perpendicular lines
to work with. As we draw these, we realize, ‘Hey, we’ve done
a problem very much like this!’ We think back to Problem 12.22,
which we solved by creating the diagram shown at right.
Maybe this diagram will give us a clue. If we can find a way to construct any of X, Y, or even Z
given only circles OO and ©P, we'll be able to construct a line tangent to both circles.
At this point, we might think:

Bogus Solution: Pick a point Z, draw OZ until it hits


©O. That gives us point X. Then con-
struct a line through X that is tangent
to ©P using the construction from
Problem 12.26.

Unfortunately, this badly fails. If we just take any old Z, we might get a diagram like the one we
want, but we probably won't. XY is indeed tangent to OP, since we constructed it to be tangent to OP.
However, this line is not necessarily tangent to ©O, as our diagram clearly points out.
So, we can’t just pick any point to be Z (or X, or Y). And we can’t just keep trying different possibilities
for Z until we get one that works! We must find a specific one that we can prove works. Looking again
at our ‘working backwards’ diagram we drew first, we see that Z is the vertex of the right angle of right
triangle AOZP. As we saw in Problem 12.26, this means that Z is on the circle that has OP as a diameter.

329
CHAPTER 12. CIRCLES AND ANGLES

So, starting from just our circles and their centers, we at least have a first
step: we construct a circle with OP asa diameter. Point Z must be on this circle.
If we can find another circle or line with Z on it, we'll be finished. Thinking
one more time about Problem 12.26 and our ‘working backwards’ diagram,
we note that OZ is the difference between the radii of the two circles. Call
this difference d. We can construct a segment with length d since we know the
radii of the circles!

After constructing a segment with length d, we


can draw a circle with center O and d as the radius.
Since Z must be d away from O, it must be on this
little circle. Therefore, the intersection of this circle
and the circle with OP as diameter is Z! Extending
OZ to hit ©O gives us X, and drawing the line per-
pendicular to OZ through X gives us the common
tangent.

Of course, you'll get to prove that this final line


is tangent to OP as an Exercise. 0

12.5.1 Given two intersecting lines, construct a circle that is tangent to both of them.

12.5.2 Prove that our construction in Problem 12.27 does produce a line that is tangent to both circles.
12.5.3 In Problem 12.27, we constructed the common external tangent of two circles. How can we
construct a common internal tangent of two nonintersecting circles (i.e., a line tangent to both circles
that intersects the line connecting the centers of the circles)? Hints: 29, 440, 378

12.6 Summary

Definition: An angle formed by two chords of a circle is inscribed in the angle it cuts off. |

Important: e Any angle inscribed in a semicircle is a right angle.

V e Aninscribed angle equals 1/2 the measure of


the arc it intercepts. For example, R
—_—

ZPQR = =.

e Any two angles that are inscribed in the same


arc are equal.

330
12.6. SUMMARY

Important: The measure of the angle formed by two intersect-

VY ing chords is the average of the measures of the


arcs intersected by the chords. For example, in the
diagram at right, we have

LPTS = LOTR = PS+OR x+y


2 many

_ Important: Two secants that meet at a point outside

VY a circle form an angle equal to half the


difference of the arcs they intercept. For
example, in the diagram we have

Important: A line tangent to a circle is perpendicular to

Y the radius drawn to the point of tangency.


Conversely, a line drawn through a point
on a circle that is perpendicular to the ra-
dius drawn to that point must be tangent
to the circle. For example, for the diagram
at right, we can write:

BC is perpendicular to radius OB if and only if BC is tangent to circle O.

| Important: An angle formed by a tangent and a

VY chord with the point of tangency


endpoint equals one-half the arc
as an
inter-
m

cepted by the angle. For example, in the


figure at right, line m is tangent to the
circle at A, so

LZAB =

The angle formed by a tangent and a secant is


half the difference of the intercepted arcs. For
example,
BD - BC
ZBAD = 5)
CHAPTER 12. CIRCLES AND ANGLES

|Important: “As showninthe diagram at right, we can draw A


Vv two tangent segments to a circle from a point |
outside the circle. These tangents are always
equal to each other in length. P
|ALS
EAVES
ES

“Important: A cyclic quadrilateral is aquadrilateral that can be inscribed in a circle.


Vv The opposite angles of any cyclic quadrilateral sum to 180°.

“Important: — Tff the length of


ofamedian ofatriangle iis half the length of the side to which
Vv it is drawn, the triangle must be a right triangle. Moreover, the side to
we this median is drawn is peeS ee of the right triangle.
SS ee =

Important: - The lengths from the vertices of


? AABC to the points of tangency of
its incircle are given as follows:
AZ = AY = sa
Bl = BX = s—b
CAS CY. = sec

where AB = c, AC =b,andBC
=a, B aes ae =
and the semiperimeter of AABC is s. : |
cS sa pesmi

Important: The length of the inradius of a right triangle with hypotenuse of length
Vv c and legs of lengths a and b is

a+b-—c

Problem Solving Strategies

Concepts: e When trying to prove something or find some general rule, investi-_
gating a few sample cases can be an excellent guide.
e When you can’t find any more lengths or angles in a problem but
you still haven’t solved it, try assigning a variable to one of the
lengths or angles. Then, find other lengths or angles in terms of that
variable. Finally, label your diagram with everything you find — this
will make it easier to see when you have enough information to solve
the problem.

Continued on the next page.


REVIEW PROBLEMS

Concepts: ... continued from the previous page

e When you have tangents in a problem, it’s often very helpful to draw |
radii to points of tangency to build right triangles. When you have |
tangent circles, connect the centers. (In fact, if you have multiple -
circles in a problem, connecting the centers will sometimes help even —
when the circles aren’t tangent.) |
When stuck on a problem, always ask yourself ‘Where have I seen |
something like this before?’ |

e Always be on the lookout for right triangles.


Symmetric systems of equations can often be easily solved by either |
multiplying all the equations or adding them.

e When tackling challenging construction problems, try drawing a


completed diagram (without straightedge & compass construction),
then using observations about your completed diagram to figure out |
how to construct it with straightedge & compass alone.

12.28 Find x in each of the following

(a) : i
b da (ofa
120° aw s
H
R

SOS
B y J
se
190°

(d) ZACB = 10° Batgvp Otpa in 30

OL)
Ais

333
CHAPTER 12. CIRCLES AND ANGLES

12.29 Find x in each of the following. (Assume that all lines that look like tangent lines are tangent
lines.)

(b)
IEG NG
Y Ww

155°
4x

(c) (d) (e) ZJLI = 43°


P ie i
Q Nae H

is I
L

I K
(eer

12.30 AC is a diameter of OO in the diagram at left below. Given ZACB = 58° and # A
BD = CD, find the following:
WO
(a) AB.
B
(b) ZBAC.
(c) ZDBC. ©
12.31 Points A, B,C, and D are ona circle in that order, and AC and BD meet at E. Given that AB = 34°,
GD i= 1027 J Ar=-109— find ZBEC:

12.32 Prove that if line m passes through a point B on circle O such that m is perpendicular to OB, then
m must be tangent to the circle. (Try this one without looking back in the book for the proof!) Hints: 302
12.33 Use your knowledge about inscribed angles to explain why the circumcenter of an obtuse triangle
is outside the triangle.

12.34 Point T is on minor arc RS of ©O. Given that ZROS = 53°, find ZRTS.
12.35 KT meets OT at Y. Given that YK = 6 and YT = 2, find the length of the tangent segment from K
tool,

334
REVIEW PROBLEMS

12.36 Point Y is on ©F and X outside ©F such that XY is tangent to F. XE intersects ©F at A and B such
that XA < XB. Given that YB = 134°, find AY, ZYXF, and ZXYA.

12.37 AT and BT are tangent to ©O as shown at left below. Given AT = 6 and AB = 120°, find OT,
ZATB, and [OATB].

A
{ih Sis
28 SSB

B (oh ae
Figure 12.7: Diagram for Problem 12.37 Figure 12.8: Diagram for Problem 12.38

12.38 In the circle at right above, AB || CD, AD is a diameter of the circle, and AD = 36. What is the
length of AB? (Source: MATHCOUNTS)

12.39 ABCDEFGHIJKL is a regular dodecagon. Use the circumcircle of the dodecagon to find the
following:
(a) ZABC.
(b) ZACD.
(CULALALI.
(d) the acute angle between AF and BI.

12.40 Given ZVWZ = 81° and ZWZX = 38° in the diagram at right, find the w_ x
following: V

(a) ZVXZ.
(b) ZWYZ. Y
(c) 2A.
(d) LXYZ.
(e) LZWXY.

12.41 ABCD is a square with area 100. OO is tangent'to all four sides of the square. Diagonal BD meets
the circle at X and Y, with X closer to D than to B. Find DX.

12.42 Right triangle AXYZ and its incircle are shown at right. As shown, Z is be
6 units from the point where theincircle touches XZ, and Y is 3 units from the
point where the incircle touches XY. Find YZ.
12.43 Prove that the incircle of a given triangle is unique. (In other words, prove
that there is only one circle tangent to all three sides of a given triangle.) Ax 3

335
CHAPTER 12. CIRCLES AND ANGLES

12.44 Circles @A and @B are tangent at point C. P ison OA and Qis on ©B such Q
that PQ is tangent to both circles. Given AC = 3 and bBG="S,nnder @: P6

12.45 A circle is inscribed in a regular hexagon and another circle is circum- cS


scribed about the hexagon (i.e. it passes through all six vertices of the hexagon).
Find the ratio of the area of the smaller circle to the area of the larger circle.
Hints: 195

Challenge Problems

12.46 In the diagram at left below, lines f and m are tangent to the circle. Prove that @ || m.

E B

A D

Figure 12.9: Diagram for Problem 12.46 Figure 12.10: Diagram for Problem 12.47

12.47 ABCD is a parallelogram. BC extended past B meets the circumcircle of AABD at point E as
shown at right above. Prove that ED = CD.
12.48 AABC is a right triangle and the radius of its inscribed circle equals 5. If the perimeter of AABC
exceeds twice its hypotenuse by N, compute N. (Source: ARML) Hints: 449
12.49 Let A, B,C, and D be points on a circle w, in that order. Let P and Q be points on w such that PA
bisects ZDAB and QC bisects BCD. Prove that PQ is a diameter of w.

12.50 Let the incircle of triangle ABC be tangent to sides BC, AC, and AB at points D, E, and F,
respectively. Given that ZA = 32°, find ZEDF. Hints: 127

12.51 AB is a diameter of OO and AX and AY are chords of the circle such that AX = AY. Prove that
AB is the perpendicular bisector of XY. Hints: 210, 289
12.52 Find XZ in the right triangle described in Problem 12.42. Hints: 352

12.53. The areas of two adjacent squares are 256 square inches and 16 square inches,
respectively, and their bases lie on the same line. What is the number of inches in
the length of the segment that joins the centers of the two inscribed circles? (Source:
MATHCOUNTS) Hints: 55, 456
12.54 Given a circle of radius 2, there are infinitely many line segments of length

336
CHALLENGE PROBLEMS

2 that are tangent to the circle at their midpoints. Find the area of the region consisting of all such line
segments. (Source: AMC 12) Hints: 49

12.55 Let Cj; and C2 be two circles that intersect at two points, A and B. Let P be the point diametrically
opposite A on C;, and let Q be the point diametrically opposite A on C2. Prove that P, B, and Q are
collinear. Hints: 87
12.56 Chord CE isa segment of the perpendicular bisector of radius OA of ©O. AX is a diameter of
©O. Find ZOCX. Hints: 264

12.57. Three of the sides of ABCD are tangent to the circle shown at left below. Given that AB = 6,
BC =7,CD = 8, and AD = 9, find CY + DX. (Source: Mandelbrot) Hints: 173

Y C
B

A
Xx
D

Figure 12.11: Diagram for Problem 12.57 Figure 12.12: Diagram for Problem 12.58

12.58 A room is built in the shape of the region between two semicircles with the same center and
parallel diameters such that the smaller semicircle is contained within the larger. The room is shown at
right above. The farthest distance between two points with a clear line of sight is 12m. What is the area
in m? of the room? (Source: HMMT) Hints: 316

12.59 The incircle of AABC is tangent to AB at Z. Show that if the inradius of AABC equals AZ, then
the angle ZBAC is a right angle. Hints: 348
12.60 The sides of right triangle AABC all have integer lengths. Prove that the inradius of AABC also
has integer length. Hints: 449

12.61 Let w be the circumcircle of triangle ABC. Let A;, B,, and C, be the midpoints of arcs Ne: CA, and
AB, respectively, of circle w. In this problem we prove that the incenter of triangle ABC is the orthocenter
of triangle A;B,C}.

(a) Let I be the incenter of AABC. Why must Al pass through A;?
(b) Why must AA, be perpendicular to ByC)? Hints: 256
(c) Combine the previous parts to conclude that the incenter of triangle ABC is the orthocenter of
triangle A,B,C).

12.62x Prove that quadrilateral ABCD is cyclic if ZA + ZC = 180°. (Note that this is the converse of
what we discussed in Problem 12.19.) Hints: 331, 37

12.63x Weare given points A, B, C, and D in the plane such that AD = 13 while AB = BC = AC= CD =
10. Find ZADB. (Source: Mandelbrot) Hints: 82, 199

337
CHAPTER 12. CIRCLES AND ANGLES

12.64x Let PO bea diameter of a circle and T be a point on the circle besides P and Q. The tangent to
the circle through point Q intersects PT at R, and the tangent through T intersects QR at M. Prove that
M is the midpoint of QR. Hints: 222, 128

Extra! Most construction problems involve the two classical tools of geometry, the straightedge
imine and compass. These tools can solve an array of construction problems, many of which
are discussed in this book, but perhaps even more interesting are construction problems
that are impossible to solve with these tools.
Three famous classical construction problems that the ancient Greeks couldn’t solve
are listed below.

(a) Squaring the Circle: Given a circle, construct a square of equal area.

(b) Doubling the Cube: Given a cube, construct another cube with twice the volume
of the first cube.
(According to Greek legend, the Athenians appealed to the Oracle for advice about
a plague that was ravaging their city. The Oracle’s answer was to have their cube-
shaped altar to the god Apollo doubled in size. The Athenians then attempted
to follow this advice by building a new altar that had twice the side length of the
original altar, but this had the effect of increasing the volume of the altar by a factor
of 2° = 8, not 2, and as a result, the plague worsened.)

(c) Trisecting the Angle: Given an arbitrary angle, divide it into three equal angles.

We now know that the ancient Greeks couldn’t solve these problems because all three
of these constructions are impossible to perform with straightedge and compass.
Modern mathematicians proved these constructions are impossible by determining
_ what lengths could be constructed if we are given segments of lengths a, b, and 1. With
these segments as a starting point, the lengths a + b, a —b, ab, and a/b can be constructed.
See if you can figure out how to construct ab and a/b. Even Ja can be constructed, as the
diagram below suggests. (See if you can figure out how this construction works!)

This means that any of the following lengths can be constructed:

4g, SANE S/N Vs eee


Continued on the next page. ..

338
CHALLENGE PROBLEMS

Extra! . . . continued from the previous page


mp In other words, any length involving the integers and the arithmetic functions +, —,
x, +,and V canbe constructed. Furthermore, these are the only lengths that can be so
constructed. So how can this fact be used to solve the three problems?
The first problem, squaring the circle, is equivalent to constructing 7. However, it
was shown by Lindemann in 1882 that 7 is transcendental, meaning that 7 is not the root
of any polynomial with integer coefficients, which also implies the same for V7. Since
every constructible number is the root of some polynomial with integer coefficients, we
know that neither z nor /7 can be constructed.
The second problem, doubling the cube, is equivalent to constructing V2. A branch
of mathematics, known as Galois theory, tells us that V2 cannot be expressed using
arithmetic functions and square roots of integers. Hence, ¥2 cannot be constructed.
For the third problem, trisecting the angle, certain angles can be trisected. For ex-
ample, given a straight angle of 180°, a third of it would be 60°, which can be easily
constructed. But can a 60° angle itself be trisected? In other words, can a 20° angle be
constructed? It can be shown that if x = cos 20°, then 8x° — 6x — 1 = 0. Again, Galois
theory tells us that such an x cannot be expressed using arithmetic functions and square
roots. This counterexample of trisecting the 60° degree angle shows that trisection of
angles cannot be done in general.
Despite the fact that these problems have been conclusively solved for years, many
amateur mathematicians still attempt to find constructions for these problems and submit
them to mathematical journals, hoping for fame. All these attempts are completely futile.
The same Galois theory that rules out these constructions also tells us which regular
polygons can be constructed. The construction of an equilateral triangle and a regular
hexagon are easy, but beyond that, which other regular n-gons are constructible?
To answer this question, we introduce the Fermat numbers. The kt Rermat number
is Fy = 2a 4 1, so Fo = 3, Fy = 5, F2 = 17, and so on. A Fermat prime is a Fermat number
that is also prime. At the time of writing, there are only five known Fermat primes,
namely Fo, Fi, F2, F3, and Fy.
A regular n-gon is constructible if and only if n is a power of 2 or is of the form

A= Fy Fue F an,
where j is an arbitrary non-negative integer and Fyn,, Fin, ..., Fin, are distinct Fermat
primes. So the regular polygon with 3 - 17 = 51 sides is constructible, but the regular
polygons with 7 sides and 9 sides are not. Gauss first proved that the regular 17-gon is
constructible, and was said to have been so proud of this fact that he requested that a
regular 17-gon be etched into his tombstone.
: Itis surprising that either one of the straightedge or compass by itself can essentially do
the same work as both of them put together. The Mohr-Mascheroni Theorem states that
any construction that can be performed by straightedge and compass can be performed
by compass alone. If the target of a construction is a line, the compass alone cannot
produce the line; however, it can produce two points on the line, which defines the line
correctly. Analogously, the Poncelet-Steiner Theorem states that any construction that
can be performed by straightedge and compass can be performed by straightedge alone,
as long as a single circle and its center are given.

339
CHAPTER 13. POWER OF A POINT

Radical Axis Theorem

Power corrupts. Absolute power is kind of neat. — John Lehman, Secretary of the Navy, 1981-1987

CHAPTER =o re

| Ah ee of a Point

In the previous chapter we explored the angles formed by lines that meet inside, outside, or on a circle.
In this chapter, we explore the lengths of segments that meet inside or outside a circle. Together, the
relationships we will prove and use in this chapter are given the lofty name ‘Power of a Point.’

13.1 What is Power of a Point?

Problems >

Problem 13.1: Chords AB and CD meet at point X as shown. In this problem we will
prove that
(XA)(XB) = (XC)(XD).
(a) Draw AD and BC. Why is ZXDA = ZXBC? : B
(b) Show that the two triangles in the diagram are similar. Use these similar triangles to prove that
XA/XC = XD/XB.
(c) Complete the proof that (XA)(XB) = (XC)(XD).
(d) Is every chord of the circle that passes through X split into two pieces that have a product equal
to (XA)(XB)? (In other words, is there anything special about chord CD or will this proof work
for every chord through X?)

340
13.1. WHAT IS POWER OF A POINT?

Problem 13.2: Point P is outside a given circle and A is on the circle such that A
PA is tangent to the circle. C and E are on the circle such that secants PC and <5
PE intersect the circle again at B and D, respectively. In this problem we prove P —
that .
= (PB)(PC) = (PD)(PE). ease ig
(a) Draw AB and AC. Why does ZACP = BAP?
(b) Use the angle equality from the first part to identify two triangles that are similar.
(c) Use the triangle similarity to prove PA? = (PB)(PC).
(d) How can we show that (PD)(PE) = (PB)(PC)?

Problem 13.3: Chords TY and OP meet at point K such that TK = 2, KY = 16, and KP = 2(KO). Find
OP.

Problem 13.4: In the diagram, CB = 9, BA = 11, and CE = 18. Find DE.

Problem 13.5: Points R and M trisect PS, so PR = RM = MS. Point U is the


midpoint of PQ, TM = 2, and MQ = 8.
U a
(a) Find RM and MS. : oo 3
(b) Use the relationship you found in Problem 13.2 to find PU. (Don’t x NIM
forget that U is the midpoint of PQ.) I

We begin this section by proving the Power of a Point Theorem in two parts. First, we consider
points inside a circle.

Problem 13.1: Given chords AB and CD that meet at point X, prove that

(XA)(XB) = (XC)(XD).
Solution for Problem 13.1: When we rearrange what we want to prove, we have C
XA/XC = XD/XB. The ratios suggest we look for similar triangles, so we draw 4
AD and BC to form triangles. Since ZB and ZD are inscribed in the same arc, we
have ZB = ZD. We also have ZAXD = /CXB, so we have AAXD ~ ACXB by AA
Similarity. Therefore, we have XA/XC = XD/XB, so (XA)(XB) = (XC)(XD). D

Note that there’s nothing special about chords AB and CD. We can use the same B
exact approach as above to show that each of this circle’s chords through X is divided into two pieces
such that the product of the lengths of these pieces equals (XA)(XB). 0
Having finished with points inside the circle, we turn to points outside the circle.

341
CHAPTER 13. POWER OF A POINT

ee Solution for Problem 13.2: We start as we did in the previous problem, drawing
ae C AB and AC to make triangles. Since ZACB = AB/2 and ZPAB = AB/2, we
P “el have APAB ~ APCA by AA Similarity (since ZP is the same in both triangles
as well). Therefore, PA/PB = PC/PA, so PA? = (PB)(PC).
D
Wes E Just as there was nothing special about the chords we looked at in the
previous problem, there’s nothing special about secant PC here. By exactly
the same reasoning, we have (PD)(PE) = PA*, so (PD)(PE) = (PB)(PC). Notice that once again there’s
nothing special about the tangent and the secants we have chosen except that they have point P in
common. 0
We can put the facts we’ve proved in the last two problems together in a single statement.
aoe
Important: Suppose a line through a point P intersects a circle in two points, U |
Vv? and V. The Power of a Point Theorem states that for all such lines, the |
product (PU)(PV) is constant. We call this product the power of point P.
For example, in the figure at right, applying Power of a A C |
Point to X with respect to the circle shown gives |
|
(XA)(XB) = (XC)(XD). D |
B
A In the figure at left, the power of point P with |
of C respect to the circle gives us |
ie | |
ei, PA? = (PB)(PC) = (PD)(PE).
ig We think of Power of a Point whenever we have
a problem involving intersecting segments and a circle.
ua

Let’s try using Power of a Point on a few problems.

Problem 13.3: Chords TY and OP meet at point K such that TK = 2, KY = 16, and KP = 2(KO). Find
OP.

Solution for Problem 13.3: A quick sketch suggests how to apply Power of a Point. From R
the power of point K, we have R
(KP)(KO) = (KT)(KY). O
Substituting the given information in this equation yields

2(KO)(KO) = 2(16),
342
13.1. WHAT IS POWER OF A POINT?

from which we find KO = 4. Therefore, KP =2KO = 8,so OP =KO+ KP = 12. o


eS ie 28
‘Problem 13.4: In the diagram, CB = 9, BA = 11, and CE= 18. Find
A 5 i
‘DE.
D
|
E
|

Solution for Problem 13.4: We have two intersecting secants, so we apply Power of a Point, which gives

(CD)(CE)
= (CB)(CA).
Therefore, (CD)(18) = 9(20), so CD = 10 and DE = CE—CD=8. 0

WARNING! A very common mistake inapplying Power 4 B C |


~~ of a Point is to write (BC)(BA)= (DC)(DE) |
when faced with the figure at right .This al- D |
leged equality is not what Power of a Point |
tells us! Note that whenever we write a
Power of a Point relationship, the same point must appear in all of the|
segments in the equation, as point C does when we write the correct |
relationship for the secants in the figure above:

(CD)(CE)
= (CB)(CA).

fproblem 13. 5: Points R a M trisect PS:soPR ee MS. Point ae


is Q
the midpoint of PQ, TM = 2, and MQ = 8. Find PU.
U

P R S
le

Solution for Problem 13.5: Circles, chord lengths, and secant lengths. This is a job for Power of a Point.
The power of point M gives us
(MR)(MS) = (MT)(MQ).
We know that RM = MS, so substitution gives MR? = (2)(8), i.e., MR = 4. Therefore, PR = MR = 4 and
PS = 3(MR) = 12. Since U is the midpoint of PQ, we have PQ = 2PU. Now we can apply the power of
point P to find:
(PU)(PQ) = (PR)(PS).
Substitution gives (PU)(2PU) = 4(12), so PU = 2 v6. 0

Extra! Never say of a branch of mathematics, ‘There’s something I don’t need to know.’ It always comes
‘maw —ack to haunt you. —Charles Rickart

343
CHAPTER 13. POWER OF A POINT

Q ee
Le W Loi 4 a

R
(a) Find BC. (b) OS = 9 and TQ < TS; find TQ. (c) Find ZY.

13.1.2

[phe eeys

B Z
6 8

A ed E Ee

(a) Find DE. (b) Given XZ = 12, find WZ.

13.1.3 Chords XY and AB intersect at Q. Given AQ = 4, BQ = 6, and XQ = 8, find XY.


13.1.4 Chord UV bisects chord ST at point M. Given ST = 12 and UV = 15, find all possible values of
UM.

13.1.5 In this problem we use Power of a Point to prove the Pythagorean Theorem. GC
A cee Wises B

(a) Chord AB is perpendicular at point X to diameter CD of circle O.


Let the radius of the circle be c, the length of AB be 2a, and OX = b.
Find CX and XD in terms of b and c.
(b) Use Power of a Point to show that c? = a? + b?.
(c) How can you use this argument to prove the Pythagorean Theorem? D
Hints: 3, 41

344
13.2. POWER OF A POINT PROBLEMS

13.1.6 Inthe figure at right, we have WX = YZ. Prove that AW = AY. Hints:
62
W
A
Ne
Z

13.2. Power of a Point Problems

We now apply Power of a Point to some more challenging problems. As we'll see, Power of a Point can
be particularly useful for proofs involving segment lengths when circles are part of the problem.

Problems >

Problem 13.6: Given that XY = 6, YZ = 5, and that point X is 9 units from


the center, O, of the circle, we wish to find the area of the circle.

(a) Extend XO to meet the circle again at V.


(b) Let the radius of the circle be r; use the power of point X to find r,
and hence, the area of the circle.

Problem 13.7:
(a) Find BF.
(b) Find ED.

Problem 13.8: In the diagram, we have BP = 8, AB = 10, CD = 7, and


ZAPC = 60°. (Source: AHSME)
(a) Find CP.
(b) Show that AACP is a 30-60-90 triangle. Hints: 317
(c) Find the area of the circle.

Problem 13.9: In this problem we wish to show that XA = BZ in the diagram x


at right, given that DY bisects ZXYZ.
(a) Use Power ofa Pointtc find expressions for XA and BZ in terms of other VIS
| segments in the diagram. o
: () Use the Angle Bisector Theorem to prove that your expressions from the Y ee
first part are equal.
CHAPTER 13. POWER OF A POINT

Problem 13.10: Point O is the center of the circle, AB . BC, AP = AD, and
AB has length twice the radius of the circle. Prove that

AP? = (PB)(AB).

(Source: AHSME)

We start where we left off last section by finding lengths with Power of a Point in slightly more
complicated problems.

Problem 13.6: Z is on ©O andsecant


XZ hits
the circle at Y such that XY = 6and YZ = 5. Given that
|X is 9 units from the center of this circle, find the area of the circle.

Solution for Problem 13.6: We start with a diagram. We continue Ye SS Z


segment XO to hit the circle again at V, so that we can use the power X =r
of point X: (XW)(XV) = (XY)(XZ). If we let the radius of the circle be W
r, substitution gives (XO —1r)(XO +1) = 6(6 +5). Since XO = 9, we have us
(9 —r)(9 + r) = 66, from which we find 17 = 15. Therefore, the area of ae
the circle is mr? = 157. O

Concept: If a problem includes a segment that abruptly stops inthe middle of a | j

| circle, consider continuing the segment until it hits the circle. Then see >
what Power of a Point gives you. |

Problem 13.7:

(a) Find BF. A VS B


(b) Find ED. ————

Solution for Problem 13.7:


(a) We can find BF with Power of a Point: (FE)(FB) = (FA)(FD), so FB = (FAKED) (FE)= 2073:
(b) While Power of a Point won’t get us anywhere with ED, our proof of Power of a Point guides us
to the answer. Specifically, since ZFDE = ZFBA and ZAFB = ZEFD, we have AAFB ~ AEFD, so
ED/BA = EF/AF. Hence, ED = (EF)(BA)/AF = 18.

We used Power of a Point in combination with similar triangles in the previous problem. Now let’s

B06
13.2. POWER OF A POINT PROBLEMS

try using it with some of our other geometric tools.

Problem 13.8: In the diagram, we have BP = 8, AB = 10, CD = 7, and A


|ZAPC = 60°. Find the area of the circle. (Source: AHSME)
|
| B

Solution for Problem 13.8: It’s not immediately obvious how we will find the radius, A
so we start by finding what we can. The power of a point P gives us
B
(PC)(PD) = (PB)(PA), “
so (PC)(PC + 7) = 144. Therefore, PC? + 7PC — 144 = 0, so (PC + 16)(PC — 9) = 0. PC = ie
must be positive, so PC = 9.

Seeing that ZAPC = 60° makes us wonder if there are any equilateral or 30-60-90 triangles lurking
about. Since CP = AP/2 and the angle between these sides is 60°, the sides adjacent to the 60° angle in
AACP are in the same ratio as the sides adjacent to the 60° angle in a 30-60-90 triangle. Therefore, AAPC
is similar to a 30-60-90 triangle by SAS Similarity, so AAPC must be a 30-60-90 triangle with right angle
at ZACP!
Since ZACD is right and inscribed in AD, we know AD is a semicircle. Therefore, AD is a diameter
of the circle. Since AC = CP V3 = 9 V3 from our 30-60-90 triangle, we have AD = VAC2 + CD? =
V¥243 + 49 = 2 V73. Finally, the radius of the circle is AD/2 = 73, so the area is (V73)*1 =7 OT E)

Concept: If you see a 60° or 30° angle (or evena 120° angle) ina problem, you should |
be on the lookout for 30-60-90 triangles.

Now we will use Power of a Point in a couple proofs.

Problem 13.9: Given that YD bisects XYZ in the diagram at right, prove xX
that XA = ZB. iS D

Solution for Problem 13.9: This problem involves lengths, an angle bisector, chords, secants, and circles.
Therefore, we should consider the Angle Bisector Theorem and Power of a Point. The Angle Bisector
Theorem applied to angle bisector YD of AXYZ gives us
KD ep
Rey
347
CHAPTER 13. POWER OF A POINT

However, we need something with XA and ZB. Therefore, we turn to the powers of points X and Z,
which respectively give us
(XA)(XY) (XD)(XZ)
(ZD)(ZX) (ZB)(ZY).
Seeing some common terms in these two, along with terms from our Angle Bisector Theorem equation,
we know we're on the right track. We solve these two equations for XA and ZB, and we have
_ (XD)(XZ)
Siti XY
fo LZ XA)
7 aac ay

We need only show these two expressions are equal. We have XZ in common, and we already have
XD/XY = ZD/ZY from our Angle Bisector Theorem. We can put these together to show
_ (XD)(XZ) ey a (22h (ZD)(XZ)
= —— os ease
se XOE ae (XY FENG Zi

HProblem 13 10: Point O isthe center of the circle, AB 1 BC, AP = AD, and
AB has length twice the radius of the circle. Prove that AP? = (PB)(AB).
(Source: AHSME)

Solution for Problem 13.10: We have a lot of information here. What we want to prove looks a lot like
Power of a Point when we have a tangent. AB sure looks tangent, but we have to prove it. Fortunately,
since diameter BC is perpendicular to AB at point B on the circle, AB must be tangent to the circle.
Therefore, AB* = (AD)(AE).

This problem has a lot of different segments. We could get lost in a blizzard of AP’s and AD’s and
DE’s and so on. So, we have to stay organized. One way to do this is making a list of what we know
and what we want to show. Letting the radius of the circle be r, such a table might look like this:

What We Know What We Want


AP=AD AP? = (PB)(AB)
AB? = (AD)(AE)
AD = 20

This has the advantage of letting us work in two directions. We can work forwards from what we
know to try to reach what we want, or we can work backwards from what we want. Let’s try a little of
both here. First, going forwards, we note that BC = DE = 2r because they are diameters of the circle.
Combining this with AB = 2r gives AB = BC = DE.
Going backwards, we write our desired relationship in terms of fewer lengths. Specifically, since
PB = AB - AP, we can write AP? = (PB)(AB) in terms of just AB and AP. We can also note that AB = 2r,
and write the equation we want in terms of rand AP. Now our table looks like:

aia
ER TILE
13.2. POWER OF A POINT PROBLEMS

| What We Know What We Want


AP.=AD AP? = (PB)(AB)
AB* = (AD)(AE) AP? = (AB — AP)(AB)
AB =.27= DE = BC AP? = (2r — AP)(2r)

_Going forwards with our new information, we don’t see anything we can do with BC, but DE is part
of AE, so we have AB? = (AD)(AE) = (AD)(AD + DE) = (AD)(AD + AB). All the stuff on our ‘What We
Want’ side has AP’s in it, so we use AD = AP to write

AB? = (AD)(AD + AB) = (AP)(AP + AB) = AP* + (AP)(AB).


Now our table looks like this:

What We Know What We Want |


AP = AD AP? = (PB)(AB)
AB? = (AD)(AE) AP? = (AB — AP)(AB)
Abe 2? =DE= BC AP? = (2r — AP)(2r)
AB* = AP? + (AP)(AB)

Now we have an equation in our ‘What We Know’ that is very close to an equation in our ‘What We
Want.’ A little rearranging turns one into the other. Subtracting (AP)(AB) from both sides of

AB* = AP? + (AP)(AB)


gives AB? — (AP)(AB) = AP?. Factoring then gives

(AB — AP)(AB) = AP”,


which is the second equation in our ‘What We Want’ column.
Ah-ha! We have a path from something We Know to something We Want! However, we can’t just
say ‘Done!’ We have to write a nice solution. We do so by retracing our steps. We have to examine our
work above to figure out three things:

(a) How we get from the given information to the equation AB* = AP? + (AP)(AB).
(b) How we get from this equation to the one in our list of ‘What We Want’: AP? = (AB — AP)(AB).

(c) How we get from AP? = (AB — AP)(AB) to what we want to prove: AP? = (PB)(AB).

After reviewing our work above, we write our solution:


Since diameter BC is perpendicular to AB at point B on the circle, AB must be tangent to the circle.
Therefore, the power of point A gives us AB* = (AD)(AE). We are given AB equals twice the radius of
the circle of which DE is a diameter, so AB = DE. Since we are also given AD = AP, we have

AB? = (AD)(AE) = (AD)(AD + DE) = (AP)(AP + DE) = (AP)(AP + AB).


349
CHAPTER 13. POWER OF A POINT

Expanding and rearranging AB? = (AP)(AP + AB) gives AB2 — (AP)(AB) = AP?. Factoring yields
(AB — AP)(AB) = AP?. Since AB — AP = PB, we have the desired (PB)(AB) = AP”.
Notice that we have some items on our ‘What We Know’ and ‘What We Want’ lists that we don’t
use. This will usually be the case for challenging geometry proofs. O

Important: When writing a solution, you should write a clean solution like what we —
Vv have after ‘After reviewing our work above, we write our solution.’ All
the work we did before that point was just our investigation to find the
solution. This often involves working both forwards and backwards, as
we saw. However, once we find our way, we write a nice clean solution
1

forwards, starting from what we are given and ending at what we want |
to prove.

Concept: Organize your work on challenging geometry proofs. Keep track of what —
()==3 ‘you know and what you can derive from what you know. Also keep track |
of what you need to prove, and work backwards from there to list more —
statements that, if proven, would mean you are finished. When you know |
how to get from a statement on your ‘What We Know’ list to a statement |
on your ‘What We Want’ list, then you can construct your proof. |

13.2.1 In the figure at left below, AB is tangent to ©O. Given that AB = 4 and BD = 12, find the area of
the circle.
Q

Figure 13.1: Diagram for Problem 13.2.1 Figure 13.2: Diagram for Problem 13.2.2
13.2.2 Find the following in the figure at right above:

(a) [PQR].
(b) ST. Hints: 69
(c) The area of the circle.
(d) [QTR]. Hints: 594

350
2 SR He a <a = ames =o. Seat evra Lin Ri ec TS ESE
13.3. SUMMARY
AP a SRR ITA SANE

13.2.3 As shown in the diagram at left below, XA is tangent to the circle at A and a secant through X
meets the circle at B and C, with XB < XC. Prove that XB < XA < XC. Hints: 91

a (é

Figure 13.3: Diagram for Problem 13.2.3 Figure 13.4: Diagram for Problem 13.2.4

13.2.4 A circle is tangent to side BC of equilateral triangle AABC at point Q as shown at right above.
The circle intersects sides AB and AC in two points each, as shown. Given that AW = AY, prove that Q
is the midpoint of BC. Hints: 103

13.3 Summary

“Important: Suppose a line through a point P intersects acircleintwo points,U


Vv and V. The Power of a Point Theorem states that for all such lines, the —
product (PU)(PV) is constant. We call this the power of point P. |
For example, in the figure at right, applying Power of a C|
é : A
Point to X with respect to the circle shown gives |

(XA)(XB) = (XC)(XD). D
B
A In the figure at left, the power of point P with |
2 C respect to the circle gives us |
it
PA? = (PB)(PC)
= (PD)(PE).
{

E We think of these whenever we have a problem i|


||
involving intersecting segments and a circle. 4

ESSE ttt Bis satan SSS geeee el

as pene eet Sereen Oe a 5


|
WW i luti
wards, when we write our solution, we write a nice clean solution for | ite a nice clean solution for-_
wards, starting from what we are given and ending at what we want to |
|

prove. |
]

351
CHAPTER 13. POWER OF A POINT

Problem Solving Strategies

Concepts: ae in the middle of


problem includes a1 segment that abruptly stops i
O== a circle, consider continuing the segment until it hits the circle. Then
see what Power of a Point gives you.
If you see a 60° or 30° angle (or even a 120° angle) in a problem, you
| should be on the lookout for 30-60-90 triangles. |

Organize your work on challenging geometry proofs. Keep track |


of what you know and what you can derive from what you know. |
Also keep track of what you need to prove, and work backwards |
from there to list more statements that, if proven, would mean you —
are finished. When you know how to get from a statement on your —
‘What We Know’ list to a statement on your ‘What We Want’ list,
then yous construct your a

Things To Watch Out For!

WARN!ING! Avvery ccommon mistake in applying Power 4 B


of a Point to a point outside a circle as in
the figure at right is to write (BC)(BA) =
(DC)(DE). This alleged equality is not what
Power of a Point tells us! Note that when-
ever we write a Power of a Point relationship, the same point must
appear in all of the segments in the equation, as point C does when
we write the correct relationship for the secants in the figure above: |
|\
(CD)(CE) = (CB)(CA).

Extra! The set of all points that have the same power with respect to
‘> im whi §~twoO given circles is a line called the radical axis of the two circles.
One particularly important use of radical axes is the Radical Axis
Theorem, which states that given three circles, the three radical axes
of the three pairs of circles are concurrent. The diagram at right
is an example of the Radical Axis Theorem for three intersecting
circles. Do you see why the radical axis of two intersecting circles
must include the chord connecting the points where the circles meet?
As an extra challenge, try proving first that the set of all points that have the same
power with respect to two given circles is a line, then use this to prove the Radical Axis
Theorem. (The first part is tougher than the second!)

352
REVIEW PROBLEMS

|
| REVIEW PROBLEMS >

icyel

B
Q R

KA ae
P
'e

D S

(a) Find EC. (b) Find PR.

13.12

S Ae

"he

(a) Find VZ (b) Find BC and CD. (c) Find PS.

13.13 Chords GH and IJ of ©O are perpendicular at M. Given GM = 2MH = 6IM = 12, find GJ. Hints:
76

13.14 Lines mandn meet at P. OO meets m at points A and B, and meets n at C and D, such that PA = 3,
AB = 9, PC = 4, and CD = 5. Why must P be outside OO?

13.15 Earth is roughly 8000 miles in diameter.

(a) I’m riding ina hot air balloon 1 mile above the surface of Earth. Approximately how far away is
the horizon? (In other words, how far away is the farthest point on the surface of Earth that I can
see.)
(b) What if I’m ina plane 6 miles above the surface of the Earth?
(c) What if I’m ina spaceship 100 miles above the surface of Earth?

353
CHAPTER 13. POWER OF A POINT

13.16 Given ZY < ZW in the diagram at left below, show that ZV < ZX.

tp
" V Ch
Figure 13.5: Diagram for Problem 13.16 Figure 13.6: Diagram for Problem 13.17

13.17 The figure at right above shows two circles that intersect at A and B. P is on AB, and PQ and PR
are tangents as shown. Prove that PQ = PR.
13.18 Is it possible for chords AB and CD of a circle to intersect at X such that X is the midpoint of AB,
but is closer to C than to D?
13.19 Is it possible for two chords AB and CD of a circle to meet at a point X such that AX = BX,
CX= 2D ena Ab = Gio4

B 13.20 Find the ratio AC/AD in the figure at left.


C 13.21 Point A is on ©O such that PA is tangent to circle ©O. Point B is on circle
©O such that PB = PA. Must PB be tangent to ©O?
A 13.22 Let P be a point in the same plane as a circle centered at O with radius r.
# F Asecant through P intersects the circle at points A and B. Use PG to prove that
if P lies outside the circle, then PA - PB = PO? — r*. What is the formula if P lies inside the circle?

13.23 Use the diagram in the figure at right to find a proof of the Pythagorean A
Theorem. aa

Challenge Problems

13.24 IncircleO, PO 1 OB and PO equals the length of the diameter of OO. Compute
PA/AB. (Source: ARML) Hints: 294, 323 P
J i.

13.25 Chords PQ and RS of a circle meet at point X. Given that PQ = RS, show that ee
PX = RX or QX = RX (or both). Hints: 364 B
13.26 Jake is working on a problem in which chords AB and CD, when extended past B and D,
respectively, meet at P. He is given PB, AB, and CD. He mistakenly uses Power of a Point improperly
by using (PB)(BA) = (PD)(DC). However, he still gets the right answer for PD. Prove that the question
Jake was answering must have had AB = CD. Hints: 385
13.27. A and B are two points on a circle with center O, and C lies outside the circle, on ray AB. Given

354
CHALLENGE PROBLEMS

that AB = 24, BC = 28, and OA = 15, find OC. Hints: 122

13.28 Circles C; and C2 have the same center, O. The radius of C) is r; and the radius of C2 is r2, where
1G) el hek

(a) Prove that it is impossible for a point P to have the same power with respect to both circles if P is
outside both circles. Hints: 104
(b) Prove that it is impossible for a point P to have the same power with respect to both circles if P is
inside both circles.
(c) Show that it is possible for a point inside C; but outside C2 to have the same power with respect
to both circles, and find all such points that have equal power with respect to both circles.

13.29 WX and YZ meet at P such that (WP)(PX) = (YP)(PZ). Prove that the circumcircle of AXYZ goes
through point W. Hints: 512

13.30 Two circles C; and C> intersect at two points, A and B. Let PQ be a chord of C; and RS a chord
of C2 such that they intersect on AB. Prove that points P, Q, R, and S all lie ona circle. Hints: 540, 397

13.31 Given two segments with lengths a and b, construct a segment with length Vab. Hints: 415

13.32 Equilateral triangle ABC is inscribed in a circle. Let P be a point on arc BC, A
and let AP intersect BC at Q. Prove that

1 1 ‘l
PO mabe serch
BSS eee
Hints: 382, 349 ene?
13.33 In the diagram at right, AB = 8, AP = 2, and PC = 4. (Source: ARML) B
A
(a) Prove that [ABE]/[CDE] = (AB/CD)?. P
(b) Prove that [PBC]/[PAD] = (PC/PA)*. c
(c)x Find [PAEC]/[BAE]. Hints: 309, 346
D
13.34x An equilateral triangle is inscribed iin a circle. Points D and E are midpoints of AB and BC,
respectively, and F is the point where DE meets the circle. Find DE/EF. (Source: ARML) Hints: 300, 252

13.35x Segment IF is tangent to the circle at point I as shown in the diagram at I <F
right. We are given that IF = 21 V2, IO = 20, ON = 12; RF = 18, and OR > GO. Find
OR. (Source: Mandelbrot) Hints: 188

13.36x Chords AB and CD of ©O are perpendicular at point P. Given CP = 2, G </


AP= 3, and PD= 6, find the following:

(a) OP. Hints: 31,90


(b) The radius of OO. Hints: 387

355
CHAPTER 14. THREE-D IMENSIONAL GEOMETRY
O
a a cc te c

Poncelet’s Porism
~

Man's mind, stretched by a new idea, never goes back to its original dimensions. — Oliver Wendell Holmes

CHAPTER a an subline ns

Three-Dimensional Geometry

So far in this book we’ve confined ourselves to zero dimensions (points), one dimension (lines and line
segments), or two dimensions (basically everything else in the book). However, we all know the world
around us seems three-dimensional — that in addition to left and right, and forwards and backwards,
there’s up and down.
In this chapter we step off the page and into the ‘real world’ by considering three-dimensional objects.
As we'll see, this isn’t much of a step; much of our work in three-dimensional geometry will consist of
reducing problems to two-dimensional situations we already know how to handle.

14.1 Planes

As mentioned in Section 1.2, a plane is a ‘flat’ two-dimensional surface that extends forever. For example,
when your book is completely flat, this page is part of a plane. In this section, we'll explore planes, as
well as how lines and planes can intersect in space.

356
14.1. PLANES

|Problems il
Problem 14.1: We know that given any two points, we can draw exactly one line through the two
points.
(a) Given two points, how many different planes pass through the two points?
(b) Given three points, is it always possible to find a plane that passes through all three points?
(c) Given any four points, is it always possible to find a plane that passes through all four points?

Problem 14.2:
(a) When two lines intersect, their intersection is a point. When two planes intersect, what kind of
figure is their intersection?
(b) Is it possible for two planes to never intersect?
(c) Can there be a line and a plane that never intersect?
(d) Remember that two lines are parallel if they are in the same plane but do not intersect. Can two
lines in space be such that they are not parallel but still do not intersect?
Given any two intersecting lines, is there always a plane that contains both lines?

Problem 14.3:
(a) How can we tell if a line is perpendicular to a plane? Give an example in the ‘real world’ of a
line and a plane that are perpendicular.
(b) How can we tell if two planes are perpendicular? Give an example in the ‘real world’ of two
planes that are perpendicular.

We start with a fundamental question about eae

Solution for Problem 14.1: If we only have two points, we can find a line through Stes
them, but there are infinitely many planes that can pass through this line (and Se
therefore pass through our initial two points). An example is shown in the figure Sci OO
at right, in which three planes are shown passing through the same line.

Given three or more collinear points (meaning they are all on the same
line), we can still find infinitely many planes that pass through all three
points, but if we have three points that are not collinear, then we have a
triangle. The plane of this triangle is the unique plane that passes through
the three points, as shown at left.

Sidenote: Why do most telescopes have three legs? Why do photographers u: use a
_ tripod, which has three legs, to stabilize their cameras instead of something |
_~ with four legs? The answer: three points determine a plane, so wherever
a you place a stand with three legs, it will be stable.

357
CHAPTER 14. THREE-DIMENSIONAL GEOMETRY

While any three points are coplanar, meaning there is a plane through all three D
of them, it is easy to find a set of four points that are noncoplanar. Three vertices B
of a triangle and a point not in the plane of the triangle give us an example in
which no plane passes through four given points. Such a situation is depicted at C
right, where BC is dashed because it is in the ‘back’ of the figure. (We wouldn’t be
able to see BC if this figure were rendered in three dimensions.) 0 A

- eae A _—- — —— ey
E|
Important: Three noncollinear points determine a plane.
be
fF
||
Now we consider how lines and planes in space can be related.
A a SS = ~ —s = = —

Problem 14.2:
_ (a) When two lines intersect, their intersection is a point. When two planes intersect, what kind of
| figure is their intersection?
(b) Isit possible for two planes to never intersect?
(c) Can there be a line and a plane that never intersect?
(d) Remember that two lines are parallel if they are in the same plane but do not intersect. Can two
lines in space be such that they are not parallel but still do not intersect?
Given any two intersecting lines, is there always a plane that contains both lines?

Solution for Problem 14.2:


(a) Consider a standard rectangular room as shown at right, where EFGH is
the ‘floor’ and ABCD is the ‘ceiling’. The walls are portions of planes.
Look at where a wall meets the floor, and you'll see an example of two
planes intersecting to form a line. Similarly, any two planes that intersect
form a line.

(b) Ina typical room, the ceiling is a constant distance from the floor. This means that if the ceiling
and the floor extended forever in all directions, they would be planes that never intersect. Such
planes are called parallel, just as lines in the same plane that never intersect are called parallel.
For example, in our figure above, ABCD and EFGH are parallel planes.

(c) The intersection of a wall and the ceiling (line AB, for example) will never meet the floor (plane
EFGH). This is an example of a line that is parallel to a plane.
(d) Lines AB and CG do not intersect and are not in the same plane. Since they are not coplanar, they
are not parallel. But they still don’t intersect! We call noncoplanar lines that do not intersect skew
lines.
(e) Yes. Let X be the intersection point of the two lines, Y be a different point on one of the lines, and
Z a different point on the other line. These three points determine a plane. Clearly, both XY and
are in the plane because XY and XZ are.

358
aaaeld 14.1.delenit
PLANES

After dealing with parallels, you know what's next.

Problem 14.3:
(a) How can we tell if a line is perpendicular to a plane? Give an example in the ‘real world’ of a
line and a plane that are perpendicular.
(b) How can we tell if two planes are perpendicular? Give an example in the ‘real world’ of two
planes that are perpendicular.

Solution for Problem 14.3: We'll use our trusty room for our ‘real world’ examples.

(a) Suppose line k intersects plane P at point X. If k is perpendicular to every


line in P that passes through X, then we say that line k is perpendicular
to the plane. The line of intersection between two adjacent walls in a
typical room is perpendicular to the ceiling and the floor of the room. For
example, AE is perpendicular to ABCD.
(b) Suppose planes P and Q intersect in line m, and let O be
a point on line m. Consider lines j and k in P and Q,
respectively, such that each of j and k is perpendicular to
m at point O. The angle between j and k, which is marked
with a 0 in the diagram, is the angle between the planes.
We call such an angle between planes a dihedral angle. If Q
this angle is a right angle, the planes are perpendicular. We
In a typical room, a wall is perpendicular to the ceiling k
and to the floor. For example, plane ABCD is perpendicular
to plane BCGF in our ‘room’ above.

TL +
14.1.1 Line m is perpendicular to plane FP at point X. Line k is in plane P and passes through X. Are
lines m and k necessarily perpendicular?

14.1.2 Triangle PAB and square ABCD are in perpendicular planes at right.
Given that PA = 3, PB = 4, and AB = 5, what is PD? (Source: AMC 12)

14.1.3 Is it true that the nearest point on line m in space to a point P in


space is the foot of the perpendicular segment from P to m? Why or why
not?
14.1.4 Is it true that the nearest point on plane P in space to a point X in
space is the foot of the perpendicular segment from X to plane P? Why or why not?
14.1.5 Two planes, Mand WN are each perpendicular to a third plane, P. M and WN intersect in line m.
Is line m perpendicular to P? (No proof necessary.)

359
CHAPTER 14. THREE-DIMENSIONAL GEOMETRY

14.2 Prisms

We move now from considering one- and two-dimensional figures in space to thinking about solid
figures. Our ‘room’ was so useful in the previous section that we'll start with it here.

Mathematically speaking, ABCDEFGH at the right is a right rectangular A


prism. Since all the boundaries of ABCDEFGH are polygons, ABCDEFGH can
also be called a polyhedron. Each of these boundary polygons is a face of the E
polyhedron. The ‘rectangular’ part of ‘right rectangular prism’ refers to the fact
that the top and bottom faces (ABCD and EFGH) are rectangles. We typically a
call the ‘top’ and ‘bottom’ the bases of the prism. The ‘right’ part refers to the fact that all the edges
connecting the two bases meet the bases at right angles.
Which leaves us with the ‘prism’ part. A prism is a three-dimensional solid figure with two congruent
parallel faces, and with parallelograms as the other faces. The faces that are not bases are sometimes
called the sides of the prism. In a right prism all of these sides are rectangles. As already noted, the
congruent parallel faces are the bases of the prism. Hence, any pair of opposite sides of ABCDEFGH
above could be considered the bases of the prism.

Te

Figure 14.1: Some Prisms

Figure 14.1 shows some more prisms. The first is a right regular pentagonal prism because the base
is a regular pentagon and the segments connecting the bases are perpendicular to the bases. The second
is a hexagonal prism because the bases are hexagons (note that we don’t call this one a ‘right hexagonal
prism’). The third has parallelograms as its bases, as well as all its sides. Such a prism is given the
special name parallelepiped. As one last bit of vocabulary, if there’s a ‘regular’ in the description of a
prism, it means the base is a regular polygon.
Make sure you understand what a prism is, but you don’t have to worry about all those other names.
One reason you don’t have to worry too much is that they aren’t used consistently. For example, the
‘right’ is often left out of the description, and you're meant to infer from the problem that the prism
is indeed a right prism. Sometimes in this book we’ll do this, such as by calling our original prism
ABCDEFGH a ‘rectangular prism’.
Unfortunately, the vocabulary lesson isn’t over yet. Just as we measure the region contained in a
two-dimensional figure with area, we can measure the space inside a three-dimensional figure with
volume. We do so in much the same way as we measure area. Instead of measuring how many 1 x 1
squares fit, we measure how many 1 x 1 x 1 ‘blocks’ (or portions of blocks) fit inside the solid.

B60)
a ee ee
14.2. PRISMS

Finally, we can still use area to measure three-dimensional figures. The total surface area of a figure
is the area of all of the surfaces that form the borders of the solid. The lateral surface area is the total
area of all the faces that are not considered ‘bases.’ We often just ignore lateral surface area and say
‘surface area,’ which means the same as ‘total surface area.’

I)
Probiems il
Problem 14.4: ABCDEFGH shown is a rectangular prism with AB = 3,BC=4, A
and AE = 5.
(a) Find BF and EH.
Find the volume of ABCDEFGH.
Find the total surface area of ABCDEFGH.
Find AC.
What kind of triangle is AACG?
Find AG.

Problem 14.5: A cube is a special rectangular prism in which all edges have the
same length (i.e., its base is a square and its height has the same length as a side of
the base).
(a) Find the formula for the volume of a cube with side length s.
(b) Find the formula for the total surface area of a cube with side length s.
(c) Finda formula for the length of SY in the diagram given that the figure is a cube with side length
S.

Problem 14.6: Once again, we consider cube STUVWXYZ.

(a) What geometric shape is SXYV?


(b) What is the area of SXYV if SV = 4?

Problem 14.7: The right regular hexagonal prism shown has sides of length
4 on base ABCDEF and a height of 8 units.
(a) Find the lateral surface area of the prism. = A D’

(b) Find the total surface area of the prism.


(c) Find the volume of the prism.
(d) Find AB’, AC’, and AD’ by building the appropriate right triangles.

361
CHAPTER 14. THREE-DIMENSIONAL GEOMETRY

We start off by finding volumes, areas, and lengths of a (right) rectangular prism.

‘Problem 14.4: ABCDEFGH show nisa rectangular prism with AB =3,BC=4, A


and AE = 5.

(a) Find BF and EH.


(b) Find the volume of ABCDEFGH.

(c) Find the total surface area of ABCDEFGH.


d) Find AC.
(e) What kind of triangle is AACG?
(f) Find AG.

Solution for Problem 14.4:

(a) All of the faces are rectangles. Since AEFB is a rectangle, BF = AE = 5. Similarly, rectangle EFGH
tells us that EH = FG, and rectangle BFGC gives us FG = BC = 4. Therefore, EH = 4. We could
also have seen that EH = BC by noting that both represent the distance between parallel faces
AEFB and DHGC.
(b) We approach this much as we did finding the area of a rectangle with 1 x 1
squares on page 85. We can organize a layer of (AB)(BC) = 121x1x1 cubes
on face ABCD. This plus four more such layers completely fills our prism,
so the volume is 3 x 4 x 5 = 60.
pe
(c)
See
To find the total surface area, we need to add the areas of all the faces. Each
face is a rectangle, but rather than finding the areas of six different rectangles, a! ; 5 “i I\

we can note that the rectangles come in pairs of congruent rectangles. For PSTN
example, ABCD = EFGH. Therefore, our total surface area is: PIN IRSIS
2([ABCD] + [BCGE] + [ABFE]) = 2[(3)(4) + (4)(5) + (3)(5)] = 94.
(d) AC is a diagonal of rectangle ABCD (or hypotenuse of triangle ABC), so
it has length V3* + 42 = 5. AC is sometimes called a face diagonal of the
prism.
(e) Since CG is perpendicular to plane ABCD, it is perpendicular to AC. There-
fore, AACG is a right triangle.
(f) From right AACG, we have

AG = VGC2 + AC2 = V¥25


+ 25 =5 v2.
Note that VAB2 + BC2 + GC2 = 5 V2 = AG. Is this a coincidence?

Segment AG is called a space diagonal of the prism. The lengths AB, AD, and AE are sometimes
referred to as the prism’s dimensions. We might use these to describe the prism as a3 X45 rectangular
prism. We can follow the exact same logic we followed in Problem 14.4 to find formulas for the volume,
total surface area, and the length of a space diagonal of a rectangular prism.

362
14.2. PRISMS

Important: _ The three dimensions of a rectangular prism are com-


WW, monly called the length, /, the width, w, and the height,
h. For such a prism, we have:

Volume = lwh
Surface area = 2(lw+wh+ lh)
Space Diagonal = Ve +ee + h? |
L z ES ait EU DSS PS SEA EN

Notice that our volume is simply the area of a base of the prism times the height of the prism. This
is true for any prism.
.
_ Important: The volume of a “prism equals the area of a base times the distance
| VV between the bases (i.e. the height).
|
|
Notice that by ‘distance between the bases’, we do not necessarily mean the
length of the edges connecting corresponding points on the bases. If the prism
is not a right prism, then the height is the length of a segment from one base to
the other that is perpendicular to both bases. For example, the height between
bases ABCD and EFGH of the prism at left is XY, not FB.

Problem 14.5: A cube is a special rectangular prism that has all its dimensions the same (ie., its base
'is a square and its height has the same length as a side of the base).
(a) Find the formula for the volume of a cube with side length s.
(b) Find the formula for the total surface area of a cube with side length s.
(c) Find a formula for the length of SY in the diagram given that the figure is a
cube with side length s.

Solution for Problem 14.5:


(a) Since a cube is a rectangular prism, its volume is the product of all three dimensions. Since all
three dimensions are the same, s, the volume is simply s°.
(b) The faces of a cube are all squares. Therefore, each of the 6 faces has area s?, so the total surface
area is 6s”.
(c) Once again, we can simply use the formulas we already found for a rectan-
gular prism. But to be a little more sporting, let’s prove the formula directly
for a cube. To find SY, we'd like to create a right triangle that has SY as a
side, so we can use the Pythagorean Theorem. Hence, we draw SY and SU,
creating right triangle ASUY. We have UY = s, so all we have to do is find
SU. SU is the hypotenuse of ASTU, so SU = s V2. Therefore,

= VSU2 + UY2 = V2s2 + 52 = 8 V3.

363
CHAPTER 14. THREE-DIMENSIONAL GEOMETRY

“Important: A cube with side length s has:


V Volume = 5°
|
| Surface area = 687
| Space Diagonal = s V3 |

[Problem 14 6 Once again, we consider cube STUVWXYZ. What is the areaof SXYV W
if SV = 4?

Solution for Problem 14.6: We start by drawing SXYV so we can figure out what sort W Z
of shape it is. Since SV and XY are perpendicular to faces STXW and VUYZ, they ann
are both perpendicular to both SX and YV. So, SXYV is a rectangle. We already
have one side of the rectangle, SV = 4. Therefore, we only need to find SX. Since
SX is a diagonal of square SWXT, SX = 4 V2, so [SXYV] = (SV)(SX) = 16 V2. O s
One way to view rectangle SXYV of the previous problem is as the intersection
of a plane and the cube, where the plane passes through vertices S, X, Y, and V r
of the cube. We call such an intersection of a plane and a solid a cross-section of the solid. Many
three-dimensional geometry problems are solved by choosing the right cross-section of the problem to
consider. Sometimes we even have to consider multiple cross-sections!
Lest you start to think that all prisms are rectangular ones, we'll try a problem involving a regular
hexagonal prism.

_a height of 8 units.
(a) Find the lateral surface area of the prism.
(b) Find the total surface area of the prism.
(c) Find the volume of the prism.
(d) Find AB’, AC’, and AD’.

Solution for Problem 14.7:


(a) Each lateral face of the prism is a 4 x 8 rectangle. There are 6 lateral faces, so the lateral surface
area is 6(4 x 8) = 192.
(b) To find the total surface area, we must add the area of the two bases to the lateral surface area.
Each base is a regular hexagon. As we saw back in Problem 9.7, we can find the area of a regular

364
14.2. PRISMS

hexagon of side length 4 by breaking it into six equilateral triangles with side length 4. Therefore,
the area of each base is
2
6 (47) = 24 3.

So, the total surface area of the prism is 192 + 2(24 V3) = 192 + 48 V3.
(c) We've already done the hard part! The volume is just the area of a base times the height, or
8(24 V3) = 192 v3.
(d) Since AB’ isa diagonal of rectangle ABB’A’, AB’ = V4? + 82 = 4 V5.
The other two are a little trickier. For AC’, we consider the cross-section
ACC’A’ and notice that AC’ is a diagonal of rectangle ACC’A’. We already
know CC’ = 8, so we only have to find AC. Since AACD is a 30-60-90 triangle
(make sure you see why), we have AC = CD V3 = 4 V3. Hence,

AC’ = VAC2 + CC? = V48 + 64 =4-7.


Similarly, AD’ is a diagonal of rectangle ADD’A’. To find AD, we recall the
dissection of a regular hexagon by its long diagonals into 6 equilateral triangles
as shown, so AD = 2(BC) = 8. Therefore,

AD’ = VAD2 + D’D2 = V64+ 64 =8 v2.

14.2.1 Describe as fully as possible a cross-section of a prism that is parallel to the bases of the prism.

14.2.2. A right rectangular prism has dimensions 2, 5, and 3 Ay2s

(a) Find the volume of the prism.


(b) Find the total surface area of the prism.
(c) Find the length of a space diagonal of the prism.
(d) Find the length of the longest face diagonals.
14.2.3 Find the volume of a rectangular prism that has a space diagonal of length 10 and two sides of
length 3 and 8.
14.2.4 Find the volume of a cube given that its space diagonal has length 6.
14.2.5 How many different space diagonals does a cube have?
14.2.6 Ifall the dimensions of a right rectangular prism are doubled, what happens to the surface area?
What happens to the volume?

aero
== = © 65
CHAPTER 14. THREE-DIMENSIONAL GEOMETRY

14.2.7 Shown is right rectangular prism ABCDEFGH. Given that AB = 4,BC =3,and A
ABGH is a square, find the following:

(a) BL: E

(b) AG.
(Cee:
(d) the volume of ABCDEFGH.

14.2.8 Bridget places a box that is 4 x 6 x 8 on the floor. She then places a 2 x 3 x 5 box on top of the first
box, forming a two-box tower. She will then paint all the surfaces she can paint without moving either
of the boxes. She wants to paint as little as possible, so she places the boxes in a way that minimizes the
amount she'll have to paint. What is the total area she has to paint? (Source: ARML)

14.3 Pyramids

If we connect all the vertices of a polygon to a point that is not in the same plane as the
polygon, we form a pyramid. This point is called the apex of the pyramid and the polygon is f
the pyramid’s base. As we can see at right, the non-base faces of a pyramid are all triangles.
The lateral surface area of a pyramid is the sum of the areas of these triangles.

A As with prisms, a pyramid is ‘regular’ if its base is a regular polygon. A pyramid


is ‘right’ if the center of the base is the foot of the altitude from the apex to the base
(i.e., the apex is directly ‘over’ the center of the base). Usually when we speak of a
pyramid, we mean a right pyramid. The height of a pyramid is the distance from
B E the apex to the base. For regular pyramids, we also define a slant height, which
\F is the distance from the apex to a side of the base. For example, at left, AG is the
C D height and AF is the slant height of the pyramid.

3 Important: The volume ofa pyramid is one-third the product of the pyramid’s height
Vv and the area of the pyramid’s base.

Deriving this formula of a pyramid’s volume is very challenging, and will be left for the Intermediate
Geometry textbook. However, the formula is not hard to apply, so it is occasionally used in introductory
three-dimensional geometry.

Extra! I have two fish tanks that are both rectangular prisms like the one
imi tywe Shown to the right. I have a fish that is very picky, and will only be
happy ina tank that has exactly 50 gallons of water. Unfortunately,
my fish tanks are both 60-gallon tanks. Worse yet, I don’t have
anything I can use to measure the sides of the tank and mark off a
‘fill line.’ All I have is a hose to pour the water. How can I fill one of my tanks with
exactly 50 gallons?

366
14.3. PYRAMIDS

_Probiems
Problem 14.8: ABCDE is a right square pyramid such that BC = 6 and AC =5 A
as shown at right.

(a) Let O be the center of the square. Why is AO perpendicular to the base F D
of the pyramid? Show that AC = AD.
(b) Find [ACD] by first finding the length of an altitude from A to CD.
(c) Find the total surface area of ABCDE.
(d) Let M be the midpoint of CD. What kind of triangle is AAOM?
(e) Find AO, then find the volume of the pyramid.

Problem 14.9: Find a formula for the lateral surface area of a right pyramid whose base is a regular
n-gon with side length s, and whose slant height is 1.

Problem 14.10: A triangular pyramid is more commonly called a tetra-


hedron. A regular tetrahedron is a tetrahedron whose edges all have the
same length. In the diagram, regular tetrahedron ABCD has side length 6.

(a) Let G be the foot of the altitude from A to face ABCD. What kind
of triangles are AAGB, AAGC and AAGD?
(b) Use part (a) to show that G is the circumcenter of ABCD. Why must
it also be the centroid of ABCD?
(c) Let M be the midpoint of CD. Find BM and BG.
(d) Find AG.
(e) Find the volume of ABCD.

We'll start by investigating a square right pyramid.

Problem 14.8: ABCDE is a square right pyramid such that BC = 6 and AC = 5. Find the total surface
area and the volume of the pyramid.

Solution for Problem 14.8: We can find the area of the base quickly: BC? = 36.
But to find the area of the triangular faces, we’ll need an altitude of one of
the faces. We start by drawing AO, the altitude of the pyramid. Because
the pyramid is right, O is the center of base BCDE. We suspect that each
triangular face is isosceles because the pyramid is right. We can prove this
by noting that since AO is perpendicular to the base, it is perpendicular to
both OC and OD. We have OC = OD because O is the center of the square,
so AAOC = AAOD by SAS Congruence. This triangle congruence gives us
AC = AD.
CHAPTER 14. THREE-DIMENSIONAL GEOMETRY

Therefore, AACD is isosceles, so the median from A to CD is also an altitude. Since AC = 5 and
CM = CD/2 = 3, we have AM = 4 from right triangle AAMC. Therefore, [ACD] = (AM)(CD)/2 = 12.
Our total surface area consists of four of these triangles plus the square, or 4(12) + 36 = 84.
Turning to the volume, we need the height of the pyramid, AO. We can build a right triangle by
connecting O to either M or to one of the base vertices. In the diagram, we have right triangle AAOM.
Since OM = BC/2 = 3 and AM = 4, we have AO = VAM? — OM? = 7. Since the area of the base is 36,
the volume of the pyramid is (36)(V7)/3 = 12 N78

Once again, we built right triangles to find the lengths we needed.


As we saw in that last problem, finding the lateral surface area of a regular right pyramid is easy
once we know the slant height and the side length. We can even build a formula for it.

Problem 14.9: Find a formula for the lateral surface area of a right pyramid whose base is a regular
n-gon with side length s, and whose slant height is 1.

Solution for Problem 14.9: Each of the lateral faces of the pyramid is a triangle with
height / and base s. Hence, each of these triangles has area s//2. There is one
triangle for each side of the base, so there are n triangles, which have a total area of
n(sl/2) = nsl/2. Note that ns equals the perimeter of the base, so the lateral surface
area of a regular pyramid is half the product of the slant height and the perimeter
ool the bases
That formula is so much easier to derive than to memorize that we won’t even put it in an ‘Important’
box.
Problem 14.10: A triangular pyramid is more commonly called a tetrahedron. A regular tetrahedron
is a tetrahedron whose edges all have the same length. Find the volume of a regular tetrahedron that
has sides of length 6.

Solution for Problem 14.10: To find the volume, we need an altitude, so we draw A
altitude AG from A to ABCD. Since AG is perpendicular to plane BCD, triangles
AAGB, AAGC, and AAGD are all right triangles. Because AB = AC = ADandAGis B
obviously the same in all three triangles, we have AAGB = AAGC = AAGD by HL
Congruence. Therefore, BG = CG = DG, which means that G is the circumcenter D
of ABCD because it is equidistant from the vertices of ABCD. Since ABCD is an
equilateral triangle, G is also the centroid of ABCD. M
We can build more right triangles by continuing BG to M. Since ABCD is C
equilateral, BM is a median and an altitude. Therefore, DM = DC/2 = 3 and BM = 3 V3 (from 30-60-90
ABMD). Since the centroid of a triangle divides its medians ina 2 : 1 ratio, we have BG = (2/3)BM = 2 v3.
Finally, we can find AG from right triangle AAGB:

AG = VAB2
SBC. = V36=
12 = Vo.
Since the area of ABCD is (DC)(BM)/2 = 9 Y3, our volume is ([BCD])(AG)/3 = 18 y2.

Similarly, we can show that the volume of a regular tetrahedron with edge length s is s? -¥2/12.0

368
| 14.3. PYRAMIDS

Important: Problem 14.10 isa typical challenging three-dimensional geometry prob- ]


lem in that our general tactic is to reduce it to a series of two-dimensional
problems. When you can work through this problem on your own,

pes LUMO SNS SORE hiSsh

j|___Exercises
14.3.1 ABCDE is a right square pyramid with base ABCD. Given that AB = 4 and AE = 8, find the
following:
(a) The height of the pyramid.
(b) The slant height of the pyramid.
(c) The volume of the pyramid.
(d) The total surface area of the pyramid.
14.3.2. When we first created Problem 14.8, we let BC = 8 instead of BC = 6 in the problem statement.
Why did we have to change it? Hints: 477
14.3.3. A regular pyramid with a square base has base edges of length 6 inches and height 4 inches.
What is the ratio of the number of cubic inches in the volume of the pyramid to the number of square
inches in its surface area? (Source: MATHCOUNTS)

14.3.4 A4” x6” x 8” rectangular solid is cut by slicing through the midpoints of three
adjacent sides as shown.
(a) Find the volume of the shaded piece that is cut off.
(b) Find the sum of the lengths of the edges of the shaded piece that is cut off. (Source: MATH-
COUNTS)

14.3.5 STUVWXYZ isa cube as shown with ST = 4.

(a) What are the volume and surface area of pyramid STUW?
(b) What are the volume and surface area of pyramid STUX? Hints: 524
(c) What are the volume and surface area of pyramid STUZ?

Extra! Back on page 253, we noted that any polygon can be dissected and rearranged to form
imam any other polygon that has the same area. You might wonder if it is possible to dissect
any polyhedron and rearrange the pieces to form any other polyhedron with the same
volume. You wouldn’t be the first to wonder this! In fact, this question was one of
Hilbert’s famous problems (see page 56). It was also the first to be solved, by Max
Wilhelm Dehn.
Dehn used the powerful problem solving technique of invariants to solve the problem.
He created a function of the edge lengths and the dihedral angles of a polyhedron and
showed that this function must be equal for any two polyhedra that can be dissected
into the same pieces. The function is different for a cube and a tetrahedron, so these two
cannot be dissected into the same pieces.

369
CHAPTER 14. THREE-DIMENSIONAL GEOMETRY

14.4 Regular Polyhedra

Figure 14.2: A Regular Tetrahedron and a Cube

We have already mentioned that a polyhedron is a solid figure that is bounded on all sides by polygons.
A regular polyhedron is a convex polyhedron in which all the faces are congruent regular polygons,
and there are the same number of edges at each vertex. We’ve already seen examples of two such regular
polyhedra — a cube and a regular tetrahedron, examples of which are shown in Figure 14.2. The faces of
the cube are congruent squares and those of the tetrahedron are congruent equilateral triangles.
These aren’t the only types of regular polyhedra, however. In this section we discover the other
regular polyhedra, and we tackle a problem involving one of the types of regular polyhedra.

ee
Problem 14.11: To make a regular tetrahedron, we might start
with four equilateral triangles in a plane as shown at right. We Cc
then fold the triangles until A’, D, and D’ coincide as shownin 4’ Aioge ipA
the regular tetrahedron. As we'll see in this problem, thinking B B
about ‘folding up’ regular polygons like this allows us to limit D
the possibilities of regular polyhedra that can be made.
(a) Consider three squares, ABCD, ABEF, and AFGH, arranged around a point as B
shown. If we ‘fold’ this arrangement so that D and H coincide, we'll be on our
way to forming what type of regular polyhedron? E D
(b) Can we fit three regular pentagons around a point in.a plane? Can we then *
_ fold the resulting figure to start to form a polyhedron? G H
(c) Can we fit three regular hexagons around a point in a plane? Can we then fold the resulting
figure to start to form a polyhedron?
(d) Can we fit four equilateral triangles around a point in a plane? Can we then fold the resulting
figure to start to form a polyhedron? How about four squares?
(e) Can we fit five equilateral triangles around a point in a plane? Can we then fold the resulting
figure to start to form a polyhedron? How about five squares?
(f) At most how many different types of regular polyhedra are there?

370
14.4. REGULAR POLYHEDRA

Problem 14.12: ABCDEF shown is a regular octahedron, which has eight


congruent equilateral triangles as faces. We can also think of a regular
octahedron as a pair of square pyramids glued together at their bases. In
this problem, let AB = 4.
(a) Find BD. B
(b) Find AF.
(c) Find [BCDE].
(d) Find the volume of ABCDEF.

There aren’t many types of regular polyhedra. With a little physical intuition, we can discover which
types of polyhedra might possibly exist.
Pa ee ——EEEe SS Se eee

Problem 14. 11: Toae a regular Meee he dren.wee might C iy A’=D’=D


start with four equilateral triangles in a plane as shown at C
right. We then fold the triangles until A, D, and D’ coincide A’ pslpitrs.9 A
as shown in the regular tetrahedron. As we'll see in this B B
problem, thinking about ‘folding up’ regular polygons like D
this allows us to limit the possibilities of regular polyhedra that can be made.
(a) Consider three squares, ABCD, ABEF, and AFGH, arranged around a point B C
as shown. If we ‘fold’ this arrangement so that D and H coincide, we'll be on
our way to forming what type of regular polyhedron? rc D
(b) Can we fit three regular pentagons around a point in a plane? Can we then
fold the resulting figure to start to form a polyhedron? G H
(c) Can we fit three regular hexagons around a point in a plane? Can we then fold the resulting
figure to start to form a polyhedron?
(d) Can we fit four equilateral triangles around a point in a plane? Can we then fold the resulting
figure to start to form a polyhedron? How about four squares?
(e) Can we fit five equilateral triangles around a point in a plane? Can we then fold the resulting
figure to start to form a polyhedron? How about five squares?
(f) At most how many different types of regular polyhedra are there?

Solution for Problem 14.11:


(a) Folding up our squares gives the figure shown, which is clearly well on its way B_C
to being a cube. = Dew

FioiG

Extra! We have to reinvent the wheel every once in a es not because.we needa lota wheels,but
-— because we needa lot ofinventors. 4 :
2 : Bruce Joyce

371
e
CHAPTER 14. THREE-DIMENSIONAL GEOMETRY

(b) We can fit three pentagons around a point with room to spare. Therefore, we can fold the pentagons
to possibly form part of a regular polyhedron. Such a polyhedron does indeed exist, as shown
below. This polyhedron is called a regular dodecahedron. It has 12 regular pentagons as its faces.

(c) We can fit three regular hexagons around a point, but since each angle of a regular
hexagon is 120°, the three fit snugly. In other words, we can’t bend the hexagons
together as we’ve done earlier with the pentagons, squares and triangles. There-
fore, there are no more regular polyhedra that have three of each polygon meeting
at a point.

(d —
Four equilateral triangles fit around a point with room to spare, so we can fold them together. The
result is a square pyramid without its base. Gluing another square pyramid to this one gives us
a polyhedron with eight congruent equilateral triangles as faces. This polyhedron, shown in the
diagram below, is a regular octahedron.

Four squares, of course, fit snugly around a point, since 4(90°) = 360°. Hence, we can’t fold the
squares to make a polyhedron.

(e) As with three and four equilateral triangles, we can fit five triangles around a point, then fold them
together. This gives us the start of a regular icosahedron, which has twenty congruent equilateral
triangles as its faces.

Obviously, we can’t fit five of any other regular polygon around a point.

372
14.4. REGULAR POLYHEDRA

(f) We can fit six equilateral triangles snugly around a point, so we can’t ‘fold’
them into the start of a regular polyhedron. Obviously, any regular polyhedron
must have at least three faces meet at each vertex, so we’ve exhausted all the
possibilities for regular polyhedra.

———___———_—

Important: There aare five regular polyhedra, which are described below.

VY Name
Tetrahedron Triangle
Cube Square
Octahedron Triangle
Dodecahedron | Pentagon
Icosahedron Triangle

These five regular polyhedra are collectively named the Platonic solids,
after the great Greek philosopher Plato.
secre |

Icosahedra and dodecahedra are pretty hard to work with, but since regular octahedra are just two
square pyramids stuck together, they’re much more manageable.

ey
Problem 14.12: ABCDEF shown is a regular octahedron with AB = 4.
(a) Find BD.
(b) Find AF.
(c) Find [BCDE].
(d) Find the volume of ABCDEF.

Solution for Problem 14.12:

(a) Since BCDE is a square, BD = BC Y2. All the sides of a regular polyhedron have the same length,
so BC = AB = 4 and BD = 4 v2.
(b) Instead of thinking of the octahedron as square pyramids ABCDE and FBCDE glued together,
we can think of it as BAEFC and DAEFC glued together. Therefore, ACFE is also a square, so
AF = BD = 4 Y2. (We can also just note that since the polyhedron is regular, the distance between
directly opposite vertices must be the same no matter which pair of opposite vertices we choose.)
(c) BCDE is a square with side length 4, so its area is 42 = 16.
(d) To find the volume of the regular octahedron, we think of it as two congruent square pyramids,
ABCDE and FBCDE. Since the altitudes from A and from F, respectively, of these two pyramids
together make up AF, the height of each pyramid is AF/2 = (4 V2)/2 = 2 V2. The area of a base is
4* = 16, so the volume of each pyramid is (16)(2 V2)/3 = 32 2/3. Therefore, our octahedron has
volume 2(32 Y2/3) = 64 V2/3.

373
CHAPTER 14. THREE-DIMENSIONAL GEOMETRY

L_txercises lg
14.4.1 Compute the quantity

(N umber of vertices) — (Number of edges) + (Number of faces)

for each regular polyhedron (cube, tetrahedron, octahedron, dodecahedron, icosahedron). Notice any-
thing interesting? Try it on other polyhedra, such as various types of prisms and pyramids and see if
your interesting observation still holds.
14.4.2 The six vertices of a regular octahedron are snipped off, leaving a square face in place of each
corner and a hexagonal face in place of each original face of the octahedron. How many vertices, faces,
and edges will the new polyhedron have?

14.4.3x ABCDEF shown at right is a regular octahedron with side length 1. Let A
O be the center of face ABC, P be the center of face ACD, and Q be the center of
face DEF.
D
(a) Find OP. Hints: 287 B
(b)x Find OQ. Hints: 59 an
(c) Find [OPQ]. Hints: 580 r

14.5 Summary

ons: ‘The volume ofathree dimensional figure is a measure of the space inside the figure.
al surface area of a figure is the total area of all the surfaces that form a boundaries of the |
solid. 1
The lateralsurface area isthe total <
area of all the surfaces that are not considered ‘bases’.

‘Definitions: . polyhedron iis a solid figure with polveuns 2as its boundaries. A prism has. two ?
| congruent ely ae as bases and all meted faces (called ee are nes Ina

The———— ofa Hone rectangular prism are


commonly called the length, J, the width, w, and the
height, h. For such a prism, we have:

Volume lwh
Surface area 2(lw + wh + Th)
Space Diagonal Vi? + w + h2
The volume of a prism equals the area of the base times the distance
between the bases (i.e. the height).

374
14.5. SUMMARY

Definition: A cube isa special right rectangular prism in which all the edge lengths W
are the same (i.e., its base is a square and its height has the same length as a side of
the base).

Important: A cube with side length s has:

y Volume II s°
Surface area = 6s”
Space Diagonal

Definitions: If we connect all the vertices of a polygon to a point that is not in the same f
plane as the polygon, we form a pyramid. This point is called the apex of the pyramid
and the polygon is the pyramid’s base. As we can see at right, the non-base faces of a
pyramid are all triangles. The lateral surface area of a pyramid is the sum of the areas of
these triangles. A tetrahedron is a pyramid with a triangular base.
The height of a pyramid is the distance from the apex to the base. If the base is a regular polygon,
the pyramid is a regular pyramid. For regular pyramids, we also define a slant height, which is the
distance from the apex to a side of the base.

Important: e The volume of a pyramid is one-third the product of the pyra-


Vv mid’s height and the area of the pyramid’s base.

e The lateral surface area of a regular pyramid equals one-half the


product of the slant height and the perimeter of the pyramid’s
base.

Definitions: A regular polyhedron is a polyhedron whose faces are all congruent regular polygons.

Important: There are five regular polyhedra, which are described below.

VY Name
Tetrahedron
|Face Type
Triangle
Cube Square
Octahedron Triangle

375
CHAPTER 14. THREE-DIMENSIONAL GEOMETRY

REVIEW PROBLEMS >

14.13 A diagonal of one of the faces of a given cube has length 4.


(a) Find the length of an edge of the cube.
(b) Find the length of a space diagonal of the cube.
(c) Find the total surface area of the cube.
(d) Find the volume of the cube.

14.14 Shown is right rectangular prism ZYXWVUTS. What type of quadri-


lateral is ZYTS and why?
14.15 A right rectangular prism has space diagonal 3 V13 and two sides of
length 3 and 7, respectively.
(a) Find the third dimension of the prism.
(b) Find the volume of the prism.
(c) Find the total surface area of the prism.

14.16 A space diagonal of cube A is an edge of cube B.


(a) Find the ratio of the surface area of A to the surface area of 8.
(b) Find the ratio of the volume of A to the volume of 8.

14.17 VWXYZ is a right square pyramid with square base WXYZ. Given YZ = 10 and YV = 13 v2,
find the following:

(a) the height of the pyramid.


(b) the slant height of the pyramid.
(c) the total surface area of the pyramid.
(d) the volume of the pyramid.

14.18 M,N,O,and Pare midpoints of edges AB, BC, CD, and DA, respectively, of right A
rectangular prism ABCDEFGH shown. Q, R, S, and T are the midpoints of edges EF,
FG, GH, and HE, respectively. Given AB = 8, BC = 10, and BF = 5, find the following:

(a) MN.
(b) MO.
(c)x the volume of MNOPQRST.
14.19 What figure is formed by connecting the centers of the faces of a:
(a) cube?
(b) regular octahedron?

376
REVIEW PROBLEMS

(c) regular tetrahedron?


(d) regular dodecahedron?
(e) regular icosahedron?

14.20 Shown is cube ABCDEFGH with side length 8.


(a) Find [ACF]. Hints: 332
(b) Find [ACG].
14.21 How many edges does a regular hexagonal prism have?

14.22 This shape at right is folded into a regular octahedron with the equilateral
triangles shown as faces. What is the sum of the numbers on the faces sharing an
edge with the face with a “1” on it? (Source: MATHCOUNTS)
14.23 ABCDEFG is a right pyramid with regular hexagonal base ABCDEF and
apex G with AB = 6 and AG = 6 Y3.
(a) Find the volume of ABCDEEG. S
(b) Find the total surface area of ABCDEFG.

14.24 Find the volume and the surface area of a regular tetrahedron with side length 9.
14.25 On the center of each face of a cube with side length 4 inches, we glue a cube of side length 1
inch (so that an entire face of each small cube is glued to the big cube). We then paint the entire resulting
figure red. How many square inches of red paint will we use?
14.26 WXYZ isa triangular pyramid with XY = YZ = ZX =9 and WX = WY = WZ = 18. Let G be the
centroid of AXYZ and M the midpoint of XY.
(a) Prove that WG is perpendicular to plane XYZ. (If you're stuck, check out Problem 14.10.)
(b) Find WM.
(c) Find ZM.
(d) Find the volume of WXYZ.

14.27 _Inacube with side length 6, what is the volume of the tetrahedron formed by any vertex and the
three vertices connected to that vertex by edges of the cube? (Source: HMMT)
14.28 Each of the three lines k, m, and n is perpendicular to the other two lines. All three lines pass
through point P. K is on line k, M is on line m, and N is on line n such that KP = 7, KM = 9, and KN = 11.

(a) Find the length of MN.


(b) Find the volume of KNMP.
(c)x Find the length of the segment from K to the midpoint of MN. Hints: 554, 371
14.29 Intetrahedron MNOP, we have MN = OP, MO = NP, and MP = NO. Prove that MNO = ZMPO.

14.30 Find a formula for the volume of a regular octahedron with edge length s.

377
CHAPTER 14. THREE-DIMENSIONAL GEOMETRY

Challenge Problems >

14.31 Base EFGH of right prism EFGHIJKL is a parallelogram with EF = 8, EG @=6 and 2E6G = ou
Given that the height of the prism is 9, find the following:
(a) [EFGH]. Hints: 542
(b) the surface area of EFGHIJKL.
(c) the volume of EFGHIJKL.
14.32 Thealtitude from vertex V of tetrahedron VWXY meets face WXY at the circumcenter of AWXY.
Prove that VW = VX = VY. Hints: 425

14.33 Each side of the cube shown has length 10. What is the volume of the solid
whose edges are formed by connecting all the midpoints of the edges of the cube
as shown? Hints: 50
14.34 A soccer ball is constructed using 32 regular polygons with equal side
lengths. Twelve of the polygons are pentagons, and the rest are hexagons. A seam
is sewn wherever two edges meet. What is the number of seams in the soccer ball?
(Source: MATHCOUNTS) Hints: 130

14.35 The figure at right with 5 square faces and 10 equilateral triangular
faces is folded into a 15-faced polyhedron. How many edges does the polyhe-
dron have? How many vertices does the polyhedron have? (Source: MATH-
COUNTS)
14.36 Anoctahedron is formed by connecting the centers of the faces of a cube.
What is the ratio of the volume of the cube to the volume of the octahedron?
Hints: 164

14.37 An insect lives on the surface of a regular tetrahedron with edges of


length 1. It wishes to travel on the surface of the tetrahedron from the midpoint
of one edge to the midpoint of the opposite edge. What is the length of the shortest such trip? (Note:
Two edges of a tetrahedron are opposite if they have no common endpoint.) (Source: AMC 12) Hints:
ileUh

14.38 STUVWXYZ is a right rectangular prism with XY = 12, WX = 3, and


4)
(a) Find WU.
(b) Find the distance from Z to WU. Hints: 64, 531
(c) Find the volume of TWXY.
(d)* Find the volume of pyramid VSUYW. (V is the vertex.)
(e)x Find the volume of WXZU. Hints: 155,573

378
CHALLENGE PROBLEMS

14.39 A cube of cheese C = {(x, y,z)|0 < x, y,z < 1} is cut along the planes x = y, y = z, and z = x. How
many pieces are produced this way? (Source: AHSME) Hints: 181, 253
14.40 EFGHIJKLisacube. How many different planes pass through at least three vertices of EFGHIJKL?
Hints: 297

14.41 In cube ABCDEFGH shown, AB = 12. Find the distance from B to plane ACF.
Hints: 93, 281
14.42 The sum of the lengths of the twelve edges of a rectangular box is 140, and the
distance from one corner of the box to the farthest corner is 21. What is the total surface
area of the box? (Source: AMC 12) Hints: 431

14.43 Congruent wedges A and B are sliced as shown. Each


wedge has a pair of parallel triangular faces. Wedge A is cut ver- LED iG 6"
tically so that slices are made perpendicular to the base; wedge B
is cut horizontally so that slices are made parallel to the base. Six 10” 10”
slices 1/4 inch thick are cut from each wedge. What is the ratio 12” (2
of the remaining volume of wedge A to the remaining volume Wedge A Wedge B
of wedge B? (Source: MATHCOUNTS) Hints: 45

14.44 ABCDEF shown at left below is a regular octahedron with edge length 1. The midpoints of edges
AB, AC, AD, and AE are M, N, O, and P, respectively. The midpoints of edges BF, CF, DF, and EF are Q,
R, S, and T, respectively. Find the volume of MNOPQRST. Hints: 96

Figure 14.3: Diagram for Problem 14.44 Figure 14.4: Diagram for Problem 14.45

14.45x Parallelogram EFGH is a base of right prism EFGHIJKL at right above. Given EF = 8, FG = 6,
EI = 9, and ZEFG = 60°, find FL and EK. Hints: 133
14.46x A right square frustum is formed by cutting a right square pyramid with a plane parallel to
its base. Suppose the original pyramid has base length 6 and height 9, and that the plane cutting the
pyramid to form the frustum is 3 units from the base of the pyramid.

(a) Find the volume of the frustum. Hints: 185, 263


(b) Find the total surface area of the frustum.

14.47x Let ABCD be a regular tetrahedron with side length 2. The plane parallel to edges AB and CD
and lying halfway between them cuts ABCD into two pieces. Find the surface area of one of these pieces.
(Source: HMMT) Hints: 211

379
CHAPTER 15. CURVED SURFACES

The Seven Circles Theorem

Everything in nature adheres to the cone, the cylinder and the cube. — Paul Cezanne

CHAPTER

Curved Surfaces

15.1 Cylinders

Instead of using congruent polygons for bases as we do with a prism, suppose we use congruent circles.
The resulting solid is called a cylinder, an example of which is shown below. The line connecting the
centers of the bases is called the axis of the cylinder (OO’ in the diagram) and the radius of a base is also
considered the radius of the cylinder.

Figure 15.1: A Right Circular Cylinder

You won't be surprised to learn that if the axis is perpendicular to the bases, then the cylinder is a
right cylinder as shown above. Furthermore, we can have cylinders in which the bases are any sort of
wacky curves. Therefore, our simple cylinder above is more precisely called a right circular cylinder,
where the ‘circular’ tells us that the bases are circles. When we speak of a cylinder, we nearly always

380
1.,CYLINDERS

mean a ‘right circular cylinder,’ instead of one of the wacky cylinders shown in Figure 15.2 below.

Figure 15.2: More Cylinders

I)Problems i
Problem 15.1:
(a) Suppose we take a cross-section of a cylinder that is perpendicular to the axis of the cylinder.
What shape is this cross-section?
(b) What is the shape of a cross-section of a cylinder that contains the axis of the cylinder?
(c) What is the shape of a cross-section of a cylinder that is parallel to the axis of the cylinder?

Problem 15.2: The figure at right shows a right circular cylinder (a.k.a. a
cylinder) with radius 3 and height 5.

(a) Find the volume of the cylinder.


(b) Suppose we cut the curved surface of the cylinder along the dotted line
shown (assume the dotted line is perpendicular to the bases) and ‘unroll’
the surface into a plane. What type of figure results?
(c). What is the lateral surface area of the cylinder?
What is the total surface area of the cylinder?
(e) Find formulas for the volume, lateral surface area, and ‘otal surface area of a linear with
radius r and height h.

Problem 15.3: The radius of a cylinder is 2/3 its height. Find the total surface area of the cylinder if its
volume is 9677.

We’ll start our investigation of cylinders by investigating a few cross-sections of a cylinder.

Problem 15.1:
(a) Suppose we take a cross-section of a cylinder that is perpendicular to the axis of the cylinder.
What shape is this cross-section?
(b) What is the shape of a cross-section of a cylinder that contains the axis of the cylinder?
(c) What is the shape of a cross-section of a cylinder that is parallel to the axis of the cylinder?

Ree
ste lll. RE
CHAPTER 15. CURVED SURFACES

Solution
for Problem 15.1:
(a) Intuitively, it seems clear that the cross-section is a circle. To
prove this, we consider A and A’ on the bases such that AA’ is
parallel to the axis OO’. We let X and Y be the points where
OO’ and AA’, respectively, meet our cross-section plane. Since
this plane is perpendicular to the axis, we have XY 1 OO’. So,
XYAO is a rectangle, and XY = OA. Hence, XY equals the
radius of the cylinder.
But there’s nothing special about the points A and A’ we
chose on the circumferences of the bases! In exactly the same way, we can show that any point on
the intersection of our plane and the curved surface of the cylinder is exactly the cylinder’s radius
away from point X on the axis. Therefore, our cross-section is indeed a circle.
(b) Since AA’ and BB’ are both parallel to the axis of the cylinder, z i Oe es Goa)
they are perpendicular to both bases. So, AA’B’B is a rectangle. ene peed
(c) Since our cross-section is parallel to the axis of the cylinder,
it must be perpendicular to both bases. Therefore, as in the B’ A!
previous part, our cross-section is a rectangle.

One result of part (a) above is that the set of all points that are a fixed distance from a given line in
space is a cylindrical surface, as shown below. The surface is basically a cylinder without bases that
continues forever along the line that is its axis.

Figure 15.3: A Cylindrical Surface and Its Axis

Now that we have some sense for what a cylinder is like, let’s investigate the area and volume of a
cylinder.
‘.
Problem 15.2: The figure shows a right circular cylinder (a.k.a. a cylinder) with radius
3 and height 5. cS
(a) Find the volume of the cylinder.
(b) What is the lateral surface area of our cylinder?
(c) What is the total surface area of our cylinder?
(d) Find formulas for the volume, lateral surface area, and total surface area of a cylinder with
radius r and height h.

382
15.1. ;CYLINDERS

Solution for Problem 15.2:


(a) Our cylinder is just like a prism — its cross-sections have the same area from bottom to top.
Therefore, its volume is just the area of a base times the height. The area of the base is 7:(37) = 971,
so the volume is (97t)(5) = 457.
(b) We don’t have any tools to deal with finding the area of a curved surface, so we'll
have to do something clever. Suppose our cylindrical surface is made of paper, so Coz o
we can cut along a segment like XX’ that is parallel to both bases (and hence the
same length as a height). Then we ‘unroll’ the surface. The top and the bottom
are always 5 units apart (the height of the cylinder), so the unrolled surface is a x’
rectangle.
The unrolled surface is shown at right. One dimension Z circumference of top
of this rectangle is the height of the cylinder; the other is the
circumference of the cylinder. For our given cylinder, the
height is 5 and the circumference is 2mr = 67. Therefore,
our lateral surface area is (5)(67) = 307.
yb circumference of bottom J
(c) For the total surface area, we add the bases to our lateral
surface area. Each base has area 971, so the total surface area is

30m + 2(97t) = 4870.

(d) We simply follow the steps we used to solve the preceding parts. The area of each base is mr”, so
the volume is 7r7h. The curved surface unrolls to form a rectangle with dimensions h and 27r (the
circumference of each base), so it has area 27rh. Therefore, our lateral surface area is 27rh and the
total surface area is 2nrh + 27.

Let’s try applying these formulas.

rr nes
CHAPTER 15. CURVED SURFACES

Solution for Problem 15.3: We let the height be h, so the radius is r = 2h/3. Since the volume is 967, we
have nr2h = 967. Substituting for r gives

2
(>) hn = 967,
)

and solving this equation for h gives h = 6. Therefore, r = 2h/3 = 4. Now we can find our total surface
area:

2nrh + 2nr* = 48n + 327 = 807.

Concept: These basic area/volume word problems are often no different from other
©=== _~_—-word problems. The key to solving them is to assign variables and use the
area and volume information to set up equations with the given informa-
tion. ; Z

Exercises

15.1.1 A cylinder has radius 8 and height 4.


(a) Find the volume of the cylinder.
(b) Find the lateral surface area of the cylinder.
(c) Find the total surface area of the cylinder.

15.1.2 The diameter of a cylinder equals its height. The total surface area of the cylinder is 1507. What
is the volume of the cylinder?

15.1.3. Shown at right is a circular cylinder that is not right. The bases have
centers A and B as shown, and point C is on the circumference of a base. Given
that AB = 8, BC = 4, and ZACB = 90°, find the following:

(a) The height of the cylinder. (Note that AB is not an altitude of the
cylinder because it is not perpendicular to the bases! Remember, the
height of a cylinder or prism is the distance between its bases.)
(b) The volume of the cylinder.

15.1.4 The lateral surface area of cylinder C equals the sum of the areas of its bases. What is the ratio
of the radius of C to the height of C?

15.1.5 A cylinder is inscribed in a square prism. As shown at left, each of the bases of
the cylinder is inscribed in a face of the prism, and the cylinder is tangent to the other
four faces of the prism. Given that AB = 4 and AE = 8, find the volume of the cylinder.
15.1.6* Does there exist a group of seven cylinders such that it is possible to arrange
them so that each cylinder is tangent (i.e. touches at only one point) to the other six?
15.2. CONES

15.2 Cones

Just as turning the bases of a prism into circles makes a cylinder, we can turn A
the base of a pyramid into a circle to get a cone. The figure on the right shows
a right circular cone. The point A at the tip of the cone is the vertex of the cone
and the distance from the vertex to the base is the height. The line connecting the
vertex to the center of the base is the axis of the cone. The radius of the base is
considered the radius of the cone, and for right circular cones, the distance from
the vertex to a point on the circumference of the base is the slant height. B

As with cylinders, cones don’t have to be ‘right’ or ‘circular.’ On


the left are a couple of these less common cones. Despite the existence
of these weird cones, when we simply say ‘cone,’ we almost always
mean ‘right circular cone.’
Having already learned that the volume of a pyramid equals one-
third its height times the area of its base, you won’t be surprised to
learn that the same holds for a cone.

Important: The volume of a circular cone with height h and radius r is |

VY Volume = ah

Problems Be

Problem 15.4:
(a) What is the shape of a cross-section of a cone that contains the axis of the cone?
(b) What is the shape of a cross-section of a cone that is perpendicular to the axis of the cone?

Problem 15.5: In this problem we will find a formula for the lateral surface
area of a cone with base radius r and slant height /.

(a) Use the result of Problem 14.9 to take a guess at the formula.
(b) Let A be the vertex of the cone and B a point on the circumference of
the cone. Suppose we cut the curved surface of the cone along AB, then
‘unroll’ it into a flat figure. What type of figure do we thus form?
(c) How long is the curved portion of the figure you make in (b)? Use this
information to find a formula for the lateral surface area of the cone.

Problem 15.6: Find the volume of a right circular cone that has radius 6 and slant height 8.

Extra! The open secret of real success is to throw your whole personality at a problem.
8 0116 —George Polya

385
CHAPTER 15. CURVED SURFACES

is 8,
Problem 15.7: A semicircle is rolled up to form a cone as shown. If the radius of the semicircle
what is the volume of the cone?

a: |
C LS
Problem 15.8: A cone with vertex A, height AB = 9, and radius BC = 12 is given.
The cone is cut in two pieces by a plane perpendicular to AB at point X, where
AX = 6.

(a) The top piece, of course, is a cone. What is the height of this cone? What is
the volume of this cone?
(b) What is the volume of the other piece?
(c) What is the ratio of the volume of the little cone to the volume of the large
cone? Is this surprising?

We start by getting a feel for cones by looking at some cross-sections.

Problem 15.4:
(a) What is the shape of a cross-section of a cone that contains the axis of the cone?
(b) What is the shape of a cross-section of a cone that is perpendicular to the axis of the cone?

Solution for Problem 15.4:


(a) A cross-section of a cone that contains the cone’s axis consists of two segments A
of equal length connecting the vertex to two points on the circumference of the
base of the cone, as well as the segment connecting these two points along the
base of the cone. So, our cross-section is an isosceles triangle, such as AABC in
the diagram at right. B C
A (b) Intuitively, it seems clear that a cross-section of a cone perpendicular to the
axis of the cone is a circle. Suppose the plane of our cross-section meets the
axis at O’. To prove our cross-section is a circle, we must show that every
point where our plane hits the curved surface of the cone is equidistant
from O’. Consider point B’, the intersection of AB and our cross-section
B’ plane, as shown. Since B’O’ and BO are each perpendicular to AO, we have
B’O’ || BO. Therefore, AAO’B’ ~ AAOB.
Since AAO’B’ ~ AAOB, we have B’O’/BO = AO’/AO. Therefore, we
find B’O’ = (AO’/AO)(BO). Since BO is just the radius of the cone and AO

386
15.2. CONES

is the cone’s height, we have B’O’ = (r/h)(AO’). Similarly, we can show that all points of the
cross-section are (r/h)(AO’) away from O’. Since AO’ is fixed, all the points of our cross-section
are the same distance from O’. Therefore, the cross-section must be a circle. (Make sure you see
why every point on this circle must be in the cross-section.)

While the proof of the volume formula for cones requires some very advanced tools, we can find a
formula for the area of the curved surface of a cone with techniques we already have.

Problem 15.5: Findaformula


forthe lateral surface areaofa right circular cone with
base radius r'
and slant height /.

Solution for Problem 15.5: Since cutting and unrolling A


ener A
the curved surface was so successful in finding the l
lateral surface area of a cylinder, we try it with a cone B B’
as well. We cut the curved surface of the cone along alec
AB, where A is the vertex and B is a point on the
circumference of the base.
Since every point on the circumference of the B
cone’s base is the same distance from the cone’s vertex (the slant height), when we unroll the curved |
surface, these points will still all be the same distance from the vertex. Hence, our ‘unrolled’ surface is a
sector of a circle as shown at right above. (B and B’ coincide when the sector is rolled up to form a cone.)

The radius of this sector is AB, the slant height of the cone. To find the area of the sector, we must
determine what portion of a whole circle the sector is. We know that the length of BB’ is just equal to
the circumference of the cone’s base. The base of the cone has radius r, so its circumference is 27r. Thus,
the length of BB’ is 2mr. Since a whole circle with radius AB = | has circumference 2nl, our sector is
(27r)/(271) = r/l of a whole circle.
A full circle with radius / has area 71/2, so the area of a sector that is r/I of this circle is (r/1)(n/?) = ml.

Recall from Problem 14.9 that we showed that the lateral surface area of a regular pyramid is half the
product of the slant height and the perimeter of the base. The proof we used there wouldn’t work for
cones, since we don’t have triangular faces as the sides of a cone. However, since cones are essentially
just pyramids with circular bases, we expect the formula to work for cones, too. Trying it, we note
that the perimeter of the base of a cone is 277, so our formula gives us (1/2)(27r)(1) = mrl for the lateral
surface area. Unsurprisingly, this matches the formula we already proved. 0

“Important: “The lateral surface area of a rightcircularcone with radius r-and slant
ae Vv ee lis mrl.
ea

on ept: AS with cylinders, problems involving the curved :surface of a cone can
ten be solved by ‘unrolling’ thea surface. As we've seen, the result |
is asector of a circle. ae

387
CHAPTER 15. CURVED SURFACES

lem
15.6:
‘Prob Find the volume of aright circular cone that has radius 6and slant height 8.
Solution for Problem 15.6: We need the height in order to find the volume. As we A
did with a pyramid, we can build a right triangle to find the height. We draw AO,
the axis of the cone, and connect A and O to B, a point on the circumference of the
base. Since OB is a radius and AB is a slant height, we have AB = 8 and OB s=6;
Since the cone is ‘right,’ ZAOB is a right angle, so AO = VAB2 — OB2 = 2V7.
Therefore, we have:

muh2 _ 76 2 )(2 v7) Serene,


Volume =
o 3
0

As we've just seen, a radius, an altitude, and a slant height make a right triangle in a right circular
cone. We could immortalize this in yet another formula, I? = r* + h?, but there’s no need to — this is just
the Pythagorean Theorem!
Back in Problem 15.5, we unrolled the curved surface of a cone to get a sector. What happens if we
go the other way?

‘Problem 15.7: A semicircle is rolled up to form a cone as shown. If the radius of the semicircle is 8,
what is the volume of the cone?

Solution for Problem 15.7: Here we need to find both the radius and the height of the cone. At first it
seems that all we are given is the slant height; however, the information about the ‘unrolled’ curved
surface can be used to find the radius in a couple different ways.

Method One: Roll it up! When we roll up the semicircle into a cone, the arc BCB’ becomes the
circumference of the base of the cone. Since this arc of the semicircle is half the circumference of a circle
with radius 8, it has length 87. Therefore, the circumference of the base of the cone is 87, so its radius
must be 4.

Method Two: Use the lateral surface area! The semicircle has.area 7(8*)/2 = 327. When rolled up, this
is the curved surface of the cone, which we know has area mrl, where r is the radius of the cone and | is
its slant height. Since the radius of the semicircle is the slant height of the cone, we have | = 8. Hence,
we can solve mrl = 327 to find r = 32/1 =

Once we have the radius, we can find the height by using right triangle AAOB in the A
cone as before. Specifically, h = V8? - 42 = 4 v3. Thus, our volume is

mr’h 64n V3
CPi. | B

388
SSS
sss SSS St
15.2. CONES
SESS

Now that we have a good understanding of cone volumes and basic cross-sections of a cone, let’s
try putting them together.

Problem 15.8: A cone with vertex A, height AB = 9, and radius BC = 12 is given. The cone is cut in|
two pieces by a plane perpendicular to AB at point X, where AX = 6. Find the volume of the two
smaller pieces thus formed.

Solution for Problem 15.8: We showed in Problem 15.4 that a cross-section of a cone A
perpendicular to its axis is a circle. So, one of our pieces is itself a cone. The other piece
is called a right circular frustum. We don’t have any tools to deal with a frustum,
but we do know how to find the volume of a cone. The original cone has volume
mtr*h/3 = 7(127)(9)/3 = 4327. We have the height of the smaller cone, AX = 6, so all Y
we have to do is find the radius. arias
Since XY and BC are each perpendicular to AB, we have ZAXY = ZABC and :
LXAY = ZBAC, so AAXY ~ AABC. (Notice that we are essentially considering the cross-section of the
cone that contains AABC here — three-dimensional problems are often just two-dimensional problems in
disguise!) Therefore, XY/AX = BC/AB, so XY = (BC/AB)(AX) = (12/9)(6) = 8. Hence, our little cone has
volume m(XY7)(AX)/3 = 1287.

To get the volume of the other piece, we merely subtract the little cone from the big one, which yields
432n — 1287 = 3047.
Notice that
Volume of smallcone 1287 _ 8 @ a, eS
— AB .

Volume of largecone 432 27 % 2


This shouldn’t be a surprise, because our cones are similar figures. 0

Important: Just as the ratio of the areas of similar two-dimensional figures is the
square of the ratio of their corresponding sides, the ratio of the surface
areas of similar three-dimensional figures is the square of the ratio of their
corresponding side lengths. Moreover, the ratio of the volumes of similar
three-dimensional figures is the cube of the ratio of their corresponding
side lengths.

a || Exercises | g a

15.2.1 Acone has height 5 and radius 2. Find the volume and total surface area of the cone.
15.2.2 Acone has a lateral surface area 547 and radius 6. Find the slant height, height, and volume of
the cone.

15.2.3. A quarter-circle with radius 4 is rolled up to form a cone.


(a) Find the lateral surface area of the cone.
(b) Find the volume of the cone.

ess.
— - 889
CHAPTER 15. CURVED SURFACES

15.2.4 Cone A has twice the radius, but half the height, of cone B. What is the ratio of the volume of
A to the volume of 8?

15.2.5 Prove that the vertex of a right circular cone is equidistant from every point on the circumference
of the cone’s base. Hints: 310
15.2.6 Is it possible for the slant height of a cone to be smaller than the cone’s radius?
15.2.7. How could we have used the fact that the two cones in Problem 15.8 are similar in order to find
the volume of the small cone quickly?

15.2.8 Inthis problem we will develop a formula for the volume of a right circular Vv
frustum, which was described in Problem 15.8. Call the bases 8; and 8» and let
their radii be r; and 12, respectively, with r; < r2. Furthermore, let the distance
between 8, and 82 be h. In Problem 15.8, we formed a frustum by cutting a cone
with a plane parallel to the base. Let’s try running this backwards — recreating a B A
cone by extending the curved surface of the frustum up to a point. Call this point
V, and let AB be a diameter of the small base and YZ be a diameter of a the large Y is,
base as shown.

(a) What do we know about AVAB and AVZY?


(b) Use part (a) to find the distance from V to 8, in terms of h, 11, and 12.
(c) What is the volume of the cone with vertex V and base 8, in terms of h, r1, and rz?
(d) What is the volume of the cone with vertex V and base 8 in terms of h, 11, and rz?
(e) What is the volume of the frustum in terms of h, r;, and rz?

15.3 Spheres

Just as a circle is the set of all points in a plane that are the same distance from a
given point, a sphere is the set of all points in space that are equidistant from a given
point. Most balls and globes are examples of spheres.
Fortunately, we can take a break from the vocabulary bombardment in this chapter. ae)
The only significant length we have in a sphere is the sphere’s radius, which is just
the distance from the center of the sphere to the surface of the sphere. The formulas for the volume
and the surface area of a sphere are very challenging to derive (we won't derive them in this book; see
page 407 for a hint as to how Archimedes did it). Archimedes considered his discovery of the formula
for the volume of a sphere the greatest of all his accomplishments, and even requested a reference to it
be inscribed on his tombstone.

- Volume |

390
15.37 SEHERES

|Problems ii
Problem 15.9: The volume of a sphere is numerically equal to twice its surface area. What is the radius
of the sphere?

Problem 15.10: In this problem, we show that if a plane passes through the
interior of a sphere, then the intersection of the plane and the sphere is a circle. >A
Let O be the center of the sphere, X be the foot of the perpendicular segment
from O to a plane cutting the sphere, and A be a point on the intersection of
the plane and the surface of the sphere (as shown).
(a) What kind of triangle is AOAX?
(b) Find XA in terms of OA and OX.
(c) Why does your answer to part (b) tell you that the intersection of the plane and the sphere is a
circle?

Problem 15.11: A plane cuts through a 26 cm diameter sphere, but the closest it gets to the center is 5
cm. What is the area of the intersection of the sphere and the plane?

Problem 15.12: A sphere is inscribed in a cube, meaning the sphere is tangent to all 6 faces of the
cube. A second cube is then inscribed in the sphere, meaning that all 8 vertices of this second cube are
on the surface of the sphere. Given that the radius of the sphere is 6, we will find the volume of each
cube.
(a) Forget the small cube for a minute. Draw a cross-section of the large cube with the sphere inside
that includes the center of the sphere and the four of the points where the sphere is tangent to
the cube.
(b) Use your cross-section to find the length of an edge of the cube. Find the volume of the cube.
(c) Forget the large cube for a minute. Draw a cross-section of the sphere and small cube that
includes four of the cube’s vertices and the center of the sphere. What part of the small cube
equals the diameter of the sphere?
(d) Use the radius of the sphere to find the length of an edge of the small cube, and then find the
cube’s volume.

We'll start our work with spheres by using the formulas for volume and surface area.

Problem 15.9: The volume of a sphere isnumerically equal to twice its surface area. What is the
radius of the sphere?

Solution for Problem 15.9: We let the radius of the sphere be r and convert the words in the problem to
an equation we can solve: Volume = 2 x (Surface Area).

Extra!
{1111
(11> 1

ee ——————————————————————————————————————————————————

391
CHAPTER 15. CURVED SURFACES

Therefore, we have
ue = 2(4nr’).
Solving this equation gives r = 6. 0
We'll continue our exploration of spheres by considering a cross-section of a sphere.
ofa sphere, then the intersection of
Problem 15.10: Prove that ifa plane passes through the interior
the plane and the sphere is a circle.

ES Solution for Problem 15.10: Intuitively, the cross-section seems to be a circle. We'll
A prove it is a circle the same way we proved similar cross-sections are circles earlier.
Let O be the center of our sphere, r be its radius, and P be our plane. Let X be the
foot of the perpendicular segment from O to P. X is the point we think is the center
of our circle. Let A be a point where the surface of the sphere meets P.
Since AOXA is a right triangle, we have XA = VOA? — OX?. OA is just the radius
of the sphere, so we have XA = Vr* — OX?. Similarly, every point where the surface of the sphere meets
P is the same distance, Vr? — OX?, from X. Hence, the intersection of the plane and the sphere is a circle
in plane P with center X and radius Vr? — OX?. Furthermore, every point in P that is on the circle with
center X and radius XA is on the sphere. (Make sure you see why this is true, and why it is important!)
O

We have now proved two very useful facts about cross-sections of a sphere.

:Important: Every cross-section of a sphere is a circle (or a point, when the cross-
_ section plane is tangent to the sphere). The segment connecting the —
center of the sphere to the center of this circle is perpendicular to the |
plane of the cross-section. |

A cross-section of a sphere that has the center of the sphere as its center is sometimes called a great
circle of the sphere.
Let’s try using our new cross-section knowledge on a problem.

Problem 15.11: A plane cuts through a 26 cm diameter sphere, but the closest it gets to the center is
5. cm. What is the area of the intersection of the sphere and the plane?

Solution for Problem 15.11: In this problem we are simply considering a specific
cross-section of a specific sphere. It’s so close to the last problem that we can Vee A
even use the same diagram. We are given that AO = 26/2 = 13 and OX = 5.
From right triangle AAXO we find AX = 13? — 5? = 12 (maybe you recognized
the 5-12-13 Pythagorean triple). We know that this cross-section is a circle, so its
area is 127m = 1447 cm*. O
Now we'll use cross-sections to examine a problem that includes more than
just spheres.

392
15.5. SPHERES

Problem 15.12: A sphere is inscribed in a cube, meaning the sphere is tangent to all 6 facesof the
cube. A second cube is then inscribed in the sphere, meaning that all 8 vertices of this second cube
are on the surface of the sphere. Given that the radius of the sphere is 6, find the volume of each
cube.

Solution for Problem 15.12: To find the edge length of the larger cube, we consider
a cross-section that includes the center of the sphere and four of the points of
tangency where the sphere touches the cube at the centers of the faces of the cube.
The cross-section of the sphere is a circle. Since the circle includes the center of the
sphere, the radius of the circle equals the radius of the sphere. The cross-section
of the cube is a square whose side lengths equal the edge lengths of the cube.
Drawing radii to the points of tangency at E and F, we see that AEFD is a
rectangle and EF is a diameter of the circle. Hence, AD = EF = 2(OE) = 12, and the volume of the large
cube is 12° = 1728. (Note that we assumed the ‘obvious’ fact that EF passes through O. For an extra
challenge, prove it!)

We have several ways to find the volume of the small cube. We'll start with a
x Y judiciously chosen cross-section. We want to include the center of the sphere since
we know the radius of the sphere. Therefore, we include the center of the sphere
and four vertices of the cube as shown at left. XY and WZ are diagonals of faces of
7 the cube, while WX and YZ are edges of the cube. Therefore W and Y are opposite
vertices of the cube, so that WY is a space diagonal of the cube. Since WY isalsoa
diameter of the sphere, we know that WY = 12. Since WY is a space diagonal of the small cube, the side
length of the small cube is WY/ V3 = 4 V3. So, the volume of the small cube is (4 v3) = 192 y3. 0

Concept Cross-sections including the center of the sphere are often useful in prob-
==> ~+(Iems involving a sphere. Typically, we want to include other important
points in the cross-section as well, such as ae where the sphere is”
tangent to other figures in the problem.

Exercisesig
15.3.1 The volume of a sphere is 367. What is the surface area of the sphere?

15.3.2 A sphere is inscribed in a cylinder, meaning that it is tangent to both bases, eee
and that one great circle of the sphere is along the curved surface of the cylinder.

(a) Find the ratio of the volume of the sphere to the volume of the cylinder. Hints:
354
(b) Find the ratio of the surface area of the sphere to the lateral surface area of the
cylinder.

15.3.3. An ice cream cone has radius 1 inch and height 4 inches. What is the number of inches in the
radius of a sphere of ice cream that has the same volume as the cone? (Source: MATHCOUNTS)

393
CHAPTER 15. CURVED SURFACES

15.3.4 Atthe end of our solution to Problem 15.10, we noted that ‘Every point in P that is on the circle
with center X and radius XA is on the sphere.’ Why is this true and why is our solution incomplete
without this?

15.3.5 A ball was floating in a lake when the lake froze. The ball was removed (without breaking the
ice), leaving a hole 24 cm across and 8 cm deep. Find the radius of the ball. (Source: AHSME)

15.3.6 In this problem we explore facts about tangency and spheres.


(a) Sphere S is tangent to plane P at point T, and the center of sphere S is O. Let X be somepointin
plane P besides T. Consider the solution to Problem 12.14 on page 318, then prove that (Tex
(b) Spheres S; and S> are externally tangent at point T. The centers ofpthe the spheres are O; and Op.
Consider the solution to Problem 10.17 on page 278, then prove that O10: Oz passes through T.
(c) Suppose S; is internally tangent to Sz at T, meaning S; is inside S» and both pass through T. Let
the centers of S; and Sz be O; and Oz, respectively. Must 0103
O; pass through T?
15.3.7x A cube is inscribed in a sphere. Prove that each space diagonal is a diameter of the sphere.
(Write a complete proof without invoking symmetry. Don’t just say, ‘It’s obvious!’ and move on.) Hints:
53, 100, 144

15.4 Problems

The following problems are challenging extensions of the material in this chapter and the previous
chapter. Each problem illustrates useful problem solving techniques for three-dimensional geometry
problems. Rather than write the terms ‘two-dimensional’ and ‘three-dimensional’ repeatedly, we'll
frequently use ‘2-D’ and ‘3-D” instead.

“Problems = As

Problem 15.13: My company produces 6 inch tall cans of paint. The cans have a radius of 15 inches.
(Yes, these are very oddly shaped cans!) We want to offer a super-size can of paint. Toby wants to
leave the can just as wide as it is, but increase the height by x inches. Maryanna thinks the fat cans
sell better, so she wants to make them even wider. She wants to increase the radius by x inches. After
arguing for a while, they decide to use the design that produces the greatest increase in paint volume.
They reach for their scratch paper and figure out how much more paint is needed to fill their new
can designs. They both use the same value of x, and they find that both designs require the same
increase in paint volume! What value of x did they use?

Problem 15.14: The areas of three of the faces of a rectangular prism are 24, 32, and 36. In this problem
we will find the volume of the prism.
(a) Let the dimensions of the prism be x, y, and z. Use the information given to find three equations.
(b) In terms of x, y, and z, what is the volume of the prism?
(c) How can you combine your equations from the first part by adding, subtracting, and/or multi-
plying them to find the volume of the prism?

394
15.4. PROBLEMS

Problem 15.15: Cube STUVWXYZ is shown. Find ZWYT.

Problem 15.16: A ball with radius 6 inches is tangent to the wall and the floor in my house. Given
that the wall is perpendicular to floor, what is the radius of the largest ball I can slide into the space
between the ball and the corner where the wall meets the floor? Hints: 495, 175

Problem 15.17: Three congruent spheres are inside a cylinder such that each sphere is tangent to the
other two spheres, to both bases of the cylinder, and to the curved surface of the cylinder. In this
problem we will find the volume of the cylinder given that the radius of each sphere is 12.
(a) Find the height of the cylinder.
(b) Draw a cross-section of the figures in the problem that contains the centers of the spheres. What
kind of triangle is formed by connecting the centers of the spheres?
(c) Let X be the point where your cross-section intersects the axis of the cylinder. How is X related
to the triangle mentioned in part (b)?
te (d) Find the radius of the cylinder, then the volume of the cylinder.

Problem 15.18: Adam the Ant is at point A of cube ABCDEFGH with edge length 1. He wishes to
walk along the surface of the cube to point G, where G is the vertex of the cube such that AG is a space
diagonal of the cube. What is the length of the shortest path Adam can take?

Problem 15.19: Adam the Ant is now on point A of the cylinder shown. He
wishes to walk along the curved surface of the cylinder to point B, where BC is a C ee eH A
height of the cylinder and C is on the top base such that AC is a diameter as shown.
Given that the cylinder has height 4 and radius 3, what is the length of the shortest
path Adam can take?

Extra! We saw on page 264 that squares, hexagons, and triangles are the only
im im ~regular polygons that will tile the plane. Pentagons, with their quirky
108° angles, simply can’t add up to 360°, no matter how many of them
get together at a vertex. But, on a sphere, pentagons get their due! We
can view the dodecahedron we discovered on page 372 (and shown at
right) as a tiling of a sphere with regular pentagons. ee
Each of the other types of polyhedra can be considered a method of tiling a sphere with
regular polygons. Notice that while there is only one way to tile a plane with equilateral
triangles, there are three ways to tile a sphere with them!

395
CHAPTER 15. CURVED SURFACES

We start off with a couple problems in which we apply algebra to geometry problems. Then we'll
move on to challenging spacial geometry problems.

(Yes, these are very oddly shaped cans!) We want to offer a super-size can of paint. Toby wants to
leave the can just as wide as it is, but increase the height by x inches. Maryanna thinks the fat cans
' sell better, so she wants to make them even wider. She wants to increase the radius by x inches. After
‘arguing for a while, they decide to go with the design that produces the greatest increase in paint
volume.
They reach for their scratch paper and figure out how much more paint is needed to fill their new
can designs. They both use the same value of x, and they find that both designs require the same
increase in paint volume! What value of x did they use?

Solution for Problem 15.13: We are given

Volume(Toby’s design) = Volume(Maryanna’s design),

so we find each volume in terms of x:

m(157)(6 + x) = n(15 + x)7(6).


After a few lines of algebra, we have x(2x — 15) = 0. Since x must be nonzero (otherwise we wouldn’t be
making much of a change to the can design!), we have x = 15/2.
We can quickly check our work by considering the dimensions of the new cylinders. Toby’s has
radius 15 and height 6 + 15/2 = 27/2, so it has volume 7(15)(27/2) = (3° -5*)/2. Maryanna’s has height
6 and radius 15+ 15/2 = 45/2, so it has volume 7(45/2)?(6) = 77(3°-5)/2. (Notice we don’t have to bother
multiplying all those numbers out. Looking at the prime factorizations allows us to quickly verify that
the volumes are the same!)

So, they must have both been using a value of 15/2 = 7.5 for x. O

Concept: For straightforward problems involving the volume and/or surface area
of
== basic solids, usually we just turn the words of the problem into equations
using volume and area formulas. Then we solve the equations, check our
work, and make sure we answer the question that is asked. — :

Problem 15.14: The areas of three of the faces of a rectangular prism are 24, 32, and 36. Find the
volume of the prism.

Solution for Problem 15.14: We start off by assigning variables and writing the information in the problem
as equations. We let the dimensions of the prism be x, y, and z, so the given information about areas of
faces becomes:

xy = 24
WZ 52
yz = 36
15.4. PROBLEMS

We seek the volume, which in terms of x, y, and z is just xyz. We notice from the nicely symmetric
form of the left-hand sides of our equations (xy, yz, zx) that if we ses 34 all three, we'll get x7y7z?.
Specifically, we find that multiplying all three equations gives x?y*z* = (24)(32)(36). Taking the square
root of this yields xyz = (24)(32)(36)= 96 V3, which is the volume of the prism.
Notice that we didn’t even find x, y and z! But now that we have xyz, we can find them quickly.
You'll get a chance to do so as an Exercise. 0

Concept: Sometimes turning a geometry problem into algebra results in a nice sym-
OO metric system of equations, as we saw in Problem 15.14. We often solve.
such symmetric systems by adding orF multiplying allthe‘given equations. |

Now we turn to using our two-dimensional problem solving strategies on three-dimensional prob-
lems.

Problem 15.15: Cube STUVWXYZ is shown. Find ZWYT.

W Z Solution for Problem 15.15: We don’t really know how to handle angles in three di-
mensions, sO we consider a two-dimensional figure that includes our angle. In other
M4 words, we consider the cross-section that contains ZWYT. We form this cross-section
by cutting through vertices W, Y, and T of the cube, as shown. Thus, our cross-section
is a triangle. Moreover, we note that since each of the sides of AWYT is a diagonal of
a face of the cube, the sides of AWYT are all equal. Hence, AWYT is equilateral, so
each of its angles, including ZWYT, is 60°. 5

anne Most 3-D problems are 2-D problems in disguise. Considering cross-
sections that include important pieces of the problem often allows us to
use our two-dimensional strategies to find the solution.

Problem 15.16: A ball with radius 6 inches is tangent to the wall and the floor in my house. Given
that the wall is perpendicular to floor, what is the radius of the largest ball I can slide into the space
between the ball and the corner where the wall meets the floor?

tra! The mathematician Johannes Kepler wondered how densely spheres «can fill space. The
diediadee stacking you see for oranges at the grocery store fills just 74% of space. Is there
a different arrangement that gets more oranges in the same space? It took nearly 400
years before mathematicians Thomas Hales and Samuel Ferguson were able to answer
. Kepler’ s question and prove in 1998 what pare have known all along: there isnfa
= way to pack oranges. : eae

397
CHAPTER 15. CURVED SURFACES

Solution for Problem 15.16: We have a 3-D problem involving spheres


that are tangent to each other and to some planes, so we consider
a cross-section including the centers of the spheres and points of
tangency. Hopefully this allows us to build helpful right triangles.
(Tangents mean right angles!)
O and P are the centers of our spheres, which are tangent at X.
These spheres are tangent to the floor at M and N as shown. Sphere
Ois tangent to the wall at L. Since the spheres are tangent to the wall
and the floor, O and P are equidistant from both. Therefore, both O
and P are on the angle bisector of ZLCM as shown. Since LCM = 7;
90°, we have ZOCM = ZPCN = 45°. So, AOCM is an isosceles C
right triangle. Similarly, APCN is an isosceles right triangle, so
PC = PN V2 = 6 V2, and XC = PC — PX = 6 V2 —6. If we let the radius of our small sphere be r, we now
know that
OC = XC-r=6V2-6-r.
However, from AOCM, we also have

OC = OM V2 =r v2.
Therefore, we have two expressions for OC, which we can set equal:

rV2=6V2-6-r.

Therefore, r +r V2 = 6 V2 -6,s0
_ 6V2-6
atsia/Da
We can rationalize the denominator by multiplying the top and bottom of the fraction by 1 — 2:

ND 6 Ml SN De 218 1242
pe V2-6 | Spe BU mena etry
Tao) ay? -1

Concept: A great many geometry problems are solved by assigning a variable to


a length, then finding two different expressions for some other length in
terms of that variable. We can then set these expressions equal to solve for
the variable.

Problem 15.17: Three congruent spheres are inside a cylinder such that each sphere is tangent to the
other two spheres, to both bases of the cylinder, and to the curved surface of the cylinder. Find the
volume of the cylinder given that the radius of each sphere is 12.

Solution for Problem 15.17: Since each sphere is tangent to the top and bottom base of the cylinder (and
these bases are parallel), the height of the cylinder must equal the diameter of the sphere. Thus, the
height of the cylinder is 2(12) = 24.

398
15.4. PROBLEMS

To get the radius of the cylinder, we start with a well-chosen


cross-section. We have spheres and tangency, so we take the cross-
section that includes the centers of the three spheres, the points R Q
where the spheres touch each other, and the points where spheres
touch the cylinders. Our cross-section is shown at right. AABC
connects the centers of our spheres, and O is where the cross-section wey,
meets the axis of the cylinder. <n
O is equidistant from P, Q and R because the cross-section of \V/
the cylinder is a circle with center O. Since OA, ©B, and @C are
tangent to OO, rays RA, OB, and PC all pass through O. O is
therefore equidistant from A, B, and C because OR = OQ = OP
and AR = BQ = CP. So, O is the circumcenter of AABC. Since
OP = OC + CP, we only need to find OC to find the radius of the
cylinder.
AABC is equilateral because each side equals twice the radius of a sphere. Therefore, O is also the
incenter and the centroid of AABC. Moreover, the points where the spheres’ cross-sections touch each
other are midpoints of the sides of AABC. Hence, AOXC is a 30-60-90 right triangle because OC bisects
ZACB and OX 1 AC (make sure you see why). Therefore,
ee
OC = 20X = = = = =8y3.
36 NS
The radius of the cylinder is then OC + CP = 8 V3 +12. Thus, our volume is:

mrh = 24n(8 V3 + 12)? = 24n(4")(2 V3 + 3)* = (24 - 16)7(21 + 12 V3) = 80647 + 46087 V3.

Concept: To solve 3-D problems involving a sphere, consider cross-se


ing the center of the sphere. If you have multiple spheres in tl
try finding a cross-section that includes multiple centers. If
tangent to anything, try including the point oftangency. _

Sometimes the 2-D problem is more subtly hidden in our 3-D problem. In these cases we need more
than a simple cross-section to coax it out.
Problem 15.18: Adam the Ant is at point A of cube ABCDEFGH with edge length 1. He wishes to
walk along the surface of the cube to point G, where G is the vertex of the cube such that AG is a
space diagonal of the cube. What is the length of the shortest path Adam can take?

Solution for Problem 15.18: We might think that Adam should walk along edge AB A D
and then along face diagonal BG. However, if we put a big glob of jelly at G and put Neds
ants at A, the ants will quickly find a shorter path. They might go diagonally across (e:
face ABFE to a point X on BF, then on to point G. We suspect point X will be the E aN
midpoint of BF, but how can we be sure? (And worse yet, what if ABCDEFGH isn'ta
cube? You'll get a shot at dealing with that complication in the Exercises, of course!) E G

399
CHAPTER 15. CURVED SURFACES

A B G We make our 3-D problem into a 2-D problem by unfolding the cube
and placing the two faces Adam crawls across in the same plane as shown.
This doesn’t change the distance Adam must craw] at all, but it does make
our answer clear. Adam’s shortest path is the straight segment from A to
G. Since AB = BC = CG = 1, we have AG = VCG? + AC? = Y5. Therefore,
le E G_ the shortest path Adam can take has length V5. Notice that since AB = FG
and AC || EG, we can use congruent triangles AABX = AGFX (by ASA) to show that BX = FX, i.e., that
Adams shortest path indeed goes through the midpoint of one of the edges of the cube. 0

Concept: The 2-D problem lurking in a 3-D problem isn’t always just a cross-section. |
Sometimes unfolding or unrolling a 3-D figure reveals a simpler 2-D prob-
lem.

Problem 15.19: Adam the Ant is now on point A of the cylinder shown. He wishes 6
to walk along the curved surface of the cylinder to point B, where BC is a height of C Pee A
the cylinder and C is on the top base such that AC is a diameter as shown. Given
that the cylinder has height 4 and radius 3, what is the length of the shortest path
Adam can take? B

Solution for Problem 15.19: When we found the lateral surface area of a Cr Sonera
cylinder, we discovered how to deal with problems involving the curved pears poe
surface of a cylinder — we unroll the cylinder. We try that here and we get |
the rectangle shown at right. B
BC still equals the height of the cylinder, so BC = 4. However, here AC is not a diameter of the
original cylinder, but rather the distance from A to C along the circumference of the base. Therefore, AC
is half the circumference of the base of our cylinder, or 37. From right triangle AABC we can now find
AB = VAC? + BC? = V16 + 97. (Note that this is not the distance from A to B in space in our original
cylinder. This is the length from A to B along the curved surface of the cylinder. Also, note that if Adam
is willing to walk across a base of the cylinder, there is a shorter path he can take!) O

Exercises

15.4.1 The areas of three of the faces of a rectangular prism are 24, 32, and 36. Find the dimensions of
the prism. Hints: 186
15.4.2 ABCD isa regular tetrahedron. G is the centroid of AABC and H is the centroid of ABCD. Given
AB = 8, find GH. Hints: 198

15.4.3 WXYZis a regular tetrahedron such that WX = 4. M is the midpoint of WX and N is the
midpoint of YZ.
(a) Prove that MN 1 WX.
(b)x Find MN.

BO ees
15.4. PROBLEMS

15.4.4 Three spherical balls are snugly in a row inside a cylindrical can such that the first is tangent
to the lid of the can and the middle ball, the second is tangent to the other two balls, and the third is
tangent to the bottom of the can and the middle ball. Each ball is fits snugly inside the can so that it is
tangent to the curved surface of the can and cannot move side-to-side. Given that the radius of each ball
is 2 inches, what is the volume of the can?

15.4.5 Annie the ant is on vertex A of right rectangular prism ABCDEFGH shown.
Given that AB = 4, BC = 6, and AE = 8, what is the shortest distance Annie can walk to
reach vertex G?

15.4.6 Spheres S and 7 have radii 6 and 8, respectively, and centers O and P, respec-
tively. OP = 10.
(a) Why must the spheres intersect?
(b) Let X be a point that is on both spheres. What is XO? What is XP?
(c) What kind of triangle is AXOP?
(d) Let the altitude from X to OP meet OP at Y. What is XY?
(e) What is OY?
(f) Suppose we pick another point Z that is on both spheres. Must XY = ZY?
(g) Use the previous part to show that the intersection of the two spheres is a circle.
(h) Find the area of this circle.

15.4.7 ABCDEFGH isa cube as shown. M is the midpoint of FG and N is the midpoint
of GH. Given that AB = 6, find the following:

(a) MN.
(b) AM.
(c)* [AMN]. Hints: 227
(d)x the volume of AEMN. Hints: 265

15.4.8 The height of a cylindrical pole is 12 feet and its circumference is 2 feet. A rope is attached to a
point on the circumference at the bottom of the pole. The rope is then wrapped tightly around the pole
four times before it reaches a point on the top directly above the starting point at the bottom. What is
the minimum number of feet in the length of the rope? (Source: MATHCOUNTS)

15.4.9* Two regular square pyramids have all edges 12 cmin length. The pyramids have parallel bases
and parallel edges, and each has a vertex at the center of the other pyramid’s base. What is the total
number of cubic centimeters in the volume of the solid of intersection of the two pyramids? (Source:
MATHCOUNTS) Hints: 318
i
Extra! We’ve seen that regular hexagons can tile a plane, and regular pentagons can tile a
imi ini —sOhere, giving us a dodecahedron. There’s one well-known example of a tiling that uses
both hexagons and pentagons — it’s commonly known as a soccer ball. This fabulous
structure has been around since before the invention of soccer, too, in the form of buck-
minsterfullerene, Cgo, a recently discovered form of carbon. Drs. Richard Smalley and
Robert Curl received the Nobel Prize in 1996 for that discovery.
ne EEUUEEUU EIEE EERE EES
401
CHAPTER 15. CURVED SURFACES

15.5 Summary

Definitions: If a prism has congruent curved figures as bases instead of polygons, we call it a
cylinder. When we talk about a cylinder, we almost always mean a right circular cylinder, in which
the bases are circles. The line connecting the centers of the bases is the axis of the cylinder.

Important: A cylinder with height h and radius r has:

Volume = 7rh
Lateral Surface Area = 2m7rh
Total Surface Area Qnrh + 2nr°

Don’t memorize these formulas! If you take the time to understand


them, they’ll always be obvious to you.

Definitions: A pyramid with a curved base is a cone. At the right is a right circular
| cone, which is usually what we mean when we write ‘cone.’ The point at the tip of
the cone is the vertex. The distance from the vertex to the base is the height and the
line connecting the vertex to the center of the base is the axis of the cone. Ina right
circular cone, the distance from the vertex to a point on the circumference of the base B
| is the slant height. -

_ |Important: Ina cone with radius r, slant height /, and height h, we have:

VY Volume = srr

Lateral Surface Area Trl


Total Surface Area mrl + mr?

|Definition: The set of all points in space that are the same distance froma given point is a sphere.
S | |Important: A sphere of radius r has:

Volume

Surface Area =

Every cross-section of a sphere is a circle (or a point, when the plane ae : . |


is tangent to the sphere). The segment connecting the center of the}
sphere to the center of this circle is perpendicular to the plane of the| —
cross-section. a

402
15.5. SUMMARY

Important: Just as the ratio of the areas of similar two-dimensional figures is the
? square of the ratio of their corresponding sides, the ratio of the surface |
areas of similar three-dimensional figures is the square of the ratio of their.
corresponding side lengths. Moreover, the ratio of the volumes of similar
three-dimensional figures is the cube of the ratio of their Sp eebones gs
side lengths.

Problem Solving Strategies

|Concepts: e Problems involving the curved surface of a cylinder can ¢ often be.
| o== solved by ‘unrolling’ the curved surface into a rectangle.
Similarly, problems involving the curved surface of a cone can often
be solved by ‘unrolling’ the curved surface into a sector.
e Basic area/volume word problems are often no different from other |
| word problems. The key to solving them is to assign variables and |
use the area and volume information to set up equations with the -
given information.
To solve 3-D problems involving a sphere, consider cross-sections ©
including the center of the sphere. If you have multiple spheres
in the problem, try finding a cross-section that includes multiple
centers. If the sphere is tangent to anything, try including the point —
of tangency. |

Most 3-D problems are 2-D problems in disguise. Usually consider- _


ing cross-sections including important pieces of the problem allow
us to use our two-dimensional strategies to find the solution.
The most useful of these 2-D strategies is the Pythagorean Theorem.
Building right triangles is just as powerful in three dimensions as in
two.

The 2-D problem lurking in a 3-D problem isn’t always just a cross-
section. Sometimes we have to manipulate our 3-D figures a little
through unfolding or unrolling to discover our 2-D problem.
A great many geometry problems are solved by assigning a variable
to a length, then finding two different expressions for some other
length in terms of that variable. We can then set these expressions
equal to solve for the variable.

403
CHAPTER 15. CURVED SURFACES

a _

Review Proptems ill


15.20 The number of cubic inches ina sphere’s volume equals the number of square inches in its surface
area. What is the radius of the sphere?
15.21 Acone has slant height 7 and height 5.
(a) Find the volume of the cone.
(b) Find the total surface area of the cone.

15.22 A sphere has diameter 8. Find the surface area and the volume of the sphere.
15.23 A right circular cylinder has radius 3 and height 6.
(a) Find the volume of the cylinder.
(b) Find the lateral surface area of the cylinder.
(c) Find the total surface area of the cylinder.
15.24 A space diagonal of cube C is a diameter of sphere S. The edges of the cube have length 8. Find
the volume of the sphere.
15.25 The radius of cylinder A is three times the radius of cone 8, but the height of cylinder A is half
the height of cone 8. Find the ratio of the volume of A to the volume of 8.
15.26 ABCD is a square inscribed in one base of a cylinder, and EFGH is a square inscribed in the other
base of the cylinder. Given that ABCDEFGH is a cube with side length 9, find the volume of the cylinder.
15.27 A 40° sector of a circle with radius 9 is rolled up to form a cone. Find the volume of the cone.
15.28 The intersection of a plane with sphere G is a circle with area 367. Find the volume of the sphere
if the center of this circle is 8 units from the center of G.

15.29 Two spheres are inside a rectangular box such that each sphere is tangent to five faces of the box
and to the other sphere. Each sphere has radius 4. Find the volume of the box.
15.30 A cross-section of cylinder C is a square in a plane parallel to the axis of C. Given that the area
of the square is 36 square units and that the plane of the cross-section is 4 units from the axis of the
cylinder, find the volume of the cylinder.
15.31 A cone with radius 9 and height 12 is cut in two pieces by a plane parallel to the base of the cone
such that the plane is 8 units from the base of the cone. Find the total surface area of each piece thus
formed.
15.32 Sphere S is tangent to all 12 edges of a cube with edge length 8. Find the volume of the sphere.

Naannnnn nner rereer reeeeeeereeeeeeeeeeseeeereeeeeeeseeeee secre SSS SS

Extra! I don't like that sort of school ... where the bright childish imagination is utterly discouraged ...
mire where I have never seen among the pupils, whether boys or girls, anything but little parrots and
small calculating machines. —Charles Dickens

404
CHALLENGE PROBLEMS

15.33 A sphere is inscribed in a cone as shown at right. The cone has radius 9 and height
12. Find the radius of the sphere.
15.34 In this problem we use volume to find the radius of a sphere inscribed in regular
tetrahedron ABCD with edge length 6. You may want to review Problem 7.14, since we use e&
a similar tactic on this problem. [X=
(a) Find the length of the altitude from D to face ABC of the tetrahedron.
(b) Find the volume of tetrahedron ABCD.
(c) Let the radius of the sphere inscribed in the tetrahedron be r and the center of this sphere be O.
Why is O a distance of r away from each face of the tetrahedron?
(d) In terms of r, what is the volume of tetrahedron OABC? Hints: 402
(e) In terms of r, what are the volumes of tetrahedra OABD, OACD, and OBCD?
(f) Find r.

15.35 The height of right circular cone C equals the cone’s radius in length. Given that A is the vertex
of the cone and XY is a diameter of the cone’s base, prove that ZXAY is a right angle.

Challenge Problems a

15.36 Right circular cone C has vertex V, radius 6, and height 6. Sphere S has center V and is tangent
to the base of cone C.

(a) What is the radius of the sphere?


(b) Let X be a point where S meets the curved surface of C. Find the distance from X to the axis of
the cone. Hints: 11
(c) Show that the intersection of S and the curved surface of C is a circle.
(d) Find the area of the circle from the previous part.

15.37 A cylinder is inscribed in a right regular hexagonal prism such that each base of the cylinder is
a circle that is tangent to all six sides of a base of the prism. Find the ratio of the volume of the cylinder
to the volume of the prism.

15.38 Dennis’s scoop of ice cream has a radius of 2 cm. It rests ina cone that has a radius
of ¥3 cmat the widest part, and the scoop is tangent to each line containing a slant height
of the cone. He eats some of the ice cream and then finds that the remainder of the ice
cream can be pushed down to fill the cone exactly. How many cubic centimeters of ice
cream did he eat? (Source: Mandelbrot) Hints: 177 Ne

Extra! Imagination is more important than knowledge.


ddd la —Albert Einstein

405
CHAPTER 15. CURVED SURFACES

15.39 An edgy spider walks along the edges from A to B of the dodecahedron
formed by the folding up the figure shown at right. What is the number of
edges in the shortest path that spider could take? (Source: MATHCOUNTS) A

15.40 Two parallel planes intersect sphere S, forming two congruent circles.
Show that the center of the sphere is the midpoint of the segment connecting
the centers of the two circles. Hints: 489, 556

15.41 Planes P; and P2 are perpendicular. Their intersection is line m. Plane


P, is 4 units from point O and plane FP? is 6 units from O. A sphere with radius
9 centered at O meets line m at points A and B. Find AB. Hints: 503, 567
15.42 X is on the circumference of one base of a cylinder and Y is on the other base such that XY is
bisected by the axis of the cylinder. Given that the diameter of the cylinder is 8 and XY = 12, find the
volume of the cylinder.
15.43 Arnav the ant is on the outside of a cylindrical glass, halfway up the glass. The glass is 8 inches
tall and has a radius of 2 inches. Arnav wants to walk to a point on the inside of the glass that is
diametrically opposite the point he’s now at (i.e., halfway up the glass, on the inside surface of the glass,
exactly opposite where he is now). What is the shortest distance he can walk? Hints: 218,529
15.44 A spiral staircase with radius 3 feet turns 270° as it rises 10 feet. What is the number of feet in
the length of the outer handrail? (The outer handrail is a curved rail along the entire staircase. It is
everywhere 3 feet from the central axis of the staircase.) (Source: MATHCOUNTS) Hints: 66

15.45 A right circular cylinder with its diameter equal to its height is inscribed in a right circular cone.
The cone has diameter 10 and altitude 12, and the axes of the cylinder and cone coincide. Find the radius
of the cylinder. (Source: AMC 10) Hints: 429
15.46 PQRS isa regular tetrahedron. The distance from the midpoint of PQ to the midpoint of RS is 6
units. Find the volume of PORS. Hints: 565

15.47 The wire frame at left below consists of three mutually perpendicular segments AD, BD, and
CD, each 3 cm in length. A quarter-circle of radius 3 is attached to each pair of segments as shown.
The curved portions of the wire frame will fit snugly against a sphere of a certain size, so that the
entire lengths of all three quarter circles make contact with the sphere. For what radius is this possible?
(Source: Mandelbrot) Hints: 389, 452
A

Vas
C
LAi\
ek See
Figure 15.4: Diagram for Problem 15.47 Figure 15.5: Diagram for Problem 15.48

15.48% One cone of radius 4 and height 12 and another cone of radius 6 and height 12 intersect as
shown at right above, so that the vertex of one coincides with the center of the base of the other. Find
the volume of the intersection of the two cones. Hints: 139, 238

406
CHALLENGE PROBLEMS

15.49

(a) In tetrahedron ABCD, let V be the volume of the tetrahedron and r the radius of the sphere that is
tangent to all four faces of the tetrahedron. Let K,, K2, K3, and K4 denote the areas of faces BCD,
ACD, ABD, and ABC, respectively. Prove that
- AA
Ys K, + Ky + K3 + Kg

Hints: 184

(b) WXYZ isa tetrahedron with XY = YZ = ZX = 9and WX = WY = WZ = 18. Find the radius of the
sphere that is inscribed in tetrahedron WXYZ. (You might remember working with this pyramid
in Problem 14.26.)

15.50* Find a formula for the total surface area of a right circular frustum
with base radii r; and r2 and slant height /. (The slant height of a frustum is CD
the distance from a point on the circumference of one base to the nearest point l
on the circumference of the other base, as shown at right.) Hints: 219, 292

15.51* A ball of radius 1 is in the corner of a room, tangent to two walls


and the floor. A smaller ball is also in the corner, also tangent to both walls,
the floor, and the larger ball. The walls are perpendicular to each other, and they are perpendicular to
the floor. Find the radius of the smaller ball. Hints: 327

Extra! We've already had a glimpse of the brilliance of Archimedes n page 293 in his proof of
‘mvt §the formula for the area of a circle. Although the mathematical and scientific accom-
plishments of Archimedes could fill several books, the feat he allegedly treasured most
was his determination of the volume and surface area of a sphere. Legend has it that
he even asked that his tombstone be inscribed with a sphere inscribed in a cylinder to
commemorate his feat.
Archimedes used the tactics we saw in his area of a circle proof, although with
considerably more complicated figures. He compared a sphere to figures he already
knew how to handle: cones and cylinders. To get a feel for the volume formula, consider
the figures below. Shown is a cone that shares a base with a cylinder such that both
the cone and the cylinder have height equal to the radius of the base. Next to these is a
hemisphere, or half a sphere, with the same radius as the base of the cylinder and cone.

py
gee a Ga
ey es
Consider the cross-sections of each taken at the same height, h, from the base. Letting
the radius of the hemisphere be 1, find the areas of the cross-section of each figure in
terms of r and h. Next, find the area inside the cross-section of the cylinder, but outside
the cross-section of the cone. Notice anything interesting?

407
CHAPTER 16. THE MORE THINGS CHANGE...

Fagnano’s Problem

The mathematical sciences particularly exhibit order, symmetry, and limitation; and these are the greatest forms
of the beautiful. — Aristotle

CHAPTER T 6

eee More Things Change...

A geometric transformation is a rule we apply to a geometric figure that usually results in the figure
being moved or changed in some way. In this chapter we will explore a few basic types of transformations
by sliding, spinning, and flipping figures. As we'll see, these transformations change the locations of
figures to which they are applied, as opposed to turning them into entirely different figures.

16.1 Translations

When we apply a translation to a figure, we simply slide it a specified distance in a given direction. For
example A’ and B’ below are the result of translating A and B, respectively, in the direction and by the
distance suggested by the arrows.

i “Ss
‘SA’
B’

Figure 16.1: A Translation


We use some special terms when performing transformations. We say that A’ is the image of A under
the translation shown above. We can also say that the translation maps A to A’.
Of course, we can translate more than just points. We can translate any figure — a segment, a line,

408
16.1. TRANSLATIONS

a triangle, etc. — and the result will be congruent to our original figure. This is one of those ‘obvious’
things that we won’t worry about proving right now. We’ll look at proving ‘obvious’ facts like these in
the next book in this series, Intermediate Geometry.

rr Problems me

Problem 16.1; A fixed point of a transformation is a point that is its own image. The identity
transformation is the transformation that maps every point to itself. In other words, the identity is the
‘do nothing’ transformation. Can a translation that is not the identity have a fixed point?

Problem 16.2: Sugneauene

(a) Locate point U on the grid shown such that RSTU isa parallelogram. _ aeeSaeeeun
(b) Is the U you found the only point that can be combined with R, S, and
T to form the vertices of a parallelogram? If not, find the others.
ae ie ee BL
BEST eseasan
BESESEa8
HAL EZaS anne

Translations are the simplest transformations, so there aren’t many interesting problems involving
translations. We’ll use these simple transformations to introduce a couple more terms.

Problem 16.1: A fixed point of a transformation is a point that is its own image. The identity
transformation is the transformation that maps every point to itself. In other words, the identity is
the ‘do nothing’ transformation. Can a translation that is not the identity have a fixed point?

Solution for Problem 16.1: Ina translation, every point is moved the same distance in the same direction.
If there is a fixed point, then that point is not moved at all. Since all points must move the same distance
in the same direction, this means none of the points is moved by the translation. Hence, the only
translation that has a fixed point is the identity. 0

oecee Understanding transformations is often more about learning what stays


the same rather than what changes. In the three basic transformations we'll
study in this chapter, everything will stay the same for each transformed
figure except its location and orientation. That may sound like a lot to
change, but it really isn’t much. Line segments remain line segments,
circles remain circles, points that are one unit apart remain one unit apart,
and so on.

Here’s one example of using translations to solve a problem:

—Eric Temple Bell


CHAPTER 16. THE MORE THINGS CHANGE...

JProblem 16.c

(a) Locate point U on the grid shown such that RSTU is a parallelogram.
(b) Is the U you found the only point that can be combined with R, S,and _
T to form the vertices of a parallelogram? If not, find the others.

flee
7 3 EG # ig jOER i re 3 SB

Solution for Problem 16.2:


(a) If RSTU is a parallelogram, then SR ||TU and SR = TU. In other words, the same translation that
maps S to R must map T to U. Since we must go down 5 and left 4 to get from S to R, we must go
down 5 and left 4 to get from T to U. Therefore, U is at the point shown in the diagram below.

(b) In the first part, we ‘completed’ the parallelogram that has /RST as an
angle. We could instead have chosen ZSRT or ZSTR as an angle of the
parallelogram. Let points X and Y, ‘complete’ parallelograms SRTX and
STRY, respectively.
To complete parallelogram SRTX, we note that we must go right 3
and up 2 to get from R to T. Therefore, we go right 3 and up 2 from S to
get to X. Similarly, to complete STRY, we see that we go left 3 and down
2 to get from T to R, so we must go left 3 and down 2 from S to find Y.
Our other two parallelograms are shown in the diagram.

Exercises.

16.1.1 ABCDEF is a regular hexagon. Why must the translation that maps E to C also map F to B?
Hints: 26, 32

Extra! Somebody came up to me after a talk I had given, and said, “You — dei lacie seem hy
‘ut > Fin.” I was inspired to tae “If it isn’t fun, why do it?”
: Ralph oe

410
16.2. ROTATIONS

16.2 Rotations

When we rotate a figure, we spin it by some angle about some point. In other words, A
the image of point A under a rotation of angle 6 counterclockwise about point O is the
point A’ such that OA = OA’ and ZAOA’ = 6, as shownat right. There are two points that _/
satisfy this definition of A’, which is why we must specify the direction of the rotation as | A>0O
clockwise or counterclockwise. Point O is the center of the rotation, and @ is the angle of \
rotation. ;
As with translations, the image of any figure upon rotation is congruent to the original fas
figure. This is one of those ‘obvious’ facts that you might try proving on your own for specific shapes
like line segments or circles.

Sidenote: The contributions of the ancient Greeks to mathematics have been immor-
talized in many ways. One of these is shown in our introduction of rotation.
It’s a common convention to use Greek letters as variables for angle mea-_
sures. Above, we use the letter 0, or ‘theta.’ The letter @ is also often used -
for angles, and sometimes the Greek letters a, 6, and y are usedto stand for
the angles of a generic triangle AABC. Greek letters «aree also fun { write!

f]___Proviems
i
Problem 16.3: Let O be the center of equilateral AXYZ.
(a) Through what angles between 0° and 360° can we rotate Y about Z such
that the result is point X?
(b) Through what angles between 0° and 360° can we rotate X about O such
that the result is point Y? Y
(c) Is there a rotation about Y that maps O to Z?

Problem 16.4: Through how many different positive angles @ less than 360° is it possible to ro-
tate a regular dodecagon clockwise about its center such that its image coincides with the original
dodecagon?

Problem 16.5: After I draw a figure, I rotate it 48° about some point P. If the image of this rotation
coincides exactly with the original figure, is it true that the image of a rotation of the figure 72° about
P must also coincide with the figure?

We'll explore rotations with a few basic problems. As you'll see, rotations are all about angles.

ee —————————————————————————— aE
DE sss

Extra! The more things change, the more they are the same.
TLdla dlindllind —Alphonse Karr
ee EEUU EEIUEEEEE EES Ere
411
CHAPTER 16. THE MORE THINGS CHANGE...

Problem 16.3: Let O be the center of equilateral AXYZ. x


(a) Through what angle(s) between 0° and 360° can we rotate Y about Z such
that the result is point X?
(b) Through what angle(s) between 0° and 360° can we rotate X about O such
that the result is point Y? Y Z
(c) Is there a rotation about Y that maps O to Z?

Solution for Problem 16.3:


(a) Since YZ = XZ and ZYZX = 60°, a 60° rotation clockwise about Z takes Y to X. Of course, a 300°
rotation the other way will also do the job!
(b) Since O is the center of equilateral AXYZ, XO and YO bisect ZZXY and ZZYX, respectively.
Therefore, ZYXO = ZOYX = 30°, so ZXOY = 180° — ZYXO — ZOYX = 120°. Since OX = OY and
ZXOY = 120°, if we rotate X 120° counterclockwise about O, it is mapped to point Y. Note that
we can also rotate X 240° clockwise around O to get to Y.
(c) By thedefinition of rotation, every rotation about Y maps O to a point that is as far from Y as O is.
Since YO and YZ are not equal in length (make sure you see why!), it is impossible for a rotation
about Y to map O to Z.

Problem 16.4: Through how many different positive angles 0 less than 360° is it possible to
rotate a regular dodecagon clockwise about its center such that its image coincides with the original
dodecagon?

Solution for Problem 16.4: Let ABCDEFGHIJKL be our regular dodecagon and LSle |
O its center. Each of the interior angles has measure 180° — 360°/12 = 150°,
and each segment from a vertex to the center bisects an interior angle, so
ZOAB = ZOBA = 75°. Therefore, ZAOB = 30°. So, if we rotate the dodecagon
30°, every vertex will be ‘shifted over’ by one vertex, and the dodecagon will B H
map to itself.
C G
Because a rotation of 30° works, we know a rotation of 30° + 30° = 2(30°) = D EF
60° will work too. A 60° rotation is just the result of two 30° rotations, and E
each of these maps the dodecagon to itself. We could also think of this as shifting each vertex over two
vertices. Similarly, a rotation of each multiple of 30° will map the dodecagon to itself. This gives us 11
positive angle rotations less than 360° about the center that will take the dodecagon to itself.
It is impossible for two different clockwise rotations between 0° and 360° about O to map A to the
same vertex, and we can’t map A to itself with such a rotation. Therefore, we know that the 11 rotations
we've found are the only clockwise rotations between 0° and 360° that map the dodecagon to itself. 0

Problem 16.5: After I draw a figure, I rotate it 48° about some point P. If the image of this rotation
coincides exactly with the original figure, is it true that the image of a rotation of the figure 72° about
P must also coincide with the figure?

a
1673: REFLECTIONS

Solution for Problem 16.5: We know that rotating the figure 48° leaves the figure unchanged, so to
think about what happens when we rotate by 72°, we start by finding what other rotations leave
the figure unchanged. Spinning the figure by 48° twice will leave it unchanged, so a 96° rotation
leaves the figure unchanged. Similarly, other angles of rotation about P that don’t change the figure
are 3(48°) = 144°, 4(48°) = 192°, 240°, 288°, 336°, 384°, 432°, 480°,... Taking a look at those ‘over 360°’
rotations, we see that one of them is 432°. A 432° rotation is equivalent to a 432° — 360° = 72° rotation.
Therefore, a 72° rotation will also coincide with the original figure. 0

|
Exercisesi
16.2.1 ABCDEFGH is a regular octagon, with the vertices labeled counterclockwise.

(a) A clockwise rotation with center B and angle 0 maps A to C. Find @ such that 6 < 360°.
(b) A clockwise rotation with center B and angle ¢ maps H to D. Find ¢ such that @ < 360°.

16.2.2 B’ is the image of B rotated 90° about A. Given AB = 5, find BB’.


16.2.3 Ois the intersection of the diagonals of quadrilateral ABCD. A 90° rotation about O maps ABCD
to itself, meaning each vertex of the image is also a vertex of the original quadrilateral. Must ABCD be
a square? Hints: 43
16.2.4* Regular tetrahedron ABCD is rotated 60° about its altitude from A to face BCD.

(a) What kind of shape is the intersection of the original tetrahedron and its image? Hints: 208
(b)x Find the volume of intersection of the original tetrahedron and its image given that AB = 6.
Hints: 561

16.3 Reflections

We've tried sliding and we’ve tried spinning; now, we'll try flipping. If you’ve looked in a mirror, you
have experienced reflection.

Mathematically speaking, the image when we reflect point A over line m is the point m
A’ such that m is the perpendicular bisector of AA’. In other words, if we folded our 4
paper along line m, A and A’ would coincide.
Of course, we can flip more than just points. In the diagram below, AA’B’C’ is the
image of AABC upon reflection over line k.

413
CHAPTER 16. THE MORE THINGS CHANGE...

If a figure maps to itself under a reflection over a certain line, that line is called a line of symmetry
of the figure. For example, every diameter of a circle is part of a line of symmetry of a circle, and each
diagonal of a square is part of a line of symmetry of the square.

[Proven
Problem 16.6: What are the fixed points of a reflection over a given line m? In other words, what
points are their own images when reflected over line m?

Problem 16.7: Given two different points, X and Y, describe all lines m such that the image of X upon
reflection over m is point Y.

|Problem 16.8: How many different lines of symmetry does a regular hexagon have? |

Problem 16.9: I own a cabin in the woods that is 4 miles directly north of a river that runs east-west. I
am currently out exploring the woods. I am 2 miles north of the river, and 3 miles west of my cabin.
I want to walk to the river, then walk to the cabin. What is the length of the shortest possible route I
can take?

Problem 16.10: My miniature golf course has only two holes, which are shown below.
- (a) On Hole 1, the ball starts at Y, and we want to hit it to X. Find the point on AB at which we
_ should aim to hit the ball to X.
(b) On Hole 2,the ball starts at W, and we want to hit it to Z. There is a barrier between W and Z,
_ as shown. Where should we aim on UT to hit the ball to Z?

One of the best ways to understand any geometric transformation is to analyze what stays the same
when everything else is changing.

|Problem 16.6: What are the fixed points of a reflection over a given line m? ‘

Solution for Problem 16.6: Clearly every point on m maps to itself, since m is the ‘fold.’ To see that there
are no other fixed points, we note that m divides the plane into two pieces. Any point on one side of m
is mapped to a point on the other side. Since a point can’t be on both sides of m, a point not on m can’t
be mapped to itself! Oo
Suppose we are given a point and its image under a reflection. How do we find the line over which

BR ee
16.3. REFLECTIONS

the point was reflected?

Problem 16.7: - Given two different points, ADC oY describe all lines m such that the image of X |
upon reflection over m is point Y.

Solution for Problem 16.7: The image of a point A upon reflection over a line is the point A’ such that the
line is the perpendicular bisector of AA’. Since the image of X in our problem is Y, the line over which
we are reflecting must be the perpendicular bisector of XY. 0
Now that we’ve mastered reflecting points, let’s reflect a more complicated figure.

Solution for Problem 16.8: Intuitively, it seems like the only lines of symmetry
are the six dashed lines shown that cut the regular hexagon in half, but how
can we be sure these are the only ones?
To make sure there aren’t any others, we note that if the reflection maps
the hexagon to itself, then A must be mapped to some vertex of the hexagon.
Furthermore, for any two different vertices, there is only one reflection that will
map the vertices to each other — the reflection over the perpendicular bisector of
the segment connecting the two vertices. Therefore, there’s only one reflection
that could possibly map A to B, and one that maps A to C, etc. Each of these reflections gives us a line of
symmetry. This doesn’t include lines of symmetry through A. AD is the only possible line of symmetry
through A, since it’s the only line through A that leaves two vertices on each side of the line. 0
Now we'll use reflections to solve a couple of problems.

Problem 16.9: I own a cabin in the woods that is 4 miles directly north of a river that runs east-west.
Iam currently out exploring the woods. I am 2 miles north of the river, and 3 miles west of my cabin.
I want to walk to the river, then walk to the cabin. What is the length of the shortest possible route I
can take?

Solution for Problem 16.9: Point M is where we start, the cabin is at C, and YZ is
the river. One approach we can take is to suppose we hit the river at point D,
x miles east of where we start. We can then use the Pythagorean Theorem to
make an expression in terms of x for how far we have to walk. Since YD = x,
we have DZ = 3 - x, so from right triangles AMYD and ACZD, we have

MD + CD = V44+x2 + 16+ (3 - x).

Figuring out the smallest possible value for this expression is going to be very
hard. We need to find a more clever approach.
Since we're stuck, we try thinking of similar, simpler problems that we know how to solve. For
example, if we didn’t have to go to the river, the answer would be easy — we just go from M to C and
we're done. Another simplified problem would be if we were on the other side of the river. Then we
could still just go straight from our starting point to the cabin, and we’d still hit the river. Aha! We can
cross the river.

415
CHAPTER 16. THE MORE THINGS CHANGE...

Suppose we reflect ourselves over the river, to point M’ as shown at right. Since
MX is the mirror image of M’X, we have MX + XC = M’X + XC. Therefore, finding
the shortest total distance from M to the river to C is the same as finding the shortest
total distance from M’ to the river to C. M

Any path from M’ to C hits the river, so clearly our desired shortest distance -
is M’C. Since M is 2 miles north of the river, M’ is 2 miles south of it. So, M’
Since M’ is also 3 miles west of the cabin, Yea y Z
is 2+ 4 = 6 miles south of the cabin. 3
M'C = V364+9=3V5.
21 :
Sey oo 3 'B
Therefore, the shortest path I can take is 3 v5 miles long. O

|Concept: If you can’t solve a problem, try thinking ofa similar, simpler problem and -
= solve that. Then try to use your solution to the simpler problem to solve |
ve harder problem. |

Next time you play miniature golf or shoot pool, you'll likely think of reflections. Here’s why:

Problem 16.10: My miniature golf course has only two holes, as shown below.
(a) On Hole 1, the ball starts at Y, and we want to hit it to X. Find the point on AB at which we
should aim to hit the ball to X.
(b) On Hole 2, the ball starts at W, and we want to hit it to Z. There is a barrier between W and Z,
as shown. Where should we aim on TU to hit the ball to Z?
A U

Solution for Problem 16.10:


(a) For our first hole, we need to bounce the ball off AB so that it goes directly to X. To figure out
where to aim the ball, we note that the path after the ball bounces is the reflection of what the path
would be ifthere were no wall AB and the ball just kept going5 Straight. Therefore, if we aim at the
image of X upon reflection over AB, the ball will bounce off AB and go straight to X.

416
SSSSS S
sss sss SS
16.3.
tS
REFLECTIONS
SSS SS

So, to find the point on AB to aim at, we reflect X over AB to get its image X’. The intersection
of YX’ and AB is the point we should target.
(b) On Hole 2, the barrier not only prevents us from hitting the ball straight from W to Z, but it also
prevents us from just bouncing the ball off TU and having it go straight to Z. We'll have to go for
a double-bank shot. Therefore, we need it to bounce off TU, then hit the appropriate point on the
top wall so that it continues to point Z.
We still don’t know where to aim along TU, but we can figure out where we
want the ball heading after it banks off of TU. After coming off TU, the ball
should be heading towards the image of Z upon reflection over the top wall.
We now know where we want the ball heading once it comes off TU, but we
still don’t know where to aim along TU. Since we need the ball to go from W
and bounce off TU so that it is heading straight towards Z’, we think to try our
reflection tactic on W as well.
Point W’ is the image of W upon reflection over TU. If we were at W’
and there were no wall TU, we would aim directly at Z’. The path W/Z’
meets TU at P. Since W’P is the image of WP, if we aim the ball at P, it
will bounce off TU, and head towards Z’. Since Z’ is the image of Z upon
reflection over the top wall, a ball heading from P towards Z’ will bounce W’s
off the top wall and head straight to Z as desired.
Have fun beating your friends next time you play miniature golf!
O

Hi Exercises | >

16.3.1 How many lines of symmetry does each of the following figures have?

(a) A rectangle that is not a square.


(b) A rhombus that is not a rectangle.
(c) A parallelogram that is not a rectangle or a rhombus.
(d) A regular pentagon.
(e) A regular dodecagon.
(f) A regular polygon with n sides.

16.3.2 Square ABCD is reflected over CD. A’ and B’ are the images of A and B, respectively. Given
CD = 4, find A’A and. A’B.

16.3.3 Lines k and £ are not parallel, and line m is the image of line k upon reflection over €. Show that
¢ bisects a pair of the angles formed by line m and k.
16.3.4 Given intersecting lines £ and m and point X not on either line, let Y be the image of X upon
reflection over ¢ and let Z be the image of X upon reflection over m.

(a) Is it possible for the reflection of Y over m and the reflection of Z over € to be the same point?
(b) Must the reflection of Y over m and the reflection of Z over € be the same point?

COAa
CHAPTER 16. THE MORE THINGS CHANGE...

16.3.5* Wecan define reflections in space through a plane similar to the way we define reflections over
a line. Specifically, given a point K and a plane P, then the image of K upon reflection through FP is the
point K’ such that P is perpendicular to KK’ and bisects KK’. A plane of symmetry of a figure is a plane
such that the image upon reflecting the figure through the plane is the figure itself. For example, any
plane through the center of a sphere is a plane of symmetry of the sphere.

(a) How many planes of symmetry does a right square pyramid have?
(b) How many planes of symmetry does a right rectangular prism have if the dimensions of the prism
are all different?
(c) How many planes of symmetry does a regular tetrahedron have? Hints: 154
(d) How many planes of symmetry does a cube have? Hints: 63
(e) Show that any plane of symmetry of a right circular cone must include the axis of the cone. Hints:
115

16.4 Dilation

The last transformation we will explore is dilation. In plain English,


a dilation results from stretching or shrinking a figure.
At right is an example of dilation. Point OO is the center of dilation.
The image of point A is the point A’ on OA such that OA’/OA= 2.
Similarly, the image of point B is the point B’ on(OB such that OB’ /OB=
2, and the image of point C is the point C’ on OC such that OC’/OC= 2.
The image of the center of dilation is itself. We can think of the dilation
as ‘stretching’ AABC away from O.
In general, the image of a point P upon dilation with scale factor k and center O is the point P’ on
OP such that OP’= k(OP) (when k is positive). In our example above, the scale factor is 2.

The scale factor need not be positive. At right we have an example "44
in which the scale factor is —2. Here, the image of point X upon dilation os xX
about O is the point X’ on the ray starting from O but going in the opposite ae *
direction from X such that OX’/OX = 2. In other words, point X’ is the Z’ <eS ee -ere pe
point such that OX’/OX = 2 and O is on XX’. x ee
pets ¥
RY

Problems |

Problem 16.11: Triangle A’B’C’ is theiimage of a dilation of triangle ABC about the centroid oftrie ngle |
ABC. The dilation has a positive scale factor. :
(a) Suppose AA’B’C’ is entirely outside AABC. Is the scale factor greater than 1 or ickythe 1
(b) Suppose AA’B’C’ is entirely inside AABC. Is the scale factor greater than 1 or less than: .
(c) What happens if the scale factor equals 1?

ic.
SSSSS
16.4. DILATION
SSS SD

Problem 16.12: Square A’B’C’D’ is the image of a dilation of square ABCD about A with scale factor
3. Suppose AB = 2.
(a) Find AA’, AB’, and BB’.
(b) Find BC’ and CD’.

Problem 16.13: Let A’ and B’ be the images of points A and B, respectively, under a dilation with
center O and positive scale factor k.
(a) Show that AAOB ~ AA’O’B’.
(b) Show that AB ||A’B’ and A’B’/AB = k.

Problem 16.11: Triangle A’B’C’ is the image of a dilationof AABC about the centroid of triangle
AABC.
(a) Suppose AA’B’C’ is entirely outside AABC. Is the scale factor greater than 1 or less than 1?
(b) Suppose AA’B’C’ is entirely inside AABC. Is the scale factor greater than 1 or less than 1?
(c) What happens if the scale factor equals 1?

Solution for Problem 16.11: Let G be the centroid of AABC. The centroid of a triangle is always inside the
triangle, so G is inside AABC.

(a) Because A’ is the image of A upon dilation about G, point A’ is on GA. If A’ is outside the triangle
on this ray, then it is beyond A on GA. So, we must have GA’ > GA, which means GA’/GA > 1.
Therefore, the scale factor is greater than 1. An example of this case is shown at left below.

Figure 16.2: Scale Factor Greater Than 1 Figure 16.3: Scale Factor Less Than 1

(b) As before, we know that A’ is on GA . However, this time, A’ is inside AABC, so we have GA’ < GA.
Therefore, we have GA’/GA < 1,s0 the scale factor is less than 1. An example of this case is shown
at right above.
(c) Ifthe scale factor equals 1, then the image of AABC is itself.

ir
CHAPTER 16. THE MORE THINGS CHANGE...

Important: If the scale factor of a dilation is greater than 1, then the dilation of
V a figure ‘stretches’ the figure. The resulting image is larger than the
original figure. If the scale factor is positive but less than 1, then the
dilation is a ‘shrinking’ of the original figure, which gives an image that
is smaller than the original figure. A dilation of a figure with scale factor
equal to 1 leaves the figure unchanged.

‘Problem 16.12: Square A’B’C’D’ is the image of a dilation of square ABCD about A with scale factor
3. Suppose AB = 2.
(a) Find AA’ AB and BB’.
(b) Find BC’ and CD’.

Solution
for Problem 16.12:
(a) At right, we have the original square ABCD and its image under the
dilation with scale factor 3 and center A. The image of A upon a dilation
about A is just itself. (The image of the center of a dilation is always p Cc
the center itself.) Therefore, we have AA’ = 0. (Yeah, that was a bit of a
trick question!)
Because B’ is the image of B upon a dilation about A with scale factor
3, we have AB’/AB = 3. In other words, B’ is 3 times as far from A as_ DP’ C’
B is. Since AB = 2, we have AB’ = 6. Points A, B, and B’ are collinear
because B’ is the image of B upon the dilation about A. So, we have AB + BB’ = AB’. From this,
we find that BB’ = AB’ — AB = 4.
(b) We seek two lengths and we have lots of right angles, so we look for ASA TB B
right triangles on which we can use the Pythagorean Theorem. In the
previous part, we found that AB’ = 6, so square AB’C’D’ has sidelength D C
6. We also found that BB’ = 4, so right triangle BB’C’ gives us

BC’ = \/(BB’)? + (CC’)2 = V16 + 36 = 2 V13.

In the same way we found BB’ = 4 in part (a), we can find that
DD’ = 4. Right triangle CDD’ then gives us

CD’ = VCD? + D’D2 = V¥4+16 =2%5.

This problem raises an interesting question about dilations. Points B’ and C’ are the images of B and
C under the dilation, and we see that B’C’ ||BC, and B’C’/BC equals the scale factor. Note that we didn’t
prove these facts; we were told to assume that the image of ABCD upon dilation is a square. Let’s see if
we can prove that these observations are not a coincidence.
Problem 16.13: Let A’ and B’ be the images of points A and B, respectively, under a dilation with
center O and positive scale factor k. Show that AB ||A’B’ and A’B’/AB = k.

@20
°C ee ea ee
16.4. DILATION

Solution for Problem 16.13: We want to prove something about a ratio, and we A’
want to show that two lines are parallel. The dilation also gives us information A
about ratios. This looks like a job for similar triangles. Our diagram at right
shows the set-up where k > 1. (The case k < 1 can be solved with the same steps O
we will use for k > 1.)

We're looking for similar triangles, and we need to find some way to use the Y
fact that A’ and B’ are the images of A and B under the same dilation about O. The B
dilation information tells us that OA passes through A’ and OB passes through B’, as shown. But that’s
not enough! The dilation also gives us information about ratios. Specifically, it tells us that OA’/OA and
OB’/OB both equal the scale factor, so
OA OBS -
OA OB
Since we also have ZAOB = ZA’OB’, we have AAOB ~ AA‘OB’ by SAS Similarity. From this triangle
similarity, we have A’B’/AB = OA’/OA = k. Wealso have ZOAB = ZOA'B’, which tells us that
AB ||A’B’.
a

Notice that we did not prove in Problem 16.13 that the image of AB is A’B’. We only proved that
A’B’/AB = k and AB ||A’B’. You'll have a chance to prove that the image of a segment under dilation is
a segment as a challenging Exercise. However, the parallel lines suggest something very useful about
dilations.

Important: A figure and its image upon dilation are similar. The ratio of corre-
sponding sides of the figure and its image equals the scale factor of the
dilation.

You'll have a chance to prove this for triangles as an Exercise. This is just the beginning of interesting
dilation properties. In Art of Problem Solving’s Intermediate Geometry, we will build on these basics to
find powerful and intriguing applications of dilation.

(RRR
16.4.1 In AXYZ, we have XY = 3, YZ = 4, and XZ = 5. Suppose AX’Y’Z’ is the image of AXYZ under
a dilation with scale factor 4.

(a) What is X’Y’?


(b) What is the area of AX’Y’Z’?
(c) Is it possible to determine XX’ with the information given in the problem?

16.4.2 Point Q is the image of point P under a dilation with center O and scale factor 5. If PQ = 20,
what is OP?
16.4.3 Show that if A’, B’, and C’ are the images of A, B, and C, respectively, under a dilation with
center O and scale factor k, then AABC ~ AA‘B’C’. (You can assume k is positive.) Hints: 6

a. hCCCCCOCCC~C~CCS
CHAPTER 16. THE MORE THINGS CHANGE...

16.4.4x In this problem, we show that the image of a segment upon a dilation is itself a segment.
Suppose that A’ and B’ are the images of A and B, respectively, upon dilation about O with scale factor
k. (You can assume k > 0; the proof for k < 0 is very similar.)

(a) Let X bea point on AB, and let X’ be the image of X under the dilation. Show that LOXA = LOX'A’
and ZOXB = ZOX’B’.
(b) Use part (a) to show that ZA’X’B’ = 180°. Why does this mean that the image of each point on AB
is a point on A’B’?
(c) Show that each point on A’B’ is the image of some point on AB under the dilation. Why is this
step necessary to complete the proof?

16.5 Changing the Question

Sometimes problems can be quickly solved by rearranging the problem or by considering special cases
of the problem. Typically this requires a little creativity and flexibility.

Problems

apee
é2 dtheRGpoRE:ofee sides of the stalles square are then
a What iis
aoe number of square inches in the area of the shaded

i In the
hefigure, ABCD
A iisa square with side length L and M and N- AL
points of AB and CD, respectively. Find the area of the shadedie a
oe . - :
16.5. CHANGING THE QUESTION

Problem 16.17: In the diagram we have two concentric circles, and chord AB of
the large circle is tangent to the smaller circle. Given that AB = 8, find the area of
the region between the two circles.

Problem 16.18: In the diagram, ZINT = 80°. The angle bisectors of two of the exterior
angles of ATIN meet at point K, as shown. Compute ZIKT.

Since rotations, reflections, and translations don’t alter lengths or areas, we can often use them to
simplify Ripe, Here are several examples.

Problem 16.14: The side length of the largest square at right is 10 inches. ‘The
midpoints of the sides of the largest square are connected to form a smaller square.
Opposite vertices and the midpoints of opposite sides of the smaller square are then
connected as shown. What is the number of square inches in the area of the shaded
region? (Source: MATHCOUNTS)

Solution for Problem 16.14: Solution 1: Each of the shaded regions is a 45-45-90 triangle, so we can find
the area of each by finding one of the legs. One leg of each triangle is half a diagonal of one of the 5 x 5
quarters of the largest square. Therefore, each triangle has a leg of length 5 V2/2, so each triangle has an
area of (5 ¥2/ 2)?/2 = 25/4. There are four such triangles, so the total shaded area is 25.

Solution 2: Instead of finding the leg lengths of each little triangle to find the area, we could have
instead noted that each of these little triangles is 1/4 of one of the 5 x 5 squares. Hence, each has area
25/4, so the total shaded area is 4(25/4) = 25

Solution 3: Since the area of a triangle doesn’t change if we slide,


spin, or flip it, we can manipulate the shaded pieces to make a region
whose area we know how to find easily. We first slide the bottom
triangles up to complete a large shaded isosceles right triangle. Then,
we can slide half of this triangle to the right to complete one of the
5 x 5 squares as shown. Clearly, then, the area of the shaded region is 5* = 25. 0

423
CHAPTER 16. THE MORE THINGS CHANGE...

Problem 16.15: A circle is inscribed in a large square and circumscribed about A


a smaller square. The area of the larger square is 8 square meters. What is the ee NN
‘number of square meters in the area of the smaller square?

——
D C

Solution for Problem 16.15: As with the last problem, we can work through several steps to the answer,
or we can make a little transformation to simplify the problem.
Solution 1: Since our large square has area 8, each side has length V8 = 2-2. Because the circle is
tangent to all four sides of the large square, the diameter of the circle equals the side length of the large
square. But the diagonal of the small square is also a diameter of the circle, so the diagonal of the small
square has length 2 V2. Therefore, its sides are each (2 V2)/V2 = 2 and its area is 4.

Solution 2: We can spin the small square without changing its area, so we spin it A mM B
so that its vertices are the points where the circle is tangent to the large square. When VN
we draw the diagonals of the small square MNOP, we divide the large square into four ie N
equal pieces. Half of each of these pieces is inside MNOP, so [MNOP] = [ABCD]/2 = 4. W
IB D O C

Problem 16.16: In the figure, ABCD is a square with side length 1,and Mand Nare A M
the midpoints of AB and CD, respectively. Find the area of the shaded region. (Source:
Mandelbrot)

Solution for Problem 16.16: MBYX is a trapezoid (why?), but it’s not clear how we'll find the height or
either of the bases. It’s also not clear how we could chop MBYX into easy-to-handle pieces. However,
it looks like MBYX is congruent to NDXY. We don’t have any tools for proving quadrilaterals are
congruent like we do for triangles. But if we can transform one quadrilateral into the other with flips,
spins, or slides, then we'll know they are congruent.
We might think to reflect AABC over AC, but that won’t map M to N. However, if we rotate AABC
180° about the center of the square, we'll have the transformation we want. (You'll prove this as an
Exercise.) This rotation maps MBYX to NDXY, showing that these quadrilaterals are indeed congruent.
Therefore, the area of MBYX is half the area of MBND. MBND is a parallelogram with base ND = 1/2
and height BC = 1, so its area is 1/2. Therefore, the shaded region has area (1/2)/2 = 1/4. O

Transformations aren’t the only ways we can manipulate diagrams to help us with problems.

Problem 16.17: In the diagram we have two concentric circles, and chord AB of the
large circle is tangent to the smaller circle. Given that AB = 8, find the area of the
region between the two circles.

424
Eas
aan ems a RR
16.5. CHANGING THE QUESTION

Solution for Problem 16.17: The problem doesn’t tell us what the radius of either the
larger circle or the smaller circle is. We only know the length of that one tangent
chord. Therefore, the problem suggests that the shaded region between the circles
has the same area no matter what these two radii are, as long as that tangent chord
has length 8. We can get a quick answer for what that area is by picking a special
case that’s easy to work with. The simplest is to shrink the smaller circle down to
a point. Then, our tangent chord becomes a diameter of the larger circle, and our shaded region is this
whole circle. The area of this region, then, is just (8/2)?7 = 167.

Notice that we have not proved that the shaded region is always 167 when our tangent chord is 8
units long. We’ve only found it for one case. However, if the area is always the same, we know it’s 167.
To prove that it’s always the same, we have to find some way to get information about the radii of the
two circles. We have a tangent line, so naturally we build a right triangle by drawing radii as shown
below.

Since OC 1 AB, OC bisects chord AB. Therefore, BC = 4. From right triangle AOBC, A ‘eis B
we have OB? — OC? = BC? = 16. Since OB and OC are the radii of our circles, the area ae
we seek is (OB?)n — (OC?)n = (OB* — OC*)n = 167. O Ge

ee - When working on a problem with a variable set-up, consider the extreme |


se Beit

WARNING! Before you start examining those extremes, make sure you really can _
*< vary the set-up. If you vary the set-up in a way that violates infor- |
mation given in the problem, then your conclusions don’t necessarily _
apply to the original problem! For example, if we are asked to find the —
area of a triangle with sides of length 13, 14, and 15, we can’t simply |
pretend tw
two0 ofthe sides aare
re perpendicular tto
0 get the answers |

Extra! In an acute triangle, the triangle formed by connecting the feet of the altitudes is the
“0 triangle with smallest perimeter that can be formed by connecting a point on each side of
the triangle. This was originally proved by Fagnano using calculus, but mathematician
_ HA Schwarz came up with the amazing proof without words offered below, using the
_property of reflection repeatedly. See if you can figure out how this proof works! os

425
CHAPTER 16. THE MORE THINGS CHANGE...

Let’s try this technique on one more problem.

Problem 16.18: In the diagram, ZINT = 80°. The angle bisectors of two of the Aj]
exterior angles of ATIN meet at point K, as shown. Compute ZIKT.

Solution for Problem 16.18: Since ZITN can vary, we'll call it x. From AINT, we find that ZTIN=
180° =.80° —x = 100° — x. Therefore, ZAM = 180" —ZTIN.= 80; +4 andJ1 5 — 180" 72. onice IK and
TK bisect these angles, we have ZKIT= 40° + x/2 and ZKTI= 90° — x/2. Finally, we can use AKIT:

LK + ZKIT + ZKTI = 180°.

Substitution gives ZK + (40° + x/2) + (90° — x/2) = 180°, from which we find ZK = 50°.

Notice that in this problem, we can place points I and T anywhere on the sides of ZANB without
violating the problem. We only need ZINT = 80°. Therefore, to quickly check the answer we found
above, we can consider the extreme case of putting J at N.

When I is at N, ZAIT is ZN and IK is the bisector of ZN. Therefore, ZKIT = A


80° /2 = 40°. Furthermore, ZITB is a straight angle, so its bisector is perpendicular to
IT. Since AKIT is a right triangle, we then have ZIKT = 90° — 40° = 50°. Notice that
this is not a proof that ZIKT is always 50° — it’s just a quick way to check the answer K
we found earlier. O Pen

y i
isaagre at wa<
si
eri ng ex tr em es 1
in pro blems with varying set-up
Pad | Consid nea Se
oo ye answer. Boe ee

aveEZ
Considering extremes does not constitute a proof! You stillha
~WARNING!!
in allcases. Za
os : _show your answer holds i

Exercises |

16.5.1 The rectangle at right is inscribed in the circle, and a rhombus is inscribed in
the rectangle by connecting the midpoints of the sides of the rectangle as shown. If the
radius of the circle is 4 ft, how many feet are in the perimeter of the rhombus? (Source:
MATHCOUNTS)

16.5.2 In the diagram, D is the midpoint of EC, and the area of AEDFis4 E D &
square centimeters. What is the number of square centimeters in the area of
rectangle ABCD? (Source: MATHCOUNTS)

A B
426
16.6. CONSTRUCTION: TRANSFORMATIONS

16.5.3. Three circular arcs off radius 5 units bound the region shown. Arcs AB and e
AD are quarter-circles, and BCD is a semicircle. What is the area, in square units, of
the region?
16.5.4 In Problem 16.16, we asserted that DNYX is the image of BMXY upon rotation
about the center of the square. Complete our solution by proving this fact. Hints: A
419
16.5.5x In trapezoid ABCD, AB and CD are perpendicular to AD, with AB + CD = BC and AD = 7.
What is AB - CD? (Source: AMC 10) Hints: 497

16.6 Construction: Transformations

As a thorough test of your spinning and flipping abilities, you'll now have the chance to spin and flip
some points given only a ruler and compass.

J
Proviems
te
Problem 16.19: Given are points C and T. Construct point U such that U is the image of
point T under a 30° rotation about C.

Problem 16.20: Given is point A and line m. Construct point B such that B is the
image of point A upon reflection over m.

Problem 16.21: Given are parallel lines m and n and point P between the
lines. Construct an equilateral triangle such that one vertex is P, one vertex
is on m, and one vertex is on n.

Let’s go for a little spin.

Problem 16.19: Given are points C and T. Construct point U such that U is the image of point T
‘under a 30° rotation about C.

Solution for Problem 16.19: Since U is the image of T rotated 30° about C, we
must have CT = CU and ZTCU = 30°. Since CT = CU, U must be just as far
from C as T is. Therefore, U must be on the circle with center C and radius
CT. So, we draw that circle. Since ZTCU = 30°, U must be on a ray through
C that forms a 30° angle with CT. We construct this ray by first constructing
an equilateral triangle, ACTX, then bisecting ZC of this triangle.

427
CHAPTER 16. THE MORE THINGS CHANGE...

The intersection of this angle bisector and our circle is the desired point U such that CT = CU and
ZTCU = 30°. Therefore, U is the image of T under a 30° rotation about C. O

Concept: In construction problems in which we must locate a single point, our


== ~ general strategy is to find two figures, usually circles or lines, which must
contain the point. The intersection of these two figures is the point we
want.

You'll flip over this one...

Problem 16.20: Given is point A and line m. Construct point B such that B is the image of point A
upon reflection over m.

Solution for Problem 16.20: Since B is the image of point A upon reflection over m,
we must find the point B such that m is the perpendicular bisector of AB. Since
AB 1 m, B must be on the line through A perpendicular to m (see Problem 6.24 for
this construction). Let this perpendicular line meet m at M. Since m bisects AB, we
must have AM = BM. Therefore, B is on the circle with center M and radius AM.
Where this circle meets our earlier line through A is the desired point B such that m
is the perpendicular bisector of AB. 0
Problem 16.21: Given are parallel lines m and n and point P
between the lines. Construct an equilateral triangle such that
one vertex is P, one vertex is on m, and one vertex is on n.

Solution for Problem 16.21: We seek point M on m and point N on n


such that APMN is equilateral. If we can find either M or N, we can
easily construct our equilateral triangle, since once we have a side of
the triangle, we can make the whole thing. Therefore, we’ll focus on
finding N.
We already have one figure that contains N — line n. Therefore,
we only need to find one more line or circle that contains N. Point P
and line n are not enough to find N; we need more. The only other
piece of information we have is that M is on m, so we need to use m
to find N.

Since APNM must be equilateral, we know that a 60° rotation B’


about P maps M to N. But we don’t know where M is! However, we
do know that M is on m, so the image of M upon rotating 60° about
P must be on the image of m when m is rotated 60° about P. Since this image of point M is point N, all
we have to do is rotate m 60° about point P to have another line that contains N!

aeTN
SSS
SS SS
16.7. SUMMARY
SS

To rotate m 60° about P, we simply pick two points on m and rotate them both. Connecting the
images of these points upon rotation gives us m’, the image of m upon 60° rotation about P. Since N
is on both m’ and n, where these lines meet gives us N! Now we can easily use PN to construct the
equilateral triangle. 0
One key step in finding our solution was thinking about what information we hadn’t used yet,
thereby focusing on using line m to find N. When we focused on how to use m to get N, we had to think
about how to relate M to N given point P. This led us to a new problem solving tactic:

Concept: When stuck on a problem involving equilateral triangles, think about 60°—|
Os rotations!

I)
_Exercises
16.6.1 Given points A, A’, and Q, construct point Q’ such that Q’ is the image of Q under the same
translation that maps A to A’.
16.6.2 Given a point P and two intersecting lines, and m, construct the image of P upon reflection
over €. Let P’ be the image of P upon reflection over €. Construct the image of P’ upon reflection over
m. Try this for a variety of points (reflecting the point over @, then reflecting its image over m). Notice
anything interesting?

16.7 Summary

Definitions: In this chapter we explored three important geometric transformations. A figure’s


image under a transformation is the result of applying the transformation to that figure. We also
say that a transformation maps a figure to its image. A fixed point of a transformation is a point
that is its own image. Four important transformations are described below.

e When we slide a figure a given amount ina specified direction, we perform a translation.
e When we apply a rotation to a figure, we spin it by some angle about some Pt The point
is the center of rotation.
i flipped over |
e The image of a figure upon reflection over a line is what results when the figure is
the line. If a figure is its own image upon reflection over a line, the line is a line of symmetry |
of the ABUT
i.Ecaeoy speaking,when we apply a dilation about a point v to ' figurewe stretchthefi 2
:shrink it towards a :

| ital oto and reflections preserve length, area, and le which means ‘that‘the
| lengths, ae andeee ina toe s image equal the corresponding Cue geiniy ae.

429
CHAPTER 16. THE MORE THINGS CHANGE...
I a

Problem soy: passes


yg ie cae Pees
“Concepts: e Understanding transformations iis often more about learning what
|
O== stays the same rather than what changes. Whenever you are learning ©
about a new transformation, focus on what stays the same. |
|

e If youcan’tsolvea problem, try thinking of a similar, simpler problem —


and solve that. Then try to use your solution to the simpler problem
to solve the harder problem.
In addition to our strategy of chopping areas into easy-to-handle
pieces, we can also sometimes rearrange them to form an easy-to-
handle whole.
e When working on a problem with a variable set-up, consider the
extreme possibilities. Considering extremes is also a great way to
check your answer. |
e Inconstruction problems in which we must locate a single point, our —
general strategy is to find two figures, usually circles or lines, which —
must contain the point. The intersection of these two figures is the |
point we want. |
|
e When stuck ona problem involving equilateral triangles, think about |
60° rotations! |

Things To Watch Out For!


WARNING!! e Before you start examining extremes in a problem, make sure
0.0
4 you really can vary the set-up. If you vary the set-up in a
way that violates information given in the problem, then your
conclusions don’t necessarily apply to the original problem!

e Considering extremes does not constitute a proof! You still have


to show your answer holds in all cases.

Extra! Suppose we have a circle OO with radius r. The geometric ttransformation called in-
we version maps each point P other than O to the point P’ on OP such that OP -OP’= 7.
To make inversion work, we define the point at infinity to be the image of the center
of inversion, O. Experiment with inversion yourself by finding out what happens to
various points upon inversion. What happens to points inside the circle? Points outside
the circle? Points on the circle? After playing with inversion for a while, turn to page 434
for more suggestions for exploring inversion.
Rn ee ee ee ee eee
430
a ca i REVIEW PROBLEMS
ad

| REVIEW PROBLEMS a?

16.22 ABCDEFGH is a regular octagon. A translation maps G to D. To what point does this same
translation map H?
16.23 OPQRSTUVWXYZ is a regular dodecagon.

(a) A rotation of 0; degrees about U maps V to T. Given 6; < 180°, find 6}.
(b) A rotation of 02 degrees about U maps X to R. Given 62 < 180°, find @.
(c) A rotation of 63 degrees about U maps W to S. Given 63 < 180°, find 63.
(d) A rotation of 64 degrees about U maps Y to Q. Given 04 < 180°, find 04.
(e) A rotation of 65 degrees about U maps Z to P. Given 05 < 180°, find 6s.

16.24 All of the angles of octagon GHIJKLMN are 135°. GH = IJ = KL = MN = 1 and HI = JK = LM=
GN = 2. How many lines of symmetry does GHIJKLMN have?
16.25

(a) GH and Glare line segments such that GH = GI. Must there exist a reflection that maps GH to GI?
(b) XY and AB are line segments such that XY = AB. Must there exist a reflection that maps XY to
AB?
16.26 ABCD isa square. M is the midpoint of CD.

(a) Is there a rotation about M that maps A to B?


(b) Is there a rotation about M that maps A to C?
(c) Is there a reflection over some line that maps M to A?

16.27 In the figure at right, the shaded region is formed by drawing two paral-
lel segments that connect the midpoints of congruent squares. Each square has
side length 1 centimeter. What is the area of the shaded region? (Source: MATH-
COUNTS)
16.28 The image upon reflection of regular hexagon QWERTY over WE is regular
hexagon Q’W’E’R’T’Y’.

(a) Why must W’ and E’ be W and E, respectively?


(b) Find ZQWQ’.
(c)x Find YT’ if QW = 6. Hints: 435
16.29 Line ¢ is a line of symmetry of AABC.
(a) Must line ¢ pass through a vertex of AABC? Why or why not?
(b) Is it possible for AABC to be scalene (i.e., all sides have different lengths)? Why or why not?

431
CHAPTER 16. THE MORE THINGS CHANGE...

16.30

(a) Point R is the image of point S under a dilation with center T and scale factor 3. If TR = 18, what
ike ros
(b) Point E is the image of point D under a dilation with center O and scale factor —3. If ED = 9, what
is OD?
(c) Point X is the image of point Y under a dilation with center A and scale factor 0.25. ibe touts ty
what is AY?
16.31 The image of circle OO under a dilation about P with scale factor 4 is OO’. The area of ©O’ is
487. What is the area of OO?

16.32 The rules for a race require that all runners start at A, touch B
any part of the 1200-meter wall, and stop at B. What is the number of A
meters in the minimum distance a participant must run? 300 m
16.33 Lines f and m are perpendicular at point X. Point A is not on 1200 m
either line. B is the image of A upon reflection over €, and C is the image of B upon reflection over m.
Show that ZABC = 90° and XA = XB = XC. Must AC pass through X?

16.34 A square sheet of paper has area 6 cm”. The front is white and the back is
shaded. When the sheet is folded so that point A rests on the diagonal as shown, the
visible shaded area is equal to the visible white area. How many centimeters is A’ from
its original position, A? (Source: MATHCOUNTS) Hints: 335
16.35 Arotation about A maps X to Y. A different rotation about X maps A to Y. Show
that AAXY is equilateral.
16.36 Point O is inside quadrilateral ABCD. A rotation of 180° about O maps the quadrilateral ABCD
to itself, meaning each vertex of the image is also a vertex of the original quadrilateral.

(a) Must ABCD be a parallelogram?


(b) Must ABCD be a rectangle?
(c) Must ABCD be a square?

16.37 The image of point A upon a 90 degree clockwise rotation about point B is point C. The image of
C upon a 90 degree rotation clockwise about point D is point A. Prove that ABCD is a square.

16.38 Let ABCD be a rhombus with /BAD = 60°. Let P and Q be points on AD D Q
and CD, respectively, such that ZPBQ = 60°. Find the other two angles of triangle P
PBQ. A G
16:39 SAABC has AB =.3, BC = 4 and AC = 5, 2
(a) Aone is formed by rotating the triangle around AB. What is the volume of this cone?
(b) Acone is formed by rotating the triangle around BC. What is the volume of this cone?
(c)x A solid is formed by rotating the triangle around AC. What is the volume of this solid?

432
CHALLENGE PROBLEMS

16.40 Show that if X’ and Y’ are the images of X and Y under a dilation with center O and scale factor
k, where k is negative, then we have X’Y’ || XY and X’Y’/XY = |k.

Challenge Problems

16.41 CD is the image of AB under some rotation. AB and CD have the same midpoint, M. Must M be
the center of the aforementioned rotation that maps AB to CD? Hints: 520

16.42 You might recognize the diagram at right from Problem 8.22 on page 226. A B
It is indeed the same problem. ABCD is a square. The circle with center A and
radius AB intersects the perpendicular bisector of AD in two points, of which O L
is the one inside the square. Use the principles we have explored in this chapter is
to quickly find ZAOC. Hints: 241, 180
16.43 Consider the octagon of Problem 16.24. D -

(a) Show that there is a point inside the octagon that is equidistant from all 8 vertices of the octagon.
Hints: 204
(b) Show that a 90° rotation about the point found in part (a) maps the octagon to itself. Hints: 246

16.44 AABC with vertices A(2,4), B(6,4) and C(4,10) is graphed in a coordinate plane. What will the
sum of the abscissas (x-coordinates) of the vertices be when AABC is reflected over the line x = 8?
(Source: MATHCOUNTS)

16.45 The image of the incircle of equilateral triangle AABC upon a dilation about the center of the
circle with scale factor k is the circumcircle of AABC. Find k.
16.46 Use a clever dissection of the section of regular octagon ABCDEFGH that is traced in bold at left
below to prove that [ABCDEFGH] = (¥2)[ACEG]. (O is the center of the octagon.) Hints: 159, 410

5
D 7G

e F A B

Figure 16.4: Diagram for Problem 16.46 Figure 16.5: Diagram for Problem 16.47

16.47 A laser beam is fired from point A in the U-shaped room at right above. It reflects from each wall
as from a perfect mirror. What is the minimum distance the beam travels before hitting a target at point
B? (A and B are the midpoints of their respective sides.) (Source: Mandelbrot)

esCCC CCC CCCCCCOCOCOC™*~—“—~S™S™OOO::””””::”~CSC(C‘C SS


CHAPTER 16. THE MORE THINGS CHANGE...

16.48 Suppose AB || A’B’ and A’B’/AB = k, where k # 1. In this problem, we prove that there is a
dilation that maps A to A’ and B to B’.

(a) Why must AA’ and BB’ intersect?


(b) Let P be the intersection of AA’ and BB’. Show that APAB ~ APA’B’.
(c) Show that there is a dilation with center P that maps A to A’ and B to B’.

16.49 Let ABCD be a square, and let M and N be the midpoints of AB and BC, respectively. Prove that
CM 1 DN. Hints: 445
16.50 The center of the cue ball on my rectangular pool table is directly above point A on the table. I
wish to bounce the cue ball off a rail such that after it bounces off the rail, the center of the ball will pass
directly over point B on the table. The radius of the cue ball is 1 in. A is 6 inches from the rail, and B is
9 inches from the rail. A is 5 inches from B. How far from the nearest point on the rail to point B do I
want the cue ball to hit the rail? Hints: 278, 234

16.51x What is the total length of the shortest path that goes from (0,4) to a point on the x-axis, then
to a point on the line y = 6, then to (18,4)? Hints: 262, 472

16.52% Laam standing at point /A in the diagram at right. I wish to walk to a


point on BC, then to a point on BD, then back to my starting point. Construct with
straightedge and compass the shortest such path I can take. Hints: 338, 370 G3

16.53% Regular octahedron ABCDEF has volume 360 cubic units. What is the
volume of that portion of the octahedron that consists of all points that are closer
to A than to any other vertex? Hints: 311

16.54x In the diagram at right, AABC, APBR, AAQR, and APNQ are equilateral. N C
Prove that BQ = RN = AP. (Source: Mandelbrot) Hints: 320

16.55x Prove that the image of a circle under rotation about any point is also a
circle. Hints: 590
Q

A B
- Extra! _ Back on page 430, we introduced the geometric transformation called inversion. Here
“nim are some more questions to guide further exploration ° inversion:

e What is theimage of ©O upon inversion with respectto @0?,


e What iis the image of a line that passes through O? Of a line that doesnot
t pass
through O?

e What is the image Ol;a circle that passes te O? Of a circle hat does not |:
through O? aoe

We'll explore these properties and more in later book in SeyArt of Problem Solving
series. oe

434
Morley’s Second Triangle

New facts often trigger new ideas. — Alex F. Osborn

CHAPTER T iy.
na at Ea Hig ate =0-) a a

Analytic Geometry

Much of the mathematics developed by the ancient Greeks was geometry. In fact, even much of the
number theory and algebra that they developed was expressed in geometric terms. However, by the
16" or 17 century A.D., algebra was recognized as an extremely important field of mathematics in its
own right, possibly equaling, or even exceeding, geometry.
In 1637, the great mathematician and philosopher René Descartes brought these two great fields of
mathematics together when he described a general method to represent geometric figures with algebraic
equations. This combination of algebra and geometry is often referred to as analytic geometry. In this
chapter, we review how to describe lines and circles with equations, then we apply these methods to
both algebra and geometry problems.

ee
a ll ll 435
CHAPTER 17. ANALYTIC GEOMETRY

Descartes’s great insight that unified geometry and algebra has become
so important that we still use Descartes’s name to describe the result. We use
the Cartesian plane to describe geometric figures with algebraic equations.
At right is the Cartesian plane. The center of the plane, where the
bold lines meet, is called the origin. On the Cartesian plane each point
is represented by an ordered pair of numbers. These numbers denote the
position of the point relative to the origin.

For example, we denote the point that is 3 steps to the right and 2 steps
up from the origin with the ordered pair (3,2). The “ordered” part is very
important! The first number always denotes how far the point is to the right
(or left) of the origin, and the second number denotes how far the point is
above (or below) the origin. We call the two numbers in an ordered pair the
coordinates of the point. By convention, we call the horizontal (left-right)
coordinate the x-coordinate and we call the vertical (up-down) coordinate
the y-coordinate.
As you might have guessed, the x-coordinate of a point is negative when
the point is to the left of the origin, and the y-coordinate is negative when the point is below the origin.
For example, the point (—5, —1) shown in the diagram is 5 steps to the left and one step below the origin.
The point (0,0) is 0 steps to the right and 0 steps up from the origin. In other words, (0,0) is the origin
itself.

When the point is neither above nor below the origin, its y-coordinate
is 0. Such a point is directly to the left or right (or on) the origin. Therefore,
the point must be on the bold horizontal line in the diagram at left. We call
this line the x-axis. Similarly, the vertical line consisting of points directly
above or below (or on) the origin is called the y-axis. We usually label the
x-axis and y-axis with an x and a y, respectively, as shown in each of our
diagrams.

Finally, we aren’t restricted to using integers for our coordinates. We


can plot any point represented by an ordered pair of two real numbers on
the Cartesian plane. For example, the diagram at right depicts the point
(3.5, -2.7), which is 3.5 steps to the right and 2.7 steps below the origin.
Points that have integers for both coordinates are called lattice points.
rrr
eS A SS A
Te
IRE
EINES
SAP

17.1 Lines

In this section, we review how equations can be used to represent lines. We cover this material in much
more detail in Introduction to Algebra. If you are unfamiliar with using equations to represent lines, you
may wish to study the fundamentals covered in Introduction to Algebra before continuing.

The graph of an equation that has x and y as variables consists of all points y
(x, y) on the Cartesian plane that satisfy the equation. For example, the graph |
of the equation 2x + 3y = 6 is shown at right. As we see, the graph is a line.
Indeed, the graph of any equation of the form Ax + By = C, where A, B, and
C are constants and A and B are not both 0, is a line. As a result, such an
equation is often called a linear equation.
The points where a graph intersects the x-axis are called the x-intercepts
of the graph, and the points where a graph intersects the y-axis are called the
y-intercepts of the graph. For example, the only x-intercept of the graph of
2x + 3y = 6 at right is (3,0) and the only y-intercept of the graph is (0, 2).
One way we describe graphed lines is by indicating the ‘direction’ of the line, which we call the slope
of the line. We define the slope, m, of the line that passes through the points (x1, y1) and (x2, y2) to be
Y2- V1
m= ;
XO XA

A line with positive slope goes upward as it goes from left to right, and a line with negative slope
goes downward as it goes from left to right.
“ y
(x2, 2) (x1, 41)
(x1, 41)
3 x

Figure 17.1: A Line With Positive Slope Figure 17.2: A Line With Negative Slope

The slope of a horizontal line is equal to 0, and the slope of a vertical line is undefined. Moreover,
the greater the magnitude of the slope of a line, the more ‘steep’ the line is.
CHAPTER 17. ANALYTIC GEOMETRY

Problems ig
|Problem 17.1: Find an equation whose graph is the line passing through the points (—1, 7) and (—5, =),

|Problem 17.2: Find an equation whose graph is the line with slope 4 and y-intercept (0, —3). 4

Problem 17.3:
(a) Find the length of the segment with endpoints (—4, —3) and (6, 1).
(b) Find a formula for the distance between the points (x1, yi) and (2, y2).

Problem 17.4: Let point A be (x1, y;) and point B be (x2, y2). Let M be the point with coordinates

epR Asn),
In this problem we show that M is the midpoint of AB.
(a) Show that AM = MB.
(b) Show that M is on AB.

|Problem 17.5: Explain why two different lines that have the same slope are parallel.

Problem 17.6: In this problem we show that if two lines are perpendicular, and neither line is vertical,
then the product of their slopes is —1.
Shown at right are perpendicular lines k and €. Let P be the point
where these lines intersect. We construct two right triangles such that their
hypotenuses are along lines k and €, respectively, as shown. Moreover, each
right triangle has a horizontal leg and a vertical leg.
(a) Show that AADP ~ ABEP.
(b) Use the similar triangles from part (a) to show that

DA EP _
DP Eo
Why does this tell us that the products of the slopes of k and ¢ is —1?
a reeeeemeeeeeeeeeee e

We start with a review of some different forms in which we can write equations whose graphs are
lines.

Problem 17.1: Find an equation whose graph is the line passing through the points (—1,7) and
(28 23).
Solution for Problem 17.1: The slope of the line through these two points is

7 =(=3)t SG) 5
-1-(-5) 4. 2
438
17.1. LINES

Therefore, if (x, y) is any point on this line other than (—1,7), then the slope between (x, y) and (—1,7)
must be 5/2. So, we must have

Multiplying both sides of this equation by x — (-1) gives us y— 7 = 3[x—(-1)]. This is an example of the
point-slope form of a linear equation.

Important: The graph of the equation

y—y1 = m(x
— x1)
is a line through (xj, y;) with slope m. This is called a point-slope form
of the equation.

While we often use point-slope form to find an equation of a line given the slope of the line and a
point on the line, we often then convert the equation to standard form.

Important: The standard form ofa linear equation i is Ax+B veve.where, ifpossible,
Wi A, B, and C are integers, A is positive, and A, B, and C have no common |
factors besides 1.

We start converting y — 7 = 3[x — (—1)] to standard form by first multiplying both sides by 2 to get
2(y — 7) = 5(x + 1). Expanding both sides and rearranging gives 5x — 2y = —19.
We can check our answer by making sure that the two points (x, y) = (—1,7) and (x, y) = (-5,—3)
satisfy our equation. Both do, so we know the equation 5x — 2y = —19 represents the line through (—1, 7)
and (-5;43)4D
One more form that is occasionally useful is slope-intercept form, which we'll investigate in the next
problem.
‘Problem 17.2: Find an equation whose
v e graph is
isstheline‘with slopeA andy-intercept0,3).
_ SRL TNA EEE IE DOL OS Si Sa SRR Seale eset aia se TSS ESS

Solution for Problem 17.2: Because the line passes ane (0,-3) and has slope 4, a 5 a a form
of the line is y — (-3) = 4(x — 0). Isolating y on the left side then gives us y = 4x — 3. Notice that the
coefficient of x equals the slope of our line and the constant term on the right equals the y-coordinate of
the y-intercept. This is the slope-intercept form of the equation:

|Important: The slope-intercept form of a linear equation is |

y=mx+b,

| Liesmiseine ae 6 oe me 2S :isPane ya oFthe y-intercept.|

As an Exercise, you'll prove that the graph of an equation of the form y = mx + b has slope m. O

ChUCC CC CC CCT... AG
CHAPTER 17. ANALYTIC GEOMETRY

We'll now review some particularly useful relationships regarding segments and lines on the Carte-
sian plane.

Problem 17.3:
(a) Find the length of the segment with endpoints (—4, —3) and (6, 1).
(b) Find a formula for the distance between the points (x1, yi) and (x2, Y2).

Solution for Problem 17.3:


(a) We start by labeling our points A and B such that A is (—4, -3)
and B is (6,1).
One of our favorite tactics for finding the length of a segment
in a geometry problem is to build a right triangle with that
segment as a side. Here, an easy way to build a right triangle
with AB as a side is to extend segments horizontally from A
and vertically from B to meet at point C, as shown. Since a
horizontal line and a vertical line meet at a right angle, ZACB
is a right angle. So, if we can find AC and BC, we can use the
Pythagorean Theorem to find AB.
The x-coordinate of C is the same as the x-coordinate of B | |
and the y-coordinate of C is the same as the y-coordinate of A. So, point C is5 (6,3). itersfore,
AC = 6 — (-—4)= 10 and BC= 1 — (—3) = 4, and the Pythagorean Theorem gives us

= VAC2 + BC2 = ¥100 + 16 = V116 = 2 V29.


(b) Building a right triangle worked well when finding the distance between two specific points, so
let’s try using that tactic to find a formula for the distance between (x1, y1) and (x2, y2). Using part
(a) as a guide, we let A be (x1, yi) and B be (x2, y2) and build a right triangle with AB as a side.
In the diagram at right, we assume that B is above and to the B (x2, y2)
right of A. As before, we draw a vertical line down from B and a ;
horizontal line to the right from A. We label the point where these Seas
lines meet C. Since C is directly below B, its x-coordinate is the
same as B’s, or x2 (in other words, it is just as far horizontally from 4
the y-axis as B is). Similarly, since C is directly to the right of A, its (x,;,y;) %2-%1 C (*2, yn)
y-coordinate is the same as A’s, y}.
Now we're ready to use the Pythagorean Theorem. We can use our coordinates to see that C is
x2 — x1 to the right of A and y2 — y; below B. So, we can use the Pythagorean Theorem to find

AB? = AC? + BC* = (x2 — x1)" + (y2 — yx).


We can take the square root of both sides of this equation to find a formula for the distance between
the points (x1, y1) and (x2, y2):

AB = 4/(x2 — x1)? + (yo — 1)”.


This is often called the distance formula. Notice that if x; = x2, then the distance between (xj, 1)
and (x2, y2) is just the nonnegative difference between the y-coordinates. Let’s see if that’s what
eee
440
17312 “EINES

the distance formula gives us when x; = x2:

V (2 = 11)? + (y2 = yt)? = (x2 — 12)? + (yo — #1)? = Vo - 1)? = ly2 — Mal
Indeed, the distance formula gives the expected result when x; = x2. (We use absolute value signs
when writing \/(y2 — yi)? = |y2 — yil because square roots, like distances, must be nonnegative.)
Similarly, the distance formula tells us that the distance between (x1, yi) and (x2, y2) when yj = y2
is the nonnegative difference between the x-coordinates.

Important: The distance in the plane between the points (x1, y1) and (x2, y2) is

(aa et (yo = 91).


This is often referred to as the distance formula.

The distance formula is essentially the same thing as the Pythagorean Theorem. In fact, because you
know the Pythagorean Theorem, you basically know the distance formula already, so you shouldn’t
have to memorize the distance formula.
Problem 17.4: Let point A be (x1, y1) and point B be (x2, y2). Let M be the point with coordinates

(= a XD YE =H)
22271 ;

Show that M is the midpoint of AB.

Solution for Problem 17.4: In order to show that M is the midpoint of AB, we must show that AM = MB
and that M is on AB. To show that AM = MB, we turn to the distance formula. We have

ate [OZB-n)
+EG2=n)=(OR) ER
p alec Coa ah Cara ae
Se aavce
So, we have AM = MB.

To show that M is on AB, we compare the slopes of AM and MB. We have

Tre ae ee at 2 cl
Slope of AM = = pv = Sr uAt

yity2
Sag eer a ee UI
Slope of MB = yy 1 LE = Sea

When x2 # Xj, the slopes of AM and MB are the same, so A, M, and B are collinear. If x2 = x1, then A, M
and B are on the same vertical line.

Since A, M, and B are collinear and AM = MB, we know that M is the midpoint of AB. O

441
CHAPTER 17. ANALYTIC GEOMETRY

sae
Problem 17.5: Explain why two different lines with the same slope are parallel.

Solution for Problem 17.5: Slope is a measure of the direction of a line.


Therefore, we expect two lines with the same slope to be parallel. We can
prove this quickly with geometry. If our two lines both have slope 0, then
they are both horizontal, and clearly are parallel. Otherwise, we can draw
a horizontal line segment with one endpoint on each line, as shown in the
diagram at right. We can build two right triangles by drawing two vertical
line segments, as shown. If we show that ZACB = ZCBD, then we can
deduce that k || €.

The only given information we have is that the lines have the same
slope, so we need to find some way to express the slope of each line to
allow us to use this information. Fortunately, our diagram gives us the answer. The slope of k is DC/BC
(change in y-coordinate divided by change in x-coordinate), and the slope of line € is AB/BC. Because
the slopes of k and ¢ are equal, we have DC/BC = AB/BC, so DC = AB. We also have ZABC = ZBCD, so
we have AABC = ADCB by SAS Congruence. Therefore, we have ACB = ZCBD, so k || €, as desired.

You'll have the chance to prove the converse, that two parallel lines have the same slope (if they are
not vertical) as an Exercise. 0

Important: If two lines have the same slope, then they are parallel. Conversely, if

WV two non-vertical lines are parallel, then they have the same slope.

We've seen that the relationship between the slopes of two parallel lines is pretty straightforward.
What about perpendicular lines?

Problem 17.6: Show that if two lines are perpendicular, and neither line is vertical, then the product
of their slopes is —1.

Solution for Problem 17.6: Because the lines are perpendicular and neither is vertical, one line has positive
slope and the other has negative slope. To see why, suppose the lines intersect at P, and that their slopes
have the same sign. Then, the smaller angle formed by the lines is smaller than the right angle formed
by the vertical line and horizontal line through P. So, if the slopes of the lines have the same sign, the
lines cannot be perpendicular.
Let the line with negative slope be k and the line with positive slope be €. Building right triangles
worked so well in the last problem that we should try it again for this problem. We’re also guided to
build right triangles since these will give us a way to express the slopes of k and €. We can use essentially
this same proof for any configuration of k and ¢ in which the lines are perpendicular and neither line is
vertical.

442
uml Ol ee nnn nn nn ee
SSS
sss Ss Ss
T7Ae LINES

We make right triangles AADP and APBE such that each has a horizontal
leg and a vertical leg. From APBE, we see that the slope of € is EP/EB. From
AADP, we see that the slope of k is -DA/DP. Make sure you see why we
need the negative sign; the slope of k is clearly negative. Now that we have
expressions for the slopes of k and €, we can write an equation we wish to
prove. We must show that

(is) (ap) =>


The ratios in this expression make us think of similar triangles. The right triangles are a pretty big
clue, too. We already have ZADP = ZBEP, so we only need to find one more pair of equal angles to
prove the right triangles are similar. We probably need to use the fact that k and ¢ are perpendicular, so
we focus on that. Since ZAPB = ZDPE = 90°, we have

ZAPD = 90° — ZDPB = ZBPE.

Combining ZAPD = ZBPE with ZADP = ZBEP gives us AADP ~ ABEP by AA Similarity. This similarity
gives us
DA EB
DP EP’
Multiplying both sides by —1 and by EP/EB gives us the desired (EP/EB)(-DA/DP) = —1. Therefore,
the product of the slopes of k and f is —1.
As an Exercise, you'll have the chance to prove the converse: if the product of the slopes of two lines
is —1, then the two lines are perpendicular. 0

| Important: “If two non-vertical lines aree perpendicular, then the product of their
= VY slopesis 1. Conversely, if the product of the ee oftwo lin ie
. then the two lines are perpendicular.

es

17.1.1 Find an equation, in standard form, whose graph is a line through (—2, 5) and (—5, 9).

17.1.2 Find the x-intercept and y-intercept of the line with slope 4 that passes through (—7, 2).
17.1.3 The distance between (—4,3) and (a,0) is 5. Find all possible values of a.

17.1.4 When finding the distance between two points using the distance formula, does it matter which
point we call (x1, y;) and which we call (x2, y2)? Why or why not?

17.1.5 Point P is (—5,2) and point Q is (—8, 8).

(a) Find PQ.


(b) Find the midpoint of PQ.
(c) Find T such that Q is the midpoint of PT.

Sl UhUCUhUCUCUCmCUmCUCUC™C™C””C”C~”~”O”C~C™””~””””””C
CHAPTER 17. ANALYTIC GEOMETRY

17.1.6 Line k is perpendicular to the graph of 4x — 3y + 14 = 0 and passes through (—3, 2). Find an
equation in standard form whose graph is line k.
17.1.7. Show that the graph of y = mx + b, where m and b are constants, has slope m.
17.1.8 Show that if the product of the slopes of lines k and is —1, thenk 1 €.

17.1.9 Suppose A, B, and C are constants and B # 0. Show that the slope of the graph of the equation
Ax + By + C =0is —A/B.

17.2 Circles

In this section, we use the distance formula to find equations whose graphs are circles.

Problems

Problem 17.7:
(a) Find an equation whose graph is a circle with center (4, —5) and radius 3 v2.
(b) Explain why the graph of the equation

(x-hy+(y-kh? =P,
: where h, k, and r are constants with r > 0, is a circle with center (h,k) and radius r.

Problem 17.8: Find the area of the region that is enclosed by the graph of the equation

2x? — 8x + 2y* + l6y = 4.

Problem 17.7:
(a) Find an equation whose graph is a circle with center (4, —5) and radius 3 2.
(b) Explain why the graph of the equation

(x—h) +(y-kP =r,


where h, k, and r are constants with r > 0, is a circle with center (h, k) and radius r.

Solution for Problem 17.7:


(a) If a point (x, y) is on the circle, then it must be 3 V2 from the center of the circle, which is (4, —-5).
So, the distance between (x, y) and (4, —5) is 3 V2. The distance formula then gives us

V(x — 4)? + (y+5)2 = 3 V2.


Squaring both sides of this equation gives

(x — 4)* + (y +5)? = 18.

444
17.2. FCIRCLES

We could multiply out both (x — 4)? and (y + 5)*, but leaving the equation in this form makes it
particularly easy to see that the graph of this equation is a circle with center (4,—5) and radius
3 V2. In our next part, we'll see why.
(b) Taking the square root of both sides of (x — h)? + (y — k)? = 1? gives us

J(x—h)? + (y—b? =r.


Since h and k are constants and r is a positive constant, this equation tells us that the point (x, y) is
a distance of r from the point (h,k). Therefore, every point (x, y) that satisfies this equation is on
the circle with center (h,k) and radius r.
Conversely, if a point (x, y) is a distance of r from the point (i,k), then the distance formula tells
us that
V(x —h? + (y-k? =r.
Squaring both sides of this equation gives us (x — h)? + (y — k)* = 17, so we see that every point on
the circle with center (h,k) and radius r satisfies the given equation. Therefore, the graph of the
equation (x — h)? + (y —k)* =r? isa circle with center (h,k) and radius r.

Important: The standard form of an equation whose graph is a circle is

(@-hY +y-h =r,


where h, k, and r are constants with r > 0. The center of the circle is (h,k)
and the radius of the circle is r. |

Sometimes we have to do a little work to put an equation in standard form.


Problem 17.8: Find the area of the region that is enclosed by the graph of the equation

2x? — 8x + 2y? + 16y = 4.


Solution for Problem 17.8: The 2x? and 2y* terms make us think of the standard form of a circle, because
expanding the squares on the left side of
(x—hy +(y-k% =P

will give us an x* and y’. So, we try to write the given equation in this form. Our first step is to divide
both sides by 2, to make the coefficients of x? and y* both equal to 1. This gives us
x? — Ax + y? + By = 2.

We then complete the square in both x and y by adding 4 and 16 to both sides:
7 —4x+4+y°+8y+16=2+4+4+16.
x2 — 4x + 4= (x — 2)? andy* + 8y + 16 = (y + 4)”, we have
Since
(x — 2) + (y +4)? = 22.
SE
Ee eee eres
445
CHAPTER 17. ANALYTIC GEOMETRY

This equation is in the desired standard form of a circle. Therefore, we know that the graph of this
equation is a circle with center (2, —4) and radius 20)

f A EA AE S RRNA |

WARNING!! Make sure you see why the center is not (—2, 4) or (2,4). The standard |
~ form of a circle is |
(x =hyY + -— 7 =r.
Comparing this to
(x — 2)? + (y+ 4)? = 22
gives us h = 2 and k = —4.
Sea eae ee

Because the graph is a circle with radius 22, its area is (V22)?n = 227. O

||Exercises
__
17.2.1 Find the center and the radius of the circle that is the graph of the equation 3x*-12x+3y*+6y = 15.
17.2.2 Find the standard form of the equation whose graph is a circle with center (—2,7) that passes
through (—5,9).

17.2.3. What is the length of the longest chord of the graph of

x +2x+y*
—6y =6
that passes through the point (—2, 4.5)?
17.2.4x Find the two points at which the graphs of 2x — y = 7 and x? — 10x + y* + 4y = —4 intersect.

17.3 Basic Analytic Geometry Problems

In this section, we begin to explore the power of analytic geometry. Because analytic geometry gives us
a tool to relate algebra and geometry, it gives us a way to apply geometric tools to algebraic problems,
and to use algebraic tools to solve geometry problems.

: (| Problems >

Problem 17.9: The area of the shaded region between line k and the axes
in the diagram at right is 36. If line k passes through (12, 0), then what is the
slope of k?

i i DA
17.3. BASIC ANALYTIC GEOMETRY PROBLEMS

Problem 17.10: Let A be (—5,6), B be (—7,9), and C be (1, 10).


(a) Find the length of the median of AABC from B to AC.
(b) Find the slope of each side of AABC. Notice anything interesting?
(c) Find the area of AABC.
(d) Find the length of the altitude from A to BC.

Problem 17.11: In this problem, we determine the number of points (a,b) such that a? + b? = 25 and
the area of the triangle with vertices (—5, 0), (5,0), and (a,b) is 10. (Source: MATHCOUNTS)
(a) What does the fact that (a,b) satisfies a* + b* = 25 tell us about where the point (a,b) is located
on the coordinate plane?
(b) What does the fact that the area of the triangle with vertices (—5,0), (5,0) and (a,b) is 10 tell us
about where the point (a, b) is located on the coordinate plane?
(c) How many points satisfy the conditions you found in parts (a) and (b)?

Problem 17.12: ABCD is a rectangle with AB =8 and AD = 12. Points X and Y are on AB and CD,
respectively, such that AX = 7/3 and DY = 1. XY meets diagonal AC at point P. In this problem, we
find BP.
(a) Set the problem up on the Cartesian plane. Let A be the origin. What coordinates can we assign
to each of B, C, and D?
(b) What are the coordinates of P?
(c) Find BP.

Problem 17.13: In AABC, we have AB = BC = 8 and ZABC = 45°. In this problem, we find AC.
(a) Why should 45° make us think of the graph of y = x?
(b) What point on the line y = x is 8 units from the origin?
(c) Let B be the origin. Find coordinates for A and C such that AABC satisfies the problem. Find
AC:

Problem 17.9: The area of the shaded region between line k and the axes
in the diagram at right is 36. If line k passes through (12,0), then what is
the slope of k?

Solution for Problem 17.9: The shaded region is a right triangle. The vertex of the right angle of this
triangle is the origin, (0,0), which is labeled A in the diagram. Vertex B is (12,0), so we know that leg AB
has length 12. Because the area of the triangle is 36, we know that half the product of the lengths of the
legs of the triangle is 36. Therefore, we have (AB)(AC) = 72, so AC = 72/AB = 6. So, vertex C is (0,6),

EI
ge En ee ae
447
CHAPTER 17. ANALYTIC GEOMETRY
ee

and the slope of k is (6 — 0)/(0 — 12) = -1/2. 0

|Problem 17.10: Let A be (5, 6), B be (-7,9), and C be (1, 10).


| (a) Find the length of the median of AABC from B to AC.
_ (b) Find the length of the altitude of AABC from A to BC.

Solution for Problem 17.10:


=St4 , O10) = (—2,8). The distance from B to this point is
(a) The midpoint of AC is (34

V[-2 - (-7)P + (8-9)? = V25 +1 = V26.

(b) Solution 1: Find the coordinates of the foot of the altitude. To find the length of the altitude from A to
side BC, we try to find the point P on BC such that AP 1 BC. First, we find the equation whose
graph passes through B and C. The slope of this line is (10— 9)/[1 — (- igs = 1/8. The line passes
through (1, 10), so an equation of the line in point-slope form is y— 10 = 3(x—1). Rearranging this
equation gives x — 8y= —79.
Let point P be (x, y). Since (x, y) ison BC, it satisfies the equation x—8y = —79. Because AP 1 BC,
the product of the slopes of these two segments must be —1. Since the slope of BC is 1/8, we know
the slope of AP is —8. So, AP passes through (x, y) and (—5, 6), and it has slope —8. This gives us

¥ =O
x—(=5)
Rearranging this equation gives 8x + y = —34.
We now have the system of equations

sO Uy -—79,
8x+y = —34.

Multiplying the second equation by 8 then adding the result to the first equation gives us the
equation 65x = —351, which yields x = —27/5. Substituting this into either equation above then
gives us y = 46/5. Therefore, P = (—27/5, 46/5), so we have

Eeley : , 236 __ [260 g165,


ar=[-Z cs} +(=e) 55 S95 V5
Solution 2: Use some geometric insight. The algebra in our first solution is a little messy. The
system of equations we have to solve isn’t very nice, and all the fractions give us plenty of
opportunities to make mistakes. Once we see all that algebra coming our way, we might stop and
think if there areey geometric ee we can use to simplify the problem.
Cc ncept: When the algebra iin an analytic geometry problem starts toBe1
OS _ using geometric facts to simplify |the problem,

448
17.3. BASIC ANALYTIC GEOMETRY PROBLEMS

Here, we seek the length of the altitude from A to BC. The altitude
makes us think about area. We can easily find the length of BC:

BC = \/(-7 — 10)?= V65.


Therefore, if we can find [ABC], we can easily find the length of AP, since
(AP)(BC)/2 = [ABC].
Is [ABC] easy to find? To answer this, we think about what kind of
triangle ABC is. If it is isosceles or right, we can probably find the area
easily. We can useslope to quickly check if it is right. We've already seen that the slope of BC is
1/8. The
ne slope cof AB is —3/2 and the slope of AC is 2/3. The product of the slopes of AB and AC is
—1,so AB 1 AC.

Concept: When given the coordinates of the vertices of a triangle, check if the triangle |
sp _ is special in any way. Most notably, check if the triangle is a right triangle.
If it is, this fact will probably simplify the problem, because we know so
much about right triangles.
Now the area of AABC is easy to find. Because BAC = 90°, we have [ABC] = (AB)(AC)/2.
Using the distance formula, we find that AB = V13 and AC =2 Vis so

[ABC]= ——* Se = 13.

Finally, we have

2[ABC] 26 26S V65__-2-V65


AP=
BC V6e5 v65 Vo 5
A little geometry sometimes goes a long way in analytic geometry problems!

Problem 17.11: Find the number of points (a,b) such that a? + b* = 25 and the area of the triangle
with vertices (—5,0), (5,0), and (a,b) is 10. (Source: MATHCOUNTS)

Solution for Problem 17.11: Let our triangle be PQR, with P = (a,b), Q = (—5,0), and R = (5,0). Side OR
is a horizontal segment with length 10. Because the area of APQR is 10, we know that the length of the
altitude from P to OR is 2. Since QR is along the x-axis, the fact that P is 2 units from OR means that P
is either 2 units above or 2 units below the x-axis. Therefore, point P is on the line y = —2 or on the line
y = 2. From here we present two solutions.
Solution 1: Algebra. Because P is on either y = 2 or y = —2, we must have either b = 2 or b = —2.
When b = 2, the equation a? + b? = 25 gives us a* = 25-b* = 21,soa=+ 21. So, two possible points P
are ( ¥21, 2) and (— V21,2). Similarly, letting b = -2 gives us two more points that satisfy the conditions
in the problem, namely, (v21, —2) and (- 2 —2). Therefore, there are 4 different points that fit the
description of the problem.

449
CHAPTER 17. ANALYTIC GEOMETRY

Solution 2: Geometry. Because (a,b) satisfies the equation x? + y? = 25, we know


that (a, b) is on the circle centered at the origin with radius 5. We also know that (a,b) is
either on the line y = 2 or the line y = —2. Each of these lines is 2 units from the center
of the circle, so both lines go inside the circle. Therefore, both lines intersect the circle
twice, as shown in the diagram at right. Each of the 4 points of intersection between
a line and the circle gives us a point that satisfies the restrictions of the problem. 0
In our second solution, we interpreted the equation a* + b* = 25 as telling us that the point (a, D) is
on the circle with center (0,0) and radius 5. Any time we see an equation that is in the standard form
of a circle, we might consider using analytic geometry. This gives us a new tool for tackling algebraic
problems:

Concept: Interpreting an equation as a geometric figure on the Cartesian plane can


| often help us solve problems.

This street goes both ways! Sometimes it is useful to put a geometry problem on the Cartesian plane
and use algebraic tools to solve the problem.

Problem 17.12: ABCD is a rectangle with AB = 8 and AD = 12. Points X and Y are on AB and CD,
respectively, such that AX = 7/3 and DY = 1. XY meets diagonal AC at point P. Find BP.

Solution for Problem 17.12: We seek a length, so we might try building right triangles and using the
Pythagorean Theorem. (Try doing so on your own.) Analytic geometry offers us another way to find
lengths: if we can find the coordinates of two points, we can use the distance formula to find the distance
between them. So, we try turning this problem into an analytic geometry problem. One reason we do
so is that rectangles are particularly easy to describe with coordinates.
We start by letting A be the origin, and letting two sides of the rectangle be along the coordinate
axes. This makes a lot of the coordinates in the problem equal to 0, and 0’s are easy to deal with.

Concept: When setting a geometry problem up on the Cartesian plane, choose the -
origin and the axes in a convenient matter. Often this means letting the
origin be the vertex of a right angle in the Pri so that thesides of the
/ _ angle are along the <
axes. * Se

In the diagram at right, we let AB be along the y-axis and AD


be along the x-axis. Because AB = 8 and AD = 12, point B is (0,8)
and point D is (12,0). Because X is on AB such that AX = 7/3, we
have X = (0,7/3). Similarly, Y is 1 unit above D, so Y = (12, 1).
Now, if we can find the coordinates of point P, we can use the
distance formula to find BP. We know that P is on AC. Because
AC has slope (8 —0)/(12—0) = 2/3 and it passes through the origin,
the equation whose graph is AC is y = 2x/3. (This is why we like
having so many 0’s in our coordinates — the resulting equations

450
17.3. BASIC ANALYTIC GEOMETRY PROBLEMS

for lines connecting points in the diagram are simpler.) Point P is also on XY. The slope of XY is
.me 7}3)/(12 — 0) = -1/9 and its y-intercept is (0, 7/3), so the slope-intercept form of the equation of this
ine is “and:
Ve= 9" aif 3"

Rearranging this equation gives x + 9y = 21. So, P is the intersection of the graphs of y = 2x/3 and
x+9y = 21. Substituting y = 2x/3 into x+9y = 21 gives x + 6x = 21, from which we find x = 3. Therefore,
we have y = 2x/3 = 2, so P is (3, 2). Finally, because B is (0,8), we have

=3 V5.
BP = \/(3 —0)2 + (2—8)2= V9 + 36 = V45
A little algebra sometimes goes a long way in geometry problems! 0

WARNING!! Don’t get carried away with using analytic geometry to solve ge-
~_ ometry problems. While our solution to Problem 17.12 was pretty
straightforward with analytic geometry, you'll find that most geom-
etry problems are easier to solve with geometric methods than with
analytic geometry.

For example, suppose we wished to find the length of the median to the hypotenuse of a right triangle
with sides of length 14, 48, and 50. It is much faster to use the simple geometric fact that the median to
the hypotenuse of a right triangle is half the length of the hypotenuse than it is to assign coordinates to
the vertices of the triangle, find the coordinates of the midpoint of the hypotenuse, then use the distance
formula.
After finishing this chapter, try flipping back through this book and finding problems that you can
solve with analytic geometry. You'll find some common trends among the ones you can solve, such as
rectangles, midpoints, and right triangles. You'll also find some trends among problems that are hard to
solve with analytic geometry, such as circles, angles that are not 45° or 90°, and complicated diagrams.
Our last sentence suggests that problems with 45° angles might be candidates for analytic geometry.
Let’s see why.

Problem 17.13: In AABC, we have AB = BC = 8 and ZABC = 45°. In this problem, we find AC

Solution for Problem 17.13: We present both an analytic geometry solution and a ‘pure geometry’ solution.

Solution 1: Analytic geometry. The 45° angle might make us think about
analytic geometry because the graph of the line y = x makes a 45° angle with
the coordinate axes. We also notice that AB = BC = 8 means that A and C are
both on the circle with center B and radius 8. In other words, if we let B be
the origin, then the coordinates of A and C are solutions to the equation
P+ = 64,

because the graph of this equation is the circle with center (0,0) and radius
8. We know that the graph of y = x makes a 45° angle with the axes, so we
let C be along the the x-axis, at (8,0), and A be on the graph of the line y = x.

451
CHAPTER 17. ANALYTIC GEOMETRY

We can find the coordinates of A either by drawing an altitude from A to the x-axis, or by using our
equation for the circle. Because A is on both the circle and the graph of y = x, we can find the coordinates
of A by substituting y = x into our equation for the circle. This gives us x* +x* = 64, from which we find
x = 4 y2. (In our diagram, A is on the right side of the y-axis, so x must be positive.) Therefore, point A
is (4 ¥2,4 V2), and we can use the distance formula to find

AC = J(42 —8)2 + (42 - 0)? = 1/32 — 64 V2 + 64 +32 = V128 —64V2


=8V2 - V2.

Solution 2: Geometry. Our analytic geometry solution might inspire a purely A


geometric solution. At the point where we were finding the coordinates for A, we
suggested drawing an altitude from A to BC. Suppose we do that right from the
beginning. (We might also be inspired to do this by the 45° angle; drawing this B C
altitude builds a 45-45-90 triangle). x
From 45-45-90 triangle ABX, we have AX = XB = AB/¥2 = 4 V2. Therefore, we have CX = CB-BX =
8 — 4 V2. We can then apply the Pythagorean Theorem to AAXC to find AC = 8 \/2 — V2, as before. O

Concept: Solving a problem in two ways isa good way tocheck your answer. _ ZZ
he
Our two solutions in the last problem also show that analytic geometry solutions and purely geo-
metric solutions to the same problem are often closely related to each other.

Exercises

17.3.1 In Problem 17.10, we used slopes to determine that AABC is a right triangle. How else could we
have determined that AABC is a right triangle?
17.3.2 Find the area of the triangle bounded by the graphs of the equations y = x, y = —x, and y = 6.
(Source: AMC 12)

17.3.3 What is the shortest possible distance between a point on the graph of x? + y? + 6x — 8y = 0 and
a point on the graph of x* — 14x + y? + 10y + 65 = 0?
17.3.4 In AABC, we have AB = 8, BC = 12, and ZABC = 30°. Find AC.
17.3.5x Find the radius of each circle that passes through (9,2) and is tangent to both the x-axis and
the y-axis. Hints: 7, 487

17.4 Proofs with Analytic Geometry

In this section, we learn how to prove geometric facts using analytic geometry.

a TRATES
17.4. PROOFS WITH ANALYTIC GEOMETRY

Problems

Problem 17.14: In this problem, we prove that the midpoint of the hypotenuse of a right triangle is
the circumcenter of the triangle.
(a) Let the right triangle be AABC, with right angle at C. Why is it useful to choose C to be the origin
of the Cartesian plane?
(b) Suppose C is the origin. Why should we choose the axes of the Cartesian plane such that the
legs of AABC are along the axes?
(c) Let A be (a,0) and B be (0,b). What is the midpoint of the hypotenuse?
(d) Show that the midpoint of the hypotenuse is equidistant from the vertices of the triangle. |

Problem 17.15: In this problem, we prove that the median of a z


trapezoid is parallel to the bases of the trapezoid, and that the length (2c, 2d) (2b,2d)
of the median is the average of the lengths of the bases. Let the
trapezoid be WXYZ, where W is the point (0,0), X is the point (2a, 0),
Y is the point (2b, 2d), Z is the point (2c, 2d), and we havea > 0,b >,
and d > 0.
(a) Why is WXYZ a trapezoid? Which two sides are parallel?
(b) Find the coordinates of the midpoints of the legs of WXYZ. Why did we use 2a, 2b, 2c, and 2d
to describe the coordinates of the vertices of WXYZ, instead of using just a, b,c, and d?
(c) Show that the median of WXYZ is parallel to the bases of WXYZ, and equal in length to the
average of the lengths of the bases.
(d) Explain why our proof is valid by describing why for every trapezoid there is some choice of
origin and axes such that the vertices of the trapezoid can be described with the coordinates
(0,0), (2a,0), (2b, 2d), and (2c, 2d).

Problem 17.16: In this problem we prove that the medians of a triangle are concurrent. Let our triangle
be ABC, where A = (0,0), B = (2a,0), and C = (2b, 2c).
(a) Find the coordinates of the midpoint of AB. Find an equation whose graph is the line containing
the median from C to AB.
(b) Find an equation of the line whose graph is the line containing the median from B to AC.
(c) Find the intersection of the lines you found in parts (a) and (b).
Prove that the intersection point you found in part (c) is on the median from A to BC.

Problem 17.17: Suppose line k divides rectangle ABCD into two pieces of equal area. Prove that line
k passes through the intersection of the diagonals of ABCD.

Problem 17.14: Use analytic geometry to prove that the midpoint of the hypotenuse of any right
triangle is the circumcenter of the triangle.

er
ee SESS
CHAPTER 17. ANALYTIC GEOMETRY

Solution for Problem 17.14: Our first step is to set up the problem on the Cartesian plane. We must be
very careful when we do so. See if you can figure out why the set-up in the Bogus Solution below is not
sufficient:

Bogus Solution: & let our triangle be ABC, and we let C = (0,0), A = (4,0), and
= (0,6). Legs AC and BC are along the axes, so they are perpen-
ae Therefore, AABC is a right triangle with hypotenuse AB.
The midpoint of AB is (2,3). Let this midpoint be M. We have

AM = \/(2 - 4)2 + (3-0)? = V13,


BM= \/(2 -0)2 + (3 —6)2 = V13,
CM = y(2- 0)? + 3-0) = V1.
ey Since M is equidistant from the vertices of AABC, point M is the
a circumcenter of AABC.

This does indeed show why the midpoint of the hypotenuse of this particular right triangle is the
circumcenter of this right triangle. However, what about other right triangles with different side lengths?
Our ‘proof’ doesn’t cover every possible right triangle.

WARNING!! When setting up aTproof on the Cartesian plane, we2 must be very
careful. Our proof must address all possible configurations of the
Z problem, so our analytic geometry ge of the problem must
= 8: r allpossible configurations. Za

We might start by letting A = (a,b), B = (c,d), and C = (e, f). But that’s six variables! Hopefully we
can find a simpler representation of AABC.
When setting up a geometry problem on the Cartesian plane, we can start with a diagram, then add
the coordinate axes. We try to do so in a way that simplifies our problem. We can choose any point to
be the origin, so we choose one of the vertices of the right triangle to be our origin.

We choose the vertex of the right angle, point C, to be the origin because this allows us to choose our
axes so that the legs of the right triangle are along the axes. So, we let A = (a,0) and B = (0, b).

454
17.4. PROOFS WITH ANALYTIC GEOMETRY

Notice that a and/or b could be positive or negative. For example, in the case below, both a and b are
negative.

(a, 0) z

B (0,01

Since C is the vertex of the right angle, side AB is the hypotenuse. Let M be the midpoint of AB, so
that
M= (5.5)
oat
To show that M is the circumcenter of AABC, we must show that M is equidistant from all three vertices.
This is a job for the distance formula:

am= i[(2-.) (8-0) = oe

BM = (4-0) + (4-») = i

We have AM = BM = CM, so M is the circumcenter of AABC.

Notice that every step in our proof is valid even if a or b or both are negative. 0

Important: e,choose Z
your origin and your coordinate axes wisely. Typically, we do so in a
way that makes the coordinates of important points in the problem as
simple as possible.

Notice that our proof in Problem 17.14 only applies to right triangles, not to all triangles. The points
we chose to represent the vertices, (0,0), (4,0), and (0, b), are always the vertices of a right triangle (when
a and b are nonzero). We cannot use these three points as our vertices to prove a fact about all triangles,
since any triangle with vertices (0,0), (4,0), and (0,b) must be a right triangle.

Problem 17.15: Use analytic geometry to prove that the median of a trapezoid is parallel to the bases
of the trapezoid, and that the length of the median is the average of the lengths of the bases.

Solution for Problem 17.15: On page 211, we saw geometric proofs of these facts. Here, we try to find an
analytic geometry proof. We might start by letting the vertices of our trapezoid be (a, b), (c,d), (e, f), and
(g,h). Yikes. Eight variables! Maybe we can do better.
a
455
CHAPTER 17. ANALYTIC GEOMETRY

As noted in Problem 17.14, we can start with a diagram, then add the coordinate axes to set up a
proof on the Cartesian plane. Here, we choose the x-axis to contain one of the bases such that the other
base is above the x-axis. We then choose the leftmost of the two vertices on the x-axis to be the origin.
So, one vertex is the origin and the other is (a,0) with a > 0. We'll call the origin W and the second vertex
on the x-axis X, as shown below.

(Gai Was

We placed the x-axis such that the endpoints of one base are on the x-axis because this forces the
endpoints of the other base to have the same y-coordinate, since the bases are parallel. By choosing the
‘lower’ base to be x-axis, we force this common y-coordinate to be positive. Therefore, the other two
vertices of the trapezoid can be represented by (b,d) and (c,d), where d > 0 and b > c. Finally, we let
Y be (b,d) and Z be (c,d), so XY and WZ are the legs of the trapezoid. Now, we have represented our
trapezoid with only 4 variables, instead of 8.
Notice that every trapezoid can be represented in this way. Because b and c can take on negative
values, letting the vertices be W = (0,0), X = (a,0), Y = (b,d), and Z = (c,d) witha > 0,d > 0,andb>c
will also represent trapezoids in which Z and/or Y end up to the left of the y-axis. Two examples are
shown below. In each case, we have W at the origin, X to the right of W on the x-axis, and YX and ZW
as the legs (because b > c).

y
(c,d) (b, d)

But what about trapezoids in which the bases are not horizontal?

There’s no reason we have to make our x- Y y (b, d)


axis and y-axis horizontal and vertical! They
only have to be perpendicular. For exam- Z x
ple, consider the trapezoid with ‘slanted’ 2 X x
bases at right. We similarly make our axes = (2,0)
‘slanted’ so that the x-axis includes one base.

456
17.4. PROOFS WITH ANALYTIC GEOMETRY

Now, we're ready for our proof. We'll use the diagram in which uv
YZ is horizontal and both Y and Z are to the right of the y-axis, as
shown at right. (However, our proof will sevalid for all set-ups we have (c,d) _(b,d)
discussed.) The midpoint of leg WZ is Gy 4) and the midpoint of leg XY
is (2, 4). a median connects these two points, so the median is on
the line y = 4. Therefore, the median is horizontal, so it is parallel to the (a,0)
bases. We ah also use the coordinates of the midpoints of the legs to
determine that the length of the median is

a+b c¢ tht = 0
ce,
Eee ere

The lengths of the bases are WX = a and YZ = b —c, so the average of the lengths of the bases is also
(a+b-—c)/2. So, the length of the median equals the average of the lengths of the bases. Notice that
every step of this proof is valid for all the other arrangements we showed.
We also could have avoided the fractions in our solution by being a little clever about assigning
variables to coordinates. Because we are going to have to work with midpoints to get information about
the median, we know we'll have to divide expressions by 2. Therefore, we might make our coordinates
W = (0,0), X = (24,0), Y = (2b, 2d), and Z = (2c, 2d). Then, the endpoints of the median are (a + b,d) and
(c,d). No fractions! See if you can finish the problem from here. 0

“Whenv we set up a geometry problem involving midpoints on nthe Cartesian


& 3 plane, we often use 2a, 2b, 2c, etc. for coordinates, rather than just a, b, c,
ete. This helps us avoid expressions inalving fractions:when wee find
midpoints of segments in the problem. oo

Speaking of midpoints, these are often a sign that an analytic geometry approach might work. This
is because coordinates of midpoints are particularly easy to find.

Problem 17.16: Prove that the medians of any triangle are concurrent. : 3

Solution for Problem 17.16: We must prove a statement about ut


medians, so we’re working with midpoints. This might make
us consider analytic geometry. First, we have to set up the C = (2b, 2c)
problem on the Cartesian plane. Let AABC be our triangle, and
let the medians be AD, BE, and CF. We let A be the origin and
choose our axes so that B is on the x-axis, as shown at right.
Since we will have to find the coordinates of the midpoints of
the sides of the triangle, we let A be (0,0) and B be (2a,0). We
have no restrictions on C except that it cannot be on the x-axis,
so we let C be (2b, 2c), where c # 0. Again, we use 2b and 2c
because we will be finding the midpoints of the sides of the
triangle. Specifically, the midpoints of the sides of AABC are
= (a+ b,c), E = (b,c), and F = (a,0).

i CC
CHAPTER 17. ANALYTIC GEOMETRY

Now that we have the coordinates of two points on each y


median, we can find the linear equations whose graphs include
the medians. We then must show that there is a point (x, y) that G@i=(2b 22)
satisfies all three equations. First, we’ll find the equation for
AD. The slope of AD is c/(a + b) and AD passes through (0,0),
so an equation whose graph is AD is

y-—O= sat —().

Notice that the right side of this equation is undefined if a+b = 0.


Since the slope of AD is c/(a + b), we only have a + b = 0 if AD
is a vertical line. Specifically, if a + b = 0, then A and D are both
on the y-axis.

WARNING!! When using analytic geometry for a proof, be extra careful not to
ry
a overlook special cases. =

To take care of this special case, we multiply both sides of the equation by a + b to get

(a+ b)y =c(x — 0).

If AD is vertical, then a + b = 0, and AD is the graph of x = 0. Our equation above becomes x = 0 when
a+b=0,so this equation describes AD even i‘a3 is a vertical line.

Notice that we use (0,0) when writing a point-slope form of the equation of AD instead of using
(a + b,c). This makes our equation considerably simpler, and is one of the reasons we like to make as
many of our coordinates 0 as possible.

Concept: Don’t make analytic geometry proofs harder than they need to be! Zz

Similarly, the slope of BE is c/ (b — 2a) and BE passes through (2a,0), so an equation whose graph is
1S
Cc
y-O= (Ker i24)s
b —2a
Multiplying both sides by b — 2a gives

(b — 2a)y = c(x — 2a).

If b = 2a, then BE is the graph of x = 2a, since BE must pass through (2a,0) and (b,c). If b = 2a, then
(b—2a)y = c(x—2a) becomes 0 = c(x — 2a), which simplifies to x = 2a. So, the equation (b — 2a)y = c(x—2a)
describes BE even if BE isa vertical line.
And finally, the slope of ee is 2c/(2b — a) and GE passes through (a, 0), so for this line we have

20
(x —a).
Yao ae

458
17.4. PROOFS WITH ANALYTIC GEOMETRY

Again, we multiply both sides by the denominator of the slope to get

(2b — a)y = 2c(x — a),

which describes re even if ae is a vertical line.

We now have the three equations

>
A Tao) = xe
=
BE: y(G—24) =. x¢—2ac,
<—-?.:
CE oeW2U =F) oar 250
= 2A:

We can find the coordinates of the intersection of the graphs of the first two equations by substituting
xc = y(a + b) into y(b — 2a) = xc — 2ac. This gives us

y(b — 2a) = y(a + b) —2ae.

Subtracting y(a + b) from both sides gives us y(b — 2a) — y(a + b) = —2ac. Simplifying the left side then
gives —3ay = —2ac. We know that a # 0, so we can divide by —3a to find y = 2c/3. Since we also have
xc = y(a + b), we find x = y(a + b)/c = 2(a + b)/3.

So, we know that AD and BE meet at (2, 2c), All we have left is to see if this point is on CF. We
do so by substituting x = 2(a+ b)/3 and y = 2c/3 into our equation for CF from above. Substituting these
expressions into y(2b — a) = 2xc — 2ac gives us a left side of

Ze 4be ~ 2a¢
y(2b— a) = 7 (2b -a) = arse

and a right side of

2 Oo) c~2c = Abc 5 _ 4bc 2ac


2xcxe —— 2ac = 2| 3 —3 3 a 3 a ae
Seas 3

We see that the two sides are the same for the ordered pair (x, y) = (2a) 2c) so this point is on our
third median, as well! Therefore, we can conclude that the medians of a triangle are concurrent. 0

Problem 17.17: Suppose line k divides rectangle ABCD into two pieces of equal area. Prove that
atline
k passes through the intersection of the diagonals of ABCD.

Solution for Problem 17.17: It’s quite easy to describe a general rectangle with
coordinates. We let A be the origin and choose the axes so that two sides are
along the axes. Because ABCD is a rectangle, it is also a parallelogram, so its
diagonals bisect each other. This means we want to prove that k passes through
the midpoint of AC. Therefore, we let B = (2b,0) and D = (0,2d) (instead of
using B = (b,0) and D = (0,d)). Point C is on the vertical line through B and on
the horizontal line through D, so C = (2b, 2d). This means the midpoint of AC
is (D,d).

ui.
aie.ti-- 3 (:\:)
CHAPTER 17. ANALYTIC GEOMETRY

Having dealt with ABCD, it’s time to take care of k. First, we have to think D om
about ways in which k could intersect the rectangle. If k intersects two consecutive
sides (excluding the vertices of the rectangle), then the area of the triangle it k
forms is smaller than the area of the triangle formed by drawing a diagonal. For 4 XY B
example, in the diagram at right, we have [AXY] < [ABD] = [ABCD ]/2. So, it is
impossible for such a line to bisect the area of ABCD.
Following similar logic, if k passes through a vertex of ABCD and bisects the area of ABCD, then
line k must contain a diagonal of ABCD. So, line k clearly passes through the intersection point of the
diagonals in this case.

The other possibility is that k intersects opposite sides of the rectan- oy k


gle (excluding the vertices). Shown at right is our set-up on the Carte-
sian plane. Line k intersects AB and CD at P = (p,0) and Q = (q, 2d) as
shown. If k bisects the area of ABCD, then it splits it into two trape-
zoids with equal area. The bases of trapezoid APQD have lengths p (0,24)
and g,and APQD has height 2d, so we have

[APQD] = (24) (P=) = dip +9).


+

The bases of PBCQ have lengths 2b — p and 2b — q, so we have


(p,0) (2b,0) *
Be DAL eed
[PBCQ] = (2a)( 2 )= tab - p~ 9).
Setting these equal gives us d(p+q) = d(4b—p—q). Dividing by d gives us p+q = 4b—p—q,so2(p+q) = 4b.
Finally, we find that p + q = 2b. (We also could have used the fact the [APQD] = 5[ABCD] to show that
p+q = 2b.)

_But how does this help? We want to show that PG passes through the midpoint of AC. The midpoint
of AC is (b,d). Now that we know that p + q = 2b, we see that the midpoint of PQ is

(254, =) =e).
+q 0+2d

Therefore, line k passes through the intersection of the diagonals of ABCD. 0

17.4.1 In part (a) of Problem 17.4.2, we will use analytic geometry to prove a fact about parallelograms.
Which of the following groups of vertices can we use to represent the vertices of the parallelogram, to
prove a fact for all parallelograms? If a group cannot be used, describe a parallelogram whose vertices
cannot be represented by the given points. You can assume all variables represent nonzero numbers.
For example, if we wished to prove a result about all triangles, we cannot use (0,0), (a,0), and (0, b)
to represent the vertices of the triangle. This is because any triangle with these points as vertices is a
right triangle. So, these points cannot represent the vertices of any triangle that is not a right triangle.

(a) (0,0), (a, 0), (b, b), (b,c) (c) (0,0), (b, b), (b + a,b), (a, 0)
(b) (a, 0), (0, b), (-a, 0), (0, =p) (d) (0, 0),(b —4, c), (b,c), (a, 0)

ia
aE DD Ct
17.5. DISTANCE BETWEEN A POINT AND A LINE

17.4.2

(a) Use analytic geometry to prove that the diagonals of a parallelogram bisect each other.
(b)x Use analytic geometry to prove that if the diagonals of a quadrilateral bisect each other, then the
quadrilateral is a parallelogram.
17.4.3 Use analytic geometry to show that if the diagonals of a quadrilateral are perpendicular and
bisect each other, then the quadrilateral is a rhombus.

17.4.4x Notice that in Problem 17.16, the sum of the equations we found for AD and BE is the equation
for CF. Why could we have used this observation to deduce that AD, BE, and CF are concurrent?

17.5 Distance Between a Point and a Line

The distance between a point and a line is the shortest distance from the point to a point on the line.

Problem 17.19: In this problem, we find a formula for the distance between
the point (xo, yo) and the line Ax + By + C = 0, where A, B, C, xo, and yo are
all constants. We'll find a formula assuming that AB # 0; you'll be asked
to show that this formula also works when AB = 0 as an Exercise.
Let k be the line, P be the point (xo, yo), and Q be the foot of the altitude
from P to k. Let R be the point on k that has x-coordinate xo.
For the following parts, assume P is above line k on the Cartesian plane,
and that k has a positive slope. (The proofs for all other possibilities are
essentially the same.)
(a) Explain why QR/PQ = —A/B.
(b) Explain why PR = (Axo + Byo + C)/B. (Hint: Remember that R is on k, and has the same
x-coordinate as P. By how much do we have to increase the y-coordinate of R to get the
y-coordinate of P?)
; : ; _ |Axo + Byo + C|
(c) Show that the distance from point P to line k is apap,

Problem 17.20: Use your formula from Problem 17.19 to check your answer to Problem 17.18.

Problem 17.21: How many ordered pairs (x, y) satisfy both 14x — 48y + 49 = 0 and x° + y* = 1?

We start by finding the distance between a specific point and a specific line.

Problem 17.18: Find the distance between the point (2,7) and the graph of the line x = Sy = 1.

= ee
CHAPTER 17. ANALYTIC GEOMETRY

Solution for Problem 17.18: Let point P be (2,7) and let line k be the graph of the
line x — 3y = 1. Let Q be the point on k that is closest to P, so we have PO Lily ¢
Therefore, the product of the slopes of these lines is -1. Writing x — 3y = 1 in
slope-intercept form gives y = x/3 — 1/3, so the slope of k is 1/3. Therefore, the
slope of PQ is —3. Since Q is on the line through (2,7) with slope —3, it is on the
graph of y — 7 = —3(x — 2). Rearranging this equation gives 3x + y = 13.
We now have two equations whose graphs pass through point Q, so we find
the coordinates of Q by solving the system of equations

x= Sy = 1,
3x + y = 13.

Solving this system of equations gives (x, y) = (4,1), so point Q is (4,1). As a quick check, we note that
the slope of PQ is (1 — 7)/(4 — 2) = —3, so PQ is indeed perpendicular to k. (If this slope had not come
out to —3, then we would have known we made a mistake.)

Concept: Checking intermediate steps while working on problems will help you
a aa oeerrors.

Now, we use the distance formula to find

PQ = V/(2-4)2+ (7-1) = V4 4 36 =2 V10.

We could follow the procedure we used in Problem 17.18 to find the distance between any point and
any line. However, the procedure requires a lot of steps, and often the numbers will get pretty ugly.
Instead, let’s see if we can find a formula to use to find the distance between a point (xo, yo) and the line
that is the graph Ax + By + C = 0.

Problem 17.19: Find a formula for the distance between the point (xo, yo) and the line Ax+ By+C = 0,
where A, B, C, x0, and yo are all constants.

Solution for Problem 17.19: Let P be the point (xo, yo) and k be the graph of Ax + By + C = 0. We could
proceed as we did in our solution to Problem 17.18, by finding the slope of k, then finding the equation
of the line ge P that is perpendicular to k. Then, we find the intersection of k and this new line,
and then..
That looks like a lot of work. Before diving into pages of algebra, let’s see if we can use some
geometric insights to simplify the problem.

Concept: - Algebra is not the only tool we have to solve problems about the
t Cartesian |
plane. Combining geometric insights with algebra can lead to very nice
___ solutions. ee

a STD SN
17.5. DISTANCE BETWEEN A POINT AND A LINE

We start by drawing a diagram with P, k, and the perpendicular segment y


from P to k. Let point Q be the foot of the perpendicular from P to k. We
seek a length, and we have a right angle, so we think about building right (x0, Yo)
triangles. We build a right triangle by drawing a vertical segment from P to
line k, meeting k at point R, as shown.
Since P and R are on the same vertical line, the x-coordinate of R is x9.
Since R is on k, its coordinates satisfy the equation Ax + By + C = 0. We can
now use this equation to find the y-coordinate of R. Let yp be the y-coordinate
of R, so R is (Xo, yr). Because R is on k, we must have

Axo + Byr + C = 0.

Solving for yr gives us


a! —Axo -C
YR = B :

It’s still not clear how we can find the coordinates of Q. Right triangles
make us think of similar triangles. Drawing the altitude QS to the hypotenuse
of right triangle APQR gives us plenty of similar triangles.
But what good are they? We haven’t used any information about the
equation of our line yet, so we focus on that. When we do so, we see that we
can relate lengths of segments in our diagram to the slope of k. Specifically,
we see that the slope of line k equals SR/QS. (In our diagram, we assume the
slope of k is positive. Essentially the same approach will work if the slope of
k is negative.) We can also write the slope of k in terms of A and B. Putting
the equation Ax + By + C = 0 in slope-intercept form gives

oA
REE AEE
so the slope of k is -A/B. Therefore, we have

ae.
Ose ie

Our similar triangles give us a way to relate AQSR to APQR. We have AQSR ~ APQR, so

SR_ QR
User
Combining this with SR/QS = —A/B gives us QR/PQ = —A/B. Rearranging this gives PQ = (—B/A)(QR).

Unfortunately, it’s not so clear how to find QR. But we can find PR. Points P and R have the same
x-coordinate, so PR equals the difference in the y-coordinates of P and R:
—-Axp-C Axo +Byo+C
PR = yo— 9x = Yo —p — = >?

Se eee
CHAPTER 17. ANALYTIC GEOMETRY

Now, we're close. We have PR, and we can relate PQ to QR. Finally, we use the Pythagorean Theorem
to finish. We have PR? = PQ? + QR* and QR = —(A/B)PQ, so we have

(Axo
+ Byy
: )+C\” = PQ* + A?
ar PO“

Multiplying both sides by B? gives


(Axo + Byo + C)? = B*- PQ? + A? - PQ? = (A” + B*)(PQ”).
Dividing both sides by A? + B’, then taking the square root of both sides, gives

PQ= |Axp + Byo + C|


VA2
+ B2
We need the absolute value on the right side because length must be positive.
Our proof does not address the cases in which k is horizontal or vertical — you'll be asked to tackle
these cases as an Exercise. Moreover, our proof assumes that the slope of k is positive, and that P is
above the line. However, the proofs are essentially the same for other possible configurations in which
k is neither horizontal nor vertical.
Also, our proof doesn’t address the possibility that P is on k. We can quickly show that our formula
works in this case. If P = (xo, yo) is on the graph of Ax + By + C = 0, then we must have Axo + Byg + C = 0.
When we substitute Axp + Byo + C = 0 in our formula, then our formula gives us a distance of 0, which
is indeed the correct distance between point P and line k when P is onk. 0

Important: The distance between the point (x9, yo) and the graph of the equation |
Vv Ax+By+C=O0is
|Axo + Byo + C| |
VA? + B2

Let’s check our formula by using it to solve Problem 17.18.

Important: Whenever you derive a formula, you should test it on a specific case. |

a
Problem 17.20: Use your formula from Problem 17.19 to check your answer to Problem 17.18 by
using the formula to find the distance between (2,7) and the graph of x — 3y = 1.

Solution for Problem 17.20: See if you can figure out where we go wrong here:

Bogus Solution: The distance between the point (2,7) and the graph of the Slee
x-—3y'= Lis

Reo 7i1 18 8 Gear


vies V0 VIO V0 5
464
17.5. DISTANCE BETWEEN A POINT AND A LINE

Our answer in the Bogus Solution doesn’t match our answer to Problem 17.18. Is our formula wrong?

No! We applied it incorrectly.

WARNING! In our formula for thedistance betweenapoint and aline, the linear
rn equation is in the form Ax + By + C = 0, not Ax + By = C. |

So, we write our linear equation as x — 3y — 1 = 0. The distance between (2,7) and the graph of this
equation is
2=3-/7—~1 20 2 v1
| |= = 0 ‘ 0 =? V10,
V1? + 32 V 10 eV LOR LO
which matches our original answer. 0
Our formula can also be used to solve less straightforward problems.

Problem 17.21: How many ordered


pairs (x, y)satisfy
both 14x—48y+49 = 0 and 2 + y?2 = 1
Solution for Problem 17.21: We could solve the equation 14x—48y+49 = 0 for x in terms of y and substitute
the result into x* + y? = 1, but that will lead to some pretty ugly algebra. Before we do so, we think
about analytic geometry, because the equation x” + y* = 1 is in the standard form of an equation whose
graph is a circle. We like circles.

Concept: If an algebra problem has an equation whose graph is a circle, think about
()=s2 iusing analytic geometry.

The graph of the equation x + y* = 1 is a circle with radius 1. The graph of the
equation 14x — 48y + 49 = 0 is a line, but does it intersect the circle? The graphs of
both equations are shown at right. Unfortunately, we can’t quite tell if the line hits
the circle once or twice. And our graph might be off by just a little bit, so we can’t
even be sure if the line hits the circle at all!
Each point on the circle is 1 unit from the origin. So, if the line passes within 1
unit of the origin, it must intersect the circle twice, since it must go inside the circle.
The distance between (0,0) and the graph of 14x — 48y + 49 = 0 is
[14-0-48-0+49| _ 49
(142 + (482/142 + (—48)2
Rather than multiplying out the squares in the denominator, we remember the Pythagorean triple
{14, 48, 50}, so we know 14? + (—48)? = 50*. Therefore, the distance between the line and the origin is
49/50. So, the line goes inside the circle, which means it intersects the circle twice. This tells us that
there are two ordered pairs that satisfy both of the given equations. O

17.5.1 Find the distance between (3,4) and the graph of the equation 4x — 3y + 7 = 0.
ee
CHAPTER 17. ANALYTIC GEOMETRY

17.5.2 Find the coordinates of the points on the graph of y = 10 that are a distance of 5 from the graph
of x + 3y = 9.
17.5.3 Our proof of the formula for the distance between a point and a line does not address the cases
in which the line is either horizontal or vertical. Complete the proof of the formula by showing that it
holds if the line is horizontal or vertical.
17.5.4 Find the distance between the graphs of 3x = 4y + 8 and 3x = 4y + 17.
17.5.5 At how many points does the graph of 2x—3y = 48 intersect the graph of (x— 3)? +(y+2)* = 100?

17.6 Advanced Analytic Geometry Problems

||___ Problems
Problem 17.22: In this problem, we find the area of the region bounded by the lines 2x + 3y = 21 and
5x + 2y = 25 and the coordinate axes.
(a) Graph the two equations. Do we have a simple formula to find the area of the region?
(b) Dissect the region into pieces you know how to handle, and use these pieces to solve the problem.

Problem 17.23:
(a) What point is the image of rotating the point (—1,3) an angle of 180° about (—6, —7)?
(b) What point is the image of rotating the point (4,5) an angle of 90° clockwise about the point
(—4, 2)?
(c) What point is the image of reflecting the point (5, -3) over the line y = 8?
(d) What point is the image of reflecting the point (3, —1) over the graph of the line 2x —y + 5 = 0?

Problem 17.24: Triangles ABC and ADE have areas 2007 and 7002, respectively, with B = (0,0),
= (223,0), D = (680,380), and E = (689,389). In this problem, we find the sum of all possible
x-coordinates of A. (Source: AMC 12)
(a) What does the information about the area of AABC tell us about the possible locations of point
Al
(b) What does the fife nation about the area of AADE tell us about the possible locations of point
A?
_(c) Combine your observations from the first two parts. How many possible points A are there?
|_(d) What figure is formed when you connect all the possible points A you found in part (c)?
(e) Use your observation in part (d) to find the sum of all possible x-coordinates of A, without actually
finding any of the points that A could be.

Extra! Well done is better than well said.


im —Benjamin Franklin
460°
OO ee
17.6. ADVANCED ANALYTIC GEOMETRY PROBLEMS

Problem 17.25: Medians XA and YB of AXYZ are perpendicular. In AXYZ, we have both ZYXZ = 90°
and XY = 8. In this problem we find YZ.
(a) Set the problem up on the Cartesian plane. Choose your origin wisely, then find the coordinates
of X, Y, Z, A, and B. If you choose your origin wisely, you should only need one variable.
(b) Use the information in the problem to find the value of the variable.
(c) Find YZ.

Problem 17.26: A point P lies in the same plane as a given square of side 1. Let the vertices of the
square, taken counterclockwise, be A, B, C, and D. Also, let the distances from P to A, B, and C,
respectively, be u, v, and w. What is the greatest distance that P can be from D if u? + v* = w*? (Source:
AMC 12)

Problem 17.22: Find the area


of the region bounded by the lines 2x +3y = 21and 5x + 2y = 25 and
the coordinate axes.

Solution for Problem 17.22: We start by graphing the lines and shading the bounded region as shown
below.

Important: Just as with most geometry problems, our first step with many analytic
geometry problems is drawing a diagram.

PQRS is a quadrilateral, but it’s not one of our special types of quadrilateral. L ea © 7:
So, it’s not immediately obvious how to find the area of the shaded region. x |
Therefore, we use a tactic from our study of funky areas: we divide the shaded
region into pieces we know how to handle. We draw diagonal PR, which cuts
the shaded region into two triangles.
We could also have cut the region into two triangles by drawing QS, but we
can easily find the areas of APQR and APSR. The graph of 5x + 2y = 25 (line k in
the diagram) intersects the x-axis at (5, 0) and it intersects the graph of 2x+3y = 21
(line in the diagram) at (3,5). So, PQ = 5 and the altitude from R to PQ is 5.
This gives us [PQR] = (5)(5)/2 = 25/2.
Similarly, the graph of 2x + 3y = 21 meets the y-axis at (0,7), so PS = 7. The altitude from R to PS
has length 3, so [PSR] = (3)(7)/2 = 21/2. Finally, we have

[PQRS] = [PQR] + [PSR] = adoF = = 23.

We also could have solved this problem by dissecting PQRS into a trapezoid and a triangle. See if
you can solve the problem this way, as well. 0
Let’s try some transformations on the Cartesian plane.

Saar a SSC ttst~—<Sstsi‘“CSO:*~*~*”*”~””””””CC MR


CHAPTER 17. ANALYTIC GEOMETRY

‘Problem 17.23:
(a) What point is the image of rotating the point (—1,3) an angle of 180° about (—6, —7)?
(b) What point is the image of rotating the point (4,5) an angle of 90° clockwise about the point
(—4,2)?
(c) What point is the image of reflecting the point (5, —3) over the line y = 8?
(d) What point is the image of reflecting the point (3,—1) over the line 2x — y+ 5 = 0?

Solution for Problem 17.23:


(a) Let X be (-1,3), let Y be (—6, —7), and let Z be the image of X upon a 180° rotation about Y. Because
the angle of rotation is 180°, we have ZXYZ = 180°. So, point Y is on XZ. Furthermore, because Z
is the image of X upon rotation about Y, we have XY = YZ. Since Y is also on XZ, we know that
Y is the midpoint of XZ. So, if Z is (a,b), we have

(SF)
a+(-1) b+ *)
= 6
(-6,—7).

From this, we find a = —11 and b = —17, so the image of X upon 180° rotation about Y is (—11, —-17).
We also could have solved this problem by noting that because Y is the midpoint of XZ, we
know that the translation that maps X to Y also maps Y to Z. To get from X to Y, we go left 5
units and down 10 units. So, to get from Y to Z, we also go left 5 units and down 10 units, to
(=6=5 2-7 —10)-= (S117 17).
(b) Let (4,5) be point A and (—4, 2) be point B as shown in the diagram at
right. If point C is the image of A upon a 90° clockwise rotation about
B, then we must have AB = CB and AB 1 CB. The slope of AB is 3/8,
so we know that the slope of CB is —8/3. This means that C is on the
line through B with slope —8/3. But where on this line is it?
We can find point C by noting that to get from A to B, we go down
3 units and left 8 units. So, to get from point B to C, we go right 3
units and down 8 units, which means C is (—4 + 3,2 — 8) = (—1,-6).
This is illustrated in the figure at right. We first build right triangle
AABX so that AX is vertical and BX is horizontal. When we rotate
this triangle 90° clockwise, we get ABYC, in which BY = BX = 8 and CY = AX = 3. Therefore,
point C is 8 units below and 3 units to the right of B.
(c) The point (5,—3) is 11 units below the horizontal line y = 8, so its image is 11 units directly above
y = 8. Therefore, the image of (5, —3) upon reflection over the graph of y = 8 is (5,8 + 11) = (5,19).
We can quickly check this by noting that the graph of y = 8 is the perpendicular bisector of the
segment with endpoints (5,—3) and (5,19). This tells us that (5,19) is the image of (5, —3) upon
reflection over the graph of y = 8.

(d) Let line k be the graph of 2x — y +5 = 0. Let P be (3, -1), and let R be its image upon reflection over
k. Because R is the image of P upon reflection over k, PR and k are perpendicular. The slope of k is
2, so the slope of PR is -1/2. Therefore, R is on the line through P with slope —1/2. A point-slope
form of this line’s equation is

y~(-1) =-5(-3).
1

ae
aT 8
17.6. ADVANCED ANALYTIC GEOMETRY PROBLEMS

But which point on the graph of this line is R? If we can find the
point where PR meets k, we can use this point to find R, because
this intersection point is the midpoint of PR. Let the intersection
point of PR and k be Q. We have the equations of both PR and k,
so we can find Q by solving the system of equations

2x = yp= 0,
if
y+1= —5&— 3).

Adding these two equations gives 2x + 6 = —3 + 3. Solving this


equation gives x = —9/5. Substituting this into either of the above
equations gives y = 7/5. So, point Q is (—9/5, 7/5).
Now we're ready to find R. Let the coordinates of R be (xr, yr). Because Q is the midpoint of
PR, we must have
Cs Se) r (2 2)
me en ao GS5E oe
Solving (3 + xr)/2 = —9/5 and (—1 + yr)/2 = 7/5 gives us (xr, yr) = (—33/5, 19/5). We could also
have found the coordinates of R by noting that to get from P to Q, we go 24/5 units to the left and
12/5 units up. So, to get from Q to R, we go units 24/5 to the left and 12/5 units up from (—9/5, 7/5)
to (—33/5, 19/5).

Problem 17.24: Triangles ABC and ADE have areas 2007 and 7002, respectively, with B = (0,0),
C = (223,0), D = (680,380), and E = (689,389). Find the sum of all possible x-coordinates of A.
(Source: AMC 12)

Solution for Problem 17.24: The huge numbers in the prob-


lem make it hard to draw our graph to scale, so we start
with a rough sketch. We’ll have to use the information
about triangles ABC and ADE to determine the possible
points A. We start with the fact that [ABC] = 2007. Base
BC of this triangle has length 223, so if we let h; be the
length of the altitude to side BC of AABC, we have

_ 2ABC] _ 2(2007) _ ,.
een) a eon
This tells us that A is 18 units from BC. Since BC is along
the x-axis, we now know that A is on the graph of either
y = 18 ory = —18.
Triangle ABC gave us some information about A; let’s take a look at AADE. The distance formula
tells us that DE = 9 v2, so if we let hz be the length of the altitude from A to DE, we have

ee 2[ADE] _ 2(7002) _ 2(778) _ 778 V2.


DE SON 2
ee eee
469
CHAPTER 17. ANALYTIC GEOMETRY

So, point A is 778 V2 away from DE. We could find the equations of the two lines that are 778 V2 away
from DE, but that looks like it might be pretty difficult. Instead, we go back to our diagram and see if
we can find anything to simplify the problem.

Concept: One reason we draw diagrams for analytic geo


O=s2 ~~might inspire geometric shortcuts.

We’ve found that A must be on one of two lines


parallel to BC, and that these two lines are equidistant
from BC. Similarly, we found that A must be on one of
two lines parallel to and equidistant from DE. We add
all four of these lines to our diagram. The four possible
points A are where pairs of these four lines intersect. We
label these A, Az, A3, and Ay, as shown. A,A2A3Ayq is a
parallelogram because the opposite sides of A;A2A3A4
are along parallel lines.
We could find the coordinates of the vertices of the
parallelogram A;A2A3A4, but that looks like a lot of
work. Instead, we think a little bit to see if we can avoid all that work. Because A;A2A3Asg is a parallel-
ogram, we think about what’s special about parallelograms, and how to use these special properties to
find the sum of the x-coordinates of the vertices.

Specifically, we focus on parallelogram facts that might be used to discover something about this
sum of x-coordinates. This gets us thinking about midpoints, since finding the midpoint of a segment
requires adding coordinates. This, in turn, leads us to the fact that the diagonals of a parallelogram
bisect each other, which means they have the same midpoint. In other words, if x1, x2, x3, and x4 are the
x-coordinates of A;, Az, A3, and Aq, respectively, then the x-coordinate of the intersection of diagonals
A,A3 and A2Ay4 equals both (x; + x3)/2 and (x2 + x4)/2. We want x1 +.X2 +.x3 + x4, 80 we know we'’re close!

We let P be the intersection of diagonals A;A3 and y


A2Ay4, and let the coordinates of P be (xp, yp). We just
saw above that xp = (x; +x3)/2 = (x2+x4)/2. So, we have
Xy+x3 = X2+X4 = 2xp, which means that our desired sum
X1 +xX2+X3+X4 equals 4xp. All we have to do now is find
P. We know P is the intersection of diagonals A;A3 and
A2A4, but we would like to find a faster way to find P
than finding all the vertices of AjA2A3A4. Our diagram
gives us our slick approach. Because P is the midpoint
of both diagonals, it is equidistant from opposite sides
A, A and A3Ay4, and it is equidistant from opposite sides
AA; and A;Aq. Aha! DE is also equidistant from A,A2
and A3A4, and BC is equidistant from A2A3 and Aj. Aj So, P is the intersection point of BC and DE. This
point is easy to find!

470
17.6. ADVANCED ANALYTIC GEOMETRY PROBLEMS

Because B and C are on the x-axis, BC is the x-axis. So, all points on BC satisfy y = 0. This means
the y-coordinate of P is 0. The slope of DE is (389 — 380)/(689 — 680) = 1, and the line passes through
(680, 380), so DE is the graph of the equation y — 380 = 1(x — 680). Rearranging this equation gives
x — y = 300. Point P is on the graph of this line, and the y-coordinate of P is 0, so the x-coordinate of P is
300. Therefore, our desired sum of x-coordinates is 4(300) = 1200. o

We've seen how geometric insights can help us with analytic geometry problems. Let’s take a look at
a couple examples of how we can use analytic geometry to tackle challenging geometric problems. Just
as we saw when using analytic geometry for geometric proofs, one of the main keys in each problem is
conveniently setting up the problem on the Cartesian plane.

Problem 17.25: Medians XA andYB of AXYZ are perpendicular. IfZYXZ = 90° and XY = 8, find |

Solution for Problem 17.25: We have two big clues to try analytic geometry:
midpoints and perpendicular lines. We let X, the vertex of the right angle,
be the origin. This allows us to place Y on the x-axis and Z on the y-axis.
Because XY = 8, we can let Y be (8,0). Then, we can let Z be (0,2z) for
some value of z. We use 2z instead of just z because we know we'll be
working with midpoints in this problem. If we find z, we can use the
distance formula to find YZ, since

YZ = V/(2z -0)2 + (0-8)? = V4z2 + 64 =2 V2 +16.


We now can find |the coordinates of A and B in terms of z. Because A
is the midpoint of YZ, its coordinates are (4,z). Because B is the midpoint
of XZ, its coordinates are (0,z). But now what?

Concept: When you’re stuck on a problem, focus on information you haven’t used
& yet.

We haven’t used the fact that XA 1 YB. Because XA 1 YB, the product of the slopes of XA and YB
is —1. The slope of XA is z /4 and the slope of YB is —z/8. Therefore, we have

OG)
This gives us —z*/32 = —1, so z* = 32. Now we can find YZ using our formula from above. We have

YZ =2Vz2 +16 = 2 V32 + 16 = 2 V48 = 8 V3.

Problem 17.26: A point P lies in the same plane as a given square of side 1. Let the vertices of
the square, taken counterclockwise, be A, B, C, and D. Also, let the distances from P to A, B, and
C, respectively, be u, v, and w. What is the greatest distance that P can be from D if u2 +0? = w*?
(Source: AMC 12)

8888
aewm=m™':—_ :_—som r,s. over
CHAPTER 17. ANALYTIC GEOMETRY

Solution for Problem 17.26: Squares are easy to represent on the Cartesian plane.
We even know each side has length 1, so we can let the vertices of our square be
A =(0,0).8 = (1,0), C = (lhl) and Di (01):
Our given equation u? + v? = w* involves squares of distances. This is another
clue that analytic geometry might be helpful: squaring the distances gets rid of
the square root sign in the distance formula. We let the coordinates of P be (x, y),
so we can use the distance formula to find:
pir owes RP Ahes = 90s)
fst ay
Un) = PBad= Agel) -aly =O)
Wa PCa eel)?
+ (y a1)
Substituting these into u? + v? = w” gives us
e+yrt(x—-1Ph
ty =(x-1)% +(y-1)?.
The (x — 1)? terms cancel, and a little rearranging gives
r+yt+2y-1=0.

The graph of this equation is a circle. To learn more about this circle, we complete the square in y on the
left. This gives us
+ (y +1) ee

This means that (x, y) is on a circle with center (0, —1) and radius v2. Moreover, we can reverse our steps
above to see that any point on the circle satisfies the restrictions placed on point P in the problem. So,
now our problem is to find the point on this circle that is farthest from point D.

We add this circle to our diagram at right. Because both the center of the circle
and point D are on the y-axis, the point on the circle that is the farthest from D is
on the opposite side of the circle from D. In other words, it is the ‘bottommost’
point of the circle, which we label as Q in the diagram. Because D is 2 units from
the center of the circle, and the radius of the circle is V2, the distance between D
and Q is 2 + V2, which is therefore our desired greatest possible distance. 0

Exercises

17.6.1 What is the image of (5,6) under each of the following transformations:
(a) A 180° rotation about the origin. (d) A 90° counterclockwise rotation about (—3, 2).
(b) The translation that maps (2, —3) to (0,6). (e) Reflection over the graph of y = x.
(c) Reflection over the graph of x = —3. (f) Reflection over the graph of 2x + 3y = —5.
17.6.2 Two nonadjacent vertices of a rectangle are (4,3) and (—4, —3), and the coordinates of the other
two vertices are integers. How many different such rectangles are there? (Source: AMC 12)

i
TDD
Ee leeS
UMMAT
17.6.3 WXYZ is a square with WX = 6. Quarter circles centered at each Z
vertex of WXYZ are drawn inside the square as shown. The intersections of
these quarter circles form square ABCD shown.

(a) Find the area of ABCD with analytic geometry.


(b) Find the area of ABCD without analytic geometry.

17.6.4 ‘Two parallel lines intersect the x-axis at points that are 5 apart. The
same two lines intersect the y-axis at points that are 12 apart. How far apart are the two lines?
17.6.5 Let point C be (3,—2). Points A and B are on the graph of 2x — y = —7 such that AABC is
equilateral. Find AB.
17.6.6 Solve Problem 17.25 without using analytic geometry. Hints: 42
17.6.7x Given that x7 + y? = 14x + 6y + 6, what is the largest possible value that 3x + 4y can have?
(Source: AHSME) Hints: 2

17.6.8x Let AC and BD be two perpendicular chords of a circle with radius 8, and let the two chords
intersect at P. Find all possible values of PA? + PB* + PC? + PD?.

17.7 Summary

ee ee ee SS eS ee ee Ss a Sao]

Important: The graph of the equation y— ne= m(x-x4) isaline


is through (x,yn) weer
slope m. This is called a point-slope form of the equation.
The standard form of a linear equation is Ax + By = C, where, if |
possible, A, B, and C are integers, A is positive, and A, B, and C have no
common factors besides 1.
The slope-intercept form of a linear equation is y = mx + b, where m |
|

i nthe plane between the points (x1,41)and (xo,pi is


Important: The ‘distance in
|

| V (x2 — x1)? + (y2 — y1)?-

| This is often referred to as the distance formula.

Important: The‘midpoint ofthe segment with endpoints (x, 1) and (2, Yy2)is

V (am ne)
2 2

473
CHAPTER 17. ANALYTIC GEOMETRY

| Important: Iftwo non-vertical lines have the same slope, ‘then they are . parallel.
Conversely, if two non-vertical lines are parallel, then they have the
same slope.
If two non-vertical lines are perpendicular, then the product of their
slopes is -1. Conversely, if the product of the slopes of two lines is —1, |
oo neetwooe are Perens

‘Important: The standard form of an equation whose graph is


is a circle is

Oy
ty er

where h, k, and r are constants with r > 0. The center of the circle is (,k) |
i Tr.
and the radius of the circle is |

Problem Solving Strategies

|Concepts: "@ When the algebra in an analytic geometry problem starts to get ugly, |
| <<] try using geometric facts to simplify the problem.

| e When given the coordinates of the vertices of a triangle, check if the-


triangle is special in any way. Most notably, check if the triangleiis.
|

a right triangle. If it is, this fact will probably simplify the problem,
because we know so much about right triangles.

Interpreting an equation as a geometric figure on the Cartesian plane


can often help us solve problems.

When setting a geometry problem up on the Cartesian plane, choose


the origin and the axes in a convenient matter. Often this means
letting the origin be the vertex of a right angle in the problem, so that
the sides of the angle are along the axes.

e Solving a problem in two ways is a good way to check your answer.

e Checking intermediate steps while working on problems will also


help you catch a lot of errors.
e When we set up a geometry problem involving midpoints on the
Cartesian plane, we often use 2a, 2b, 2c, etc., for coordinates rather
than just a, b, c, etc. This helps us avoid expressions involving
fractions when we find midpoints of segments in the problem.

ae Continued on the next page. . .

474
A
ead)
a REVIEW
ae eerPROBLEMS
MEE MO

Concepts: .. . continued from the previous page

e Algebra is not the only tool we have to solve problems about the» |
Cartesian plane. Combining geometric insights with algebra can
lead to very nice solutions. |
e When stuck on a problem, focus on information that you haven't |
used yet. |
If an algebra problem has an equation whose graph is a circle, think|
about using analytic geometry.

| e One reason we draw diagrams for analytic geometry problems is |


pasSata ae Hacks ae ee |

HL_Review Prostems ll
17.27. Find an equation, in standard form, whose graph is the line through (—5, 2) that is perpendicular
to the graph of 5x — 2y + 7 = 0.
17.28 Find the area of the triangle with vertices (3, 4), (8,4), and (—6, —6).

17.29 Let A be (—2,4), B be (—7,1), and C be (—1, —5).

(a) Find an equation whose graph is a line that contains the median from A to the midpoint of BC.
(b) Find the length of the median from A to BC.
(c) Find the length of the altitude from A to BG
(d) Find the area of AABC.

17.30 Find the constant a such that the graph of (2a — 3)x + 3a — 1)y = 3 is parallel to the segment with
endpoints (2, —4) and (—1, 2).

17.31 Graph the equation x? — 6x + y” + 4y = —3. Find the area enclosed by your graph.
17.32 Let P be (-2,5) and Q be (8, -3). Find the standard form of the equation of the circle with PQ as
a diameter.
17.33 Let point X be (10,1), and let C be the graph of (x - 1)* + (y+ 2)* = 10.

(a) Let Y be the point on C that is closest to X. Find XY.


(b)x Find the coordinates of Y.

17.34 Find all possible values of a such that the distance between the point (a,3) and the graph of
5x —12y = 8is 7.
17.35 Point A is (—3,5). The midpoint of AB is (4,2) and the midpoint of AC is (3,1). What is BC?
a C—O
CHAPTER 17. ANALYTIC GEOMETRY

17.36 Point O is the origin and point B is (0, 13). Point A has positive coordinates such that AO = 5 and
AB = 12. Find the coordinates of A.
17.37 What is the image of (5, —2) when reflected over each of the following:
(a) The x-axis.
(b) The graph of x = —3.
(c)x The graph of 2x — y = -1.
17.38 Find the length of the segment that is a chord of both the graph of x? + y* = 36 and the graph of
x2 +2x
+ y? = 30.
17.39 Use analytic geometry to show that if AB = AC in AABC, then the altitude from A to BC is also
the median from A to BC.
17.40 Two adjacent vertices of a square are (—4,5) and (—3,7). What are all possible points besides
these two that could be vertices of this square?
17.41 Show that the image of the point (a,b) upon reflection over the line y = x is (b, a).
17.42 Point X is (2,3) and point Y is (6,3). Find all possible points P such that AXYP is equilateral.
17.43 Use analytic geometry to show that the sum of the squares of the lengths of the diagonals of a
parallelogram equals the sum of the squares of the lengths of the sides of the parallelogram.
17.44 Find the equation whose graph is the circle that passes through the points (—1,5), (4, 4), and (5, 9).
17.45 In triangle ABC, let D, E, F be the midpoints of BC, AC, AB, respectively.
(a) Show that triangles ABC and DEF have the same centroid.
(b) Let G1, G2, G3 be the centroids of triangles AFE, BDF, CED, respectively. Show that triangles ABC
and G;G2G3 have the same centroid.

17.46 Points X and Y are on AB such that AX = XY = BY. If Ais (0,9) and B is (4,0), what is the slope
of the line through X and the origin?
17.47 Point P is (6,2). Points Q and R are on the x-axis such that ZPQR = 30° and ZPRQ = 60°. What is
the area of APQR?

17.48 Point X is (4, —3), point Y is (—2,5), and point Z is (c, 3).

(a) For what values of c is AXYZ isosceles?


(b) For what values of c is AXYZ a right triangle?

17.49 Find an equation whose graph is a circle that passes through the origin and both the x-intercept
and the y-intercept of the graph of x — 3y = 18.

RO ea eee
CHALLENGE PROBLEMS

17.50

(a) The graph of the equation 3x — y = 0 is rotated 90° clockwise about the origin to produce line k.
Find an equation whose graph is line k.
(b) The graph of the equation 3x — y = 6 is rotated 90° clockwise about the origin to produce line k.
Find an equation whose graph is line k.
17.51 The graphs of x* + y* = 4+ 12x + 6y and x? + y? =k +4x + 12y intersect when k satisfies a < k < b,
and for no other values of k. Find b — a. (Source: AMC 12)
17.52 How many points are equidistant from the x-axis, the y-axis, and the graph of the equation
x+y = 2? (Source: AMC 12)

17.53 Point X is on diameter PQ of a circle. Prove that if AB is a chord of the circle parallel to PQ, then
XA? + XB? = XP* + XQ?.
17.54 Let U be the point (—4, 3) and V be the point (2, —1).

(a) Let A be (—1,3). Let A’ be the image when A is rotated 90° clockwise about U. Let A” be the image
when A’ is rotated 90° clockwise about V. Find A’’.
(b) Let B be (7,—2). Let B’ be the image when B is rotated 90° clockwise about U. Let B” be the image
when B’ is rotated 90° clockwise about V. Find B”.
(c) Find the midpoint of AA” and the midpoint of BB”. Notice anything interesting?
(d)x Let C be (a,b). Let C’ be the image when C is rotated 90° clockwise about U. Let C” be the image
when C’ is rotated 90° clockwise about V. Show that your observation in part (c) can be used to
quickly find C” for any a and b.

17.55 Let ABCD bea square, and let P be a point onside CD. Construct ae B
squares BPWX and APYZ externally on BP and AP, respectively, as x
shown. Let O, O;, and O2 be the centers of squares ABCD, BPWX, and Zz
APYZ, respectively. Show that quadrilateral OO; PO? is a parallelogram.
P
17.56x Find the largest value of y/x for pairs of real numbers (x, y) that Ww
satisfy (x — 3)* + (y — 3)* = 6. (Source: AHSME) Y
17.57*x Let A,A2 and B,B2 be two perpendicular chords of a circle with radius r, each passing through a
fixed point P. Show that (A;A2)* + (B1B2)? is independent of the position of the chords. (In other words,
show that for all pairs of perpendicular chords of this circle that meet at P, the sum of the squares of the
lengths of the chords is the same.)

eo would Iturn back the clock and devote to Lhe or some ae languag .

There iis oneShean anil elena ints my head


| ‘aick et ‘tobe i
o ofmy |brain since that early time: “The square on the hypotenuse ofa ri
t sides!” There
a es| fothesumathe bbeon theother two it sticks, bu
CHAPTER 18. INTRODUCTION TO TRIGONOMETRY

Law of Cosines

New facts often trigger new ideas. — Alex F. Osborn

CHAPTER ARTA OT EE Lanne


ee ae

Introduction to Trigonometry

Trigonometry begins with the relationships among the angles and the sides of a triangle. We’ll start by
exploring these relationships for right triangles, and then we’ll expand our study to all triangles.
The main purpose of this chapter is to give you a little taste of trigonometry, so that it won’t be
brand-new when you dive more deeply into the subject. We’ll explore trigonometry in much more
detail in later textbooks. Hopefully, at that time, this taste will make you a little more comfortable with
trigonometry as you learn to apply it more broadly.

18.1 Trigonometry and Right Triangles

. Problems >

Problem 18.1: You may use a calculator on this problem.

oe ae ne
(a) Inthe diagram above, BC ~ 3.5. What is EF to the nearest tenth?
(b) Use the given information about AABC to find YZ/XY to the nearest 0.01.

478
18.1. TRIGONOMETRY AND RIGHT TRIANGLES

Problem 18.1: You may use a calculator on this problem.

A LU A C1 4
C D E "ss Z
(a) In the diagram above, BC ~ 3.5. What is EF to the nearest tenth?
(b) Find YZ/XY to the nearest 0.01.

Solution for Problem 18.1:


(a) We have AABC ~ ADEF by AA Similarity. Therefore, we have BC/AB = EF/DE, so

EF =(D(DE)
E) (45)
BC
(==) ==(12)
02) (>)
(5) =47~47.
3.5

(b) Wehave AABC ~ AXYZby AA Similarity. Therefore, we have YZ/XY = BC/AB ~ 0.39. Notice that
EF/DE is also approximately 0.39, which shouldn’t be surprising because our triangle similarities
mean that YZ/XY = BC/AB = EF/DE.

The ratio
length of a leg opposite an acute angle of a right triangle
length of the hypotenuse of the right triangle
is So common and important that we give it a name. This ratio is called the sine of the acute angle. We
denote the sine of ZA as sin A.

In AABC at right, we have ZC = 90°, BC = a, CA = b, and AB = c. So, our B


definition of sine gives us
_ length of leg opposite A _ a & a
~ lengthofhypotenuse = c_

In Problem 18.1, we used similar triangles to show that in all right triangles b C
with an acute angle of 23°, the ratio of the length of the leg opposite the 23°
angle to the length of the hypotenuse is the same. Specifically, we found that this ratio is approximately
0.39. So, we write sin 23° ~ 0.39 to indicate that in any right triangle with an acute angle of 23°, the ratio
length of the leg opposite the 23° angle
length of the hypotenuse

is approximately 0.39.
We used similar triangles in Problem 18.1 to see that the ratio denoted by sin 23° is the same for an
acute angle measuring 23° in any right triangle. Similarly, we can show that for any value of x between
0° and 90°, the ratio denoted by sin x is the same in any right triangle with an acute angle measuring x.
aCC CC CO
CHAPTER 18. INTRODUCTION TO TRIGONOMETRY

This is why we define sine for an angle measure, not for a specific triangle — in any right triangle with
acute angle 23°, the side length ratio referred to by sin 23° is the same.

As noted earlier, we denote the sine of ZA as simply sin A, rather than writing sin ZA. Because
sine is a function of an angle measure, we'll assume that the Z symbol is implied when writing sin A.
However, we typically include the angle symbol when identifying an angle with three letters, so we'll
write sin ZXYZ rather than sin XYZ. These conventions are common but not universal. Some sources
will write sin XYZ instead of sin XYZ, and some write sin ZA instead of sin A.

The ratio denoted by sine is not the only useful ratio in a right triangle. The cosine of an acute angle
in a right triangle equals
length of the leg adjacent to the angle
length of the hypotenuse

We denote the cosine of ZA as cos A, so in our diagram at right, we have B

length of the leg adjacent to the angle _ b


cosA = =.=. C
length of the hypotenuse ‘6 a

Finally, the tangent of an acute angle ina right triangle is the ratio of the sine 4
of the angle to the cosine of the angle. We denote the tangent of ZA as tan A, so
we have Dae
sin
tan A = :
ie cos A

Sine, cosine, and tangent are examples of trigonometric functions. We often refer to problems
involving trigonometric functions as an area of math called trigonometry.
Just as we did with sin x, we can show that the ratio denoted by cos x is the same in any right triangle
with an acute angle with measure x, as is the ratio denoted by tan x.

Therefore, the value sin 23° is a number, just like v7 is anumber. The same is true for any trigono-
metric function of an acute angle. For most angles, we need to resort to a calculator to quickly find
the value of the sine, cosine, or tangent of the angle. But there are some angles for which we can use
our understanding of special right triangles to find the sine, cosine, and tangent of the angle without a
calculator.
Problems i

|Problem 18.2: Show that the tangent of an acute angle in a right triangle equals the ratio

length of the leg opposite the angle


length of the leg adjacent to the angle’

Problem 18.3: Suppose AABC is a 45-45-90 right triangle with ZC = 90°. Use AABC to find sin 45°,
cos 45°, and tan 45°.

<o°:CUCNStS”S”t*~<‘“ ‘SOO eee eee


18.1. TRIGONOMETRY AND RIGHT TRIANGLES

Problem 18.4:
(a) Find sin 30°, cos 30°, and tan 30°.
(b) Find sin 60°, cos 60°, and tan 60°.

Problem 18.5: In APQR, we have ZP = 90°, PQ = 3, and OR = 7.


(a) Find sinQ and cos R.
(b) Find tan Q.
(c) Find (sin R)? + (cos R)*. Note: We often write (sin R)? as sin? R, and we do likewise for other
positive powers of trigonometric functions.

Problem 18.6: Suppose that ZA is an acute angle.


(a) Explain why sinA = cos(90° — A).
(b) Explain why sin? A + cos? A = 1.

Problem 18.7: Let AABC be a right triangle with ZC = 90°.


(a) Show that sinA and cosA must be between 0 and 1.
(b) Show that tanA must be greater than 0, but that tanA has no upper bound. In other words,
show that for any positive number x, there is an angle A such that tanA = x.

|Problem 18.2: Show that the tangent of an acute angle ina right triangle equals the ratio
}

length of the leg opposite the angle


length of the leg adjacent to the angle’

Solution for Problem 18.2: Let AABC be right triangle with ZC = 90°, BC = a, B
AC = b, and AB = c, as shown at right. Since tangent is the ratio of sine to
cosine, we have

sinA a length of the leg opposite the angle


t A = =
see cosA Tha length of the leg adjacent to the angle’
aAIsS|a18
A
b G
Oo

Putting the result of Problem 18.2 together with our definitions of sine and cosine, we have the
following for right triangle AABC above:

Important: length of leg opposite A a


length of hypotenuse c
length of leg adjacenttoA _ 0b
length of hypotenuse C
length of leg opposite A
length of leg adjacent to A
CHAPTER 18. INTRODUCTION TO TRIGONOMETRY

Since these are definitions, we have no choice but to simply memorize them. One common mnemonic
for memorizing them is ‘SOHCAHTOA‘’:
Sine = Opposite/Hypotenuse; Cosine = Adjacent/Hypotenuse; Tangent = Opposite/Adjacent.
We can’t quite do SOHCAHTOA justice in print — it’s a much more effective memorization tool when
spoken, “So-cah-toe-ah.”

We can find the values of trigonometric functions of some angles with our understanding of special
right triangles.

Problem 18.3: Find sin 45°, cos 45°, and tan 45°.

Solution for Problem 18.3: At right is 45-45-90 triangle ABC with right angle atC. The B
ratio of a leg of a 45-45-90 triangle to its hypotenuse is 1/-V2, so we have iS

sin45° = sinA = ae ale bs


7 AB y2
We usually try to avoid writing fractions with a square root in the denominator. DS A
We can write 1/2 without a square root in the denominator by multiplying the
numerator and denominator by Y2:

et eee emer) weet)


sin 45° = —- = —= - —= =—
es Pen Chee
aay P
Similarly, we have cos 45° = cosA = ACs ara 2 aoe
Ae a 5; Bs Finally, since the legs of a 45-45-90 triangle have
the same length, we have
BC
an 45°
t = tan A = AC
— = 1|

Solution for Problem 18.4: We start with AABC at right, in which ZA = 30°, ZC = 90°, A
and ZB = 60°. Because AABC is a 30-60-90 right triangle, we have BC/AB = 1/2, so

: es] Oe 1 «eG!
sing = fig and cos 60° = ARs: 30

We also have AC/AB = 3/2, so

iy
sin 60
ae
eo NG AC_ v3
AB ae9 and cos 30 OS ee
AB a7 : 60° f
ie
Since AC/BC = ¥3/1, we have

AC
tan 60° = — = v3 and ta 30° eee ee oe eS ENS
BC ACs 3 37°N3° 33

482
18.1. TRIGONOMETRY AND RIGHT TRIANGLES

The table below shows the values of sine, cosine, and tangent for the special angles we have explored
in this section. The 45-45-90 triangle and the 30-60-90 triangle below exhibit the relationships among
the sides of each triangle that we used to find these values.

Angle | sin | cos | tan


A
BU Da] AP ae Ge et ie
° 3 v3

ee o |ete
| yi v2 1 2

cr | 8} i |vB NS tee
VA V2 (a V3

You shouldn’t have to memorize these! These are just another way of expressing the ratios of sides
in 45-45-90 triangles and in 30-60-90 triangles.
If we have enough information about the sides of a not-so-special right triangle, we can evaluate the
trigonometric functions of its angles, too.

Problem 18.5: In APQR, we have ZP = 90°, PQ = 3, and QR = 7.


(a) Find sinQ and cosR.
(b) Boda.
(c) Find (sin R)* + (cos R)?.

Solution for Problem 18.5:


(a) We first use the Pythagorean Theorem to determine that PR = 2 Nate 38)
Then, we draw our diagram with the sides labeled as at right. We have
7
sinQ = PR/QR = 2 V10/7 and cosR = PR/QR = 2 10/7. 3
(b) We have R
= PR _ 2Vi0
tan g =i Bu
x 2 V10

(c) We first find sin R= PQ/QR = 3/7. Then, we have


2
“4 R)*R\2 + (cosR)*Die 3 ie (2010
ab 9 ugeae:
40
(sin = (=) +( ,Esp Tes
49cont
+ 49 as1

We typically write (sin R)* and (cos R)? as sin?R and cos? R, respectively, and similarly for other
positive integer powers of trigonometric functions. So, we’d write the result we found in part (c) as
sin?R+ cos?R = 1.
Parts (a) and (c) above suggest a couple interesting relationships between trigonometric functions.
Let’s investigate.

ee
a SS CC Cl AS
CHAPTER 18. INTRODUCTION TO TRIGONOMETRY

Problem 18.6:Suppose
that ZAisan acute angle.
(a) Explain why sinA = cos(90° — A).
_ (b) Explain why sin’ A + cos* A = 1.

Solution for Problem 18.6:


(a) At right is right triangle AABC with acute angle ZA. We have sinA = B
BC/AB. We also have ZB = 90° — ZA, so

5 BC
cos(90° — A) = cosB = AB’ 3 A

Therefore, we have sinA = cos(90° — A) for any acute angle ZA.


(b) We have sinA = BC/AB and cosA = AC/AB, so

aD) ee BC2 AC? = BC*+AC :


sin’ A + cos°A = AB2 + AB2 AB?

The Pythagorean Theorem gives us BC? + AC? = AB’, so we have

Gay) 2
sin* A + cos’ A =
BE2BAC2
AB?
=Se ABRT
Se
AB?

A trigonometric identity is a statement involving trigonometric functions that is true for all values
of the angles in the statement. In Problem 18.6, we proved two trigonometric identities for acute angles
Tae

Important: ; sin? Ae cos* A=,

sinA = cos(90° — A).

Once again, note that sin* A = (sin A)? (and likewise for all trigonometric functions raised to positive
integer powers).

Problem 18.7: Let AABC be a right triangle with ZC = 90°.


(a) Show that sinA and cosA must be between 0 and 1.
(b) Show that tanA must be greater than 0, but that tanA has no upper bound.

Solution for Problem 18.7:


(a) In AABC at right, we have sin A = BC/AB. Because AB is the hypotenuse B
of AABC, it is longer than both legs. Specifically, we have AB > BC,
so BC/AB < 1. Furthermore, both lengths are positive, so BC/AB > 0.
Combining these inequalities gives us 0 < sinA < 1.
Similarly, we have cos A = AC/AB and AB > AC > 0,s00 < cosA <1. Cc

484
18.1. TRIGONOMETRY AND RIGHT TRIANGLES

(b) Because tanA = BC/AC and BC and AC are positive, we have tanA > 0. To see that tanA can
take on any positive value, let AC = 1. Then, we have tanA = BC. We have no restrictions on
BC. For any positive number x, we can build a right triangle with legs AC = 1 and BC = x, so
that tanA = x. Therefore, tan A can take on any positive value, so there is no upper bound on the
possible values of tan A.

i acute, then 0<sinA < ie0 < cos A < 1 and tanA > iy There iisad|
Important: | nie LA is
NW no upper bound on the value of tan A. |

Note that while we proved that tanA can take on any positive value, we did not show that sinA and
cos A can equal any positive number between 0 and 1. You'll have a chance to prove this in the Exercises.
We’ve seen that we can evaluate trigonometric functions for some special angles without a calculator.
However, we use a calculator or a computer to find a value of the trigonometric functions of most angles.
For the next set of problems, you are free to use your calculator.
aaa

WARNING! Degrees are not the only units for angles. If you use your calculator to
‘“S compute sin 30° and you don’t get 0.5 (or 1/2) as the result, then either
you aren’t using your calculator correctly, or your calculator is set to _
use units besides degrees for angles. If your calculator is set to use
some other units of measure besides degrees, reset your calculator to
degrees before continuing.
ee eS

||
Problems
ig
[Problem 18.8: Find AC and BC in the diagram at right to the nearest tenth. =

Problem 18.9: Right triangle ABC has 2ABC= 90°. Let the midpoint of AB be M. Thepecan
bisector of AB intersects AC at D, and sin ZMDB= 1/5. If MD= 1, then what is ACE a

Problem 18.10: I am 500 feet from a building. When I look at the—


top of the building, I look upward at an angle of 7° from horizontal.
|When I look at the point where the building touches the ground, I
look downward at an angle of 2° from horizontal. How weis the |
building to the nearest foot?

ee eee
CHAPTER 18. INTRODUCTION TO TRIGONOMETRY

| Problem 18.8: Find AC and BC in the diagram at right to the nearest tenth. C

Solution forProblem 18.8: We have sinC = AB/AC, so we have AC = AB/(sin C). Since sin C = sin59° ~
0.857, we have a, re
= —— ~ ~—— 7140.
Sit)ton aeOZ, bey

We can now use either the Pythagorean Theorem or trigonometry to find BC. Since we're in the
trigonometry chapter, we'll use cos C. Since cos C = BC/AC, we have BC = AC cos C. Since C = 59°, we
have cos 59° ~ 0.515, so BC = (14.0)(.515) = 7.2. O

Problem 18.9: Right triangle ABC has ZABC = 90°. Let the midpoint of AB be M. The perpendicular
bisector of AB intersects AC at D, and sin ZMDB = 1/5. If MD = 1, then what is AC?

Solution for Problem 18.9: Oh no! There’s no diagram! What will we do? A
We start by drawing our own diagram. Don’t let wordy, seemingly com-
plicated geometry problems scare you. Take the time to draw a diagram for 4 D
the problem, label it with information you know, then get to work. As you
discover more about the diagram, add what you learn to the diagram.
aE c
Here, we start with the perpendicular bisector of AB. The perpendicular B
bisectors of a triangle pass through the circumcenter of the triangle. The circumcenter of a right triangle
is the midpoint of its hypotenuse. So, point D is the midpoint of AC. Therefore, if we find AD, DC, or
BD, we can easily find AC.
We focus on the information we haven’t used yet: MD = 1 and sin ZMDB = 1/5. We’d like to find
BD. From sin ZMDB = 1/5, we have
NIB ASS
BD 5
Unfortunately, we don’t know MB. However, we do know DM, and we know that

DM
Bp = cos ZMDB.

So, if we can find cos ZMDB, we can find BD from this equation. We aren’t given cos ZMDB, but we do
know sin ZMDB, and we know that

cos* ZMDB + sin? ZMDB = 1.


Therefore, we have cos? ZMDB = 1 — sin? ZMDB = 24/25. We must have cos ZMDB > 0, so we have
cos ZMDB = 2 ¥6/5. From DM/BD = cos ZMDB, we have BD/DM = 1/(cos ZMDB), so

i aad a pee Aa eG
cosLMDB 26 276 V6 12°
486
18.1. TRIGONOMETRY AND RIGHT TRIANGLES

Finally, we have

aes
ol
AC = 2AD = 2BD =

Our process of finding BD using MD = 1 and sin ZMDB = 1/5 in right triangle MDB is an example
of the fact that:

|Important: If we have the length of a side of a right triangle and the value ofa
| NW trigonometric function of either of the acute angles of the triangle, then |
a we can find all the side lengths of the triangle. |

Trigonometric functions are used for surveying land and measuring large distances. Here’s an example
how.

Problem 18.10: I am 500 feet from a building. When I look at the 3


top of the building, I look upward at an angle of 7° from horizontal. ee
When I look at the point where the building touches the ground, I ee
look downward at an angle of 2° from horizontal. How tall is the 2°
building to the nearest foot?

Solution for Problem 18.10: We label the diagram as shown at right. c


We seek the length of CD. While we can’t find CD directly, we
can use trigonometry to find BC and BD. Specifically, we have i
tan ZCAB = BC/BA, so BC = BAtan7° ~ 61.4. We also have - aS eee
tan ZBAD = BD/BA, so BD = BAtan2° ~ 17.5. Therefore, to the 2s D
nearest foot, the height of the building is CD = CB + BD ~ 79 feet. O

Sine, cosine, and tangent are not the only trigonometric functions. Three more trigonometric func-
tions are cosecant (denoted csc), secant (denoted sec), and cotangent (denoted cot). These are defined
as follows:
q
CSC as
sin x
1
SeG a=
cos x
i
COta—
tan x

There’s no need to memorize these right now — you'll have enough experience with them in later texts
that you'll come to know them well.
re fg
iz Exercises —

18.1.1 Let AABC be a right triangle with ZABC = 90°, AB = 12, and AC = 16. Find sinA, cos A, and
tan A.

ee
CCC CCC ;FSSC
CHAPTER 18. INTRODUCTION TO TRIGONOMETRY

18.1.2 Find the missing sides in the triangles below to the nearest 0.1.
P
x

Oy O yb,
Q 16 A us
18.1.3 Letxbeareal number such that 0 < x < 1. Prove that there exists an acute ZA such that sin A = x,
and that there exists an acute angle ZB such that cos B = x. Hints: 1
18.1.4 Show that if 0 is the measure of an acute angle, then cos 6 = sin(90° — 8).

18.1.5 In triangle ABC, we have ZB = 90° and sinA = 5/7. Find tanC.

18.1.6 Let AB be a diameter of @O, where AB = 2. Suppose AX is tangent to ©O, and that AY bisects
LXAO. The angle bisector of 4YAB intersects ©O at point Z (where A and Z are different points). Find
AZ to the nearest hundredth.
18.1.7x Without using a calculator, evaluate sin 15°. Hints: 513, 536

18.2 Not Just For Right Triangles


So far, we have only defined trigonometric functions for angles in right triangles. However, these
functions are useful for far more than just right triangles. But to see why, we'll start with a special right
triangle.

Problems _

ABN
a aa
18.2. NOT JUST FOR RIGHT TRIANGLES

1Problem 18.11: In AABC, let ZC = 90°, AB = 1, and ZA = @. Find. expressions for AC and BC in terms |
of
L
0.

Solution for Problem 18.11: Because AB = 1, we have sinA = BC/AB = BC. B


Therefore, BC = sinA = sin @. Similarly, we have cos A = AC/AB = AC, so
AC = cosA = cos @. These side lengths are shown in the diagram at right. : sin 6
Oo a]
We can extend our observation in Problem 18.11 to build a definition of A cos 8 C
sine and cosine for angles that are not acute. First, we'll take a slightly different look at depicting the
cosine and sine of an acute angle.

|Problem 18.12: We ‘commonly refer to the circle centered at the« origin with radius 1 as the unit circle. |
Let O be the origin, A be the point (1,0) and B be a point on the unit circle such that both coordinates
of B are positive and ZBOA= 0. Find the coordinates of B in terms of 0.

Solution for Problem 18.12: To find the coordinates of B in terms of 6, we


draw altitude BX from B to the x-axis. Doing so builds a right triangle
with an acute angle with measure @. Furthermore, the hypotenuse, OB,
of this triangle is a radius of the unit circle, so OB = 1. Now, we have
the same problem as in Problem 18.11, and we have OX = cos @ and
XB = sin @. Therefore, the coordinates of B are (cos 8,sin 9). O

Rather than using right triangles to define cosine and sine, we can
extend our observation in Problem 18.12 to use the unit circle to define
cosine and sine.

Important: Let point A be (1,0) and point B be on the unit circle such that B is @ |
NW degrees counterclockwise from A. Then, we define cos 0 and sin @ to be
the x-coordinate and y-coordinate, respectively, of point B.

For example, point B at right is on the unit circle, 60° counter-


clockwise from (1,0). Drawing altitude BP from B to the x-axis forms
30-60-90 triangle ABPO, from which we have BO = 1/2 and BP = V3/2.
Therefore, the coordinates of B are (1/2, 3/2), so we have cos 60° = 1/2
and sin 60° = ¥3/2.
Let’s see how this definition allows us to define sine and cosine of
angles that are not acute.

Problems ‘a

=150° and suca


1 ,118.113: Let point Abe (1,0) and let point Cbe on the unit circle such that AC
is150° counterclockwise from A along the circle. Find the coordinates of GC. Ne

489
CHAPTER 18. INTRODUCTION TO TRIGONOMETRY

Problem 18.14: Let point A be (1,0) and let point D be on the unit circle such that AD = 225° and such
that D is 225° counterclockwise from A along the circle. Find the coordinates of D.

Solution for Problem 18.13: Following our definition of cosine and sine, we
let A be (1,0) and we let point C be the point on the unit circle that is 150°
counterclockwise from A. The coordinates of C then are (cos 150°, sin 150°).
To find these coordinates, we first determine where point C is along the unit
circle. The point (0,1) is 90° counterclockwise from A and (—1,0) is 180°
counterclockwise from A, so C is on the minor arc from (0,1) to (—1,0). At
right, we have our unit circle, the origin, O, and points A and C.

Once again, we build a right triangle by drawing the altitude from C to


the x-axis. Because AC = 150°, we have ZAOC = 150°. Therefore, we have
ZCOP = 180° — 150° = 30°. So, ACOP is a 30-60-90 triangle. Since OC = 1 and ZCOP = 30°, we have
OP = ¥3/2 and CP = 1/2. Because C is to the left of the y-axis and above the x-axis, its x-coordinate
is negative and its y-coordinate is positive. So, the coordinates of C are (— V3 /2,1/2), which means
cos 150° = — ¥3/2 and sin 150° = 1/2.
The coordinate axes divide the Cartesian plane into four quadrants. We Yy
usually label these quadrants with Roman numerals, as shown at right. We
refer to them as the first, second, third, and fourth quadrant, respectively. So,
point C in Problem 18.13 is in the second quadrant. All points in the second I I
quadrant are to the left of the x-axis and above the y-axis, so all points in the x
second quadrant have a negative x-coordinate and a positive y-coordinate.
We sometimes refer to angle measures with quadrants, as well. When we = =
say that a 150° angle is a ‘second quadrant’ angle, this means that the point
that is 150° counterclockwise from (1,0) on the unit circle is in the second
quadrant. Similarly, because 60° is between 0° and 90°, a 60° angle is a ‘first quadrant’ angle.

Problem 18.14: Evaluate cos 225° and sin 225°.

Solution for Problem 18.14: Again, we let A be (1,0), and we let point D be the
point on the unit circle that is 225° counterclockwise from A. Since (—1,0) is
180° counterclockwise from A and (0,—1) is 270° counterclockwise from A,
we know that D is in quadrant III, as shown at right.

We draw altitude DP from D to the x-axis, forming right triangle DOP.


Because AEF is 180° and AED is 225°, we have DP = 45°, so ADOP is a
45-45-90 triangle. Since OD = 1, we have OP = DP = y2/2. Point D is
2/2 to the left of the y-axis and 2/2 below the x-axis, so its coordinates
are (— ¥2/2,— V2/2). Therefore, we have cos 225° = sin 225° = — V
2/200
Now that we have a handle on sine and cosine, we can revisit tangent. Once we have defined sine

490
18.2. NOT JUST FOR RIGHT TRIANGLES

and cosine in terms of the unit circle, we define the tangent of any angle @ for which cos 0 # 0as

If cos 6 = 0, then tan @ is undefined.

||____Problems ite
Problem 18.15:
(a) Use the unit circle to find sin0° and cos 0°, then evaluate tan 0°.
(b) Find sin 90°, cos 90°, and tan 90°.

Problem 18.16: Evaluate cos 300°, sin 300°, and tan 300° without a calculator.

|Problem 18.17: Explain why sin(180° — @) = sin @ for any acute angle 0.

Problem 18.18: Show that for any triangle AABC, we have [ABC] = +(AC)(BC) sinC.

Problem 18.15:
(a) Find sin0°, cos 0°, then evaluate tan 0°.
(b) Find sin 90°, cos 90°, and tan 90°.

Solution
for Problem 18.15:
(a) The point that is 0° counterclockwise from (1,0) is simply (1,0). Therefore, we have cos 0° = 1 and
sin 0° = 0, so tan 0° = (sin 0°)/(cos 0°) = 0.
(b) The point that is 90° counterclockwise from (1,0) is (0,1). So, we have cos 90° = 0 and sin 90° = 1.
Therefore, we have tan 90° = (sin 90°)/(cos 90°) = 1/0 = --- Uh-oh! We can’t divide by 0, so tan 90°
is undefined.

Sidenote: When we put the values of sine and cosine of 0° and 90° together with
ay those of 30°, 45°, and 60°, a curious pattern emerges:

Problem 18.16: Evaluate cos 300°, sin 300°, and tan 300° without a calculator. i

491
CHAPTER 18. INTRODUCTION TO TRIGONOMETRY

Solution for Problem 18.16: We let A be (1,0), and we let point B be the point Ms
on the unit circle that is 300° counterclockwise from A. The point (0,—1)
is 270° counterclockwise from A, and going 360° counterclockwise from A
takes us back to A, so we know that B is in quadrant IV, as shown at right.
(We could also have determined that B is in quadrant IV by noting that 300° x
counterclockwise is the same as 60° clockwise.) = A

We draw the altitude from B to point P on the x-axis. Because B is 300°


counterclockwise from A, we have AB = 60°, so ZBOA = 60. From 30-60-90 B
triangle BPO, we have BP = ¥3/2 and OP = 1/2, so the coordinates of B are
(1/2, — ¥3/2). What’s wrong with this conclusion:

Bogus Solution: Because B is (1/2, — 3/2), we have sin300° = 1/2 and cos300° =
i ee
We have sine and cosine backwards here! Cosine is the x-coordinate and sine is the y-coordinate.

WARNING!! Be careful not to get sine and cosine backwards. Once you have found |
“S the appropriate point on the unit circle, an easy way to remember
which coordinate is cosine and which is sine is to use alphabetical |
order: cosine is before sine and x is before y. |

So, because B is (1/2, — V3 /2), we have cos 300° = 1/2 and sin 300° = — 3/2. Therefore, we have

sin300° 2 — ¥3/2 eer


tan 300° =
AR cos 300° 1/2 v3

We can use the unit circle to ‘see’ trigonometric relationships.

Problem 18.17: Explain why sin(180° — @) = sin @ for any acute angle 0.

Solution for Problem 18.17: Let A be (1,0) and B be (—1,0). Let C be


(cos 0, sin @), so that C is 6 counterclockwise from A. Let D be the point
(cos(180° — @),sin(180° — @)), so D is 180° — 6 counterclockwise from
A. Because (—1,0) is 180° counterclockwise from A and D is 180° — 6
counterclockwise from A, point D is 0 clockwise from (—1,0), as shown
in the diagram. We draw altitudes CX and DY to the x-axis, since these
lengths equal sin 0 and sin(180° —@), respectively. Because OD = OC = 1,
we have AOYD = AOXC by SA Congruence for right triangles. This
gives us CX = DY, so sin @ = sin(180° — 8). O
With a little bit of casework, we can extend this proof to show that
sin(180° — @) = sin @ for all angles 0, not just acute angles. This relation-
ship is not worth memorizing. After working with trigonometric functions and the unit circle more, this

492
18.2. NOT JUST FOR RIGHT TRIANGLES

relationship, and others like it, will become automatic to you. We'll be exploring how in later books in
the Art of Problem Solving series.

Problem 18.18: Show that for any triangle AABC, we have [ABC] “ 1(AC)(BC) sinC.

Solution for Problem 18.18: We consider separately the cases in which C is acute, right, or obtuse.

Case 1: ZC is acute. To find the area, we seek an expression for the altitude from A
A to BC. So, we draw altitude AX, forming right triangle AAXC. From this triangle,
we have sinC = AX/AC, so AX = ACsinC. Therefore, we have

1
[ABC] = 5 (AX)(BC) = 5(AC\(BC) sin C. x

Case 2: ZC is right. If ZC is right, then we have both sinC = 1 and [ABC] = 5(AC)(BC). So, we have
[ABC] = 3(AC)(BC) sin C.

Case 3: ZC is obtuse. If ZC is obtuse, then altitude AX is outside AABC, as A


shown at right. We have sin ZACX = AX/AC, so AX = ACsin ZACX. Because!
ZACX = 180° — ZACB, we have

sin(180° — ZACB). Xx ge B
[ABC] = 5(AX)(BC) = 5(AC\(BC) sin ZACX = 5(AC\(BC)

Because sin(180° — ZACB) = sin ZACB for any angle ZACB, we have [ABC] = 5(AC)(BC) sin ZACB, as
desired.
We have covered all three possible cases for ZC, so we have shown that

Important: Let BC = aand AC = b in AABC. Then, we have

[ABC] = absin.

Using our unit circle, we can also evaluate trigonometric functions


for angle measures greater than 360°, as well as for negative angle
measures. For example, since going 360° counterclockwise from (1,0)
brings us back to (1, 0), the point that is 360° + 60° = 420° counterclock-
wise from (1,0) at right is point B shown. From the coordinates of B, we
see that cos 420° = 1/2 and sin 420° = 3/2. To evaluate trigonometric
functions of negative angles, we go clockwise from (1,0) rather than
counterclockwise. So, because B is 300° clockwise from (1,0), we have
cos(—300°) = 1/2 and sin(—300°) = ¥3/2.

493
CHAPTER 18. INTRODUCTION TO TRIGONOMETRY

I) Exercises ie
18.2.1 Evaluate each of the following:
(a) sin 120° (c)mtan 135. (e) cos210°
(b) sin 330° (d) cos(—45°) (f) tan720°

18.2.2 Explain why sin* 6 + cos* @ = 1 for any angle measure 0.


18.2.3 How many angle measures 6 such that 0 < 0 < 360° satisfy the equation sin @ = 0.31?
18.2.4 Explain why cos(360° + @) = cos @ for any angle 0.
18.2.5 Explain why cos(180° — @) = —cos @ for any acute angle 0.

18.3. Law of Sines and Law of Cosines

SAS Congruence tells us that if two sides of one triangle and the angle between them are equal to the
corresponding sides and angle of another triangle, then the two triangles are congruent. Therefore, the
third sides of these two triangles have the same length. In other words, if we know two side lengths of a
triangle and the measure of the angle between those sides, then there is only one possible length of the
third side of the triangle. Hmmm... Can we find that length if we know the two side lengths and the
angle measure?
Similarly, ASA and AAS Congruence tell us that if two angles and a side of a triangle equal the
corresponding angles and side of another triangle, then the two triangles are congruent. In other words,
if we know two angles and a side in one triangle, then there is only one possible value for each of the
remaining two side lengths. Can we find these two side lengths if we know the angles of the triangle
and the other side length?
In this section, we use trigonometry to answer both of these questions.
Note: you can use your calculator for the problems in this section.
>
I Problems

Problem 18.19: In AABC, let AC = 15, BC = 12, and ZC = 34°. In this problem, we find AB to the |
nearest hundredth.
(a) Draw altitude BX from B to AC. Find CX and BX to the nearest hundredth.
(b) Find XA to the nearest hundredth.
(c) Use parts (a) and (b) to find AB to the nearest hundredth.

Extra! Mathematical knowledge adds a to the mind, frees it from prejudice, credulity, and supersti-
ia spam —£7077,

—John Arbuthnot

aEEIT Te
18.3. LAW OF SINES AND LAW OF COSINES

Problem 18.20: Let AABC be an acute triangle with a = BC, b = AC, and c = AB. In this problem, we
prove that
ct = a* + b* —2abcosC.
(a) Draw altitude BX from B to AC. Express BX and CX in terms of a, b, and/or trigonometric
functions of ZC.
(b) Express AX in terms of a, b, and/or trigonometric functions of ZC.
(c) Use AABX to show that c* = a? + b? — 2abcosC.
What happens in the equation c? = a” + b* — 2abcosC if ZC is a right angle?

Problem 18.21: In APQR, suppose PR = 12, ZQOPR = 66°, and ZPRQ = 63°. In this problem, we find
PQ and QR.
(a) Let T be the foot of the altitude from P to OR. Find PT to the nearest hundredth.
(b) Use PT to find PQ to the nearest hundredth.
(c) Find QR to the nearest hundredth.

Problem 18.22: Suppose that AABC is an acute triangle with a = BC, b = AC, andc = AB. Prove that

ee
sinA sinB sinC

Problem 18.23: Find BC, CD, and BD to the nearest tenth in the diagram
at right.

Problem 18.19: In AABC, let AC = 15, BC 12,


= and /C = 34°. Find AB to the nearest hundredth.

Solution for Problem 18.19: It’s not immediately obvious how to find AB. We B
know how to use trigonometry to find side lengths in right triangles, so we 12
start by drawing an altitude from B to AC as shown. This creates right triangle
ABCX with the 34° angle as one of its acute angles. From right triangle ACBX, ¢ i: A
we have 7g 1
BC sinC ~ 0.559,
so BX ~ 0.559(BC) ~ 6.71. Similarly, we have CX/BC = cosC ~ 0.829, so CX = 9.95. This doesn’t tell
us AB yet, but we now have the length of one leg of ABXA. If we can find the other, we can use the
Pythagorean Theorem to find AB. Fortunately, XA is easy to find: XA = AC —- CX * 5.05. Now, we can
use the Pythagorean Theorem to find
AB = VBX? + XA? = 8.40.

495
CHAPTER 18. INTRODUCTION TO TRIGONOMETRY

In Problem 18.19, we were given two side lengths of a triangle, and the measure of the angle between
these two sides. We then found the third side. There was nothing particularly special about the side
lengths or the angle. We might be able to follow essentially the same process for any triangle. Let’s give
it a try.

‘Problem. 18.20: Let AABC be an acute triangle with a = BC, b = AC, and c = AB. Suppose we know
a,b, and ZC. Find a formula that we can use to find c.

Solution for Problem 18.20: We use Problem 18.19 as a guide. In fact, this problem B
is essentially the same as Problem 18.19, but with variables a, b, and ZC in place
ieee a c
of the numbers that were given in that problem. We can use the same diagram,
and we'll use the same steps. We draw altitude BX from B to AC. Then, wehave a
sinC = BX/BC, so BX = BCsinC = asinC. We also have cosC = CX/BC, so x
CX = BCcosC = acosC. Therefore, we have AX = AC —CX = b-acosC. Next
Pere ae,
we apply the Pythagorean Theorem to AABX to find AB” = BX? + AX?. Substituting our expressions for
these three sides gives us

c= a-sin-C + (b—acosC)y-
a’ sin®C + b? — 2abcosC + a* cos*C
a’(sin? C + cos* C) + b? — 2ab cos C.

Since sin? C + cos? C = 1, we have


c? =a? +b —2abcosC.
bal

We can test our new formula with the data from Problem 18.19.

Concept: Check formulas you prove by trying them on specific examples you have
solved without the formula.

In Problem 18.19, we have a = 12, b = 15, and ZC = 34°, so we have

c =a? +b? — 2abcosC ~ 70.546.


Taking the square root of both sides gives c ~ 8.40, which agrees with our answer from Problem 18.19.
With a little more casework (which you'll supply as an Exercise), we can show that this equation
holds for any triangle ABC, not just for acute triangles.

Important: ‘Leta = BC, b = AC, and c = ABin ABC.


The Law of Cosines states that
ct =a? +b? — 2abcosC.

Notice that when ZC = 90° in the Law of Cosines, we have cos C = 0, so the law becomes c2 = a2 + b,
which is just the Pythagorean Theorem.
Problem 18.19 is just a specific example of Problem 18.20, so we call Problem 18.20 a generalization
of Problem 18.19.

Sa
REE
18.3. LAW OF SINES AND LAW OF COSINES

Concept: Whenever we have to prove a general statement like the Law of Cosines, |
Os it’s often useful to first consider a specific example of the general statement. |
If we can solve a specific example, we can sometimes use that solution as
a guide to prove the generalization.
|
Now that you've learned the Law of Cosines, you’re probably thinking there must be a Law of Sines,
too. Of course, you've read the section title, so you know you're right. We'll discover the Law of Sines
the same way we discovered the Law of Cosines — we’ll start with a specific example.

ale Oe ae PR = 12, ZOPR= 66°,


and ZPRQ = 63° in APQR. Find PQ and RQ to the}
nearest hundredth.

Solution for Problem 18.21: We start by building a right triangle, since we know Ie
how to use trigonometry to find lengths of sides in a right triangle. We draw
altitude PT to side QR. From right triangle APTR, we have PT/PR = sinR, so
PT = PRsinR ~ 10.69. We also have TR/PR = cos R, so TR = PRcosR ~ 5.45. 12

Now, we can use right triangle APQT to find lengths QT and PQ. First, we Q - R
find that ZQ = 180° — 66° — 63° = 51°. We have PT/PQ = sinQ, so we have jh
PQ = PT/(sinQ) ~ 13.76. We also have PT/QT = tanQ, so QT = PT/(tanQ) ~ 8.66. Finally, we have
OR = OTe TR 141.0
Let’s try using our approach in Problem 18.21 to produce another ‘law.’

Problem 18.22: Suppose that AABC is an acute triangle with a = BC, b = AC, and c = AB. Prove that

ie Scoot b ae
sinA sinB sinC’

Solution for Problem 18.22: We use Problem 18.21 as a guide. We draw altitude AT to A
BC. From right triangle AATC, we have sin C = AT/AC, so AT = ACsinC = bsinC.
From right triangle AABT, we have sinB = AT/AB, so AT = ABsinB = csinB. b E
These two expressions for AT must be equal, so we have b sinC = csin B. Dividing
this equation by sin B and by sin C, we have
ashcaeclel G T B
sinB sinC’ |

We can follow essentially the same steps starting with the altitude from C to AB to show that
b/(sin B) = a/(sin A). So, we have ‘
a C
sinA sinB sinc’

‘Important: Let a = BC, b = AC, andc = AB in AABC. The Law of Sines states that =

a b oc
‘sinA sinB. sinC’

497
CHAPTER 18. INTRODUCTION TO TRIGONOMETRY

As with the Law of Cosines, you'll tackle the cases in which AABC is right or obtuse in the Exercises.

Sidenote: The | Law of Sines gives us another look at why ‘SSA Congruence’ is not a
| valid theorem to prove triangle congruence. Suppose we have ZA = 30°,
|
i

BC =3,and AC = 4 in AABC. The Law of Sines then gives us


13]Be eS
sinA sinB’
from which we find sin B = (AC/BC) sinA = 2/3.
Next, suppose we have ZD = 30°, EF = 3, and DF = 4in ADEF. Again,
the Law of Sines gives us
EF DE
sinD sinE’
from which we find sinE = (DF/EF)sinD = 2/3. So, we have sinB = |
sin E = 2/3. Do we have ZB = LE?
Not necessarily! There are two angles @ for which y (0,1)
0° < 6 < 180° and sin @ = 2/3. The two intersections z
of the graph of y = 2/3 with the unit circle correspond
to these two angles; these are shown at right. So, we
cannot deduce that ZB = ZE, and therefore we cannot |
conclude that AABC = ADEF.
L
Let’s give the Law of Cosines and the Law of Sines a try:

Problem 18.23: Find BC, CD, and BD to the nearest tenth in the diagram C
at right.

Solution for Problem 18.23: In AABC, we know two side lengths and the angle between these sides, so we
can use the Law of Cosines to find BC. This gives us

BC? = 8* + 12? — 2(8)(12) cos 66° ~ 129.9,


so BC ~ 11.4. Turning to ABCD, we have ZBCD = 180° — 65° — 85° = 30°. Now, we know one of the
side lengths of ABCD and all of the angle measures of ABCD, so we can use the Law of Sines to find the
missing side lengths. Specifically, we have

BG pen Lee CR
sinD sinZBCD sin ZCBD’
which means we have
11.4 BD CD
sin65°—s sin30°——s sin 85°"
ATO ae
a 18.3. LAW OF SINES AND LAW OF COSINES

Solving for BD, we have


BD = ———
singpe -sin30° = 6.3,
3D 30° ~
and solving for CD gives
11.4
Daa
sing5e |SDSIN 85° BO 712,0:
El

Exercises
18.3.1 Find the missing sides below to the nearest tenth.
(a) ‘ (b) i

R 10 14
Q hae
12
u
V
18.3.2 Find the missing sides below to the nearest tenth.
7 ve
(a) 43° (b) ,

: 9

C ie
712
B
18.3.3 In this problem, we prove that the Law of Cosines works when the given angle is obtuse. Let
AABC be an obtuse triangle with ZC > 90°. Let AB = c, CA = b, and BC = 4, as usual.

(a) Let D be the foot of the altitude from A to BC. Find AD, BD, and CD in terms of a, b, c, and
trigonometric functions of ZC.
(b) Show that c* = a? + b? — 2abcosC.

18.3.4 Prove that the Law of Sines holds for right triangles and for obtuse triangles.
18.3.5 A surveyor is 3 kilometers from the base of a mountain. The mountain’s face slopes up at an
angle of 30° from horizontal. If the surveyor measures 10 degrees from horizontal to the top of the
mountain, approximately how tall is the mountain to the nearest hundredth of a kilometer?

499
CHAPTER 18. INTRODUCTION TO TRIGONOMETRY

18.3.6 Let AABC be a triangle with BC = a, AC = b, AB = c, and with circumradius R.

(a) Draw AABC and its circumcircle. Let point D be on the circumcircle such that AD is a diameter.
Find sin ZADB in terms of the side lengths of AABD.
(b) What angle of AABC equals ZADB, and why?
(c) Show that
dese le es Cie
sinA © sinB sinc
This is the Extended Law of Sines.

18.3.7x Suppose AC = 5, BC = 6, and AB = ¥31. Find ZBCA.

18.4 Summary

We write the cosine of an angle 6 as cos 6 and the sine of 6 as sin 0. We define sine and cosine using the
unit circle, which is the circle on the Cartesian plane with radius 1 centered at the origin.

Important: Let point A be (1,0) and point B be on the unit circle such thatB is 0 |
E . . . . .

degrees counterclockwise from A. Then, we define cos 6 and sin 6 to be


the x-coordinate and y-coordinate, respectively, of point B.

. oe sin @
We write the tangent of an angle @ as tan 9, and define it as tan 9 = ——~ . Sine, cosine, and tangent
os 6
are examples of trigonometric functions.

For an acute angle ZA of a right triangle AABC, we have: B

ees length of leg opposite A _ 4 é [


length of hypotenuse c
oer length of leg adjacent to A b ss
length of hypotenuse c b é
length of leg opposite A a
tanA : x
length of leg adjacent to A b

We proved several trigonometric identities, which are statements involving trigonometric functions
of angles that are true for all angles. The most important is:

‘Important
sin?A + cos?A = 1

BG
NO
a ar a a ll all lt heREVIEW
Ne te PROBLEMS
al

While we proved several other identities, it is more important to understand how these identities
are derived than to memorize all the identities.

We then used trigonometry to find three important triangle relationships:


—_ A )
_Important: Let BC =a, AC = b, and AB = c in AABC. Then, we have

e [ABC]= 5ab sinC,

ec 2 = q* + b? — 2abcosC. This is called the Law of Cosines.


a b
This is called the Law of Sines.
sinA ~ sinB re

Problem Solving Strategies

|Concepts: ° If we havethe length ofaside of aright triangle and the value of aae |
OO trigonometric function of either of the acute angles of the triangle,
| then we can find all the side lengths of the triangle. |
|
e Check formulas you prove by trying them on specific examples you |
| have solved without the formula. |
i

| e Whenever we have to prove a general statement like the Law of |


| Cosines, it’s often useful to first consider a specific example of the |
| general statement. If we can solve a specific example, we can some- |
times use that solution as a guide to prove the generalization. |

I|_REview Proslems il s
18.24 In AABC at right, find sin A, cos A, and tan A. A
18.25 In APQR, let ZQ = 90°, ZP = 71°, and PR = 16. Find PQ and QR to the a
nearest tenth. i B

18.26 Show that if AXYZ is a right triangle with 2X = 90°, then (tan Y)(tan Z) = 1.
18.27. In APQR, we have ZQ = 90° and sinP = 1/4. Find sin R.

18.28 Evaluate each of the following:


(a)) sin, 135; (c) _cos(—120") (e) tan300°
(b) sin630° (d) cos315° (f) tan 150°

501
CHAPTER 18. INTRODUCTION TO TRIGONOMETRY

18.29 The angle of elevation is the angle above the horizontal at which a viewer must look to see an
object that is higher than the viewer. Similarly, the angle of depression is the angle below the horizontal
at which a viewer must look to see an object that is below the viewer.

Object = Horizontal
Seah’ Angle of Depression

Angle of Elevation
Nigel Horizontal Object

Answer each of the following problems to the nearest foot.

(a) A surveyor measures the angle of elevation from her feet to the top of a building as 5°. The
surveyor knows that the building is 500 feet tall. Assuming the ground is flat and level between
the surveyor and the building, how far away is the surveyor from the building?
(b) I’m standing at the peak of a mountain that is 14,000 feet above sea level. The angle of depression
from this peak to a nearby smaller peak is 4°. On my map, these two peaks are represented by
points that are 1 inch apart. If each inch on my map represents 1.2 miles, and there are 5280 feet
in a mile, then how many feet above sea level is the second peak?
(c)x A bee is ona hill looking at a building. The building is 400 feet tall. The angle of elevation from
the bee to the top of the building is 4° and the angle of depression from the bee to the bottom of
the building is 2°. What is the shortest distance the bee will have to fly to reach the building?

18.30 Let 0; and @2 be the two values of 0 such that 0° < @ < 180° and sin @ = 0.48. What is 0; + 62?

18.31 Suppose that AB = 7, BC = 8, and ZABC = 45°. Find [ABC].

18.32 If tan @ is negative, then which quadrant(s) could 0 possibly be in?


18.33 Recall that sec x = —-. Show that for any angle x for which cos x # 0, we have tan? x +1 = sec? 28

18.34 ‘Two strips of width 1 overlap at an angle of a as shown. Show that the
area of the overlap (shown shaded) is 1/(sina).

18.35 Point A is on the unit circle in the first quadrant such that A is 0 degrees
counterclockwise from (1,0). Point B is on the x-axis such that AB is tangent to
the unit circle. Show that tan@ = AB. (Do you now see why we use the name
‘tangent’ for this trigonometric function?)
18.36 Find the missing side lengths in the triangles below to the nearest tenth.

(a) R (b) r
10

502
CHALLENGE PROBLEMS

18.37 Find the missing side lengths in the triangles below to the nearest tenth.

(a) (b)

Y 31 Z Ex

18.38 If I walk north 3 miles, turn 32°, then walk another 2 miles, how far will I be from my initial
starting point (to the nearest 0.1 mile)?
18.39 What's wrong with the following proof of the Pythagorean Theorem:

In the text, we proved that if a = BC, b = AC, and c = AB, then c* = a* + b? — 2abcosC. If
ZC = 90°, then we have cos C = 0. Therefore, if ZC = 90°, we have c2 = a? + b?.

18.40 In APQR, we have PQ = 3, QR = 5, and PR = 6.

(a) Find cos P.


(b)x Find sinQ.

Challenge Problems

18.41 Show that if the sides of AABC have lengths a, b, and c, and AABC has circumradius R, then

abc
BC] =—.
[ABC] AR

18.42 Two rays with common endpoint O form a 30° angle. Point A lies on one ray, point B on the
other ray, and AB = 1. What is the maximum possible length of OB? (Source: AHSME)
18.43 How many triangles have area 10 and vertices (—5,0), (5,0), and (5 cos 6,5 sin @) for some angle
0? (Source: AHSME)

18.44 Four congruent 30-60-90 triangles are constructed on the sides of a square
as shown at right. The hypotenuse of each of these triangles has length 2. The
outer vertices of these triangles are connected as shown to form quadrilateral
ABCD. What fraction of ABCD is shaded?
18.45 A circle centered at O has radius 1 and contains the point A.Segment AB
is tangent to the circle at A and ZAOB = @. Suppose point C is on OA such that
BC bisects ZABO. Show that OC = 1/(1 + sin @). (Source: AMC 12)
CHAPTER 18. INTRODUCTION TO TRIGONOMETRY

18.46 Ifsinx = 3cosx, then what is (sinx)(cos x)? (Source: AHSME)

18.47 Line ¢ intersects the x-axis at an angle of 50°. Line k makes an angle of 140° with the x-axis. The
intersection of € and k has a y-coordinate of 10. Find all possible values of the distance between the
x-intercepts of € and k.
18.48 Right triangle ABC has inradius 1 and sin A = 12/13. Find the length of the hypotenuse of ABC.
Hints: 593
18.49 What’s wrong with this proof of AA Similarity:

Suppose we have AABC and ADEF with ZA = 2D and ZB = ZE. The Law of Sines gives us
BGate AG FF EF _ DF
snA sinB sinD sinE
Therefore, we have BC/AC = (sin A)/(sin B) and EF/DF = (sin D)/(sin E). Since ZA = ZD and
ZB = LE, we have (sin A)/(sin B) = (sin D)/(sin E). So, we have

BC. sin. AS sin DEE


AC) (asi. Bie sin cee
Because ZA = ZD and ZB = ZE, we have ZC = ZF. We can follow essentially the same steps
as above to deduce BC : AC: AB = EF: DF: DE.

18.50* Points A, B, C, and D are on a circle of diameter 1, and X is on diameter


AD. Suppose BX = CX and 3ZBAC = ZBXC = 36°. Show that B

X = (cos 6°)(sin 12°)/(sin 18°). A D

(Source: AHSME) Hints: 589, 576 C

18.51% An object moves 8 cm ina straight line from A to B, turns at an angle


of a, where 0° < a < 180°, and then moves 5 cm in a straight line to C. What is the probability that
AC < 7? Hints: 526

18.52 Equilateral triangle ABC has been creased and folded so that vertex A A
now rests at A’ on BC as shown. If BA’ = 1 and A’C = 2, then what is the length Ne
of crease PQ? (Source: AMC 12) Hints: 555, 544 Pp’ ne Q

VAWAS
A’ &

since youare now studying geometry ¢and trigonometry, Twill give youa
0 ae Tt
oeBoston with aie oe wool.isMeeesO

504
Archimedes’ Circles

Let no one ignorant of geometry enter here. — Inscription above Plato’s Academy

CHAPTER Ngo ly ae: pet el

Problem Solving Strategies in Geometry

In this chapter we review many of the most powerful geometry problem-solving strategies we have
learned in the text by tackling some challenging problems with them. We also apply many of our
geometric tools to proofs that require multiple insights.
This chapter is meant to extend the lessons of this book, so many of the Exercises and Challenge
Problems in this chapter are significantly more difficult than most of the problems elsewhere in this text.
One excellent strategy for mastering the problems you cannot solve on your first try is to review the
solutions, then try the problems again on your own a few days later.

19.1 The Extra Line

We cleverly added extra lines to diagrams to solve various problems throughout this book. In this
section we reinforce the most common indications that extra lines might be helpful.

J
Proviems
ite
iina thombus TUTE Ie
Problem 19.1: Find the radius of a circle thatis inscribed
ame of
> 16and 30._

505
CHAPTER 19. PROBLEM SOLVING STRATEGIES IN GEOMETRY

Problem 19.2: In obtuse triangle ABC shown, AM = MB, MD 1 BC, and : A


EC 1 BC. If the area of AABC is 24, what is the area of ABED? (Source: M
AMC 12) Hints: 27,541

2 Herc
Problem 19.3: In triangle ABC, ZABC = 120°, AB = 3 and BC = 4. If lines ©
perpendicular to AB at A and to BC at C meet at D, then find CD. (Source:
AMC 12) B

Problem 19.4: AB is tangent at A to the circle with center O, point D is interior


to the circle, and DB intersects the circle at C. If BC = CD = 3, OD = 2, and
AB = 6, then find the radius of the circle. (Source: AMC 12)

One very common type of ‘extra line’ we draw is a perpendicular from a point to a line. Usually, our
goal in doing so is to build a useful right triangle. Here’s a classic example.

|Problem 19.1: Find the radius of a circle that is inscribed in a rhombus that |
has diagonals of length 16 and 30.

Solution for Problem 19.1: Circles + tangent lines = draw a radius to


a point of tangency. We draw radius OE because OE 1 AB. Since
the diagonals of a rhombus are perpendicular and bisect each other,
AAOB is a right triangle with legs of length OB = 8 and OA = 15.
Therefore, AB = 17. From here, we can solve the problem in several
ways. Here are a couple:
Solution 1: Use similar triangles. Lots of right angles usually
means there are similar triangles. Here, we have

AAOB ~ AAEO ~ AOEB.

Therefore, OE/OA = OB/AB, so OE = (OA)(OB)/AB = 120/17.

506
19.1. THE EXTRA LINE

Solution 2: Use area. Since [AOB] = (AO)(OB)/2 = (OE)(AB)/2, we have OE = (AO)(OB)/AB = 120/17.

(Yeah, I like the area approach better, too.) O

_Concept: When in doubt, build right triangles. One very common way to do so is
— OJ to draw radii to points of tangency. Nearly always try this when you have |
| circles and tangents in a problem. |

Problem 19.2: In obtuse triangle ABC Ehownt AM = MB, MD uyBC, and :


EC 1 BC. If the area of AABC is 24, what is the area of ABED? (Source: E
AMC 12) M

Solution for Problem 19.2: We have a problem about area, and we unfortunately can’t find anything about
[BDE] easily. We do, however, know how to relate [AMC] and [BMC] to [ABC]. Therefore, we connect
M to C. (We might also have thought to do this because CM is a median, and we know something about
medians.)

Since CM is a median of AABC, we have [BMC] = [AMC] = 12. Now A


we look for ways to relate either of these to what we want, [BED]. Triangles M E
ABED and ABMC overlap. When we remove ABDM from both, we are left
with two triangles that share side DM: AEDM and ACDM. Since DM ||EC,
the altitudes to side DM of these triangles are the same. Therefore, [EDM] = B
[CDM], which means [BED] = [BMC] = 12. 0 DC

Concept: Connecting points that are originally not connected in a diagram can be ex-
(
=== tremely useful! This doesn’t mean you should connect everything in your
diagram immediately, however. Look for segments to draw that will be
helpful, particularly those connecting important points, or those that form
segments, triangles, or angles you know something about immediately. _
—— es

Problem 19.3: In triangle ABC, ZABC = 120°, AB = 3 and BC = 4. If lines ‘e


perpendicular to AB at A and to BC at C meet at D, then find CD. (Source:
AMC 12) ;

Solution for Problem 19.3: We start by noting that ZD = 360° — 120° — 90° — 90° = 60°. Next, we might
think to draw BD, since that will give us a couple right triangles. However, we only have one side of
those two right triangles, and we don’t know anything about the acute angles of them. We’d like to
make the 120° and 60° angles useful. This gets us thinking about 30-60-90 triangles.

507
CHAPTER 19. PROBLEM SOLVING STRATEGIES IN GEOMETRY

With a right angle at ZBAD and a 60° angle at 2D, we see that extending DC past E
C and AB past B to meet at point E will give us a 30-60-90 triangle. Moreover, since
ZCBE = 180° — 120° = 60° and ZBCE = 90°, ACBE is also a 30-60-90 triangle. Now we
can solve the problem. C

From 30-60-90 ACBE we have BE = 2BC = 8 and EC = BC V3 = 4 V3. From ADAE F


we have DE = AE(2/¥3) = (AB + BE)(2/V3) = 22 V3/3. Therefore, CD = ED — EC =
10 ¥3/3. O A D
"Concept: 00%, 30°, and even 120° angles are often good clues to build 30-60-90 trian-
gles by dropping altitudes or extending segments.

Problem 19.4: ABis tangent at A to the circle with center O, point D is interior
to the circle, and DB intersects the circle at C. If BC = CD = 3, OD = 2, and
AB = 6, then find the radius of the circle. (Source: AMC 12)

A bs
Solution for Problem 19.4: We have a problem with segment lengths and a circle, E
so we think of Power of a Point. We have a tangent, but only part of a secant.
We continue BD past D until it hits the circle at E. We do this not only because it
lets us use Power of a Point, but because BD seems to end rather abruptly in the

S
middle of the circle. Segments that seem to end suddenly in a diagram (i.e. ones
that, if continued, will hit an important circle or segment) are often candidates to
be extended. The power of point B gives us
(BC)(BE) = BA2, A B
and substitution gives
3(3 + CE) = 36.
Therefore, CE = 9. Here are a couple ways we can finish:

Solution 1: Build right triangles. We know the length of chord CE and we’d E
like to find the length of a radius. We draw radius OC and drop a perpendicular
from O to CE. This gives us a couple right triangles in AODF and AOCF. Since
OF is part of a radius that is perpendicular to chord CE, it bisects CE. Therefore, QS
CF = 9/2 and DF = CF - CD = 3/2. So, aX
V7
OF ==VOD? 2 ep
=DF = ees etoC
Finally, we have = 2

OC = VOF2
+ FC2 = c+ = V2.

508
19.1. THE EXTRA LINE

Solution 2: Keep doing what worked. Extending a line that suddenly E


stopped worked once, so try it again! OD stops rather suddenly. So,
we extend it in both directions to hit the circle at J and K, as shown.
Now, we have two intersecting chords, so we can use Power of a Point! J
The power of point D gives

(JD)(DK) = (CD)(DE).
If we let the radius of the circle be r, we have oe K

(r + 2)(r — 2) = (3)(9 — 3). Crx3

Solving for r, we find r = v22. 0 A 6 B

Concept: - Segments that stop suddenly inside figures (particularly triangles, quadri-
Os laterals, or
ees) aress candidates to be extended.

Concept: Ifyou successfully use


u: a certain tacticinaa problem to get some
so information,
©==2 but you still haven’t solved the problem, try using that same tactic again
ina en See SEs it still has more information to give!
ee
ca aaa Ss

Notice that our path to the solution is easy to see in the final diagram of the last solution. This is
because the side lengths are labeled.

Concept: Label lengths in your diagram as you find them, even if you have to label
©=== | them in terms of an important variable.

(RESETS
19.1.1 A square is inscribed in a circle of radius 1 as shown at left below. Circles OP and ©Q are the
largest circles that can be inscribed in the indicated segments of the circle. The segment joining the
centers of circles P and Q intersects the square at A and B. Find AB. (Source: ARML) Hints: 10

ne

Figure 19.1: Diagram for Problem 19.1.1 Figure 19.2: Diagram for Problem 19.1.2

19.1.2 Inthe diagram at right above, a quarter-circle centered at one vertex of the square connects two
other vertices of the square. A small circle is tangent to the large circle and to two sides of the square as
shown. Each side of the square is 4 units long. What is the radius of the small circle? Hints: 19

509
CHAPTER 19. PROBLEM SOLVING STRATEGIES IN GEOMETRY

19.1.3 Around table is pushed into a corner as shown in the diagram. Point A is on the outer edge of
the table and is 2 inches from one of the walls. Given that the radius of the table is 37 inches, how many
inches is point A from the other wall? (Source: MATHCOUNTS) Hints: 30

eg VE

Figure 19.3: Diagram for Problem 19.1.3 Figure 19.4: Diagram for Problem 19.1.4

19.1.4x* WXYZ isa parallelogram, B is the midpoint of XY, and A is on WB such that ZA 1 BA. Prove
AY = YZ. Hints: 34

19.2 Assigning Variables

In many problems, finding the answer is not simply a matter of performing routine calculations. Often
we need to discover relationships between angles or between lengths. Usually, these relationships will
be equations of some sort. One very helpful way to keep track of this information is to assign variables
to lengths and/or angles. We can then write equations that are easy to read, and we can use the variables
to keep track of the information in our diagram.
As you'll see, we often use this strategy together with our ‘extra line’ tactic from the last section,
particularly when we build right triangles to use the Pythagorean Theorem.

19.5: Inthe figure at right, AD =AE, and Fis the intersection of ED with
xe.If2B =36", ow oe ee are there in <CFE?

lem 19.6:‘Points A, B, C, and D lie ona line, in that order, with AB = CD and BC =12, Point Eiis
or line,and oneCE = 10. The perimeter of AAED is twice the perimeter af—_ Find /AE.

i WXYZi is a rectangular sheet of|


paper with WX =10 and XY =12.The
ep

ncides with the midpoint of XY. What is the length of thefold?eee

MIO
Nea eee
19.2. ASSIGNING VARIABLES

Variables are great help in angle-chasing problems. As you might guess, the fact that the angles of
a triangle add to 180° is our most commonly used equation-forming tool. Try assigning variables and
chasing angles in this problem.

Problem 19.5: In the figure at right, AD = AE, and F is the intersection of ED with
the bisector of ZC. If ZB = 36°, how many degrees are there in CFE?

Solution for Problem 19.5: We can’t deduce the measures of any more
angles directly from 2B = 36°, so we'll have to use relationships among
the angles to learn more. We could start off by writing a bunch of
equations like:

ZB+ZACB+ZA 180°
LADED =ei ALD
ZEDB + ZDBC + ZBCE+ ZDEC = 360°,

but there are so many different angles that it’s hard to keep track of them
all. Instead, we assign variables. We assign a variable to what we want,
ZCFE = x, and we also pick variables for angles that are easy to relate to
other angles in the diagram. We are given that AADE is isosceles, so we
let ZADE = ZAED = z. Similarly, our angle bisector gives us ZACF = ZFCB, so we call these both y.
Now we look for other angles we can label in terms of x, y, or z. Triangle AADE gives us ZA = 180°—2z.
Then, AABC gives us ZA + ZB + ZACB = 180°, so

180° — 2z + 36° + 2y = 180°.

Therefore, y = z — 18°.

Now we can reduce the number of variables in our diagram. If we


don’t see our path to solution now, we can redraw the diagram using only
x and z. Since /FEA is an exterior angle of AFEC, we have

LCFE + ZFCE = £DEA.

Therefore, x + z — 18° = z,so x = 18°.

We could also have written ZDEC = 180° —z in our diagram, then used
the sum of the angles of AFEC to solve the problem. 0

511
CHAPTER 19. PROBLEM SOLVING STRATEGIES IN GEOMETRY

=I

Concept: Assign ‘variables to important angles and use your mastery of basic ge- |
©==3 _ ometry to express other angles in terms of those variables. Typically, you ||
should assign variables to angles you seek and to angles you know a lot”
about first. Once you start chasing angles, reach for all the angle tools -
you know, most notably the facts you know about triangles, straight lines,
parallel lines, and angles that intersect circles.
[

We can also chase lengths with equations. Our main tools here are the Pythagorean Theorem, similar
triangles, and Power of a Point. As we’ve seen throughout this book, the Pythagorean Theorem is the
most heavily used of these three for finding lengths.

Problem 19.6: Points A, BC and D lie ona line, in that order, with AB = CD and BC = 12. Point E
is not on the line, and BE = CE = 10. The perimeter of AAED is twice the perimeter of ABEC. Find
AE. (Source: AMC 10)

Solution for Problem 19.6: We start with a diagram. We draw E


everything we are given in the problem. Then we look for
more lengths we can find in the diagram and we’re almost
immediately stuck. So, we start assigning variables. Since
we want AE, we let AE = x and look for other sides we can A B 12 C i}
express in terms of x.
We suspect all these equal side lengths (AB = CD and BE = CE) will mean AE = DE, so we try
tosprove it. Since BE = CE, ZEBC.=./ECB..; Therefore, AEBAs= ZECD, so AABEL.&, ADICE Dy 5A
Congruence. So, we have DE = AE = x, too.

This isn’t enough, so we look for more lengths we can express in terms of x. AB and CD are all we
have left. We look back at the problem for information we haven’t used yet and see that bit about the
perimeter of AAED being twice that of ABEC. From this, we have

AE+ED+AB+4
BC+ CD =2(00 + 10+ 12):

Substitution gives 2x + 12 + 2AB = 64, so AB = 26 -x.

Adding all this to our diagram, we're still stuck. The


isosceles triangles are a clue to build right triangles and use
the Pythagorean Theorem. We draw altitude EF, thereby
building right triangles AEFC and AEFA. From AEFC we
have EF = 8. Now wecanbuild an equation forxby applying 4 COs b eG = D
the Pythagorean Theorem to AEFA. EF? + AF* = AE?, so ‘ for, Oo ae

64 + (32 —x)* = x’.


Therefore, 64 = x* —(32—x)?. We factor the right side of this equation as the difference of squares to find:

64 = [x — (32 — x)|[x + 62 =x)] = (2x —32)(32):

Solving for x gives x = 17.

nnn
19.2. ASSIGNING VARIABLES

Make sure you follow that nifty use of the difference of squares factorization. This manipulation can
often save you a few extra steps of nasty algebra (and will reduce careless mistakes). 0

Concept: Ifyou need a\ length iinaa problem, try building right triangles. Some of.|
©O==s= | the best tools for building right triangles are drawing altitudes of angle |
and trapezoids, and arawite Fadil,tone Uea Peay,

Problem 19.7; WXYZ is a rectangular sheet of paper with WX = 10 and XY = 12. The paperis
folded so that W coincides with the midpoint of XY. What is the length of the fold?

Solution for Problem 19.7: We start with a diagram. Instead of Xx Dvwen aVVi £7 AB. y
D
just drawing the folded paper, we also include the paper as it
originally was, since we know so much about rectangles. Our : ee
diagram shows WXYZ, and the fold AB that leads to W coinciding A
with W’, the midpoint of XY. Since AW’AB isjust the ‘folded over’
(i.e. reflected) version of AWAB, we have AW’AB = AWAB. We
only initially know XW’ = W’Y = 6 and XW = YZ = 10. We'll 19-*
have to assign some variables. We let AX = x and BZ = y, so we
then have AW = 10 —x and WB = 12 - y.
We then use our congruent triangles to note that AW’ = AW =
Ms Day By“
10 —x and W’B = WB = 12 - y. Right triangle AXAW’ gives us an equation for x:
x74 67= (10% x).

Solving this equation for x gives x = 16/5.

Unfortunately, we don’t havea right triangle that quickly gives Ww’ 6 y g y


us y. So, we borrow a tactic from last section and build one. Y
We draw BC such that BC 1 XY as shown. Since BCYZ is a
rectangle, we have CY = y, BC = 10, and CW’ = 6— y. We could A
use the Pythagorean Theorem as before on AW’CB. However,
we can find the answer a little faster by noting that ZCW’B =
180° — ZAW’B — ZAW’X = 90° — ZAW’X = LXAW’. Therefore, 10-x
right triangles AW’XA and ABCW’ are similar. So, we have
AK OxW! Ww
WAS
NED eae eo.
Substitution gives
iksviey Ws
peanvamed Oi
so y = 2/3.
Therefore, WB = BW’ = 12 — y = 34/3. In right triangle AAWB, we now have BW = 34/3 and
AW = 10— x = 34/5, so we can find AB:

i—af) ep eer34?( 2
1 2
(=) + (5) ere
1)? 34V
2 34

AB = VAW2 + WB? = A 342 (5) = 34

513
CHAPTER 19. PROBLEM SOLVING STRATEGIES IN GEOMETRY

Notice once again that we use a little algebraic manipulation, in this case factoring out the 34, to simplify
our work. 0

HL
Exercises
19.2.1 Triangles ABC and ABD are isosceles with AB = AC = BD, and BD intersects A
AC at E. Given that BD AC, find all possible values of ZC + 2D. (Source: AMC 12)
19.2.2. A point ona circle inscribed in a square is 1 and 2 units from the two closest D
sides of the square. Find the area of the square. (Source: HMMT)
19.2.3 ABCD isa rectangular piece of paper with AB = 8 and AD = 12. We fold the # G
paper so that B coincides with D. What is the length of the fold? Hints: 92

19.2.4x Square ABCD has side length 1.


Point E is chosen on side BCso that AE+EB = 3, A B
and point F is chosen on side CD so that AF bisects ZDAE. Find DF. (Source: Mandelbrot) E
Hints: 566, 588, 592

D Fad

19.3 Proofs

Geometry proofs present a challenge by offering us so many different avenues of exploration. Our task
is to narrow down the possible approaches to the ones that are most likely to bear fruit. Before trying
the problems in this section, read through Problem 13.10 again, noting particularly how we keep track
throughout the problem of What We Know and What We Want. Then, try applying this approach to
these problems. We will also use some of our techniques from the first two sections of this chapter,
especially adding extra lines.

| Problems >

Problem 19.8: Mand N are the midpoints of sides BC and CD, respectively, Af
of square ABCD. AM and AN meet BD at x and Y, ass shown. Show that
OSG YD. :

_ Extra! A ‘proof without words’ of the Pythagorean Theorem is shown at


‘mm right. Alexander Bogomolny describes it as an ‘unfolded variant’ c
__a proof by abu’ I’Hasan Thabit ibnue barven al’Ha
a2 ae oe .

514
19:3.,FROOES

Problem 19.9: ©A and OB are externally tangent. D is on OA


and E on ©B such that DE is a common external tangent of the
two circles. OC is tangent to ©A, OB, and DE as shown. Given
that a is the radius of OA, b is the radius of ©B, and c is the
radius of OC, prove that

Problem 19.10: In AABC let L be the foot of the angle bisector


from A to BC, and let M be the midpoint of BC. We also draw
the circumcircle of AABC. Finally, we construct the circle through
points A, L, and M. This circle intersects the rest of the diagram in
several remarkable ways, so we label points Z, Y, and X where the ©
circle meets AB, AC and the circumcircle of AABC, respectively.
(a) Prove that BZ = CY.
(b) Prove that AXBZ = AXCY.
(c) Prove that XL is the diameter of the circumcircle of AALM. :
(Source: Mandelbrot)

In some of the following problems, we will explore in search of the solution. After some of these
problems, we will present brief solutions based on our discoveries. The purpose of the exploration is to
give you some insight into how an experienced geometer thinks about problems. The purpose of the
solutions following the exploration is to give you a feel for how to write up clear solutions once you
find them.

of square ABCD. AM and AN meet BD at X and Y, as shown. Show that


BX = XY = YD.

Solution for Problem 19.8: We make our list of what we know and what we want. We want to prove a
statement about lengths, so we start with information we know about lengths.

What We Know What We Want


AB] Bos CD = DA BX XY eS XD
BM = MC = CN = DN

515
CHAPTER 19. PROBLEM SOLVING STRATEGIES IN GEOMETRY

It sure seems reasonable that BX = YD, but we have to prove it. Our main tool for proving segments
equal is congruent triangles, so we look for congruent triangles in our diagram. AABX and AADY look
congruent, so we go after these two triangles.

Important: Draw precise diagrams. Triangle congruences, parallel lines, collinear


vv points, and much, much more stand out in precise diagrams.

Since ABCD is a square, we have AD = AB and ZADB = ZABD = 45°. We just need another angle
to prove congruence. The angles at X and Y don’t look easy to work with, but angles ZXAB and ZDAY
are also parts of right triangles ABAM and ADAN. Since AD = AB and BM = DN (both are half the side
length of the square), we have ABAM = ADAN by LL Congruence (or by SAS Congruence). So, we have
LXAB = LMAB = ZNAD = LYAD, which means AABX = AADY by ASA Congruence.
We have more information for our table now. Most importantly, we’ve reduced our problem to just
proving BX = XY. We quickly brainstorm for a few other statements that are equivalent to BX = XY and
we see that BY = 2BX or BY = 2DY will also give us what we want. Also, if we show BX = BD/3 or
XY = BD/3, we'll be finished.

| What We Know What We Want


Ap =e Goes DA BX = XY
BMi= MG= GN = DN BY =2Bx
Bxe= Xo BYL=2DYi
AADY = AABX AY,= BDIS
|eeee a oe)

This gives us a lot to shoot for. We don’t have any more congruent triangles that are interesting to
investigate, but the ratios in our “What We Want’ list suggest looking for similar triangles. Parallel lines
mean similar triangles. Specifically, we have ADYN ~ ABYA because DN ||AB. Seeing BY = 2DY in the
‘What We Want’ list, we focus on what ADYN ~ ABYA tells us about BY and DY:

gigs DIN nag GDI? ail


BY be eh eee
We've found something we want! Now we retrace our steps to write a nice solution:
Since AB = AD, ZABM = ZADN and DN = BM (each is half the length of a side of the square), we
have AADN = AABM by SAS Congruence. Therefore, ZDAY = ZDAN = ZMAB = ZXAB. Together
with AB = AD and ZABX = ZYDA (since each is 45°), we have AABX = AADY by ASA Congruence.
Therefore, BX = DY.
Se
Extra! Josey and Beth are standing 100 feet apart with a 100 foot rope connecting their ankles
mii such that the rope is pulled taut 1 inch above the ground between them. Josey steps
one foot towards Beth, so the rope is relaxed along the ground. Is it now possible to lift
the center of the rope high enough that a person can walk under it without ducking or
moving either of the girls? Is it possible for Josey to lift the rope above her head without
her and Beth getting closer (and without her lifting her foot)?

TE
CS
19:3: 7PROGFS

Because DN || AB, we have ZNDY = ZYBA and /YAB = ZYND, so ADYN ~ ABYA. Therefore,
DY/BY = DN/AB = (CD/2)/AB = 1/2. Therefore, DY = BY/2, so BX = BY/2 and XY = BY - BX = BY/2
also. Thus, BX = XY = YD. (See if you can find other solutions!) 0

Problem 19.9: ©A and ©B are externally tangent. D is on @A


and E on ©B such that DE is a common external tangent of
the two circles. OC is tangent to OA, ©B, and DE as shown.
Given that a is the radius of ©A, b is the radius of ©B, and c is
the radius of OC, prove that

&Fr

Solution for Problem 19.9: There’s not much working backwards we can do here, but we can at least get
rid of the fractions by mutliplying our desired equation by Vabc. This gives

Vab = Vbc + Vac.

Going forwards, we can use all the information we know about tangent circles and lines. We connect
centers and draw radii. The perpendicular lines thus formed and the square roots in our target equation
both point in the same direction: the Pythagorean Theorem. In looking for a good right triangle to build,
we remember Problem 12.22, in which we built a right triangle to find the length of a common tangent
to two circles. Inspired by this, we draw a line through C perpendicular to AD and BE. Rather than
listing everything we know ina table, we add it to our diagram:

A large diagram inwhich we keep close track of what we discover isa very
oflective, way to.keep track of VWhat We Know.
Now we have our right triangles. Since XDFC and CYEF are rectangles, we have XD=CF=YE=c.
Therefore, AX = a—cand BY = b—c. Since AC = a +c, we can apply the Pythagorean Theorem to AAXC
Sia
CHAPTER 19. PROBLEM SOLVING STRATEGIES IN GEOMETRY

to find

1hE VAC2 — AX?


(a +c)? -(a-c)?
= V4ac
=) RYT

Our result is part of the equation we want to prove! Since XCFD is a rectangle, we have DF = 2 vac as
well. Therefore, the common external tangent of tangent circles with radii a and c has length 2 ac. We
can apply this result to our other two pairs of tangent circles to find FE = 2 Vbc and DE = 2 Vab. Since
DE = DF
+ FE, we have
2 Vab = 2 Vac + 2 Voc.
Dividing this equation by 2 Vabc gives the desired
ites 1 " 1
ve Va” Vo
Notice that we used both of our tactics from earlier in this chapter. We drew many extra lines to build
right triangles and we labeled lengths we could find in terms of our variables. As we did so, we found
lengths that are in our expression we sought. This brought us right to the solution. 0

“Concept: Always compare new problems to problems you have already solved. |
| O= Problem 19.9 is very similar to Problem 12.22, in which we found the _
length of a common tangent. Thinking of this common external tangent |
problem gave us a quick path to the solution of our new problem.

Problem 19.10: In AABC let L be the foot of the angle bisector x


ESS ;
from A to BC, and let M be the midpoint of BC. We also draw
the circumcircle of AABC. Finally, we construct the circle through
points A, L, and M. This circle intersects the rest of the diagram in
several remarkable ways, so we label points Z, Y, and X where the
circle meets AB, AC and the circumcircle of AABC, respectively.
(a) Prove that BZ = CY. /\
(b) Prove that AXBZ = AXCY. B POSS
(c) Prove that XL is the diameter of the circumcircle of AALM.
(Source: Mandelbrot)

Solution for Problem 19.10:


(a) We don’t have an obvious pair of congruent triangles to go after to show that BZ = CY. (We
suspect that we'll need BZ = CY to tackle the triangles in the second part, so we don’t go after
those triangles immediately.) We pull out our other length tools that this diagram invites us
to use. The circles suggest Power of a Point. Point B gives us (BZ)(BA) = (BM)(BL) and C

518
SSS 19.3. PROOFS
lence

gives (CY)(CA) = (CL)(CM). These equations include our target lengths! The Angle Bisector
Theorem gives us AB/BL = AC/CL, which has one of our target lengths, and the midpoint gives
us BM = CM. We put all this information together:
_

| What We Know What We Want


(BZ)(BA) = (BM)(BL) B= CY
(CY)(CA) = (CL)(CM)
AB/BL=AC/CL
BM = CM

Solving our first two equations in the What We Know column for BZ and CY gives us BZ =
(BM)(BL)/(BA) and CY = (CL)(CM)/(CA). We’d like the expressions on the right sides of these
equations to be equal:

What We Know [ What We Want


(BZ)(BA) = (BM)(BL) Bz GY
(CY)(CA) = (CL)(CM) (BM)(BL)/(BA) = (CL)(CM)/(CA)
AB/BL= AC/CL
BM
= CM

Since CM = BM, our second line of what we want reduces to BL/BA = CL/CA. This is just the
reciprocal of the third line in What We Know! Now we have a path from What We Know to What
We Want, so we can write our proof:

From the power of point B, we have (BZ)(BA) = (BM)(BL), or BZ = (BM)(BL)/(BA).


The Angle Bisector Theorem applied to bisector AL of AABC gives BL/BA = CL/CA, so
BZ = (BM)(BL/BA) = (BM)(CL/CA). BM = CM because M is the midpoint of BC, so
BZ = (CM)(CL/CA). Finally, the power of point C gives us (CY)(CA) = (CL)(CM), from
which we have CY = (CM)(CL/CA) = BZ.

(b) The previous part gave us BZ = CY, so if we find two pairs of xX


equal corresponding angles in ABZX and ACYX, we will prove
the triangles are congruent.
We have circles, so we look for angles that are inscribed in
the same arc. Angles 2XBZ and ZXCY are the same as /XBA
and ZXCA, respectively. These two angles are both inscribed
in XA of the larger circle, so we have ZXBZ = ZXCY. Unfor-
tunately, we can’t do the same for any other pair of angles of B
our triangles. We therefore start chasing angles, looking for a
pair of equal angles that we might relate to the angles in our
triangles.
We continue looking for equal inscribed angles, and find ZAZX = ZAYX since both are inscribed
in AX of the smaller circle. We can quickly relate each to angles in our triangles:
ZBZX = 180° — ZAZX = 180° — ZAYX = ZXYC-.

Therefore, by ASA Congruence, we have AXBZ = AXCY. Notice how marking equal angles and
sides as we find them makes the congruent triangles stand out.
ET a eee ae eee ee
Ba AS
CHAPTER 19. PROBLEM SOLVING STRATEGIES IN GEOMETRY

(c) We don’t have a lot of tools for proving a segment is a diameter


of a circle. One possible approach is to show that one of the
angles inscribed in either XYL or XZL is a right angle, thus
making the arc 180°. We consider the labeled points on the
small circle, looking for a point to be the vertex of our sought-
after right angle.
Marking the equal sides given by AXBZ = AXCY helps
guide us. Our congruent triangles tell us BX = CX, so AXBC
is isosceles. Therefore, midpoint M of BC is also the foot of the
altitude from X to BC. Since ZXMC is a right angle, ZXML =
ZXMC, and ZXML is inscribed in XYL, we have XYL = 180°.
Therefore, XL is a diameter of the small circle.
As an Exercise, you'll find another approach to this part.

Exercises Be

19.3.1 In triangle ABC, let the bisector of angle BAC intersect BC at D and the circumcircle of triangle
ABC at E. Prove that AE - DE = BE?. Hints: 260
19.3.2 In the diagram, ZA bisects ZBZC, and YA bisects ZBYX. Prove that ZYCZ + Xx
LYBZ =2Z2YAZ; Hints? 134

19.3.3 Prove ZLZX = ZLYX in Problem 19.10 without using the fact that XL is a € B
diameter of the small circle. Use this to find an alternate proof that XL is a diameter >
of the circumcircle of AALM. Wa WN

19.3.4 Quadrilateral WXYZ has right angles at ZW and ZY and an acute angle at W Z
/X. Altitudes are dropped from X and Z to diagonal WY, meeting WY at O and ER,
P as shown. Prove that WO = PY.
19.3.5x Let A, B, and C be points on a line, in that order. We draw semicircles
on segments BC, AC, and AB. Then the figure they enclose is called an arbelos, V¢
which is shaded in the diagram below at left. The arbelos has many interesting
properties, two of which we prove in this problem.
= "SOs ye? x
(a) Take point D onarc AC such that DB 1 AC. Prove that the arbelos has the same area as the circle
with diameter BD. Hints: 340
(b)x Line BD from the previous part divides the arbelos into two parts. Inscribe a circle in each part.
Prove that the two circles have equal radius. Hints: 295, 330, 284, 221

520
19.4. SUMMARY

19.4 Summary

This entire chapter is about problem solving strategies. Here are several of the most useful approaches
we used to solve problems throughout this book.

e When in doubt, build right triangles. One very common way to do so is to draw radii to points of
tangency. Nearly always try this when you have circles and tangents in a problem. Altitudes of
triangles and trapezoids also often produce useful right triangles.

e Connecting points that are originally not connected in a diagram can be extremely useful! This
doesn’t mean you should connect everything in your diagram immediately, however. Look for
segments to draw that will be helpful, particularly those connecting important points, or those
that form segments, triangles, or angles you know something about immediately.

60°, 30°, and even 120° angles are good signs to try to build 30-60-90 triangles by dropping altitudes
or extending segments.

Consider extending segments that stop suddenly inside figures such as triangles, quadrilaterals,
or circles.

e If you successfully use a certain tactic in a problem to get some information, but you still haven’t
solved the problem, try using that same tactic again in a different way. Maybe it still has more
information to give!

Label lengths as you find them, even if you have to label them in terms of an important variable.

e Assign variables to important angles and use your mastery of basic geometry to express other
angles in terms of those variables. Typically, you should first assign variables to angles you seek
and to angles you know a lot about.
e Once you start chasing angles, reach for all the angle tools you know, most notably the facts you
know about triangles, straight lines, parallel lines, and angles that intersect circles.

e When trying to find a geometric proof, keep careful track of what you know and what you want.

Draw large, precise diagrams — triangle congruences, parallel lines, collinear points, and much,
much more stand out more clearly in large, precise diagrams than in small, scribbled ones.
e Compare new problems to problems you have already solved.

“Challenge Problems

19.11 AXYZ isa right triangle with right angle 2X. P is on XZ. The triangle is folded over YP so that
point X lands on side YZ. Given that XY = 6 and PY = PZ, find, with proof, UXZ Ye

$$
hie
CHAPTER 19. PROBLEM SOLVING STRATEGIES IN GEOMETRY

19.12 Square LMNP is inscribed in right triangle ABC as shown. Given


PN = 6, compute (AP)(NC). (Source: ARML) ih M

19.13 Four circles of radius 2 are arranged so that each is tangent to two
others, and their centers are the vertices of a square of side length 4. A small
circle is inside the square such that it is tangent to the four circles. Whatis P Nec
the radius of the small circle?

19.14 AABC and APQR are equilateral in the diagram at right. AQ, RC, and A
BP are congruent and bisect their respective angles. The four interior regions
AQPB, AQRC, BPRC, and QPR all have the same area. Given that AB = 20,
what is PB? (Source: MATHCOUNTS)

19.15 Point P is inside rectangle ABCD. Prove that AP? + CP? = BP? + DP?.
Hints: 344, 380 5 LP en aR C
19.16 WXYZ isa parallelogram. M is the midpoint of WZ and O is the midpoint of YZ. MO and XZ
meet at N. Find ZN/NX.

19.17. In AABC, ZBAC = 60°, ZACB = 45°, and D is on BC such that AD = 18 and ZBAD = ZCAD. Find
the area of AABC. Hints: 411, 443, 473

19.18 In the diagram at right, the circle and the square have the same center. If the
area of shaded region ABC equals the area bounded by CD and minor arc CD, compute
the ratio of the side of the square to the radius of the circle. (Source: ARML)
19.19 In quadrilateral ABCD, AB || CD. AC and BD meet at E. Points M and N are the
midpoints of AE and DE, respectively. BM and BE trisect ZABC, and CE and CN trisect
ZBCD. Prove that ABCD is a rectangle. Hints: 501, 539

19.20 Four congruent circles are tangent to each other and tangent to the edges of
a sector as shown. If the straight edges are joined to form a right circular cone with
vertex at P, the radius of the base would be 2/3 the slant height of the cone. Compute
the ratio of the radius of the sector to the radius of each circle. (Source: ARML) Hints:
562, 535
19.21 ABCD isa square. Parallel lines m,n, and p pass through vertices A, B, and C, respectively. The
distance between m and n is 7 units, and the distance between n and p is 9 units. Find the number of
square units in the area of square ABCD. (Source: MATHCOUNTS) Hints: 502, 468
19.22. Chords XY and TU of OO bisect each other. Furthermore, XY = TU. Prove that the two chords
meet at the center of the circle.

19.23
Inthe figure, points B and C lie on line segment AD, e G
and AB, BC, and CD are diameters of circles O, N, and P, eee yt.
respectively. Circles O, N, and P all have radius 15, and the a GRAD BaD. X,
line AG is tangent to circle P at G. If AG intersects circle N D
at points E and F, then find EF. (Source: AHSME) Hints: ee ey
502, 418

See
CHALLENGE PROBLEMS

19.24 Find the volume of a sphere that is inscribed in a regular octahedron with side length 12. Hints:
337, 279, 242
19.25 Inthe diagram at left below, O is the center of the circle. Find the area of the shaded region given
that ZAOC = 120° and OM = MC = 6.

Figure 19.5: Diagram for Problem 19.25 Figure 19.6: Diagram for Problem 19.26

19.26 Squares are erected externally on the sides of AABC as shown at right above. The altitudes from
vertices A, B, and C are drawn and extended to divide these squares into rectangles. Let R;, forl <i <6,
be the areas of the rectangles as shown. Prove that R2 = R3, R4 = Rs, and Rg = R;. What does this result
say if ZACB = 90°? Hints: 496, 446

19.27 Shown at left below is right rectangular prism ABCDEFGH. Prove that AACF is acute. Hints:
396, 365

BA A B
g

a ras
y G D G

Figure 19.7: Diagram for Problem 19.27 Figure 19.8: Diagram for Problem 19.28

19.28 M and N are the midpoints of diagonals BD and AC, respectively, of trapezoid ABCD at right
above. AB ||CD, AB = 8, and MN = 6.
(a) Prove that MN || CD. (Find a rigorous proof — this is one of those ‘obvious’ facts that requires a
careful proof.) Hints: 333
(b) Find CD. Hints: 146, 492

19.29x A circle in the plane has center O. Two chords, with midpoints M and N, <Op
intersect at P. Prove that MN < OP. When does equality occur? Hints: 206, 239 -*

19.30x Let R and S be points on the sides BC and AC, respectively, of triangle
ABC, and let P be the intersection of AR and BS. Determine the area of triangle ABC
if the areas of triangles APS, APB, and BPR are 5, 6, and 7, respectively. (Source:
USAMTS) Hints: 286, 251

523
CHAPTER 19. PROBLEM SOLVING STRATEGIES IN GEOMETRY

19.31 In the diagram to the right, points A and B are on OO. Point C is on major
arc AB. Line ¢ is the angle bisector of ZACB and line m is the perpendicular bisector
of AC. Lines € and m meet at point P, point M is the midpoint of minor arc AB,
and minor arc AB has measure 0. Prove that ZAPM = 0/2. (Source: Mandelbrot)
Hints: 313
19.32 Sphere Z is tangent to all 6 edges of regular tetrahedron ABCD. Given that
each edge of the tetrahedron has length 12, find the radius of sphere Z. Hints:
205, 230, 268
19.33x AABC is an equilateral triangle with side length 6. Prove that for any point
P we choose inside AABC, the sum of the distances from P to the sides of AABC is the same. Hints: 110

19.34% Uand Care points on the sides of AMNH such that N


Mu = s, UN = 6, NC = 20, CH = s, HM = 25. If AUNC
and MUCH have equal areas, what is s? (Source: HMMT)
Hints: 275
19.35% Let C; and C2 be two circles that are externally
tangent, and let their centers be O; and O2, respectively. Let M
P be a point on C and let Q be a point on C2 such that PQ
is acommon external tangent to circles C; and C2, and let M
be the midpoint of PQ. Prove that ZO;MO> = 90°. Hints: 283, 308

19.36 Four balls of radius 1 are all tangent to each other. What is the radius of the smallest sphere
that encloses them? Hints: 334
19.37x Diagonals AC and BD of regular heptagon ABCDEFG meet at X. Prove that AB + AX = AD.
Hints: 359

BOM
Ne ae ere eee
HINTS TO SELECTED PROBLEMS

Hints to Selected Problems

. Build a right triangle in which sinA = x.

. Ifa line is tangent to @A at the point P, then the line is perpendicular to AP.

. Start with right triangle ATUV with right angle at U. Build a circle like the one we used in the
previous parts.

. To show that AC = BC if CM bisects ZACB, try using the Angle Bisector Theorem.

. Find the area of the whole figure two different ways.

. What is A’B’/AB? What about A’C’/AC and B’C’/BC?

. The center of each circle must be on the graph of what equation?


. Do you notice anything special about AE and DF?

. Consider the SSA examination we did in Problem 3.14.

. Find the radii of the little circles. Draw segments from the center of the big circle to the points
where the little circles are tangent to the big circle.

. Let VX hit the base of the cone at Z. What kind of triangle is VXZ?
. (For 3YZ > XY.) Note that AXAB is equilateral. Apply the Triangle Inequality to AAYB and AYZB.

. How many little triangles do we add in the first step? In the second? In the third? In each step after
the third? What is the ratio of the area of each triangle added to the area of each triangle added in
the previous step?

525
HINTS TO SELECTED PROBLEMS

14. Let the smallest exterior angle have measure x.

ahs), Why must ACDF be a rectangle? (Don’t forget to use AD = 25.)

16. Let the legs be a and b. And don’t forget the expansion of (a + b)?!

Wes How did we find the sum of the exterior angles of a triangle? Can we try essentially the same tactic
here?
18. Draw altitude BX of AABC. What kind of triangles are AABX and ABXC?

i), Draw the diagonal of the square that passes through the center of the little circle. Find the length
of this diagonal in terms of the radius of the little circle.

20. Connect A, B, and C to the center of the circle of which the path is a part. Call this center O.

Pa Let the triangle be AABC, and the point on the circumcircle be P. Let X and Y be the feet of the
altitude from P to AB and AC, respectively. Describe the circumcircles of APXA and APYA. (Proving
the existence of the Simson line is pretty tough! We'll explore it more in the next two volumes of
this series.)

pap Build a right triangle with XC as one of the sides.

29. Let WZ = x. Label the other arcs in terms of x based on what we are given in the problem.

24. Prove that ABDF = AEFD. Can you show that any other triangles are congruent to AEFD?

2D: Notice that median AM is half as long as the side to which it is drawn.

26. If a translation maps E to C and F to B, then what must be true about EC and FB?

27 Notice you have a midpoint of one side of AABC — what do you know about midpoints and areas?
What segment does this suggest drawing?
253 How are the sides of the pentagons related?

20: Try working backwards. Let our circles be OO and ©P with Y on ©O and Z on OP such that YZ is
tangent to both circles. Furthermore, let ro and rp be the respective radii of the circles. Build right
triangles.
30) Build a useful right triangle with the radius to A as hypotenuse.

SE Build more right angles.

Sys Show that EC = FB and EC ||FB.

33. Are our target angles corresponding angles of triangles we can prove are congruent?

34. Extend AB past B.

BS. Extend AQ to point Z on BC.

36. For the second part, notice that if we can find one set of a, b, and c for which it fails, then we are
finished.

BNae ae eae aa
HINTS TO SELECTED PROBLEMS

O/. Suppose D is inside the circle. Extend AD and CD to meet the circle again at X and Y, respectively.
In terms of arcs of the circumcircle, what is ZA + ZC? Can you prove this must be less than 180°?
What if D is outside the circle?

38. What do we get when we multiply the two areas together? What lengths are included in this
product?

. What is [ABC]?
. Start with the Pythagorean Theorem, a* + b? = 73+. Solve for a”. Can you factor the result? Use
clever trial and error to make your resulting expression a perfect square.

. Let T be the center of the circle. Extend VU to hit the circle again and extend UT to hit the circle
twice. Use Power of a Point.

. Let A be the midpoint of YZ, B be the midpoint of XZ, and G be the centroid. Let AG = x and
BG = y. Find other lengths in terms of x and/or y.

. What kind of triangles must AAOB, ABOC, ACOD, and ADOA be?

. How did we prove that the area of a triangle equals its inradius times its semiperimeter? Can we
do something similar here?

. Find the ratio of the volume of each remaining piece to the volume of its original wedge.

. Let ZC = x and ZD = y. Label as many angles as you can in terms of x and y until you can write an
equation.

. Prove AQ = RP. Can you find some congruent triangles now?


. Can you find ZBCA by considering what you know about AABC?
. Draw one of the segments. How far are the endpoints of the segment from the center of the circle?
. Can you find the volume of each piece we cut off?
. If you were given several pieces that were shaped like the region described in the first hint, and
you were given one piece shaped like the shaded region, could you build the square?

523 Deal with the hour hand and the minute hand separately. Where does the minute hand point? The
hour hand?

53. Must all the interior diagonals pass through the same point? How far is this point from each vertex
of the cube?
54. Write some equations involving ZRQZ. For example, it can be combined with ZZQP to make ZRQP.

DO: Build some right triangles.

56. What do you know about the little triangles on the outside?

a. Build a 30-60-90 triangle by dropping a well-chosen altitude.

eS ~~
HINTS TO SELECTED PROBLEMS

58. Find similar triangles to express PX and YR in terms of sides whose lengths we know.

SEP The centers of all the faces of the octahedron together are the vertices of what kind of polyhedron?

60. Consider how we built a right triangle for Problem 19.1.3.

61. Draw the altitude to the side of length 6. Can you find the length of this altitude?

62: Start with the power of point A and note that AX = AW + WX and AZ = AY + YZ. Let WX = YZ
as given. Expand, rearrange, and factor, factor, factor.

63. Why must each plane of symmetry intersect one pair of faces in lines that are lines of symmetry of
those faces?

64. What cross-section should you consider?

65. Solve for a and c in terms of b and d, then use the Pythagorean Theorem.

66. Unroll the rail!

67. This problem is the same as asking: ‘I have a 7 by 7 square whose sides are blue. How can I cut it
into 7 pieces of equal area such that each piece has the same total length of blue segments from the
original square in its perimeter?’

68. Let the three polygons have a, b, and c sides, respectively. If we add the measure of an interior angle
of each, what should we get? What should we get if we add the exterior angles?

69. What is ZSTQ?

TAS), Find more similar triangles. Draw DB and EC.

TAN Find and mark equal angles. See any congruent triangles?

Vig Prove that the perimeter of the second triangle is half the perimeter of the first triangle.

Vis We did a very similar problem in the text. Go back and study it for guidance.

74. In terms of s, the side length of the hexagon, what are the areas of the regions inside the hexagon
but outside ABCE?

[sy No matter where C is, AB is always the same. Given that AB = 2, what must we determine to find
the area of AABC?

76. What kind of triangle is AGJM?

we What is [XYC]/[XYZ]?

78. To find the area of AAOB, draw altitude OX from O to AB. What kind of triangle is ABOX?

79. OB and OA are radii of the same circle.

80. Let and


P Q be the circumcenters of triangles ABE and BCE. What is the relationship between PO
and BD?

i
HINTS TO SELECTED PROBLEMS
eerie tiered

81. Draw a picture. To do so, figure out the distance from the center of the man’s circle to the closest
point of grass beneath the man’s hat, then figure out the distance from this center to the farthest
point of grass beneath the man’s hat.

82. What does BC = AC = DC tell us?


83. Look at how you solved Problem 12.1. Try drawing the same extra line here.

84. (For 3YZ > XY.) Draw AXYA and AXZB outside AXYZ such that AXAY = AXYZ = AXZB. What
kind of triangle is AXAB?

85. How are 2WYX and ZWXY related? How are 2WYX and /ZYX related?

86. Is AADF equilateral?


87. What is ZABP?

88. Do you see any triangles that look congruent?


89. What is [BXY]/[BXA]?
90. Connect O to the midpoints of AB and CD.

91. Write the Power of a Point relationship in terms of ratios.

92. Be careful; this is a little different than the similar problem in the text. Specifically, the fold connects
points on opposite sides of the rectangle!

93. Can you consider the desired length as the altitude of a tetrahedron?

94. Look at the diagrams for constructions in this section. See any 90° angles?
95. Length ratios and medians should make us think of using centroids. Draw median CK.

96. Why is MN ||BC?


97. Consider triangles AACO and ABDO.
98. Write [AXC] and [BXC] in terms of the areas you used for your area ratios in the previous hint.
Use some clever algebra and the ratio statements you came up with in the last hint to show that
[AXC]/[BXC] = AF/FB.

99. Draw the altitude from E to AB. See any similar triangles?

100. To show all the diagonals pass through the same point, consider the midpoint, O, of one of the
interior diagonals. Show that this point is mid-way between each pair of opposite faces of the cube.
101. Start with the Pythagorean Theorem, a” + b” = 977. Solve for a”. Can you factor the result? Can you
find a value of b that makes both factors perfect squares?

102. Call the length of one side of the original poster x. What are the areas of the old poster and the new
poster in terms of x?

103. Use Power of a Point to show that WX = YZ and BX = CZ.

529
HINTS TO SELECTED PROBLEMS

104. Consider Problem 13.22. Find the power of the point with respect to each circle. Can they possibly
be the same?

105. When you draw BCD and its image B’C’D’, you should have 6 outer triangles and a hexagon in
the middle. What do you know about those 6 outer triangles?

106. Problems involving regular hexagons can often be made easier by dissecting the hexagon into 6
equilateral triangles.

107. Let AF and EG meet at X. Look at the angles of ACEX.

108. What do we know about triangles AWXY and AYZW that might be useful?

109. Extend AB past B; call the point where this extension meets m point H.

110. Consider the areas of triangles PBC, PCA, and PAB.

ite Unfold the tetrahedron.

112: We want to show that (AB + CD)/MN = 2. Find ratios equal to AB/MN and CD/MN.

113: The order of the vertices in the statement AABC ~ AADB is important! Use them to write an
equation in terms of side lengths.
114. Connect the vertices of the hexagon to the center of the circle. What kind of triangles do you form?

aller Under reflection through a plane of symmetry, what is the image of the vertex of the cone? The
base of the cone? The center of the base of the cone?

116. What kind of triangle is AABC?

147. Take another look at Problem 8.31.

118. What do you know about the diagonals of a kite?

119; Let our legs have lengths x and y. Write two equations for x and y.

120. Let the regular polygon be A;A2A3 -- - A, and let O be the point where the angle bisectors of 2A,A 1A?
and £A;A2A3 meet. Can you prove OA; = OA?? Can you use this to prove OA2 = OA3?

E21: Let AC and BD meet at Y, and let BD meet AC at X. Start with sector ABC and take out pieces you
know how to handle.
122. Consider Problem 13.22.

123. What is the second angle? The third? The nth? What is the sum of these measures? What must
this sum equal?
124. What is the ‘blue’ perimeter of each piece? Area of each piece?

125: PD is an angle bisector.

126. This problem is not nearly as hard as it looks. Don’t try to find the nonoverlapping areas; try to
find their difference.

aaa
RRR STEIT
HINTS TO SELECTED PROBLEMS

127. In terms of arcs of the incircle, what is EDF? How about ZA?

128. Show that MQ and MR both equal MT.

129. Build a 45-45-90 triangle by dropping a well-chosen altitude.

130. If we count all the sides of the pentagons, and all the sides of the hexagons, how many times do
we count each seam?

131. Draw the altitudes from the endpoints of the shorter base.

132s Draw a line through B)and the vertex of the square from the first hint that is not on a side of
AABC. Let this line hit AC at F. Can you build the desired square with F as a vertex? (And more
importantly, can you prove this is your desired square?)

i. Find EG and FH.


134. Find both ZYCZ + ZYBZ and ZYAZ in terms of angles at Y and Z. (For example, consider AYCZ.)

135. Let the smallest angle be x. What are the other two angles in terms of x?

136. Find ZACD and ZBCF in terms of ZA and/or ZB. (Don’t forget what you know about the median to
the hypotenuse of a right triangle!)
13/7, What Pythagorean triples have 50 as the hypotenuse? (We also strongly suggest trying to find an
algebraic solution! Let x be the initial height of the top of the ladder and y the initial distance from
the wall to the base.)

138. Forget about everything after C; and just find CC; and AC}. Next, just look at right triangle CCiA
with altitude CC and just find C;C2. Then take another step and find C2C3.

139. Consider a cross-section that contains the common axis of the cones.

140. What kind of triangle is AABC?

141. Each segment in the first diagram is broken into how many segments in the second diagram? What
is the ratio of the length of each segment in the first diagram to the length of each segment in the
second?

142. Review the diagrams that showed us that SSA failed. See if you can play with the diagrams to find
the cases where SSA succeeds. There’s one tricky case that’s not suggested in these diagrams. In
what case could we determine another pair of corresponding angles are equal?

143. You have two cases to consider. Can the hypotenuse be odd if both legs are even? Can we have an
even hypotenuse if one leg is even and the other is odd? (Use the Pythagorean Theorem!)
144. Let ABCD and EFGH be opposite faces of the cube, with AE, BF, CG, and DH as edges of the cube.
Let N be the midpoint of EG. Show that ON = AE/2 and that ON is perpendicular to face EFGH.
. Don’t forget that OA and CD are parallel!
. Let AC meet BD at E. Find AE/EN. Find EN/NC.
Cee
E nnn EE a-<i
HINTS TO SELECTED PROBLEMS

. Can you find some similar triangles?

. Don’t forget there are three different angles that could be the vertex angle!
. Reinterpret the problem in terms of area and perimeter.

. Let AB = x. What is BD? What is AD? What is CD in terms of x?

. Let the triangle be AABC, with ZA as the largest angle. Write an equation with the information
given.

. Extend RY and RX to meet PQ at Cand D, respectively.

. The parallel lines of the previous part give us similar triangles.


. Is it possible for a plane of symmetry to pass through exactly 1 vertex of the tetrahedron? How
about 3? 4? 0?

. Is there another tetrahedron with the same volume?

. Use your equations to write ZZQR in terms of ZPQR — ZPRQ.

. Try adding all your equations together.

. Are XN and YN corresponding sides of congruent triangles? Which triangles? Why are they
congruent?

. Draw a square with opposite vertices B and O.

. Note that (a + b)* = a* + 2ab + b?.


. What is the length of the altitude to the side of length 10 cm of the first triangle?

. Let the point where XC and AB meet be M. What is [XAM]/[XYC]?

. From the last hint, or from the diagram in Problem 3.14, you might have deduced that BC;C
is isosceles. Show that ZBC,;A + ZBC2A = 180°, then consider the sum of the angles in your two
possible triangles.
. Let the cube have edge length s. What is the length of an edge of the octahedron?
. Draw BD. Do you have a pair of congruent triangles?

. Use your equation to find DE/AB. Don’t forget that AB = CD = CE + DE!


. Are triangles XAY and YBX congruent?

. Show that AAA’A” = AXX’X”.


. In the previous part, you should have shown that if X is on BC, then AB? < AC? + BC2, which is
the opposite of what we are given. Follow similar steps to show the same is true if X is beyond B
on CB. What does this tell us?
. Find two pairs of similar triangles. You may need to use a little algebra!

i a TDL TN
HINTS TO SELECTED PROBLEMS

171. There are lots of parallel lines. How are the two triangles in this part related?

bY 4. Note that RQ = RZ — QZ, which looks a lot like what we want.

173. What is AX + BY?

174. The region described in the first hint is part of sector AOD.

175, Consider the cross-section that also includes the points where the spheres touch the wall and the
floor.
176. Show that DQ = AP = PB = CR.
ae Let O be the center of the scoop, A be the vertex of the cone, and B be a point on the circumference
of the cone. What kind of triangle is AOBA? What is ZOAB?
178. Can you prove that VY = VZ?

179. Show that AD/AE = AB/AC. Note that AB = AD + BD and AC = AE + EC, and that you are given
an equation you can use to get an expression for AD.

180. Consider the reflection of ZAOC over XY.


181. Approach 1: Is there a single line through which all three cuts pass?

182. Look back at the ways to prove two lines are parallel. Then consider ZZ and ZXYZ.

183. M is the midpoint of BD if BM = MD. Are these segments corresponding parts of congruent
triangles?

184. In the Review Problems of this chapter, we found the radius of a sphere inscribed in a regular
tetrahedron. Will the same approach work here?

185. Consider a cross-section that contains the apex of the original pyramid, the center of the base, and
the midpoints of opposite sides of the base.

186. Let the dimensions of the prism be a, b, and c. Write equations for the given information, then find
abc and use that to find each of the dimensions.

187. Consider a circle centered at M with radius 8.

188. Find FG first, then find RG.

189. Use the first part to find NM/BM. Compare this ratio to AM/AB. Do you have a pair of similar
triangles?

190. How is [FOB] related to [DOG]?

191) Draw BE.

192: Let each of the marked angles have measure x. Find ZBAE in terms of x.

193. What do you know about the diagonals of a kite?

CCCCC COCOCC”™~—~———”—C
HINTS TO SELECTED PROBLEMS

194. Continue YN to hit XZ at R. What do we know about YR?

i), Draw a radius of the small circle to a point of tangency, and a radius of the large circle to a vertex
of the hexagon. What kind of triangle can you thus form?

196. What do you know about the third angle of each triangle?

EE, Do any triangles in the diagram look congruent? (You may have to consider points that aren't
labeled!)

198; Show that AGEH ~ AAED, where E is the midpoint of Be.

198 Draw a circle with center C and a judiciously chosen radius.

200. What are the exterior angles in terms of ZA, ZB, and £C?

201 Can you construct a square that has one side on BC and one vertex on AB? Try using this square to
find the square you want.

202. Chase both angles. Relate 4QCB and ZQBP to other angles in the diagram.

PSs Draw a line segment and place the points on it. Label all the distances you can determine.

204. Let O be the intersection of the perpendicular bisectors of HI and GH. Show that AOIJ = AOGH.

205. Let W, X, Y, and Z be the midpoints of AB, AC, CD, and BD, respectively. What kind of quadrilateral
is WXYZ? What is WX?

206. What is ZOMP?

207, What kind of triangle is AEGH?

208. Focus on the base BCD. What happens when this is rotated 60° about its center?

207) What is 2X + ZY + 4Z? How are ZX and ZY related?

2105 Let XY meet AB at M. Show that AAMX = AAMY.

alle Each resulting piece has five faces. Carefully draw a diagram and use it to figure out what sort of
shapes these faces are. (Don’t forget what shape each face of the regular tetrahedron is!)
212. What is [AMB]?

213; What is ([ADF] + [BFC])/[ABCD]?

214. Can you prove that Y is the midpoint of FH?


215. Build a 45-45-90 triangle by dropping a well-chosen altitude. You may have to extend a side.

peep For the ‘only if’ part, draw altitudes from the short base to the long base and find congruent right
triangles.

UE, Let X be the leftmost point of the shaded region. Can you find the area of the region bound by AX,
BX, and AB?

nee
oo“ a ee ee
HINTS TO SELECTED PROBLEMS

218. Just unrolling the outside of the cylinder isn’t enough; Arnav has to go inside the glass!

21. Look back at how we have done problems in this chapter with frustums.

220. Show that AXYL = AXZL. (Why must YL = ZL?)


PPANE Let O be the center of the largest semicircle. What are the following in terms of r,s, and t: the radius
of the largest semicircle, OX, OB, and OY? What kind of triangle is AOXY?

22m What is ZPTQ?

228; Find [PXQ]/[ABC] in terms of PQ and BC.

224. Did you see a similar problem in the text?

tee Let there be k right angles. Note that each of the other angles must be less than 180°. So, the sum of
the angles must be less than what quantity?
226. Can you find QD?

APE Can you find the lengths of the sides of the triangle? What kind of triangle is it?

228. Since O is tangent to AB and AC, what do we know about the segment connecting A to the center
of the circle?

od, What kind of triangle is AABC?

230. The first hint was for the slick method. The rest of the hints are for a more mechanical route. How
far is the center of the sphere from a face of the tetrahedron? (Note: This is the same as asking
what the radius of the sphere inscribed in the tetrahedron is.)

Bol, What two area ratios do we know are equal to AF/FB?

BOL. Combine your expressions for ZACE and ZBCD to find ZDCE.

205. Still don’t have it? Time for a different approach. Let O be the center of each square. Connect O to
each vertex of ABCDEFGH. Can you find the areas of the triangles you form?

234. When the ball bounces off the rail, how far will the center be from the rail?

Za. What kind of triangle is AAGB? Can you find an expression for BG? How about CG and DG?

236. Draw the figure. Pay close attention to the vertices of the second square.

VE Draw OD. What kind of triangle is AAOD?


230: Your desired region is two cones with the same base. What segment in your cross-section corre-
sponds to the radius of this common base of the two cones? What does your diagram for finding
this radius have in common with Problem 5.17?

Zod. Consider the circle with diameter OP. Through what other points in the diagram must this circle
pass?

aa
| lUhUCCCUCUCUCUCUCUC™C™”™~“C~s~“—s—s™s™STS*t*tSstS~CS~s«aS
HINTS TO SELECTED PROBLEMS

. What general type of quadrilateral might be useful?

. Is there an angle in the diagram equal to ZAOC that is easy to find?

. Let ABC and ADE be faces of the octahedron with centers G and H, respectively. Let M and N be
the midpoints of BC and DE. Find the following: MN, AG/AM, GH.
. Prove ASPD = AQPA = ASRC = AQRB.

. What is [WPX]/[WXYZ]?

. Let one polygon have a sides and the other have b sides. Write two equations using the given
information.

. Consider quadrilateral OIJHG.


. We have lots of right angles, and we’re looking for an angle. Maybe the Pythagorean Theorem will
help. Build right triangles.

. Look back at Problem 2.14.

. What type of cevian is BE in AABC?


. What kind of quadrilateral is ABCD?
. Let [SPC] = x and [CPR] = y. What is [SPC]/[BPC]? Use this to build an equation.

MDE = DEER:
. Approach 2: How do our regions of cheese after the cuts correspond to orderings of x, y, and z?

. Prove that ARYX ~ ARCD. (You may need to find some congruent triangles first!)
. What do you get if you add this ratio to the ratio in the previous part?

. Find the angle between AA, and B,C; in terms of arcs of the circle. Don’t forget that B,A = Be
ee 7 BCy etc.

. Find EH/EF using similar triangles.

. Find two pairs of similar triangles. Write two equations involving x, y, and AC.
. Don’t just hunt blindly for solutions once you have an equation set up — some of the solutions are
very surprising. Find an organized way. Here’s a hint: If + + ,+ + =1,and x, y, and z are positive
integers, then at least one of these integers must be 3 or smaller (otherwise, the sum i Me SEs I will
definitely be smaller than 1). :
260. Find a pair of similar triangles such that the segments in BE* = (AE)(DE) are among the sides of
the triangles.
. What is [XPQ]/[XBC] in terms of PQ and BC?
SSS HINTS TO SELECTED PROBLEMS
sll SS

262. Forget about A for a minute and pretend we know where B is. What is the shortest path that goes
from B to a point on y = 6, then on to point D?
263. Similar triangles will help you find the side length of the second square base of the frustum.

264. Let CE meet OA at Y. What type of triangle is AOCY?

265. Can you find the area of AMEN?

266. Let AB < AC. (The proof for AC < AB is essentially the same.) Arrange the triangles so that A and
A’ coincide and B and B’ coincide. Does AC’ intersect BC? Why or why not?
267. Draw OX and PY. Draw a line through P parallel to XY. Consider the right triangle you thus form.

268. Consider the triangle formed by connecting the center of the sphere, the center of face ABC, and
the midpoint of AB. The hypotenuse of this triangle is the desired radius.
269. Find some similar triangles. You may want to rearrange the equation you're given.

270. Let OO be the circle tangent to the four given circles. Apply the Triangle Inequality to AOPR.

evi. Let X be the point where the wall meets the floor and let M be the midpoint of the ladder. How is
MxX related to the length of the ladder?
ae2. The region described in the last hint is part of sector ABC.

273. Focus on a single side of the first triangle. What happens to it as we go from the first figure to the
second figure?
274. Can you find three congruent triangles that have DH, HF, and DF as corresponding sides?

ek. Draw MC. What is [MUC]/[NUC]?

276. Let one of the legs have length x. Write an equation.

277. Extend the sides of the octagon; what type of quadrilateral is formed?

278. Make sure you track the path of the center of the ball, not the edge of the ball. You shouldn’t be
aiming at the reflection of B over the rail, because the ball will bounce before the center reaches the
rail!

ary. The points of tangency are the vertices of what figure?

280. Show that AADE, AEDF, and AFDC are congruent.

281. What are two ways you can find the volume of tetrahedron ABCF?

282. For the hour hand, can you figure out how much of the distance from 11 to 12 the hour hand has
covered by considering how much of the 11 o’clock hour has transpired?
209. Connect M to the midpoint of O; 02.

284. You have to use the fact that the little circle is tangent to the largest semicircle somehow. What does
this fact tell us? What segment should we draw to use this fact?

A CC CCC CSCSCts*C~—“‘CS;3RSTCté<;<CS;7}F*é<S~S~*;*;*;:C
HINTS TO SELECTED PROBLEMS

. What is [ABX]/[ABC]?

. Draw CP.

. Let M be the midpoint of BC and N be the midpoint of CD. How is AAOP related to AAMN?
. Let ZABO = x, ZOBC = y, and ZOAC = z. Find other angles that equal these. Find x + y + z.
. What arcs must you show are equal in order to show that ZXAM = ZYAM?

. What kind of triangle is AMOY? ANOX?

. Connect the endpoints of the chords to the center of the circle. What kind of triangles do you form?
What is ABCD?

. View the frustum as the result of chopping a small cone with radius 7; off the top of a cone with
radius 1p.

. To show that XL is a diameter of the circle, note that XZLY is a cyclic quadrilateral. What does this
tell us about ZXZL + ZXYL?

. Let the radius of the circle be r. Find PB in terms of r. How can you use Power of a Point?

. Let X be the center of the small circle on the right, and let Y be the foot of the perpendicular from
X to AC. Let r be the radius of the little circle, and let s and t be the radii of the smaller semicircles.
Find XY in terms of r, s, and t in two different ways.

. What triangle is similar to AEMC?

. Break the problem into cases in terms of the types of cross-sections formed by the intersecting
plane.

. Prove that both AP and RQ equal YZ/2.

. See Problem 8.4.3.

wirovetiatioh Eb = Dr) DB = DE/DE.

. If you answered the first part using trial and error, go back and read those hints!
. Let point X be some point besides B on line m. Consider AOBX and show that OX > OB. Does this
prove that m can’t hit the circle a second time?
. Lots of right angles and parallel lines mean lots of similar triangles. Use similar triangles to find as
many lengths as you can.

. To find the area of AAOX, draw an altitude from A. What is ZAOX?

. Let O be the circumcenter of AJKL. Why must point O be on the altitude from J to KL? Let OK = x.
Find some other lengths in terms of x.

. You're looking for a ratio, so find some useful similar triangles.


. How is CC; related to BC? How is CC) related to CC;? And so on.
eee
HINTS TO SELECTED PROBLEMS

308. Let the midpoint of the last hint be N. What does MN equal?

309. [PAEC])'= [PAE] +[PEC].


310. Let V be the vertex of the cone, O be the center of the base, and X a point on the circumference.
What kind of triangle is AVOX? What is VX?

311. There’s nothing special about A!

312. Draw an altitude from Q to PR.


313. Show that ZPAC = ZPCA = 6/4.

314. Are the triangles similar?

315. Find some congruent triangles with YX and WZ as corresponding sides.

316. Draw a segment representing one of the longest lines-of-sight. Build a right triangle by connecting
the common center of the semicircles to the point of tangency of the segment to the smaller
semicircle, and connecting the center to an endpoint of the segment.

317. Compare the sides of AACP to those of a 30-60-96 triangle. What similarity theorem can you use?

318. Find the edge lengths of the area of intersection. What sort of shape is the intersection space?

319. What is [AHF]/[AEF]?

320. Consider a 60° rotation about R.

321. What's left over when you take AEDC and AFBD away from AABC? What portion of this is AAEF?

322. The triangle is isosceles, so the perpendicular bisector of AC iis also an altitude, median, and angle
bisector. Can you find the length of the altitude from B to AC?

323. Extend PO to hit the circle again at Y.

324. Write the given area equation in terms of sides (or parts of sides) of the rectangle.

325. Let ZX meet AY at B and ZD meet AY at C. Show that AZBC = AZYC.


326. Can you dissect the original triangle and rearrange it to make another triangle with two sides of
length 13?

327. Look at how we did a very similar problem in the chapter, then try to find the distance from the
corner of the room to the center of the sphere in two different ways.

328. What kind of quadrilateral is WXYZ?

329. Let the smallest square have side length x. Find all of the lengths in the diagram in terms of x.

330. Let P be the center of the semicircle with BC as diameter. What kind of triangle is AXYP?
331. Start with the circumcircle of AABC.

ae os = = === seg
HINTS TO SELECTED PROBLEMS

332. What kind of triangle is AACF?

333. Show that both M and N are on the median of the trapezoid.
334. What sort of shape do we form if we connect the centers of the little spheres?
335. What other portions of the diagram have area equal to the shaded area?

336. Write each of the arcs in terms of PQ. Can you find the measures of the arcs now?
337. At what point on each face is the sphere tangent to the octahedron?

338. Pretend you already know what point on BC to go to. What point should you aim fcfor to find the
point on BD to go to? Once you know that, how can you figure out what point on BC to go to?

339. Parallel lines mean similar triangles. Let the diagonals meet at point E.

340. Let the radii of the small semicircles be s and t. Draw the full circle with diameter AC. Continue BD
to meet this circle again at E. What do we know about BE and BD? What tool considering lengths
and circles can we use to find BD in terms of s and ft?

341. Show that ABCD and A’BCD’ are congruent trapezoids by showing that all the corresponding
angles and sides of the two are the same.

342. What kind of triangle is AXQZ?


343. Show that PQ = PR and PX = PY. Use these to find another pair of equal lengths.

344. Squares of side lengths and a rectangle suggest building right triangles. Lots of them.

345. After substituting your expression for AB into your expression for AC*/MO*, you should be able
to factor out BC? in the numerator. Try multiplying both the numerator and the denominator of
the resulting expression by NO?. You should then have an NO* — MN? term in both the numerator
and denominator. Cancel them!

346. Triangles AAEB and APEA share an altitude from E.

347. Let the ladder be AB. What kind of triangle is AABX? How is XM related to it?
348. Let I be the incenter of AABC and X be the point where the incircle is tangent to AC. What are the
lengths of the sides of IXAZ?

349. Use similar triangles to show that CQ/PQ = AC/PB.

350. Find similar triangles that have CD and BD as corresponding sides.


351. Let ©O be the circle that is tangent to our given three circles. Either ZOQR or ZOQP is at least 90°.

352. You should already have YZ. Let the distance from X to the point where the incircle touches XZ be
x and don’t forget that AXYZ is a right triangle.
353. ‘Try using area.

540
HINTS TO SELECTED PROBLEMS

354. Let the radius of the sphere be r. What is the radius of the cylinder? The height of the cylinder?

BOD: We know a lot about triangles. Consider building a triangle by extending one of the segments.

356. What is the angle between where the minute hand points and a line from the center of the clock to
the ‘12’ on the clock? How about the hour hand?
ois Let C; and C; be the two potential point Cs. What kind of triangle is BC;C2? What does this tell us
about ZBC,A and ZBC>A?

358. We know a whole lot about the angles of a triangle now. Build some triangles.

359. Rotate the heptagon so that the image of D is on AC.

360. What angles equal ZZQW? What angles equal ZR?

361. Find the ratios of [PBS], [PQA], and [ARB] to [PQRS]. How is the sum of these ratios related to
[ABP]/[PQRS]?

362. Let the side length of the hexagon be s. Find the ratio of the area of a square to the area of the
hexagon. How about the area of one of the triangles? (Try doing these without actually finding s!)
363. What triangle similarity would allow you to deduce DE ||BC? What ratio of sides must you show
is equal to AD/AE to deduce these triangles are similar?

364. After using Power of a Point, write everything in terms of PQ, PX, and RX. Rearrange and do some
clever factoring.
365. Find the squares of the side lengths of AACF in terms of edges of the prism.

366. Proving that EY will pass through G is the same as proving that EG passes through Y.

367. What is the ratio of the area of the large equilateral triangle to the area of one of the small equilateral
triangles?

368. Must the diagonal connecting the incenter of AWXA to the incenter of AYZA pass through A?

369. Can we combine Sue’s and Barry’s answers in a way such that Sue’s base is multiplied by Barry’s
altitude and vice versa?

370. Consider the reflection of A over BD.

o71. Connect P to the midpoint of MN. How long is this new segment? (Remember, AMPN is a right
triangle!)

3/2. Prove that A’B/A’C = AB/AC. What happens if you add 1 to both sides of that?

373. What is [BFC]/[ADF]?

374. At how many different points can two circles meet? How about two lines? How about a line and
a circle?

379. Let the medians meet at G. What do we know about XG and YG?

iaa a a
HINTS TO SELECTED PROBLEMS

376. What is [ABC]/[ACD]? What is [ABC] + [ACD]?

O77. Angle bisectors and side lengths. Try the Angle Bisector Theorem.

378. Let X be the point where OP meets YZ. In terms of our radii, what is OX/XP?

572: Parallel lines mean similar triangles!

380. Draw altitudes from P to the sides of ABCD.

381. Draw altitudes from P to each of the sides of the rectangle.

382. Find a couple pairs of similar triangles.

383. In any given hour, how is the total amount of water that flows through the trapezoid related to the
combined total amount of water that flows through the holes?

384. We have a 30° angle. Build a useful 30-60-90 triangle by drawing the altitude from B to AC.

385. Write the correct Power of a Point relationship. Combine it with Jake’s accidentally correct equation.

386. What kind of quadrilateral is ACNM? (Don’t just guess — you have to prove it!)

387. We know the power of point P.

388. What piece of information did you not use in the first part? Can you use this along with the result
of the first part to find similar triangles?

eiekey Connect the center of the sphere to the centers of the circles of which the wires are arcs.

390. What is [QPX]/[WPX]?

CEM Can we still cut the quadrilateral into two triangles?

oo Using your two equations, try to find two different approaches to get the solution. First, try guessing
integer solutions (use the equation for xy first!) or use your knowledge of Pythagorean triples.
Then, for an alternative solution, expand (x + y)* and use your equations.
B95: What is (AF)(AD)?

394. Let the parallelogram be ABCD. Draw altitudes from A and B to CD. See a rectangle?

OID: (For XY > 2YZ.) Start anew diagram. We want XY > 2YZ, so if we pick a point Q on XY such that
YQ = YZ, all we have left to show is that XQ > YZ.

396. What inequality can we use regarding the sides of a triangle in order to show that the triangle is
acute?

B97. What did you learn in the previous problem?

5903 Show that in each step after the first step, the number of triangles added is 4 times the number of
triangles added in the previous step. How much smaller is each triangle added in a given step than
the triangles added in the previous step?

BA ea
HINTS TO SELECTED PROBLEMS

9! Mark equal angles and find lengths equal to DZ and lengths equal to CY.

400. Do we know the sum of the interior angles of the figure?

401. Find ZCAB, ZACD, and ZBCD in terms of ZB.

402. What is the length of the altitude from O to face ABC?

403. Find CF first.

404. What kind of triangle is AEFG?

405. Let the semicircles have centers Q and R, and let S be the point besides O where they meet. Draw
QS and SR.

406. Can you build a useful right triangle with EI as hypotenuse?

407. What are segments BP and MN in AAMB?

408. Look back at the tactic we took for a similar problem in the text.

409. Find OS a few different ways.

410. The remaining two pieces of the region enclosed by the bold line after you draw the square are
triangles. Can you find any triangles congruent to these?
411. Find AC and the length of the altitude from B to AC.

412. How is [XAD] + [XBC] related to the area of ABCD? What about [XCD] — [XAB]?

413. Can you think of a right triangle that must be similar to a triangle that satisfies the conditions of
the problem? (Make sure you include why the triangles are similar!)

414. The ratio of the sides of one pentagon to corresponding sides of another is always the same. Each
of the angles of one pentagon equals the corresponding angles of the other. What does this suggest
about the two pentagons?

415. Can you sketch a Power of a Point-like diagram in which we have segments of lengths a, b, and
Vab? Now use your straightedge and compass to recreate this diagram.

416. Draw radii from the centers of the circles nearest A and C to the points where these circles are
tangent to the square.

417. Can you find some congruent triangles?

418. Draw the perpendicular from N to EF and connect G to P.

419. What is the image of BN upon rotation about the center of the square?

420. Use similar triangles to find the distance from E to each vertex of the trapezoid.

421. What kind of triangles are AABD and ABCD? Mark all the angles you can find.

422. Are there other diagonals equal in length to BH that are easier to compare to AE?

—————E
COC~™—S«S
ee Ph
HINTS TO SELECTED PROBLEMS

. Note that (2n*)(2m7) is a perfect square.


. What is [ADB]/[ABC]?

_ Let O be the circumcenter of AWXY. What do we know about OW, OX, and OY?

. These two triangles share a side. What do we have to show equal to conclude that their areas are
equal?
. Find all the angle measures you can. Label the diagram. Keep going until you get to x!

. Connect the centers of the circles to points of tangency and to each other. Have you built figures
you know how to handle?

429. Consider a cross-section that includes the axis of the cylinder and the cone. Note that the top base
of the cylinder is parallel to the base of the cone.

430. Call our two points inside the triangle P and Q. If we draw PG, we will hit two sides of the triangle
(possibly at a vertex). Let PQ hit sides AB and AC at points P’ and Q’, respectively. We want to
show PQ < (AB + BC + AC)/2. Start by noticing that PQ < P’Q’.

431. Expand (J + w +h).

432. Can you combine the ratios from the first two parts in some way to get the ratio in this part?

433. Use ZROS.

434. What is [EHG]?

435. What kind of triangle is AYTT’?

436. Let ZBAC = x. Find as many angles as you can in terms of x. Particularly, use the angles you find
to get expressions for ZABC and ZACB in terms of x.

437. You can break quadrilateral P’Q’CB into triangles. Can you then prove P’Q’ is less than the sum of
the lengths of the other three sides?
438. Let E be the intersection of the diagonals. Can you find the areas of the four small triangles that
each have a vertex at E?
439. Let X be the foot of the altitude from O to AB and Y be the foot of the altitude from O to AD. Show
that AAOX = AAOY. Why does this mean O is on the diagonal through A?
440. Create similar triangles that will allow you to construct a point X on OP such that OX/XP = ro/rp.

441. Can you write an equation involving ZPRQ and ZZQR? How about ZPZQ and ZZQR?

442. Let one side of a little triangle be x and the other y. Write some equations. And don’t forget that
(x + y)* — (x? + y?) = 2xy.
443. What is ZB? ZBDA?

co.
CSCSee
HINTS TO SELECTED PROBLEMS

444, What do you know about the sum of the exterior angles of a triangle? If you don’t remember, look
back in the text, or try to figure out something about the sum on your own!
445, Is there a rotation that maps CM to DN?

446. Let the foot of the altitude from A to BC be D and the foot of the altitude from B to AC be E. Find a
triangle that is similar to AADC. What is R2 in terms of segment lengths in your diagram?
447. Use your side length information to get information about angles.

448. What other segment lengths can you find? Draw some other significant segments that might be
useful!

449, Look back for a useful Exercise in Section 12.4.

450. Here are some pieces to consider: sector ABC, sector YBC, segment BY of OC, sector XCB, and
AACB. You should know how to handle these pieces. How can you assemble them to get the
shaded area?

451. Let the image of A be A’ and the image of B be B’. Let C be on AB. Show that the image of C is on
A’B’. What type of quadrilateral is AA’C’C?
452. B is on the surface of the sphere.

453. What is (Va + Vb)??


454. Let O be the circumcenter of AGHI. Can you prove that AGOH = AGOI?

455. Focus on AP and RC. Are there any triangles that look congruent that have these as corresponding
sides?
456. This problem has a lot in common with the problem in the text in which we found the length of a
common external tangent. Here, we know what the length of the common tangent is and we want
to find the distance between the centers. A similar tactic will probably work.

457. Don’t forget that the diameter of the semicircle is part of the semicircle’s perimeter!

458. Use OQSR to find the area of the shaded region that has O on its boundary. How is the area of this
shaded region related to the area of the other shaded region?
459. How is the diagonal in the first hint related to WAX?

460. Let AB < CD. Draw altitudes from A and B to CD. Use these altitudes, and the fact that AB and
CD have the same perpendicular bisector, to find some congruent triangles. (Marking all the right
angles in your diagram will help a lot!)

461. Find congruent triangles, then mark equal angles. Now find more congruent triangles!

462. What information given in the problem have you not used yet? Can you combine it with the first
two parts to find more similar triangles?

463. Here’s a starting point: Let AABC and ADEF satisfy the AAS criteria, such that ZA = ZD, 2B = ZE,
and AC = DF. Starting from here, prove these triangles are congruent using ASA Congruence.

545
HINTS TO SELECTED PROBLEMS

464. M is the midpoint of hypotenuse AC of AAHC.


465. Let O be the center of the wheel, A be the bottom, and B be the point on the wheel that is 10 feet
above the ground. Draw the altitude from B to OA so you can use the information that B is 10 feet
above the ground.

466. Let the diagonals meet at K. To show that ABCD is isosceles if ZABD = ZBAC, use AABK and ACDK
to show that AC = BD.

467. What is the length of XZ? Now can you find congruent triangles?

468. Draw altitudes from A and C to n.

469. Draw an altitude from B to AC.

470. Use the two area relationships we are given to find BS/RS and QA/QR. Can you use these to find
[ARB]/[PQRS]?
471. Draw the altitude from O to AC.

472. Reflect D over the line y = 6. This may not be the only reflection you need!

473. Draw altitudes from B and D to AC.

474. Write the area of AABC in terms of the areas of triangles I,AC, [,BC, and I,AB

475. How many little triangles are in the top row of little triangles? How many in the next row down?
The next row after that? And so on.
476. The diagonals of a parallelogram bisect each other!

477. Let O be the center of the base, as in our solution. If BC = 8, then what are the side lengths of
AOCA? What kind of triangle is AOCA?
478. What is ZPDO?

479. Connect the centers of the three circles. What kind of triangle do you form?

480. It’s a word problem. Try assigning a variable to one of the angles.

481. Write all the angles you can find in terms of x and y. Can you use your resulting diagram to write
some equations in terms of x and y?
482. Can you show that AD = BC? That AD = BF? That AE = DC?

483. To show that X cannot be beyond A on BA, investigate what happens to the angles of ACXB if X is
beyond A on BA.
484. Argue by contradiction. Assume XZ > XY. What happens if ZY > ZZ or ZY = ZZ?

485. If you're having trouble with the region bound by AX, AB, and BX, try finding ZACB.

486. What is the area of AEFG?

BAG ee
HINTS TO SELECTED PROBLEMS

487. Why must the center of each circle be (a,a) for some constant a? In terms of a, what is the radius of
the circle?

488. Don’t forget that AAXC is a right triangle.

489. Let O be the center of the sphere, and let C; and C2 be the centers of the circles. First, why must
OC; be perpendicular to the planes of both circles? Does this show that OC; goes through C2?
490. Can you find [ABC]? Are there any other triangle areas you can find?

491. What is the sum of the measures of the exterior angles of a triangle?

492. N is the midpoint of AC. Use this fact with your ratios from the first hint.
493. Suppose Orion and Michelle pick different numbers. Pretend you are Joshua — can you choose a
number such that your three numbers together violate the Triangle Inequality?

494. What is the area of the garden? Of the garden and the path together?

495. Consider a cross-section that includes the centers of both spheres. What other useful points should
you include in the cross-section?
496. Find some similar triangles.

497. What if AB = CD? (Make sure you go back and prove your answer works for all trapezoids that fit
the problem! Try building a right triangle.)

498. (For XY > 2YZ.) Label all the angles you can find in your diagram.

499. Find both angles in terms of ZABC.

500. Draw AC and AE.


. Note that CN is a median and an angle bisector of AECD. What does that tell us about AECD?

. Build right triangles.


. Draw perpendiculars from O to P; and to P2.
. Look at the diagrams from constructions in this section. See any 30° angles?

. Let XO = x and YO = y. What is XY in terms of x and y?

. LN is an altitude of AKLM.
. We used SSS Similarity in a similar problem in this chapter!

. Find the radius by finding the area of AAOB in two ways.

. Let AABC be our triangle and I be its incenter such that AI 1 CI. Find ZBAC + BCA.
. How is the perimeter we want to find related to the perimeters of all the triangles together?
. Find EA and ED.

547
HINTS TO SELECTED PROBLEMS

. Let the point where XP meets the circumcircle of AXYZ be point Q. Where do XQ and YZ meet?
What is QP in terms of PX, YP, and PZ?

. Start with right triangle AABC with AB = 1, C = 90°, and ZA = 15°. What must we find in this
diagram to evaluate sin 15°?

. Let OX = x. Find the area of the region bound by AX, DX, and AD in terms of x.

. When your wheels spin once, how far does your car go? How far does your speedometer think
you go?

. Cut the desired region into triangles.

. What is the sum of the interior angles?

. Can you prove ZVYZ = ZVZY?

. Note that ZABD = ZBDE. What does that tell you about AB and DE?

. Show that the center of rotation is on the perpendicular bisector of AC and the perpendicular
bisector of BD.

> Pind’Gr and AE:

. Solve one of your equations for AC in terms of x.

. Use the Pythagorean Theorem, a? + b* = c*. What happens if a and b are odd?
. Describe in words each of the sides of AXUS. What kind of triangle does that make AXUS?

. Use the Triangle Inequality a few times.

. What is a when AC = 7?

. Right triangles: Try the Pythagorean Theorem! Let AX = x and BX = y. Write two equations and
solve for x and y. Or, make a clever guess at x and y and see if your guess works.

. EU and NT are 5 units apart. How does this restrict the length of EN?
. Unroll the outside and the inside. How should you place these unrolled surfaces next to each other
so that Arnav’s walking path can represent going over the edge of the glass, from outside to inside
the glass?

. Suppose the two circles meet in three points — what is the circumcircle of the triangle formed by
connecting these three points?

. What type of triangle is AWZU?

. What does the first part tell you about QC? About QB?
. For proving 3YZ > XY, note that 3(20°) = 60°. Can you take advantage of this fact by constructing
two more copies of AXYZ, one on either side of the original triangle?

BAG SS ae
HINTS TO SELECTED PROBLEMS

534. IsAD = BR?

30: Focus on one circle. What is the angle of the sector in which one circle is inscribed? Build a right
triangle by drawing radii to points of tangency.

536. We don’t know much about 15° angles or 75° angles. Can you break AABC into an isosceles triangle
and a right triangle that we know how to deal with?
ae Where is the circumcenter of a right triangle?
538. Draw EB.

539. Prove that ABME ~ ACNE, and use this to show that ZABC = ZBCD.

540. Find the power with respect to both circles of the point where the chords intersect.

541. Find [BMC]. How are [BMC] and [BED] related?

542. What kind of triangle do you form when you draw an altitude from G to EF?

543. Draw diagonal AC. Find AC in terms of the desired radius.

544, Notice that cos 60° = 1/2. This makes the Law of Cosines particularly easy to use on triangles with
60° angles.

545. Let A be a point on ©O. Let A’ and O’ be the images of A and O, respectively. Show that A’O’ = AO
(you can’t assume this!). What type of quadrilateral is A’AOO’?

546. Draw a diagram. Mark equal lengths. See any isosceles triangles?

547. Make sure you investigate all possible diagrams!

548. Try to build a right triangle with XY as one side such that you know the lengths of the other two
sides.

549. The circumcenter, O, is on the altitude from B to AC. Let OA = x. What is OB? How far is O from
the midpoint of AC?

oye,O Solve for RZ and QZ in terms of RQ and the lengths in the expression we want to prove.

Dol We are given an inequality in terms of sides, and want to prove an inequality in terms of angles.
Use the side information to derive an inequality involving angles.

Doe. Draw altitudes from B and C to AD.

253. How far can C get from AB and still be on the circle?

554. Build a right triangle with the desired segment as a hypotenuse.

DDD. Let AP = x and AQ = y. Can you find all the rest of the side lengths in the diagram in terms of x
and/or y?
556. Let X be on @C; and Y be on @C>. Find some congruent triangles to prove OC; = OC2.

549
HINTS TO SELECTED PROBLEMS

557. Look at Problem 4.35.

558. What kind of trapezoid is WXYZ?

559. Where is the center of the circle?

560. We’re looking for a length. Try building a useful right triangle.

561. The altitude from A of your intersection is the same as the altitude from A of tetrahedron ABCD
(look back to the 3-D geometry chapter if you don’t remember how to find the length of this
altitude).

562. First find the central angle of the sector.


563. Let H be the foot of the altitude from R to ST. What kind of triangles are ARHS and ARHM?

564. Look at the diagram in the solution to Problem 9.8. What kind of triangle is ABXC?

565. Let M be the midpoint of PQ and N be the midpoint of RS. Consider the cross-section containing
AMRS.
566. Find BE first by letting BE = x.

567. Draw perpendicular segments from the feet of the perpendicular segments you drew in the first
hint to AB. Why must these meet AB at the same point? Call this point Q. What is OQ?

568. Let the point on Z that is initially touching A be point P. Where on Z is P after Z has moved
one-quarter of the way around A?
569. What kind of triangle is AAOC?

570. Label the points of tangency. Write each side of the equation you wish to prove in terms of segments
with these points as endpoints.

571. If the interior angles have integer measure, so do the exterior angles.

572. Perpendicular lines mean right triangles, and what you want to prove has squares of lengths. What
should you try?

573. Through what other point does plane WXZ pass? How does this help with finding the distance
from Z to this plane?

574. Use ABEA to find ZBEA. What does this mean ZDEB is?

575. Show that ABAF = AEDF.

576. What kind of triangle is AABD?


977. Cut the figure into two pieces you can find the area of: a right triangle and a rectangle.

578. Compare ZBC’C to ZBCC’.

579. Show that X cannot be beyond B on AB, and that X cannot be beyond A on BA.

so
HINTS TO SELECTED PROBLEMS

580. Why is AOPQ a right triangle?

Dol, Let the middle angle have measure x and the common difference between angles be y. Write an
equation!

582. What is the measure of ZEBF? Of ZEAD?

583. [PQA], [PBS], [ABP] are all part of the problem. What piece must we combine with these to make
[PQRS]?

584. What strategy did we take for a similar looking problem in the text (look back in this section).

585. Draw the altitude from A to BC.

586. Half the product of the diagonals is not the only way to find the area of a rhombus.

587. What is the sum of all the angles? What portion of this sum is the largest angle?

588. Let G be on AE such that GF 1 AE. Let DF = y.

589. Prove that AD bisects ZBXC and ZBAC.

590. Let O be the center of the original circle and A be a point on the circle. Let the images of O and
A under the rotation be O’ and A’, respectively. Find a pair of congruent triangles to show that
O’A’ = OA.

591, Build right triangles. Draw OA, OP and BP. See any similar right triangles?

oe fe Find two expressions in terms of y for EF.

O90. Let the hypotenuse of the triangle be AC. Find BC and AB in terms of AC. To use the inradius
information, draw the incircle and the inradius. Do you see any other segments equal in length to
the inradius?

594. Does AQTR share an altitude or a base with another triangle whose area you know?

ae
ccc
INDEX

Index

©, 231 Angle-Angle-Side Congruence Theorem, 61


aw | angle-chasing, 29
m1, 285 Angle-Side-Angle Congruence Theorem, 61
3-D Geometry, 356-407 angles, 13-48
30-60-90 triangle, 144 acute, 18
45-45-90 triangle, 142 adjacent, 19
alternate exterior, 27
AA Similarity, 101 alternate interior, 27
proof of, 119 between a tangent and a chord, 319
AAS Congruence, 61 between a tangent and a secant, 320
acute, 18 between two chords, 312
adjacent angles, 19 between two secants, 313
alternate exterior angles, 27 central, 286
alternate interior angles, 27 complementary, 22
altitudes, foot of, 86 corresponding, 27
AMC, vii dihedral, 359
American Mathematics Competitions, see AMC exterior, 36
American Regions Math League, see ARML inscribed, 304-310, 320
analytic geometry, 435-477 interior, 36
angle bisector, 171 reflex, 19
Angle Bisector Theorem, 181, 201 remote interior, 37
angle of depression, 502 right, 17
angle of elevation, 502 same-side exterior, 27
Angle-Angle Similarity, 101 same-side interior, 27
proof of, 119

S02
INDEX

straight, 21 congruent, 49
supplementary, 22 conjecture, 10
trisection, 338 construction, 7
vertical, 23 angle bisector, 197
Annairizi of Arabia, 162 copy an angle, 121
apex, 366 equilateral triangle, 72
apothem, 252 parallel lines, 121
arc, 6, 284 perpendicular bisector, 73
Archimedes, 289, 293, 407 perpendicular lines, 161
area, 83-88 contradiction, 41, 143
similar figures, 117 converse, 40
ARML, viii convex, 206
Art of Problem Solving, v, 562 coordinate geometry, 435-477
ASA Congruence, 61 coordinates, 436
axiom, 10 coplanar, 358
corresponding angles, 27
base, of a triangle, 86 cosecant, 487
bisects, 54 cosine, 480
buckminsterfullerene, 401 cot, 487
cotangent, 487
Cartesian plane, 436
CPGCIC, 54
central angle, 307
cross-section, 364
centroid, 183
csc, 487
chord, 6
cube, 363
bisected by radius, 54
cyclic quadrilateral, 322, 323
circles, 5, 284-304
cylinders, 380-384
area, 290
axis, 380
center, 5
chord, 6, 340 degenerate triangle, 275
circumference, 284 degree, 14
diameter, 6 Dehn, Max Wilhelm, 369
radius, 5 Descartes, René, 435
secant, 340 diagonal, 245
tangent, 340 diagonals, 207
circular segment, 294, 295 diameter, 6
circumcenter, 175 dihedral angle, 359
circumcircle, 175 dilation, 418-422
circumference, 284 dimensions, 2
circumradius, 175 distance formula, 441, 473
circumscribed quadrilateral, 327 dodecahedron, 372
collinear, 4 doubling the cube, 338
compass, 7
complementary angles, 22 eight-point circle, 212
concave, 206 Elements, 189
concurrent, 4, 174 endpoints, 2
cones, 385-390 equidistant, 3

553
INDEX
er

equilateral triangle, 68 if and only if, 227


Eratosthenes, 302 ft
Euclid, 189 image, 408
Euler line, 191 incenter, 180
Euler, Leonhard, 191 incircle, 180
excenter, 205 Inner Napoleon Triangle, 214
excircle, 205 inradius, 180
exradius, 205 inscribed angles, 304-320
Extended Law of Sines, 500 interior angles, 245
exterior angles, 36, 246 invariants, 369
inversion, 430
face diagonal, 362 irrational numbers, 143, 285
Fagnano’s Problem, 423 isosceles trapezoid, 213
Fermat numbers, 339 isosceles triangle, 67
Fermat prime, 339
Fermat’s Last Theorem, 151 kite, 240
Fermat, Pierre de, 151
Fibonacci sequence, 19 lateral surface area, 361
fixed point, 409 lattice points, 436
foot, 86 Law of Cosines, 494-500
frustum, 379 Law of Sines, 494—500
right circular, 389 Leg-Leg Congruence, 153
Leg-Leg Similarity Theorem, 155
Garfield, President James A. , 316 legs, of a right triangle, 133
generalization, 496 Lincoln, Abraham, 189
geometric mean, 117 line, 3
golden ratio, 12, 19 line of symmetry, 414
Golden Ratio Spiral, 12 line segment, 2
golden rectangle, 12 linear equation, 437
graph, 437 LL Congruence, 153
LL Similarity, 155
Harvard-MIT Math Tournament, see HMMT locus, 5
height, 86 lune, 294, 296
hemisphere, 407
Heron’s Formula, 159 major arc, 6
Hilbert, David, 56, 369 Mandelbrot Competition, vii
Hippasus, 143 map, 408
HL Congruence, 153 MATHCOUNTS, vii
HL Similarity, 155 medial triangle, 185
HMM, vii medians, 183
homothety, 422 Midline Theorem, 186
hypotenuse, 133 midpoint, 3
Hypotenuse-Leg Congruence, 153 Millennium Problems, 216
Hypotenuse-Leg Similarity, 155 minor arc, 6
Mohr-Mascheroni Theorem, 339
icosahedron, 372
identity, 409 Napoleon’s Triangles, 214

BER TT a
INDEX

nine-point circle, 193 Poncelet-Steiner Theorem, 339


noncollinear, 176 postulates, 10
noncoplanar, 358 Power of a Point, 340-355
nondegenerate triangles, 275 prisms, 360-366
problem solving, iii
obtuse, 18 proof by contradiction, 41, 143
octahedron, 372 protractor, 14
octave, 157
pyramids, 366-369
ordered pair, 436 Pythagoras, 142
origin, 3, 436 Pythagorean Theorem, 134-140, 147-152, 163
orthocenter, 189 Pythagorean triple, 147
orthodiagonal, 236 Pythagoreans, 142
Outer Napoleon Triangle, 214
OED 311
parallel lines, 24 quadrants, 490
Parallel Postulate, 26 quadrilaterals, 206-243
parallelepiped, 360 circumscribed, 327
parallelograms, 215-219 concave, 206
parity, 132 evclic, 323
Penrose tilings, 265 diagonal, 207
pentomino, 131 parallelogram, 215
Perelman, Grigori, 216 rectangle, 221
perimeter, 81-83
perpendicular, 17 radian, 289
perpendicular bisector, 72, 73, 170 radical axis, 352
perpendicular, foot of, 86 Radical Axis Theorem, 352
pi, 285 radius, 5
plane of symmetry, 418 bisects a chord, 54
planes, 4, 356-359 Ramanujan, 298
parallel, 358 rational numbers, 143
perpendicular, 359 ray, 3
Plato, 373 rectangles, 83, 221-224
Platonic solids, 373 area, 85
Poincaré Conjecture, 216 reflections, 310, 413-418
point, 2 reflex angles, 19
point at infinity, 430 remote interior angles, 37
point-slope form, 439, 473 resources, V
polygons, 244-266 rhombi, 219-221
angles, 246-249 rhombus, see rhombi
area of a regular, 252 right angle, 17
interior angles, 245 right rectangular prism, 360
regular, 245 right triangles, 84, 133-169
polyhedra, 360 area, 84, 85
regular, 370-374 inradius of, 327
polyhedron, see polyhedra rotations, 411-413
polyomino, 131
same-side exterior angles, 27

eer
ee ee
INDEX
ee LLL

same-side interior angles, 27 length of common, 325


SAS Congruence, 57 tangrams, 243
SAS Similarity, 111 tessellation, 264
scale factor, 418 Thales of Miletus, 306
scalene triangle, 71 Thales Theorem, 306
sec, 487 theorems, 10
secant, 310, 487 three-dimensional geometry, 356-407
secant line, 6 tiling, 264
sectors, 290, 291 total surface area, 361
segment, 2 transformations, 408-434
semicircle, 14 translations, 408—410
semiperimeter, 159 transversal, 26
Side-Angle-Side Congruence Theorem, 57 trapezoids, 208-214
Side-Angle-Side Similarity, 111 base angles, 213
Side-Side-Side Congruence Theorem, 53 bases, 208
Side-Side-Side Similarity, 113 isosceles, 213
similarity, 100 legs, 208
Simson line, 318 median, 208
sine, 479 Triangle Inequality, 274-279
skew lines, 358 triangles, 31
slant height, 366 30-60-90, 144
slope, 437 45-45-90, 142
slope-intercept form, 439, 473 altitudes, 86, 188-192
SOHCAHTOA, 482 angle bisectors, 177-182
space, 4 area, 87
space diagonal, 362 centroid, 183
spheres, 390-394 cevians, 170-205
surface area, 390 circumcircle, 175
volume, 390 congruent, 49-80, 152-156
spherical geometry, 80 degenerate, 275
squares, 83, 224-226 equilateral, 68
squaring the circle, 338 height, 86
SSA Congr... .oops, 65 incenter, 180
SSS Congruence, 53 incircle, 180
SSS Similarity, 113 inradius, 180
standard form, 439, 473 isosceles, 67
straight angle, 21 medians, 183-187
straightedge, 7 nondegenerate, 275
subtend, 287 orthocenter, 189
supplementary angles, 22 perpendicular bisectors, 173-177
symmetry, 52, 256 right, 84, 85, 133-169
scalene, 71
tangent, 6, 316, 480
similar, 99-133, 152-156
common external, 325
trigonometric identity, 484
externally, 278 trigonometry, 478-504
internally, 279

~.:C~S< Esti‘ié‘é‘’ySC®™*C#™SCsé‘étRNSO ogee


INDEX

unit circle, 489


USA Mathematical Talent Search, see USAMTS
USAMTS, viii

vertex angle, 67
vertical angles, 23
volume, 360

Without loss of generality, 276


WLOG, 276

x-axis, 436
x-coordinate, 436
x-intercept, 437

y-axis, 436
y-coordinate, 436
y-intercept, 437

Ei ai... ©. °. .57
|
= - <= pa ial #- at «We = oF © + ©; net

Cth shri tin


ait Pair cr (male) lesdarngtinM Ae .

/ is KIM

Ve lot xene?
~ cS zaluca laos
ern) Say ‘3? ry TOY :

. OU. tinronsp lo eel tee


| , Tv Oa
; oan ~
omaery
iidiael > zien
pained

| dE oO TA
f*AD Oe Ss ia . |, Reine , + ee) A arn ve Joovrsind-s

a ands 04 a3 Kha Weil Cane oe sae, @ :

mano teks Cnet Therese, 25 bie) R. ~ Alb coos a


oie Gro Saniaaiicy, 11° im lame, O22 86} visnifyouay
ice, ke : lowes, Die? ey depen
ee apenas
1, ee: range itenlagiy, OU-Pe
meBee, 08 Tings, 31 ee
hah St | BAL) 0 se, se
a | ; ABMS, 1 .
a thutes, 6A, Lae 9
sage Bleorsory, i?7-182
PIO) D SFI ‘ih
s
¢ } . 4
rj

& ee 4059 C ‘ ‘

i * A= 4 {

Siete § Vea) vi regs") ‘i labia nite


yy . fiw Us a ! Ti ; lt.
toby 1 ‘Wi

: = < t
7 af\ 1) “pone : whit “wv mats bb) = i view A i

ae | Va ve >’ as LMur - a val é ¥


7

¥ 3*¥ ‘ ‘ dj Fiat : ' ‘ ‘+3 ’ Ar ie


ow v Mars ere ® 4 tn, = bey ’ ‘ oe
7
ma
- s Plt ae) my
-
ve >
\

aero Wend @ swe whe! ae owehime idsg " 7


On? Qeculut seilhoad | ; :

stirs! Wek 4 ne hand Aig ie cainlact nvtaen ghee eee:


if

.
ae ie aaneg alkvabtySeat) oF s(AEN, Maggie arenes vei .
fairattany Virste dete 8-9 Oke, a ad
CYL Te, * areas Iriaign
5
www.artofproblemsolving.com
The Art of Problem Solving (AoPS) is:

e Books

For over 15 years, the classic Art of Problem Solving books have been used by students as a re-
source for the American Mathematics Competitions and other national and local math events.
Every school should have this in their math library.
— Paul Zeitz, past coach of the U.S. International Mathematical Olympiad team

Our new Introduction series of textbooks — covering Algebra, Counting & Probability, Geometry,
and Number Theory - constitutes a complete curriculum for outstanding math students in grades
6-10.
The new book [Introduction to Counting & Probability] is great. I have started to use it in my
classes on a regular basis. I can see the improvement in my kids over just a short period.
—Jeff Boyd, coach of the 2005 MATHCOUNTS National Championship team from Texas

e Classes

The Art of Problem Solving offers online classes on topics such as number theory, counting,
geometry, algebra, and more at beginning, intermediate, and Olympiad levels.

All the children were very engaged. It’s the best use of technology I have ever seen.
— Mary Fay-Zenk, coach of National Champion California MATHCOUNTS teams

e Forum

As of July 2009, the Art of Problem Solving Forum has over 64,000 members who have posted
over 1,490,000 messages on our discussion board. Members can also participate in any of our free
“Math Jams”.

I'd just like to thank the coordinators of this site for taking the time to set it up... I think this is a
great site, and I bet just about anyone else here would say the same. . .
— AoPS Community Member
e Resources

We have links to summer programs, book resources, problem sources, national and local com-
petitions, scholarship listings, a math wiki, and a IATgX tutorial.

I'd like to commend you on your wonderful site. It’s informative, welcoming, and supportive of
the math community. I wish it had been around when I was growing up.
— AoPS Community Member

e ...and more!

Membership is FREE! Come join the Art of Problem Solving community today!
INDIANAPOLIS-MARION COUNTY P.L.

the Art of Probiem Sl AK 3 1978 08131 9212

SS

The Art of Problem Solving Introduction Series constitutes a complete curriculum for
outstanding math students in grades 6-10. The books in the series are:
Introduction to Algebra by Richard Rusczyk
Introduction to Counting & Probability by David Patrick
Introduction to Geometry by Richard Rusczyk
see
Introduction to Number Theory by Mathew Crawford
Together these books give students a solid background in basic problem-solving
mathematics and prepare them for prestigious competitions such as MATHCOUNTS
and the American Mathematics Competitions.

Praise for Introduction to Geometry


“[My son] loves your textbook! As a former teacher I know how difficult it is to find a fantastic
textbook but yours nailed it! [His] words were ‘I love that book.’ In my twenty years of teaching,
I’ve never heard a student say that about a textbook before. When asked why he loved it he said
because it was easy to follow, and had lots of examples.”
Diana Cobbe, parent and teacher

Richard Rusczyk is the founder of www.artofproblemsolving.com. He is co-author of the Art of Problem Solving,
Volumes I and 2, and author of Introduction to Algebra. He was a national MATHCOUNTS participant in 1985, a
three-time participant in the Math Olympiad Sunimer Program, and a USA Math Olympiad Winner in 1989.

yISBN 978-1-934124-08-6

9" 781934" 124086


Digitized by the Internet Archive
in 2022 with funding from
Kahle/Austin Foundation

htips://archive.org/details/introductiontogeO0000rusc

You might also like